0% found this document useful (0 votes)
17 views

AMB MCQ

The document contains a compilation of multiple-choice questions (MCQs) related to medical scenarios, covering topics such as diagnosis, management, and treatment options for various conditions. It includes cases of cellulitis, testicular torsion, hypertension, and more, along with appropriate management strategies and diagnostic tests. The questions are designed for exam preparation for medical students or professionals in the healthcare field.

Uploaded by

M Elossaily
Copyright
© © All Rights Reserved
We take content rights seriously. If you suspect this is your content, claim it here.
Available Formats
Download as PDF, TXT or read online on Scribd
0% found this document useful (0 votes)
17 views

AMB MCQ

The document contains a compilation of multiple-choice questions (MCQs) related to medical scenarios, covering topics such as diagnosis, management, and treatment options for various conditions. It includes cases of cellulitis, testicular torsion, hypertension, and more, along with appropriate management strategies and diagnostic tests. The questions are designed for exam preparation for medical students or professionals in the healthcare field.

Uploaded by

M Elossaily
Copyright
© © All Rights Reserved
We take content rights seriously. If you suspect this is your content, claim it here.
Available Formats
Download as PDF, TXT or read online on Scribd
You are on page 1/ 345

AMB 5X2 Compiled MCQs

Updated till 10th Batch Group C


Collected Exam
AMB - Group C - 10th

1. Similar Q to this a case of cellulitis: A 60-year-old man with a past medical


history of diabetes and hypertension presents with swelling in his right leg after he
scraped his calf on the corner edge of his coffee table. On physical exam, there is a
poorly demarcated 10 cm red and tender plaque on his right calf. Some parts resemble
an orange peel. There is a superficial scrape in the middle of the plaque. What is your
management?
a) Cephalexin
b) Amoxiclav + doxycycline
c) Vancomycin
2. Similar Q Case of testicular torsion: A 15-year-old boy presents to the
emergency room with severe lower abdominal pain that awoke him from sleep about 20
minutes. The pain is sharp and radiates to his left thigh. While in the emergency room,
the patient experiences one episode of vomiting. His temperature is 99.3°F (37.4°C),
blood pressure is 126/81 mmHg, pulse is 119/min, respirations are 14/min, and oxygen
saturation is 99% on room air. Physical exam normal!, what will you do?
a. Urgent urinalysis
b. Consult urology
3. Female 50+ y/o, never done screening before, what to do screening?
colonoscopy, biennial mammography, cervical cancer (pap smear every 3 years)
Other options included: breast self-examination and abdominal US

4. 48 asking about AAA screen? Done once at age 65-75 in a smoking male
5. Patient taking metformin? stop when GFR <30

6. Patient with appendicitis, stable, RLQ pain & rebound tenderness, high WBC
which investigation you want to do????
a. CT abdomen / pelvis
b. Open laparotomy
c. Douplex US of testicles
*(no US in choices, nor laparoscopic / open appendectomy I cant remember the
age of the patient maybe 18 or 21??)

7. Fatigue case? ask what do you mean by fatigue

8. Post chemo fever? same as concept - not sure of the answer

9. 2 cases of IDA

10.
riedel's thyroiditis

11. VT (with pic) unstable (not sure), mng? Unsynch cardioversion


IV Amidarone 150

12. Panic attack case, sx resolved after 10 minutes, no abnormalities in tests.

13. PID case, management?


a) Metronidazole for 3 days
b) Doxycycline IM + azithromycin for 14 days (not sure)
c) Ciprofloxacin

14. Hypertensive emergency cases choose the right one?

15. Horizontal Nystagmus caused by? 6 or 4 CN

16. How to assess liver function and acute liver injury? PT or hypobilirubinemia
17. Not done post dural headache? Fluid restriction

18. Reduce wt? Glp alone

19. Case of dm uncontrolled by diet & risk assessment >15%? Take metformin and statin

20. tension pneumothorax case, mng? needle decompression

21. What is a sign of severe obstruction? Unable to speak or cry

22. Good history and documentation in the emergency department helps in? continuity
of care, avoid mistakes (not sure of this q)

23. Primary (idiopathic) is the most common cause of osteoporosis what the second
most common cause? multiple myeloma, hypoPTH, Calcium/vit D, and Progesterone
+ Another Q about what causes osteoporosis? Estrogen

24. hypothyroid typical case, what will you also find? delayed relaxation of reflexes

ion?
Non-

26. GABA agonist


a. Propofol
b. Etomidate
c. Rocuronium

27. A patient went rock climbing where he fell landing on his feet with his knees
extended. the knee was completely separated. An x-ray was done and it showed no
fractures. Which of the following is the most serious immediate complication?
a. compartment syndrome
b. Popliteal a. injury
c. Patellar ligament torsion
· No ACL in choices

28. During investigation of polyuria and polydipsia in 38 old female, BMI 28, her
fasting blood glucose is 6.6 urine analysis is normal and no glucose in urine?
a) Impaired glucose regulation
b) DM2
c) DM1
d) Normal patient
e) Diabetes insipidus

29. A Cocaine abuser came to ER with chest pain and tachycardia. What would you give
this patient to relieve his sx?
a. Benzodiazepine
b. Beta blocker
c. Calcium channel blocker

30. Pt taking lots of pain killers + pinpoint pupils, vitals were also given,
antidote is?
a. Naloxone
b. Flumazenil
c. Atropine
d. Cocaine

31. (2X) Child with sore throat, fever, maculopapular rash all over, in hands
and feet, conjunctival injection, strawberry tongue. Dx?
a. Kawasaki
b. Coxsackie (hand foot mouth) disease
32. Secondary prophylaxis for migraine?
A. Amitriptyline
B. Sumatriptan

33. Woman with pain at sole of the foot, worse in morning, better with long distance walking,
what is the Dx?
a. Plantar fasciitis
b. Osteomalacia
c. Calcaneal epiphysisitis
33. Preg and htn? Labetalol

34. IHD? Start with statins as he has established heart disease

35. Primary prevention in a patient with known COPD? Flu vaccine

36. Patient with first toe finger joint pain (proximal phalanx), urine sample showed ve
birefringent crystals Dx?
a. Gout

37. Young woman what to give for htn? Acei (name of drug ending with pril)

38. Sitalagptin

39. Toxicity of paracetamol, since yesterday, what to give? N acetyl

40. A man complains of 7 months of colicky abdominal pain, there is no blood in his stools, one
of his relatives died of colon cancer at age 75, what is your diagnosis?
a. Irritable bowel syndrome (IBS)
b. Inflammatory bowel disease
c. Colorectal cancer

41. Old patient dysphagia started with solid and now liquid he lost weight ?
A. Esophageal ca
B. Achalasia

42.

43. A 70 y/o male had blunt trauma. On


examination he has contusions the right side
measuring 2 cm. Both kidneys are of normal
size. RFTs al
a. Simple renal cyst
b. RCC
c. PCKD (Polycystic Kidney Disease)
d. Recurrent UTI
e. Prostatic nodular hyperplasia

44. PE case same as concept

45. Ci to ngt? Facial trauma

46. An 18 y/o healthy young man presented to ER with c/o palpitation and chest pain.
His chest is clinically positive for bilateral crepitations and observations are as follows:
Pulse 170/min, BP: 70/50 mmHg RR: 24/min. ECG showed the following. What is the next
management step?
a. Vagal maneuver
b. Adenosine 6 mg [One Senior Group, also can be used for regular narrow

Crepitations can indicate asthma or interstitial lung disease in this young patient
so Adenosine is CI]
c. Adenosine 12 mg
d. Synchronized cardioversion
47. Porter (Similar to custodians & janitors job) presents with medial
epicondyle pain and swelling. X-rays are normal. What is the most likely
diagnosis?
a. Golfer Elbow
b. Tennis elbow
c. Bursitis
48. STEMI, what drug decrease mortality? Aspirin
49. Sever episode of depression she had mild to sever depression 18 months ago and
was treated for 6 months what is the duration she has to be on ssri now ?
A. 2 yaers
B. Till symptoms improve
C. 4 weeks
D. Life long
50. Which of the following is the most sensitive test for diagnosing thyroid disorders?
a. TSH
b. T3
c. T4
d. Ferritin
51. Patient fractured his tibia 3 days ago comes back complaining of pain
and paresthesia. What is the most appropriate management?
a. Check compartment pressures
b. Reapply splint and follow up with orthopedics after 24 hours
c. Give analgesia and send home
d. Review radiographs
52. A 150 premature babies were followed since birth till school years. And they
were compared with 5000 normal born babies to see who had better
performance in the school. This is what kind of study?
a. Cohort
b. Cross sectional
c. Case control
d. RCT
e. Case series
53. Room spinning when the patient moves his head. What is the diagnosis?
a. BPPV
b. Meneiere
54. (1X) A study showed that simvastatin decreases the incidence of MI with a RR
of 0.8 and a CI of (0.6 1.2) what is your conclusion?
a. It is statistically not significant
b. It is clinically significant
c.

55. A 21 y/o lady just came from Africa and diagnosed with malaria what is the most
specific test to make Dx?
a. Thick and thin blood film for malaria
56. elderly BP 180/90 twice, what to do? start meds, measure again at home, use
ambulatory?
57.
58. A 58 y/o male has history of hypertension and wants to reduce his risk of
developing an MI. His HDL is 32 (normal is 40). What can increase his HDL?
a. Exercise (Mayo Clinic says: Get more physical activity. Within 2 months
frequent aerobic exercise can increase HDL cholesterol by ~5% in otherwise
healthy adults)
b. Fenofibrate
59. A man with hepatitis B develops ascites, jaundice, fever & chills, he looks ill, what is
your management?
a. Paracentesis, cultures & 3rd generation Cephalosporins
60. Which of the following tests would you do for a 60 years old male, nonsmoker,
with no medical history of note?
a. BMD
b. Mammogram
c. PSA
d. FOBT
e. US abdomen
61. In an unresponsive victim of trauma suspected head and neck injury, what is the
best method of opening airway?

a. Head tilt neck lift


b. Chin lift - head lift
c. Jaw thrust
d. Head tilt - chin lift
e. Do not attempt to open the airway
62. 46-year-old female presents to the walk-in clinic with severe epigastric pain for 36
hours, radiating to her back. Clinically, she has upper abdominal tenderness and
guarding. Her serum amylase is markedly raised. Which of the following is the most
likely diagnosis?
a. Acute gastritis
b. Oesophagitis
c. Acute Cholecystitis
d. PUD
e. Acute Pancreatitis

63. of the following drugs acts as antagonizing the NMDA Receptor?


a. Propofol (GABA)
b. Dexmedetomidine (alpha 2 agonist)
c. Ketamine
d. Midazolam (GABA)
e. Etomidate (GABA)
64. A woman is anxious, and she had tried psychotherapy sessions, but she's
still anxious. You will give her medical therapy. The first line would be?
a. SSRI (fluxetine)
65. Which of the following is correct regarding end tidal CO2?
a. 50 mmHg considered to be normal
b. the value increases with hypotension
c. with hypoventilation the value decreases
d. the sample analyzed taken from the blood
e. with hyperventilation the value decreases
66. a 69-year-old man presents with recurrent rectal bleeding with passing stool with
feeling of incomplete and straining. His wife has also noticed significant weight loss.
Which of the following is the most likely diagnosis?
a. Diverticulosis
b. IBS
c. Hemorrhoids
d. Colorectal cancer
e. Sigmoid volvulus
67. A study demonstrates that drug X for the treatment of hypertension has a greater
reduction of stroke than drug Y. the authors conclude that this should make drug X a
better choice with hypertension than drug Y. this study is a good example of which of
the following?
a. Observational analysis
b. EBM
c. Biased analysis
d. Patient oriented evidence
e. Disease oriented evidence
68. 50 years old female patient controlled DM, HTN going for elective laparoscopic
cholecystectomy here ASA Physical status is:
a. ASA 3
b. ASA 1
c. ASA 5
d. ASA 2
e. ASA 4
69. The most appropriate first line management for low back pain in a young and
healthy individual with no red flags is:
a. Bed rest
b. Analgesia with back care advice
c. Reassurance
d. Analgesia with lumbar spine X ray
e. Analgesia with MRI Scan
70. During IV insulin treatment in a patient with DKA, the most likely electrolyte
imbalance is:
a. hypokalemia
b. hypocalcemia
c. hypophosphatemia
d. hyponatremia
e. hypomagnesemia
71. Which investigation is the gold standard for diagnosis of COPD?
a. Spirometry
b. Peak flow diary
c. CXR
d. Urea breath test
e. Oxygen saturation
72. 31-year-old lady presents with generalized abdominal discomfort for 6 months.
The pain improves with defection. Abdominal examinations are normal except for
some mild distension. What is the most likely diagnosis?
a. Hemorrhoids
b. Crohns disease
c. GERD
d. Irritable bowel syndrome (IBS)
e. Inguinal hernia
73. A 32-yea-old FEMALE has been having headaches for 1 year. The pain is unilateral
and severe, associated with blocked nose and watery eyes. Attacks last for few hours,
and might occur daily for the period of a week or so. Then he would be asymptomatic
for 3-4 months, before the next bout of attacks. What is most likely the diagnosis?

a. Recurrent tension type headaches


b. Recurrent acute closed angle glaucoma
c. Cluster headache
d. Trigeminal neuralgia
e. Migraine with aura
74. A left frontal lobe stroke (Cerebrovascular Accident) in a 57-year-old lady would
likely lead to which of the following?
a. Anosmia
b. Cranial nerve palsy
c. Visual impairment
d. Hearing deficits
e. Changes in emotion and behavior
75. DNAR code status patient may not receive CPR and:
a. G-Tube feeding
b. NG Feeding
c. Intubation
d. IV Fluids
e. IV Antibiotics
76. 1:30 min for PCI
77. In a patient with acute anaphylactic reaction, the most appropriate treatment is:
a. Methylprednisolone IV
b. Epinephrine 1:1000 0.5 mg IM in a large muscle
c. Antihistamine IV
d. Attend breathing and intubation
e. Epinephrine IV
78. a study was conducted to evaluate the accuracy of Abdominal US compared to CT scan in
the diagnosis of Acute Appendicitis. This study is an example of which study design?
a. RCT
b. A cross sectional study
c. A case control study
d. A cohort study
e. A case series
79. 82-year-old female complains of headache and left painful red eye suddenly after
watching a movie in a theater. Next most appropriate step is:
a. gentamicin ophthalmic drops every 4 hours
b. ketorolac 30 mg IV Stat
c. pilocarpine intra-ophthalmic drops stat
d. acyclovir ointment stat
80. MAC is affected by the following except:

a. Hypokalemia

b. Hypothermia

c. Hyperthermia

d. Pregnancy

e. Hypernatremia?

81. 58 y/o male with hypertension and no comorbidities. Management?

1. ACEi
2. Loop diuretic
3. CCB

82. Midazolam antidote

a. Flumazenil

83. Facial abnormality which is true

a. Malar flush is in aortic stenosis

c.

84. Bells palsy loss of


posterior or pupillary reflex
Posterior 1/3 loss of taste
Difficulty drinking and chewing

85. Side effect of propofol (repeated)

a. Decrease in CO , BP , CMR & ICP


86. Difficult intubation and desatting

a. LMA

87. Difficult to mask ventilate you changed head position still no improvement what to do next
?

a. Oral airway

88. To confirm intubation

a. Change color of EtCo2

89. Difficult airway preparation

from teammates)

90. Door to balloon time

a. Less than 1 hr & 30 min

91. 35, healthy, what vaccine to discuss (depends on scenario but we think tetanus is the better
choice here)

a. Tetanus

b. HPV

92. Diabetic + Hypertensive. Which antihypertensive will you give?

a. ACEi

93- scenario about female patient that recently moved to new place, complaining of
generalized fatigue, recently she is having heavy menstrual cycle. What could be the cause of
the fatigue ? Iron deficiency anemia ??

93- parent brought his son to the emergency department, the son has typical presentation of
panic attack. Asking about the diagnosis
-Cardiogenic pulmonary edema, how to prevent intubation? Nasal cannula, non-rebreather
mask, bipap

-Which of the following is considered hypertensive emergency?

Options: Asymptomatic patient came for medication refill BP: 210/90?

Known patient of HTN came with laceration and BP is also high in 200s

Patient with high BP and creatinine increased from 1 to 2.5 within 36 hrs

-Asymptomatic Saudi male, no previous visits, came for checkup. BP was checked twice in the
clinic with the reading of 210/110 or something? Which of the following is the most appropriate
next step?

Options: Give Amlodipine because this is confirmed HTN, tell him this is normal BP for his age,
arrange for Ambulatory BP monitor, or ask him to monitor it at home.

-Child got a 3cm laceration on his hand, tetanus toxoid vaccine was taken 2 weeks ago, what do
you do?

Options: give tetanus toxoid vaccine, give tetanus antibodies, do nothing

-Patient k/c of alcoholic cirrhosis presented with Cincinnati FACE criteria, which of the following
is the best next step?

Answers: LFTs, Fasting glucose levels, Brain MRI, and Brain CT angio

^I think this is the same as the PT q?

-A female, k/c of ESRD on dialysis, presented with a demarcated rash on her forearm
(description of the rash), which antibiotic to use?

Answers: Oral Vancomycin (correct because catheter-associated infections are


indications for using vanco)

^ this is the same q as the first one


AMB MCQ Compiled [3rd to 7th Batch]
Green = Drugs | Pink = Genetic | Yellow = Key Points | Blue = Diagnostic Point/Clue
[R] = Ready (but need minor review)
[C] = Newly Corrected/ or Answered (if no answer was provided by our seniors)
So you can Press Ctrl or CMD + F and type [C] to find all those question
New Questions 4
Family Medicine 8
[R] General: 8
[R] Vitamins: 10
[R] GIT Cases: 11
Pancreatitis 11
Cholecystitis & Appendicitis 11
H.pylori 11
Duodenal Ulcer 11
Diverticulosis/UC/Gastroenteritis 12
Pinworms 12
Diarrhea 12
Fissure & Hemorrhoids 12
IBS 13
GIT X-ray Findings: 14
[R] Ethics / Epidemiology 15
[R] Screening 17
[R] Headache 20
[R] Asthma / COPD 22
[R] Medical Errors 25
[R] Dysphagia 26
[R] Psychiatry / Depression 27
[R] Hypertension (HTN) 29
[R] Diabetes Mellitus (DM) 30
[R] Obesity 32
[R] Obstructive Sleep Apnea (OSA) 32
[R] Thyroid 33
[R] Infections: 34
TB 34
UTI & Deep Space Infection 34
SIRS (Systemic Inflammatory Response Syndrome) 34
Lyme Disease (Deer Tick Bite) 34
EBV, Measles, Coxackie 35
Cystic Fibrosis 35
RSV 35
Scarlet Fever & Kawasaki 35
Back to TOC
Page 1 of 121
Scabies 35
[R] Prenatal Visit vaccination: 36
[R] HDL & LDL Management: 37
[R] Hepatitis: 38
[R] Musculoskeletal: 39
[R] Sexually Transmitted Disease (STD): 42
[R] Centor Criteria (for GAS) 43
[R] Obstetrics & Gynecology (Emergencies) 44
[R] OCPs (Side effects) [Family Medicine] 46
Anesthesia: 47
[R] General 47
[R] EtCO2 & Parkland Formula 49
[R] Pain-Killer & Post-OP/Procedures 50
[R] Spinal Anesthesia 52
[R] Drug Administration 53
[R] Minimum Alveolar Concentration (MAC) 56
[R] Resuscitation 57
[R] Airway 58
[R] Epiglottitis: 60
[R] Anesthesia ER (Mixed to Edit) 60
Emergency (ER): 61
[R] Primary Survey (ABCDE): 61
[R] Triage: 62
[R] Trauma 63
[R] Needle Stick injury 65
[R] Wound 65
[R] Toxicity 66
[R] Sepsis 70
[R] Environmental (Emergencies) [Heat Exhaustion/Stroke/Frostbite/Diving/ 72
[R] Fluid Therapy / Blood transfusion 73
[R] DKA 75
[R] Genitourinary (Emergencies) 77
[R] GIT (Emergencies) 78
[R] ENT (Otitis Media & Externa) 79
[R] Neurology (Emergencies) 80
[R] Respiratory (Emergencies) 84
[R] Oncology (Emergencies) 85
[R] Ophthalmology (Emergencies) 86
Glaucoma 86
Subconjunctival Hemorrhage 86
Back to TOC
Page 2 of 121
Orbital Cellulitis & fractures 86
Eye Viral Infections 86
Miscellaneous 87
[VIP] Cardiology & ACLS & Electrolytes (Emergencies) 88
Pulmonary vs Cardiac 88
AAA 88
Hyper & Hypo Calcemia 88
Hyper & Hypo Kalemia 88
Hyper & Hypo Natremia 88
PE / DVT 88
MI & Angina & Heart Failure 89
Arrhythmias & Tachy & Bradycardia 90
[R] Hypertension (Emergencies) 95
[R] Anaphylaxis 96
[R] Palliative Care 97
[R] Procedures 98
Others: 99

Back to TOC
Page 3 of 121
New Questions
1. (NEW) Which of the following isn t a candidate for thrombolysis?
a. INR > 3.5 (>1.7)
2. (NEW) Atypical Chest Pain? Female & elderly
a. Female or Diabetics (we don
3. (NEW) Scenario about opioid toxicity what antidote to give?
a. Naloxone
4. (NEW) Absolute Contraindication for spinal Anesthesia?
a. Scoliosis (relative)
5. (NEW) Door to Balloon time for PCI [Acute STEMI]?
a. 90 min
So, in Summary: Door Balloon (PCI) = 90 min; Door Needle (TPA) = 30 min
6. (NEW) Heart rate is 220 bpm and delta wave on ECG. What medication is indicated? WPW 1c
(flecanide/propafenon)
a. Procainamide (anti-arrhythmic 1A)
b. Adenosine
c. Beta blocker (2nd line)
7. (NEW) Which of the following is TRUE regarding HH? (hyperosmolar hyperglycemia state?) Tx: aggressive fluid
& insulin | High mortality rate | more seen DM-II | no acidosis
a. Aggressive glucose reduction and dehydration may cause cerebral edema?
b. Lower mortality than DKA
c. Usually present with IDDM
8. (NEW) Glucose is 600, normal Na is 140. What is the expected (True) sodium level in this patient
a. 132 [One group answer this]
b. 146
c. 150
d. 120, 123
i. Corrected Serum Na+ = [Na+] +
ii. Should be Sodium + [0.016*(Glucose 100)] = 148 mEq/L
9. (NEW) What is NOT a risk factor for PID?
a. C section
b. Adolescent (RF)
c. Multiple sexual partners (RF)
d. IUD (FR)
10. (NEW) Patient came back from a trip to Asia with symptoms of diarrhea and fever how would you treat him?
a. Ciprofloxacin (traveler diarrhea)
b. Augmentin
c. Vancomycin
d. Cefuroxime
e. Metronidazole (if you are suspecting protozoa)
11. (NEW) Which of the following is TRUE regarding frostbite management
a. Rapid rewarming with water 40 degrees
b. Debride the hemorrhagic blisters (DON T RUB IT)
c. Don t clean the clear blisters
d. Refreeze the injured area until definitive treatment is available
12. (NEW) Glucagon is used for
a. Sulfonylurea overdose (treating hypoglycemia)
b. Metformin overdose (will not cause hypoglycemia)
c. Hypoglycemia induced by insulinoma (insulinoma is treated surgically)
d. Hypoglycemia induced by chronic liver failure
13. (TO CHECK NEW) Most common cause of metabolic acidosis?
Back to TOC
Page 4 of 121
a. Lactic acidosis? (yes in hospitalized pts.)
b. DKA?
c. Toxin ingestion
d. Alcohol ketoacidosis
i. Couldn t find any
14. (TO CHECK NEW) Creatinine >200 (high), what drug should you Stop?
a. Metformin
b. Sitagliptin (DPP-1 inhibitors)
c. Ramipril
d. Sulfonylurea
15. (TO CHECK NEW) In case of a disaster, what is the first problem you will encounter?
a. Jurisdictional dispute?
b. Communication problem?
c. Insufficient volunteers?
d. Water contamination?
e. Lack of back resources? [More with this option
16. (NEW) Hypokalemia what do you see
a. Flaccid paralysis (hyporeflexia)
b. J wave (seen in hypothermia)
c. Peaked T wave (seen in hypERkalemia)
17. (NEW) Hypothermia ECG sign
a. J wave
b. Osgood wave [This was an answer we got in the exam. Correct answer should be Osborn not Osgood
lol]
c. Delta wave (seen in WPW)
18. (NEW) Athletic run and jump intensely and he noticed a new onset pain that is increasing gradually in his
foot? (jones fracture?)
a. Stress fracture (WRONG)
19. (NEW) Which of the following is TRUE regarding Jones fracture
a. Fracture in the diaphysis of the 5th metatarsal [ JONES = 5 Words so 5th Metatarsal]
b. Avulsion fracture in the base of the 5th metatarsal
c. No need for a splint (you have to do splint/ORIF)
d. Lower risk of nonunion than stress fracture (associated with non-union)
e. Not common in athletes (common)
20. (TO CHECK NEW) Distal Radial fracture with dislocation of radio ulnar joint (X-ray picture). what is the
diagnosis?
a. Galeazzi fracture (G= ground)
b. Monteggia fracture (M= mountain)
c. Radial nerve injury is very common in these fractures. (median n.)
21. (NEW) Patient had a cat bite in his hand (picture is shown, it was flexor tenosynovitis) Which of the following
is one the four signs that correlate with the diagnosis?
a. Pain on palpation on the proximal sheath
b. Pain on flexion (on extension)
c. Finger is held in extension (flexed)
22. (NEW) Salter harris fracture, which one is TRUE?
a. Prognosis gets worse from Type 1 to 5
b. All types are involved with growth plate. (not all)
23. (NEW) Patient fractured his tibia 3 days ago comes back complaining of pain and paresthesia. What is the
most appropriate management?
a. Check compartment pressures (5 P s)
b. Reapply splint and follow up with orthopedics after 24 hours
Back to TOC
Page 5 of 121
c. Give analgesia and send home
d. Review radiographs
24. (NEW) Patients with altitude sickness with crackles in his lungs and confusion (HAPE & HACE), Tx?
a. Immediate descent
b. Steroids (only for HACE)
c. Mannitol (only for HACE as Prophy)
d. Nitroglycerin
25. (TO CHECK NEW) Mechanism of action of a drug that is used for high altitude
a. Is an effective analgesic?
b. It induces metabolic alkalosis, and increases the respiratory drive and oxygenation?
c. Unknown mechanism?
d. Increase CSF production?
i. by inducing metabolic acidosis >>
hyperventilation >> increase O2 in blood]
26. (NEW) A 33 y/o lady is asking for an IUD (type is not specified). You are supposed to tell her that the coil
should be removed after?
a. 1 years
b. 2 years
c. 3 years
d. 5 years??? [Could be, depend on the IUD if with hormone or without] (without copper only is 5 year)
e. Until Menopause
27. (NEW) Subarachnoid hemorrhage how would it appear on a lumbar puncture?
a. High protein, Normal glucose with a bloody appearance with xanthochromia
b. High protein, low glucose, >90% PMN (bacterial meningitis)
c. Low protein, low glucose, >80% Lymphocytes
d. Frank blood coming out from LP needle (~ traumatic LP)
i. Xanthochromia:
1. Yellow discoloration indicating the presence of bilirubin in the CSF
2. Used to differentiate in vivo hemorrhage vs. traumatic LP
28. (NEW) Which of the following tests can be used to confirm suspected calcaneal tendon rupture
a. Thompson s test (if it is Achilles tendon rupture?)
b. Lachman test (ACL injury)
c. Hoover sign (leg paresis)
29. (NEW) Which of the following inhaled anesthetics causes transient increase in BP and heart rate?
a. Desflurane
b. Sevoflurane
c. Nitrous oxide
d. Halothane
30. (NEW) Patient going to awake craniotomy, now you want to give him a drug that keeps him breathing alone,
awake but cooperative
a. Dexmedetomidine (used in neurosurgery)
b. Thiopental
c. Ondansetron
d. Propofol
31. (TO CHECK NEW) Most common complication in anesthesia
a. Respiratory events
b. Cardiac events
c. Analgesic overdose
32. (TO CHECK NEW) Which of the following is the definition of difficult airway?
a. Difficult Intubation and/or ventilation?
b. Difficult Intubation of the trachea
Back to TOC
Page 6 of 121
33. (NEW) Which of the following is considered a difficult airway intubation?
a. Restricted head and neck movement
b. Lack of teeth (this will be a difficult patient to apply face mask on. No effect on intubation)
c. Long neck
34. (NEW) Which of the following is NOT a definitive airway?
a. Tracheostomy
b. Supraglottic airway
c. Endotracheal tube below the glottis
d. Endotracheal tube that is fixed with tape to the face
35. (NEW) Anesthesia question what is the most important thing to do if there is an airway difficulty?
a. Maintain Oxygenation & Ventilation
b. Look for other tools and instruments
36. (NEW) A 150 premature babies were followed since birth till school years. And they were compared with
5000 normal born babies to see who had better performance in the school. This is what kind of study?
a. Cohort
b. Cross sectional (no comparison group)
c. Case control (always retrospective)
d. RCT
37. (NEW) A Young patient experiences pain after Propofol injection. What could be the cause of that?
a. Small vein
b. Fast infusion
c. Young patient
d. Arterial line
38. (NEW) Sickle cell with osteomyelitis, it depend on the
a. Salmonella [Associated]
b. Staph A [Most Common]
c. E. Coli
39. (TO CHECK NEW) Bradycardia of 38, BP is 132/85 and the patient wants to talk with her son on the phone
and doing fine (mentally stable)?
a. Observe?
b. IV atropine
40. (TO CHECK NEW) QSofa what is not part of it:
a. Temperature higher than 38.5
b. Tachypnea >22
c. Systolic blood pressure less than 100
d. Altered mental status
41. (NEW) Hypertensive emergency and papilledema?
a. Admit
b. do ambulatory testing, c) Follow up in the clinic, d) Reassure
42. (NEW) What is TRUE regarding FAST in trauma
a. Predicts if you need to do exploratory laparotomy
b. Fast abdominal sonography of trauma
c. More accurate than DPL
d. It checks/views 5 sites for hemoperitoneum and hemopericardium (four sites)
e. It can t detect hemoperitoneum (it can)
43. (NEW) Women lost her husband 8 months ago. Suffers from insomnia and she is feeling low. What would you gi
her?
a. Antidepressant
b. Anxiolytic, c) Antipsychotic
44. (NEW) Room spinning when the patient moves his head. What is the diagnosis?
a. BPPV
Back to TOC
Page 7 of 121
b. Meneiere
45. (NEW) Which of following sites for central lines has the greatest risk for pneumothorax?
a. Subclavian Vein
b. Internal Jugular, c) Femoral, d) Brachial
46. (NEW) Which of the following patients would you consider to have decision-making capacity?
a. 57 y/o patient with breast cancer and mets. And she understands her disease very well and
understands the risks and benefits of proposed management.
b. 34 y/o patient oriented to time and place, but is not understanding the risks and benefits of the management
and procedure.
c. 3 y/o
d. patient is not oriented to time and place and ...
47. (TO CHECK NEW) Bells palsy question
a. Antiviral (not always)
b. Immune modulator? (steroid is not an immune modulator?!)
c. Artificial tears, eye patch (must do eye care for dry eye) MAYBE THE ANSWER
48. (NEW) Patients with obstructive sleep apnea are at increased risk of?
a. Hypertension
b. Dyslipidemia (NO)
c. Migraine (NO)
d. Infertility (NO)
49. (NEW) Patient comes with seizures. He was previously diagnosed with psychogenic polydipsia. Which of the
following findings correlate with this diagnosis?
a. Na: 115 with maximally diluted urine
b. Na: 150 with maximally diluted urine
c. Na: 120 with maximally concentrated urine
d. Na: 148 with maximally concentrated urine
e. Na: 150 with maximally concentrated urine

Back to TOC
Page 8 of 121
Last Group 2019 First Draft
IV Steroid (Complete Right sided facial wekaness 2 days ago) [Other option: Antiviral/Observe]
Propofol side effect
New Fracture (Dr.Butt Lecture) - Gelazzi fracture + Salter Type 2 +
Lawsuit something answer was duty to act (other option was good samaratin breech)
2 ECG
Airway tube/mask question
GABA agonist (demitdinineie) ?????
Ketamine is NMDA antagonist
To reduce Drowsy during OP what to give? Etomidine, Propofol (?), Naloxone
Ulcertaive collitis 37 yo female + 8 time bowel movement + Bloody Dirrhea (2 week onset)
Patient 66 with IBS picture, for 6 week, answer was consider malignancy to rule it out first
Pregnancy safe drug? Heparin (true) vs CHloramphnicol (no) vs Ibuporfen (Acetaminophen is safe) (with 3rd
trimester for Ibuprofen)
Salmonela + Sickle cell patient + X-ray with peritosela ==> answer was Salmonela
(?) Kanaval sign is? Positive pressure cause tendon somthing
Case Control
Pubmed and portal medline Primary Soruce
Breaking bad new what to do? (ensure enough Tx time, bring someone with him support, tell him that there is bad
news to prepare him ahead)

Back to TOC
Page 9 of 121
Family Medicine
[R] General:
50. (2X) A 15 y/o old girl claims she can t see her mobile?
a. Malingering
b. Hyperopia
51. (2X) Definition of malingering?
a. Malingering: is the intentional faking or creating of illness in
order to obtain secondary gain (e.g. workers compensation,
disability payments, avoiding work or jail time, pain
med
52. (2X) Recurrent epistaxis Pic of the mouth?
a. Telangiectasia
53. (2X) Epistaxis with pic of lip & tongue have dots?
a. Hereditary hemorrhagic telangiectasia
b. SLE
54. (2X) Young man with nasopharyngeal tumor +epistaxis what is the next step?
a. Imaging
55. (1X) A woman visits multiple hospitals to follow up regarding her health & she doesn t have a primary care giver
this is an example of:
a. Fragmentation of care
b. Continuity of care
56. (1X) The most common cause of death worldwide is: CVD
a. Coronary artery disease
b. Diarrheal illness, c) Lung cancer
57. (1X) Which of the following is the most common cause for chest pain?
a. MSK pain
b. GERD, c) Heart attack
58. (2X) Most cause of chest pain in family medicine:
a. MSK
b. GERD
59. (TO CHECK 1X)
a. Kaposi sarcoma (in AID +ve, HHV-8, affects skin and mouth reddish purple macules)
b. Hairy leukoplakia (CD4, EBV, painless un-scrapable white plaque on lateral tongue, HIV +ve)
c. SCC (Scale)
60. (TO CHECK 1X) How to manage this tongue condition?
a. Surgery (if it is SCC)
b. Surgery with neck dissection
This case is for the next 2 question: A 35 y/o female patient presenting with a complaint of dizziness, she
describes her dizziness as like she is out of this world , all results of labs and investigations were negative
except, however she have accepted a new job which is very competitive and stressful.
61. (3X) Which is the least next action to be taken in this condition ?
a. Lab work + imaging
62. (3X) What is the most probable diagnosis ?
a.
63. (2X) A 71 y/o man diabetic, feel dizziness when he wakes up from the bed for the last 7 days he had syncope
, he is on aspirin, furosemide and beta blocker what is the cause :
a. Orthostatic Hypotension
64. (TO CHECK 1X) How can you be cost effective in family practice?
a. Generic Prescription
b. Drug formulations with multiple drug.
65. (1X) CVS risk assessment in a low risk individual and some tests were already done and asking what other tests t

Back to TOC
Page 10 of 121
do?
a. Do nothing
66. (TO CHECK 1X) A 71 y/o lady wants to undergo a stress test after she learned that one of her friends had an
MI, her previous ECG done 2 years ago was normal & she has no complaints, which of the following would you
order to access her cardiovascular risk?
a. Lipid profile (>45 female)
b. CT coronary calcium score (ordered only if DM & over 40)
67. (2X) During a patient s interview:
a. Use open ended questions
b. Never use closed ended questions, c) Interrupt the patient early to keep the interview on track
68. (2X) Kid has 3 months hx of abdominal pain, now he can t attend school, which of the following would hint
towards an organic rather than a functional cause?
a. Suprapubic pain
b. Family hx of abdominal pain??, c) Headache, d) Anxiety
69. (2X) smoking cessation: [incomplete?]
a. NRTS (Nicotine Replacement Therapy)
70. (1X) Which of the following is the first line treatment for smoking cessation?
a. Nicotine replacement therapy
b. Fluoxetine c) Venlafaxine d) Mirtazapine
71. (5X) Smoker patient failed nicotine patches treatment, next step?
a. Bupropion
b. Counselling?
72. (2X) which of the following is most effective regarding smoking cessation?
a. Advise the patient every visit.
73. (TO CHECK 3X) Tobacco clinics Saudi?
a. 70 in Saudi
74. (1X) A man taking warfarin 4 mg + INR 3.5. Currently does not have any bleeding or problems. What would
you do?
a. Decrease warfarin from 4 to 2
b. Stop warfarin and start aspirin
c. Stop warfarin and check INR next day (>3.5 you stop and re-check 3-5 days later)
d. Continue Warfarin and check INR next day
75. (1X) A 60 y/o fell at home and was brought with hip fracture, what is your next step in management?
a. Take to OR (high risk of AVN)
76. (TO CHECK 2X) What is TRUE about the 4:2:1 formula? (it is not used for pediatrics, i don t remember the other
answers)?
a. It is used in peds, need to look up.
77. (1X) A mother brings her child complaining of chronic eczema. What do you do?
a. Give emollients and topical steroids for flare ups
b. Consult derma for allergy test, c) Topical emollients only, d) Topical steroids only
78. (TO CHECK 2X) Which of the following can be done to a DNAR (Do not attempt resuscitation) patient?
a. Morphine (not sure).??? (YES)
79. (3X) Contraindication to fly on airplanes?
a. Pneumothorax
80. (2X) Which one of the following is an absolute contraindication for flying in a commercial flight?
a. Pneumothorax
b. Pregnant 30 months
Travel by air is normally contraindicated in the following cases:
1. Infants less than 48 hr old.
2. Women after the 36th week of pregnancy (32nd week for multiple pregnancies).
3. Those suffering from:
Back to TOC
Page 11 of 121
a. sickle-cell anaemia;
b. angina pectoris or chest pain at rest;
c. any active communicable disease;
d. decompression sickness after diving;
e. increased intracranial pressure due to haemorrhage, trauma or infection;
f. infections of the sinuses or of the ear and nose, particularly if the Eustachian tube is blocked;
g. recent myocardial infarction and stroke (elapsed time since the event depending on severity of
illness and duration of travel);
h. recent surgery or injury where trapped air or gas may be present, especially abdominal trauma and
gastrointestinal surgery, craniofacial and ocular injuries, brain operations, and eye operations
involving penetration of the eyeball;
i. severe chronic respiratory disease, breathlessness at rest, or unresolved pneumothorax;

Back to TOC
Page 12 of 121
[R] Vitamins:
81. (3X) What is TRUE about bariatric surgeries?
a. Nutritional deficiencies can be replaced with supplements
82. (NEW) Vitamin D recommended dosage for infant that is being breastfed?
a. 400
b. 600
c. 800
d. 1000
e. 900000
83. (1X) Breastfeeding lady complains of breast engorgement and discomfort. What do you advise her?
a. Continue feeding and warm compress
b. Stop breastfeeding
c. Topical Abx
84. (1X) Breast engorgement for 3 days. No redness or temperature
a. Teach the patient the proper breastfeeding technique
b. Stop breastfeeding for some time
c. Oral antibiotics
85. (2X) How long should you wait for doing a surgery on a baby that has just been breastfed?
a. 2 hours (if clear liquid fluid)
b. 4 hours
c. 6 hours (if formula milk or solid food)
86. (4X) Supplementation of which vitamin has known health benefits?
a. Vitamin D
87. (5X) Vitamin B12 deficiency happen in?
a. Gastrectomy
88. (2X) Which one of the following Vitamins need to be replaced Post gastrectomy?
a. Vitamin B12
b. Vitamin C
c. Vitamin K
d. Thiamine

Back to TOC
Page 13 of 121
[R] GIT Cases:
Pancreatitis
89. (2X) Pancreatitis
a. High amylase & Lipase (>3X Normal) (lipase is more specific)
90. (2X) Pancreatitis:
a. High lipase (more specific) & Amylase, Emergency.
91. (2X) A patient presents with an epigastric pain radiating to the back & elevated amylase, what is the
diagnosis?
a. Pancreatitis
92. (2X) Which of the following statements is CORRECT?
a. Acute Pancreatitis is an emergency
Cholecystitis & Appendicitis
93. (1X) When the patient hold his inspiration on pressure applied to the right upper quadrant of the abdomen is
this a sign is?
a. Cholecystitis
94. (2X) A 33 y/o obese man comes to the emergency department because of abdominal pain nausea and
vomiting. O/E rebound tenderness and guarding in Rt iliac fossa. You provisionally make a diagnosis of acute
appendicitis. Which of the following will be essential before a patient goes to OR?
a. SAMPLE history, exam, CT scan and surgical consent
b. Labs and overnight NPO
c. Labs and CT scan of abdomen and pelvis with contrast
d. Antibiotics, US abdomen, IVFs
e. SAMPLE history, exam, labs surgical consent and surgical consult
95. (1X) A classic scenario of acute appendicitis (right lower quadrant pain with rebound tenderness), what is
the diagnosis?
a. Acute appendicitis
96. (2X) RLQ pain periumbilical, localized to RLQ with N,V, woman
a. Appendicitis
97. (TO CHECK 1X) A 30 y/o obese female presented with sudden onset of fever, severe RUQ abdominal pain,
nausea, and vomiting. You ordered basic labs: Labs: Hb:12 g/dL | WBC: 17.5 x 109/L | Platelets: 110 x 109/L |
Lipase: 46 U/L (N) | AST: 90 U/L (H) What is the investigation of choice?
a. US abdomen (for the dx of cholangitis?) (correct?)
b. CT scan abdomen with contrast Note CT is diagnostic for appendicitis!
c. CT scan abdomen without contrast
d. Acute abdominal series x-ray
e. Barium swallow
98. (2X) Patient presents with right lower quadrant pain that started off as periumbilical pain, anorexia, N&V
(acute appendicitis), next step?
a. Immediate admission and surgical consult
99. (TO CHECK 2X) Patient with appendicitis, which investigation you want to do????
a. CT
b. Laparoscopy (not sure but maybe CT).
100. (2X) Woman with classic appendicitis with no rebound tenderness. What will you do next?
a. Consult surgery?
b. Do more labs and stool culture
c. Give IV paracetamol
d. Ask her to go home and come back in 24hrs
101. (1X) Woman with classic appendicitis with no rebound tenderness. What will you do next?
a. Refer and admit under surgery
b. Do more labs and stool culture
c. Give IV paracetamol
Back to TOC
Page 14 of 121
d. ask her to go home and come back in 24hrs
H.pylori
102. (2X) 55 y/o patient with dyspepsia what is your first line investigation?
a. Urea breath test
b. Endoscopy (only if patient > 60 y/o or at least 1 alarm feature)
Duodenal Ulcer
103. (3X) Which of the following statements is CORRECT?
a. Duodenal Ulcer cause episodic epigastric pain
Diverticulosis/UC/Gastroenteritis
104. (1X) Diverticulosis:
a. Increase fiber intake
b. Reassurance
105. (1X) Fever, GI symptoms?
a. Ulcerative colitis/Gastroenteritis
Pinworms
106. (5X) Child can't sleep well due to anal itching, it is due to:
a. Pinworms
b. Ascariasis, c) Giardiasis.
107. (1X) Child with itching around anal region, cotton-like stuff in stool:
a. Pinworm
b. Tapeworm , c) Hookworm
Diarrhea
108. (NEW) Best way to prevent Traveler s diarrhea
a. Only eat cooked food and drink boiled water
b. Metronidazole, c) Ciprofloxacin
109. (1X) Patient from India trip came back with diarrhea for last 4 days. Most likely organism?
a. E Coli [correction by 6th batch]
b. Rotavirus [One senior group answer]
c. Shigella
110. (3X) Patient from India trip came back with diarrhea for last 4 days. Most likely organism?
a. E.Coli
b. Rotavirus
c. Shigella
Fissure & Hemorrhoids
111. (TO CHECK 3X) anal fissure vs fistula (loose stools) [incomplete]
a.
112. (1X) A young pain presents to the ER with sudden anal pain. He is known to have chronic constipation. On
examination, there is tenderness on the posterior anal region. What is the most likely diagnosis?
a. Anal fissure
b. External hemorrhoids
c. Diverticulitis
113. (TO CHECK 1X)[C] Chronically constipated, anal pain and itching:
a. Hemorrhoids (one senior answer)
b. Fissures (Fissure more likely)
114. (1X) A lady presents with perianal itching, on examination there is a reducible mass, what is the diagnosis?
a. Hemorrhoids

Back to TOC
Page 15 of 121
IBS
115. (1X) Young male with bloating, cramps & changing bowel habits:
a. IBS
116. (3X) Young patient with bloating, abdominal pain relieved by defecation, no red flags. What is the
diagnosis?
a. Irritable bowel syndrome (IBS)
b. CRC
117. (TO CHECK 2X) IBS : [incomplete??]
a.
118. (1X) Which of the following is TRUE regarding the evidence to treat IBS?
a. Peppermint oil is clinically proven to be beneficial
b. Lactose free diet has strong clinical evidence
c. Low FODMAP diet has strong clinical evidence
d. Gluten free diet
119. (1X) TRUE about IBS?
a. Better with defecation
120. (1X) A man complains of 7 months of colicky abdominal pain, there is no blood in his stools, one of his
relatives died of colon cancer at age 75, what is your diagnosis?
a. Irritable bowel syndrome (IBS)
b. Inflammatory bowel disease
c. Colorectal cancer
121. (3X) IBS will show:
a. Mucus stool
b.Hard to flush
c. Float stool
d.Pale/clay stool
122. (2X) IBS in a > 50 y/o female?
a. Rule out ovarian pathology
123. (2X) IBS worsened by?
a. Caffeine
124. (1X) First line medication for IBS?
a. See picture above
125. (TO CHECK 3X) IBS treatment?
a. Mebeverine (antispasmodic)
GIT X-ray Findings:
126. (TO CHECK 1X) X-ray picture after multiple surgeries : (adhesion?)
a. Small bowel obstruction (?)
b. Bowel perforation
Back to TOC
Page 16 of 121
i. No X-ray provided by our seniors
127. (3X) A lady presents with abdominal pain, she had multiple surgeries for crohn s disease, her x-ray is shown
what is the diagnosis?
a. Small bowel obstruction
b. Large bowel obstruction

Back to TOC
Page 17 of 121
[R] Ethics / Epidemiology
128. (1X)[C] Definition of Evidence Based Medicine
a. An approach to medical practice intended to optimize decision-making by emphasizing the use of
evidence from well-designed and well-conducted research.
129. (1X) EBM?
a. Research + experience + patient values
130. (1X) New test detects cancer earlier, but patient dies at same age so does not increase life expectancy. This
is an example of? (study)
a. Lead-time bias
131. (1X) Screening detect 10 years earlier? (if detecting and no change in mortality lead)
a. Length time bias (if detecting earlier due to slow progression)
132. (2X) A study was designed by assigning 100 patients to either US or CT to compare the diagnostic accuracy
of in diagnosing appendicitis. What is the study type?
a. A randomized control trial
b. A case control study, c) A case report, d) A cohort study, e) A case series
133. (1X) Question describing a study with 2 groups of children with Pb exposure and without Pb exposure and
following them up to see who develops anemia, what type of study: [Side Note Pb = lead element]
a. Cohort study (exposure outcome)
b. Cross sectional (outcome exposure)
c. Case Control
134. (1X) What is a cross-sectional study?
a. Disease prevalence
135. (1X) What is a case-control study?
a. Odds ratio
136. (1X) Which of these scenarios reflects EBM:
a. Using drug X instead of drug Y after proven to be better from RCT
137. (1X) What is Disease oriented evidence refers to?
a. Refers to Outcomes of studies that measure physiologic or surrogate markers of health... Improvements
in these outcomes do not always lead to improvements in patient-oriented
138. (2X) If a drug is proven to lower LDL but does not affect patient s life, this is an example of?
a. Disease oriented study (not patient oriented)
139. (1X) Drug which reduces FBG and HBA1c?
a. Is it Disease or patient oriented?
140. (2X) Drug X was found to improve some lab values compared to drug Y. What type of evidence is this?
a. Disease oriented evidence
b. Person oriented evidence
141. (TO CHECK 1X)[C] Drugs X and Y were compared to study the incidence of stroke in hypertensive patients.
Based on the results, the physicians chose to give drug X to their patients. What is this practice called?
a. Evidence based medicine
b. Disease based?
142. (2X) cross sectional study (diagnostic) [incomplete] (prevalence?)
a.
143. (NEW) 5000 people were enrolled in study to do colonoscopy for screening of colon cancer. What does this
study tell?
a. Prevalence
b. Incidence
c. Sensitivity
d. Specificity, e) The secret of life
144. (1X) Findings from a RCT showed you that the follow up rate was 60%. What does that tell you about the
study?
a. Low validity (it should be >80%)
Back to TOC
Page 18 of 121
b. High validity
c. Low reliability
d. High reliability
145. (1X) Study statistical significance losartan with CI 95% (0.65-0.95):
a. The study shows statistical significance (it did not cross 1)
b. The study showed clinical significance
c. The study shows no statistical significance
d. There is insufficient information to know the statistical significance
146. (1X) A study showed that simvastatin decreases the incidence of MI with a RR of 0.8 and a CI of (0.6 1.2)
what is your conclusion?
a. It is statistically not significant (it crossed 1)
b. It is clinically significant
c. Can t judge since the P value is not mentioned
147. (1X) A positive D dimer value of >500, sensitivity 95%, specificity of 80%, PPV of 85%. Which of the
following statement is CORRECT?
a. 85% of the patients with positive test will have the disease
b. 95% of the patients with positive test will have the disease
c. 90% of the patients with a positive test will have the disease
148. (1X) A scenario about a diagnostic test mentioning the sensitivity, specificity, PPV & NPV (80%), the test
came back negative, what does that mean?
a. There is 80% chance that the patient does not have the disease
149. (TO CHECK 3X)[C] A study to see the effect of statin post MI. What is the number needed to treat (NNT)?
[Calculate using the formula, calculators weren t provided

a. 48 [If you take the ]


b. 56 [if you take the ]
c. 82
d. 100
i. NNT = (1/[CER-EER])
150. (4X) NNT calculate with given table of numbers [different number]
a. Same as above
151. (4X) The principle of do no harm is called?
a. Non-Maleficence
b. Beneficence
c. Autonomy
d. Maleficence
152. (Again 1X) Which principle does Do no harm refer to?
a. Non-Maleficence
153. (And again 3X) As a health care provider, we must at least act and not hurt any patient. This is referred as:
a. Non-Maleficence
b. Non-beneficence, c) Maleficence, d) Beneficence, e) Standard of care
154. (1X) Non-maleficence means?
a. Do no harm
155. (1X) Which of the following is a primary source of evidence?
a. Pubmed
156. (2X) Pubmed is an example of?
a. Primary source

Back to TOC
Page 19 of 121
157. (1X) Midline access through pubmed is an example of:
a. Primary resource
b. Secondary resource, c) Electronic Library
158. (1X) Cochrane library is an example of:
a. Primary source
b. Secondary source
c. Electronic library
d. Both primary and secondary source
159. (NEW) Scenario about a resident in an elevator who told his junior to Tell Mr.Ibrahim to continue on his
usual HIV medications . Junior knew there were drugs that could have better response. There were other
healthcare professionals in the elevator. Which rule was violated?
a. Confidentiality
b. Knowledge

Back to TOC
Page 20 of 121
[R] Screening
160. (TO CHECK 3X) A question about the differences between primary vs. secondary vs. tertiary prevention?

161. (2X) Secondary screening:


a. Hypertension (YES?)
b. Cervical cancer (YES?)
162. (TO CHECK 4X)[C] What is an example of Tertiary prevention?
a. Screening fundoscopy
b. Diabetic control (to prevent complications)
163. (1X) Tertiary prevention for cerebrovascular disease:
a. Antiplatelets
b. Speech and verbal therapy
164. (TO CHECK 7X) A 28 y/o female not married (not sexually active), want to do cervical screening. What do
you tell her?
a. Do HPV test (HPV >30 Y/O)
b. She can't do it (out of the screening people)?
c. Need to check this

165. (TO CHECK 1X) Anisocytosis?


a. Electrophoresis
b. Thyroid function test
166. (1X) Koilonychia?
a. Because of iron deficiency
167. (4X) A 21 y/o lady just came from Africa and diagnosed with malaria what is the most specific test to make
Dx?
a. Thick and thin blood film for malaria
168. (2X) Megaloblastic anemia?
a. Folic Acid Deficiency (TRUE? If no neurological Sx)
b. B12 deficiency (TRUE?)
169. (2X) Man with fatigue and shortness of breath Hb is 7.5 macrocytosis what type of deficiency?
a. Folic Acid Deficiency
170. (1X) A lady has low Hb, serum iron & ferritin, what is the diagnosis?
a. Iron deficiency anemia (IDA)
Back to TOC
Page 21 of 121
b. Anemia of chronic disease
171. (1X) Iron deficiency Anemia?
a. Ferritin
172. (2X) Ferritin for
a. IDA
173. (1X) Patient for pre-employment screening + fatigue & weakness, sister has anemia. What investigation
would help?
a. Hb electrophoresis (to check for thalassemia & SCD)
174. (1X) Which of the following will help in the diagnosis of iron deficiency anemia?
a. Low Iron
b. Low Ferritin
c. Low Hemoglobin
175. (NEW) A 42 y/o woman comes for first time and wants to be screened for breast cancer, what could be a
risk factor?
a. Family history in sister
b. Late menarche [protect] (early menarche is RF)
c. Multiparity [protect] (Nullparity is RF)
d. Breastfeeding [protect]
e. Early first pregnancy [protect]
176. (Again 3X) A 55 y/o man for follow up:
a. FOBT (Since he is 55 y/o) (screening age for colon cancer from 50-75)
177. (TO CHECK 1X) In someone who has a relative who developed colorectal cancer at age 65 when should he
start screening?
a. 50
b. 55 [it skip generation so 10 year earlier, so with number it should be 55, but we would screening as early as possible]
178. (1X) A 60 y/o lady presents with changing bowel habits and some cramps with bleeding and ?
a. Advice colonoscopy as the colon CA risk increases above 60 years of age.
b. Reassure as this likely IBS (Red flags are bleeding and elderly)
179. (1X) A classic scenario for CRC cancer asking for diagnosis?
a.
180. (TO CHECK 1X)[C] A 74 y/o male per rectal blood?
a. Colon cancer
b. Rectal cancer [One Seniors Answer]
181. (1X) Bleeding PR and weight loss. Incomplete defecation and straining?
a. Rectal cancer
b. Cecal cancer
182. (TO CHECK 2X) Breast cancer what is CORRECT regarding screening?
a. Most patients find it extremely painful
b. It has a low radiation exposure
i. Both are correct (but B is objective, and A is more subjective, so I would go with B)
183. (5X) Risk for breast cancer?
a. Alcohol Consumption
184. (To CHECK 5X)[C] Breast cancer history treated, came with back pain symptoms?
a. MRI spine [6th batch answer]
b. CT spine [Most Probable Answer, as can be due to other
etiology]
c. Lumbar X-ray
185. (2X) Breast cancer patient with fever
a. Send for MRI
186. (TO CHECK 2X) A man with full bladder and previous lung cancer : [incomplete]
a. Urgent MRI
Back to TOC
Page 22 of 121
187. (2X) At what age do you start screening for breast cancer with mammogram according to NICE and Task
Force?
a. 30
b. 40
c. 45
d. 50
e. 65
188. (1X)[C] Who to screen for AAA with US ?
a. 67 y/o male smoker [For AAA: Age > 65 for example + Smoker || For Lung cancer: Smoker for 30-40
packyear]
189. (1X) Which of the following is NOT included on premarital screening in KSA?
a. Illicit drug use
b. HIV
c. Hep C
d. Hep B
e. Sickle cell anemia
190. (NEW)[C] A patient comes to you and wants you to check his prostate by measuring PSA levels, what would
you do?
a. Discuss pros and cons of PSA
b. Suggest doing TRUS
c. Do PSA
i. Guideline for this might change in the near future (so to our juniors please read uptodate for
updates they are to do PSA with Multi-axial MRI of prostate mp-MRI and scoring risk)
191. (2X)[C] A 60 year old man came to the clinic asking to get screened for prostate cancer, what would you tell
him?
a. PSA screening is NOT recommended
b. Do PSA for him
c. PSA is recommended according to many guidelines
d. PSA high means prostate cancer
i. Guideline for this might change in the near future (so to our juniors please read uptodate for
updates they are to do PSA with Multi-axial MRI of prostate mp-MRI and scoring risk)
192. (5X) Question about a man with prostate cancer
a. Prostate cancer
193. (2X) You are evaluating a 56 y/o African-American man complaining of fatigue. He describes this as a lack of
stamina but still has motivation to do things. Sleep Refreshes him, but he tires quickly at work. His physical
examination is unremarkable. Which of the following should be included in your initial workup to help
ascertain the diagnosis?
a. Prostate cancer screen
b. Chest x-ray
c. ECG
d. HIV test
e. Drug screen
Initial Lab workup for an uncertain Dx of fatigue include:
i. CBC, sedimentation rate, urinalysis, chemistry panel, thyroid testing, pregnancy testing (if female)
ii. Age/gender appropriate cancer screening. (>50)
iii. So in this case a prostate screen would be appropriate.
iv. Chest x-ray, ECG, HIV test, and a drug screen would be appropriate if the initial screen is negative.
194. (1X) A man complains of urgency hesitancy & drippling without during urination, what is the best
investigation?
a. Prostate ultrasound

Back to TOC
Page 23 of 121
[R] Headache

Back to TOC
Page 24 of 121
195. (3X) A case of headache (description is that of tension) for which patient is now using around 200 pills a
month:
a. Chronic daily HA (medication overuse headache)
196. (1X) Patient with migraine asks for prophylaxis to prevent more episodes. What do you give?
a. Atenolol
b. Sumatriptan (this is a treatment not prophylaxis)
197. (1X) Which of the following is used for migraine prophylaxis?
a. Beta-blockers
b. Calcium channel blockers (not 1st line prophylaxis)
198. (1X) Prevention of migraine?
a. Beta Blockers & Topiramate.
199. (1X)[C] Acute migraine?
a. Sumatriptan or Dihydroergotamine (if mod/severe)
b. (if mild)
200. (1X) A lady complains of band like headache for which she takes simple analgesics but recently it has
increased in frequency & is interfering with her exams, what is the best next step?
a. Amitriptyline (used as prophylaxis)
b. Continue simple analgesics
201. (NEW) Man with headache behind the eye that was worse at night and woke him up from sleep. Had
history of these headaches that would go into remission and relapse later
a. Cluster Headache
b. Tension Headache
c. Migraine Headache
202. (6X) Cluster headache?
a. Stays for < 3 hr on daily basis (30-90 mins)
203. (2X) A patient presented with a complaint of repeated episodes of headache which are very severe and
associated with rhinorrhea and lacrimation. It is not associated with any aura. What is the diagnosis?
a. Cluster Headache
b. Tension type headache
c. Migraine

Back to TOC
Page 25 of 121
[R] Asthma / COPD

Back to TOC
Page 26 of 121
204. (2X) What would be suggestive of a diagnosis of asthma
a. Triggered by aspirin and beta-blockers
205. (2X) Which of the following goes with the diagnosis of asthma?
a. Diurnal variation of symptoms 1-2 wk on PEF)
b. Prominent light headedness, tingling sensation in fingers and fainting episodes
206. (NEW) What is the most effective medication for long term control in asthma?
a. ICS
b. LABA
c. SABA
207. (4X) Asthmatic on Ventolin, has exacerbations at night, next step? STEP2 is preventer therapy ICS
a. Add low dose inhaled Corticosteroids (low dose ICS)
208. (2X)[C] Asthma patient who takes Ventolin PRN has to take his medications at least 2 times a week, and
multiple times at night. What should you do for him?
a. Add low dose ICS + LABA
b. Add inhaled fluticasone
c. Add LABA
209. (5X)[C] Asthma treatment
a. Stepwise [Enviromental Control > SABA > Low dose ICS + LABA > Dose Increase +/- LTRA > Prednisone +/- Omaluzomab]
210. (3X) Patient on rescue salbutamol and max dose inhaled steroids, still not controlled, next step? Add on
therapy
a. Add LABA
211. (5X) White male comes in with asthma, you gave short acting beta 2 agonist, however he is not improving.
What do you give him as an outpatient?
a. Inhaled corticosteroids
212. (2X) Patient with mild asthma that only requires rescue salbutamol presents now with worsening of
symptoms?
a. Add inhaled steroid
213. (5X) Asthma question, answer was:
a. To decrease allergens
214. (4X)[C] Asthmatic, should decrease exposure to dust & other indoor allergens by?
a. Increase humidity
b. Not washing beds
i. Both of the above increase Asthma
215. (2X)[C] Which of the following should be recommended to asthmatic patients for secondary prevention of
asthma?
a. Use air humidifier [This is wrong, should be use air filter (i.e. Nano-filters) to filter the dust and allergens]
b. Use hypoallergenic pillow covers
i. We don
216. (6X) Case of bronchial asthma and hypertensive patient, which antihypertensive best avoided in this case?
a. Labetalol
217. (TO CHECK 4X) Uncontrolled severe asthma:
a. FEV1/FVC ratio less than 70%, FEV1/FVC ratio is normal
218. (1X) Which of the following is a characteristic finding in asthma?
a. FEV1/FVC ratio less than 70%
219. (3X) Asthma patient with no control on ICS?
a. LABA
220. (2X) Asthmatic patient on inhaled CS (high dose already) & short acting B-agonist, not managed well. What
to do?
a. LABA
b. Oral steroids
221. (4X) Patient with mild asthma that only requires rescue salbutamol presents now with worsening of
Back to TOC
Page 27 of 121
symptoms?
a. Add inhaled steroid
222. (2X) An asthmatic patient is already taking his regular medication (SABA) but his symptoms are still not
improving. Which of the following medication should you add?
a. LABA + Steroid
b. Oral steroid
223. (1X)[C] Asthma management:
a. Decrease allergens (a) decrease humidity (b) washing beds (scabies can exacerbate asthma)
224. (1X) Patient s symptoms no longer control with short acting beta-agonist?
a. ICS
225. (1X) Patient s symptoms no longer control with short acting beta-agonist (Albuterol) + High dose ICS?
a. Add LABA (Albuterol) or oral steroids.
226. (1X) Which antihypertensive should be avoided in bronchial asthma patients?
a. Labetalol
227. (1X) Uncontrolled severe asthma? FEV1 < 60 & FEV1/FVC reduced > 5%
a. FEV1/FVC ratio less than 70%
b. FEV1/FVC ratio is normal.
i. True about asthma? Increased with b-blocker and aspirin
228. (3X) What is the most-potent and proven effective long-term treatment of Asthma?
a. Inhaled corticosteroids
b. SABA, c) LABA
229. (4X) Most common cause of chronic cough?
a. Post-nasal drip
230. (1X) Pneumonia CURB-65 question? 2 inpatient; 3 ICU
a. Admit if it was 2-3
231. (1X) CURB-65
a. Used for assessment
b. For Abx
c. Decision for admission/mortality
i. Confusion of new onset (defined as an AMTS of 8 or less)
ii. Urea - Blood urea nitrogen (BUN) > 7 mmol/l (19 mg/dL)
iii. Respiratory Rate 30 Breaths/minute or greater
iv. Blood pressure 90 mmHg systolic or diastolic blood pressure 60 mmHg
v. 65 of Age or older
232. (3X) In a patient with pneumonia, the CURB 65 score will assist you with:
a. Need for hospitalization and mortality risk
b. Choice of antibiotics drug therapy
c. Follow up appointment with primary
d. Need for lab orders and ABG
e. Intubation decision and early goal directed therapy
233. (1X) A smoker for the past 20 years has chronic cough. His CXR shows hyperinflation. What is the most
likely Dx?
a. COPD
b. Asthma
c. Pneumonia
234. (3X) A patient with COPD.
a. Check Previous culture.
235. (1X)[C] Which of the following is a gold standard for diagnosing COPD?
a.
b. Spirometry
236. (4X) What can help you in diagnosing COPD?
Back to TOC
Page 28 of 121
a. Spirometry
237. (1X) An obese man complains of shortness of breath when he wakes up & towards the end of the day,
what is the cause of his shortness of breath?
a. Obesity
b. Asthma

Back to TOC
Page 29 of 121
[R] Medical Errors
238. (3X) To prevent framing/being framed should?
a. ISBAR
b. SOAP
239. (1X) Which of the following is used to avoid framing when signing out?
a. ISBAR
b. ICE, c) SAMPLE
240. (1X) Counselling a patient to come to ER in case of alarming symptoms is called?
a. Safety netting
b. ICE, c) ISBAR
241. (1X) Which of the following can prevent framing?
a. Proper sign out
242. (2X) To avoid framing the method of communication used is?
a. ISBAR
b. ICE, c) SAMPLE
243. (Again 1X) What should you do at sign out?
a. ISBAR
244. (And again 2X) What you do at sign out?
a. ISBAR
245. (NEW) Framing is one of the problems that can happen in ER. Which of the following will help prevent this
problem?
a. Documentation about everything you do in ER
b. Taking consent
246. (TO CHECK 1X) CORRECT statement about emergency department physician documentation is:
a. Clinical documents may communicate to other healthcare providers the care rendered
b. It is important to document well so lawsuits can be prevented [You cannot prevent lawsuits from
happening]
c. Documentation of events during ED stay by the nurses is sufficient
d. There is no need to document a procedure if there were no problems with it
e. Sole purpose of documentation is justification of billing
i.
247. (TO CHECK 1X) CORRECT statement about emergency department physician documentation is:
a. It is important to document well so lawsuits can be avoided [You can however avoid lawsuits]
b. Electronic documentation is slow.
c. Documentation of events during ED stay by the nurses is sufficient.
248. (3X) Documentation question.
a. To communicate with other doctors
249. (3X) Why should you always document in ER?
a. To avoid any lawsuits
250. (2X)[C] What s the most common mistakes:
a. Wrong medication
b. Wrong patient [One senior answer]
c. Wrong diagnosis
251. (NEW 4X) Most common error that is cause of morbidity and mortality worldwide?
a. Wrong diagnosis [according to dr butt?]
b. Wrong patient, c) Wrong treatment, d) Wrong dose
252. (3X) IPSG 1 (International Patient Safety Goals) & IPSG 2 are?
a. Define patient correctly & Improve communication
253. (3X) Medical Trainee JC s AMC Orientation Includes:
a. International Safety Goals
b. Any Ongoing Required Education, c) Infection Control Program, d) Medication Safety Program
Back to TOC
Page 30 of 121
254. (7X) Which of the followings best define Never events?
a. "Adverse events that are serious, l argely preventable, and of concern to both the public and health care
providers for the purpose of public accountability." Are 29 Particularly Shocking Medical Errors Causing
Death or Significant Injury of Patient or Staff, classified to 7 categories. [IPSG 4 is geared toward Never
Event]
255. (Again and Again 2X) What you do at sign out?
a. ISBAR

Back to TOC
Page 31 of 121
[R] Dysphagia
256. (NEW) 26-year-old man with 10 pack smoking history comes in for dysphagia. What is the appropriate
management
a. Discuss Ideas, Expectations, Concerns [ICEE]
b. Consult him on a trial of smoking cessation and see response
257. (1X) A patient initially had difficulty swallowing solids but can t swallow liquids now. He also has been
experience weight loss. What is it most likely? [Don t remember the exact details but it was classic scenario for
esophageal cancer]
a. Esophageal Ca
b. CRC cancer
c. IBS
258. (7X) Question about Dysphagia in an old patient: [incomplete]
a. Esophageal cancer
b. Achalasia

Back to TOC
Page 32 of 121
[R] Psychiatry / Depression
259. (NEW) Which of the following is the most common cause of chronic fatigue?
a. Anemia
b. Depression
c. Hypothyroidism
260. (8X) What's TRUE about depression?
a. Maximum treatment is reached at 2-8 weeks of therapy
b. No need for a follow up appointment after taking medications (F/U after 1 month)
c. Treatment should be given for 2 years after a single episode of depression (2 yrs if recurrence)
261. (5X) Anti-depression meds will start to be effective in?
a. 2-8 weeks
262. (4X) Treatment of depression and treatment of anxiety (know first lines)?
a. SSRIs for all
263. (4X) A woman is anxious and she had tried psychotherapy sessions, but she's still anxious. You will give her
medical therapy. The first line would be?
a. SSRI
264. (2X) Patient with GAD is on CBT but is not managed. What do you do next?
a. Prescribe Fluoxetine (SSRI)
265. (1X) A depressed patient was started on Fluoxetine 20 mg & did not show improvement what is the next
step?
a. Increase the Fluoxetine does to 40 mg (2nd line is to inc. the dose if there is some improvement)
b. Switch to SNRI (3rd line)
c. Switch to another SSRI (since there is no improvement)
266. (6X) The loss of a loved one results in grief as universal human behavior, the following Is the most
appropriate statement regarding the stages (Denial, Anger, Bargaining, Depression, and Acceptance):
a. Everyone must go through the stages in order
b. One may go through grieving process without passing through all stages of grieving
c. Medications should be used to shorten the grieving process
d. The stages of grief have been validated in randomized control trials
e. Normally, grief should be over by 1 month
267. (1X) End of life care/Life support question.
a. The answer is that NOT all patients pass through all the stages of grief
268. (TO CHECK 1X) Which is TRUE about the grief process?
a.
269. (2X) Depression on SSRI, partial improvement?
a. Increase dose
i. If there is an inadequate response but good tolerability after four weeks of treatment at the
recommended minimum effective dose, the dose can be slowly titrated upward. Further dose
increases can generally be made at one to four- week intervals as needed.
270. (2X) Mild depression?
a. CBT
271. (2X) GAD?
a. CBTs
272. (2X) What is the first line treatment for generalized anxiety disorder (GAD)?
a. Cognitive behavioural therapy
b. SSRI
c. SNRI
d. Antipsychotics
273. (2X) A woman is anxious, and she had tried psychotherapy sessions, but she's still anxious. You will give her
medical therapy. The first line would be?
a. SSRI
Back to TOC
Page 33 of 121
274. (2X) What is TRUE about mild depression?
a. It is best to wait and watch
b. Increase the doses of the drug
(1X) Which one is a symptom in panic attacks?
c. Chest tightness
275. (4X) Female who lost her husband 4 months ago, and is sad, hallucinates about him. Tx?
a. Bereavement clinic for 1 year
b. SSRI
c. Antipsychotics
276. (1X) Lady's husband died 4 months ago, she has stopped eating, 2% weight loss, can hear her husband's
voice, Tx?
a. Bereavement clinic for 1 year
277. (TO CHECK 1X) Psychotic illness, except when fully controlled. [question incomplete]
a.
278. (2X) Suicide what is TRUE?
a. Men widowed has highest risk
b. Suicide is more common in winter
c. Women are more successful (men)
d. Men attempt more (women)
279. (4X) Agoraphobia means? (being in a place where nobody can safe them like an open space desert ,
outside home)
a. Fear of Insects such as spiders
b. Fear of Closed places
c. Fear of Crowds
280. (1X) How to differentiate between Delirium vs. Dementia?
a. Onset and speed of progression of symptoms (abrupt and fluctuate and stays wks in delirium)
b. Visual hallucinations
281. (2X) More with delirium > dementia? (visual hallucination is more)
a. Auditory hallucination (others: loss of attention, not oriented, not alert, disoriented speech)
i. Hallucination are more common with an acute cause, which delirium is.
282. (2X) Which of the following is more in favor of organic psychosis vs. psychiatric?
a. Auditory hallucinations
b. Visual hallucination
i. Auditory hallucinations Psychiatric (functional) Psychosis.
ii. Visual hallucinations Organic (medical, drugs, tumor) psychosis.
283. (2X) Auditory hallucinations are commonly associated with which of the following psychiatric illness?
a. Schizophrenia
b. Major depression
c. Borderline personality disorder
d. Mania
e. Dementia

Back to TOC
Page 34 of 121
[R] Hypertension (HTN)
284. (TO CHECK 3X)[C] Which of the following drugs is NOT present as an oral preparation for HTN?
a. Sodium Nitroprusside (infusion)
b. Atenolol [Oral]
c. Nifedipine (oral)
d. Thiazide (oral
e. Furosemide (PO, IV, IM)
f. Captopril (oral)
285. (1X) With which of the following drugs do you have to monitor kidney function and electrolytes?
a. ACE inhibitor (as it may cause renal failure and hyperK)
b. B blocker
c. Clonidine
286. (3X) African American patient diagnosed with uncomplicated HTN what is the 1st line treatment?
a. Amlodipine (CCB) (also thiazide is a good choice)
b. Carvedilol
c. Hydralazine
287. (3X) Black male develops HTN, what do you prescribe?
a. CCB
288. (2X) Diabetic develops HTN, first line drug to treat his HTN?
a. ACEI
289. (2X) Diabetic with high BP on 3 settings and +ve microalbuminuria, drug of choice?
a. ACEI
290. (1X) A patient is hypertensive and diabetic, what drug would you give?
a. ACE inhibitors
b. CCB
c. Aspirin
d. Beta blocker
291. (1X) Hypertensive patients & Risk of renal failure:
a. ACE inhibitors
292. (TO CHECK 1X) A diabetic patient has proteinuria & HTN, which antihypertensive to start him on?
a. Enalapril
b. Amlodipine
293. (TO CHECK 2X) Filipino lady came in with diabetes mellitus and hypertension. What is your choice of
medication?
a. Lisinopril
294. (3X) Lady with HTN got pregnant. The physician changed her drug which has teratogenic effect to another
safer drug?
a. Captopril changed to Methyldopa
295. (1X) Hypertension patient with risk for coronary artery disease (Probably also had high LDL). What do you
give?
a. Simvastatin
296. (1X) Patient with HTN, DM what medication can help reduce mortality?
a. Simvastatin
297. (1X) Initial HTN routine investigation?
a. Urinalysis
298. (4X) Routinely for newly diagnosed hypertension with no previous investigations:
a. EKG (others: UA, Funduscopy, Lipid profile, FBG)
b. Renin
299. (1X) A 35 y/o male newly diagnosed with HTN, which investigation would you order?
a. Resting ECG
b. Renal ultrasound
Back to TOC
Page 35 of 121
c. Stress test
300. (3X) Man with HTN is on ACEI, develops chronic cough so doctor changes his medication to which?
a. Losartan (ARB)
301. (1X) Patient who had inconsistent BP readings in 6 visits some of them were high, how to proceed?
a. 24-hour BP monitor + Lifestyle modification
302. (2X) A heart failure patient on aspirin and ACE inhibitor. Which of the following can be added for 2ndry
prevention?
a. Beta blocker
b. Nitroglycerin

Back to TOC
Page 36 of 121
[R] Diabetes Mellitus (DM)

Back to TOC
Page 37 of 121
303. (2X)[C] DM patient to increase insulin sensitivity, give?
a. Thiazolidinediones
b. Metformin
i. Note: Both Increase insulin sensitivity
304. (1X) Which one of the following drugs increase insulin sensitivity?
a. Metformin
b. Glyburide
c. Sitagliptin
d. Insulin
305. (1X) Which of the following insulin preparations has the longest half-life?
a. Glargine (+Determir)
b. Aspart (short 1/2t)
306. (1X) Diabetic woman who is taking only metformin with no major complications came in for follow up. Her
HbA1C was high. Which of the following is TRUE?
a. Adding sitagliptin to her medication regimen is a reasonable choice
b. Prescribe insulin
This case is for next 2 questions: Patient with DM, he developed dizziness.
307. (2X) Which history question is essential in the start of history taking?
a. What do you mean by dizziness, the dizziness happens in which circumstances?
308. (2X) What physical examination you do to narrow your differential diagnosis?
a. Measure BP supine, sitting, standing
309. (1X) A lady complains of dizziness when she looks up (positional), she is hypertensive & takes a thiazide,
what is the cause of her dizziness?
a. Benign paroxysmal positional vertigo (BPPV)
b. An adverse drug reaction (possible if when standing)
310. (2X) Which of the following would indicate poor DM control?
a. HbA1c of 10%
311. (4X) Which of the following is the most sensitive test for diabetic nephropathy?
a. Albumin:Creatinine ratio
b. Serum creatinine
312. (1X) What is TRUE about diabetic neuropathy?
a. It's associated with neuropathic pain
b. Neuro stimulatory test diabetic patients will have decrease in the amplitude
c. It can be acute and subacute
d. It happens due to demyelination of the nerves
313. (1X) A patient with DM & HTN his GFR was 60 3 months ago and now it is 45, what should you do?
a. Refer to nephrology (ACE-I is renoprotective)
b. Follow up in 3 months (F/U q3-4 mo if normal)
314. (2X) man with DM2 who failed life style modifications, which medication to start him on?
a. Metformin
315. (3X) A 20 y/o diabetic patient brought to ER suffering from hypoglycemic coma, the treatment of choice is:
a. 50 milligram Glucose IV
- if severe/LOC IV 25-50% dextrose or IM/SC glucagon; if mild/alert Oral 15mg glucose
316. (TO CHECK 2X) DM woman : [incomplete?]
a. ACE I
317. (TO CHECK 3X) A 25 y/o girl who ate her grandmother s diabetes pills (sulfonylureas) and became
hypoglycemic. Was given glucose and water but still hypoglycemic. What medication would inhibit insulin
release?
a. Octreotide (binds on somatostatin receptor II & V thus inhibiting GH secretion)
b. Somatostatin
c. Glucagon
Back to TOC
Page 38 of 121
- Beta Blocker, Immunosuppressant, Antipsychotic, (direct inhibitor diazoxide )
318. (1X) Drug decrease insulin production?
a. Octreotide
319. (1X) For hypoglycemic coma, the treatment of choice is:
a. 50 milligram glucoses/dextrose IV.
320. (1X) A patient with BMI 28 (over weight) with serum blood glucose of 6.5mmol/L (RBG 11.1; FBG 7), What
would you advise him?
a. You need to lose weight in order to prevent diabetes mellitus
b. You are obese and need to lose weight (over weight)
c. You are obese and have diabetes mellitus (over weight and diabetic yet)
321. (2X) Prediabetic advise?
a. Lose weight to not become diabetic
b. rescreen within one year
322. (7X) Which of these diabetic drugs increase weight:
a. Glyburide (Sulfonylurea)
b. Metformin (dec. weight)
c. Acarbose
d. Sitagliptin (wt. neutral)
323. (1X) A patient with HTN, DM & dyslipidemia was started on many medications to correct these disorders,
over the past period he gained 5 kg of weight, which medication was responsible for this?
a. Gliclazide (SU)
b. Metformin
324. (1X)[C] Which of these diabetic drugs increase weight?
a. Liraglutide (GLP-1) [wrong senior answer- As this decrease weight]
i. Answer: Any Sulfonylurea (Glyburide, Gliclazide) increase weight
325. (1X) Which of the following agents decreases weight?
a. GLP-1
326. (TO CHECK 2X) Glargine; longest duration available in KFSH, not sure.
a.
327. (2X) 1st line drug for the treatment of newly diagnosed DM?
a. Metformin
328. (1X) Tertiary prevention for DM? screen for complications
a. Retinopathy screening.
[R] Obesity
329. (1X) Obese patient with BMI of 32, without comorbidities. What would you advise him?
a. Lifestyle modification (1st line)
b. Medications to lose weight
c. Surgery
330. (1X) A 35 y/o (BMI 42) with poorly controlled diabetes, hypertension and low back pain presents to you in the
clinic. He has tried all possible ways to reduce his weight for the past 2 years and nothing seemed to work. What
would you advise him?
a. Referral to bariatric surgery (BMI >40) or (BMI> 35 with co-morbidities)
b. Intensive diet therapy and exercise for 6 months
c. Intensive diet therapy for 6 months
331. (TO CHECK 1X) Osteoarthritis + Obese?
a. Bariatric ?? (BMI> 35 + arthritis)
[R] Obstructive Sleep Apnea (OSA)
332. (2X) Complication of OSA ->
a. CHF (others: MI, HTN, RTA, A. fib, VTE, DM-II, NAFL)
333. (2X) First step in the management of obstructive sleep apnea?
a. Weight Loss

Back to TOC
Page 39 of 121
[R] Thyroid
334. (1X) Symptoms of tiredness, weight gain, fatigue, menorrhagia. What is the diagnosis?
a. Hypothyroidism
335. (2X) Patient with low PTH and hypocalcemia, what to suspect/expect cause be?
a. Post-thyroidectomy patient
336. (2X) Which of the following is the most sensitive test for diagnosing thyroid disorders?
a. TSH
b. T3
c. T4
d. Ferritin
337. (2X) Single blood test that would be expected to be the most sensitive for determining whether the patient
is euthyroid, hypothyroid or hyperthyroid?
a. TSH
338. (1X) A patient who has hypothyroidism is being treated with levothyroxine. She came in for a follow up.
Which of the following will inform you about the effectiveness of the treatment?
a. TSH
b. T3
c. T4
d. Ferritin
339. (4X) A 36 y/o female after recent thyroid surgery presented with h/o fever, nausea, vomiting, restlessness,
agitation and extreme confusion. Her observations revealed: Temp: 40°C, Pulse: 140/min, BP: 190/110
mmHg, RR: 36/min, O2 saturation: 94%. Which of the following is used for acute treatment of above
condition?
a. Aspirin
b. IV propranolol
c. IV thyroxine
d. IV hydrocortisone
e. IV acetaminophen
340. (1X) A patient with thyroid storm, what is the most important immediate treatment?
a. IV propranolol

Back to TOC
Page 40 of 121
[R] Infections:
TB
341. (TO CHECK 1X) Which of the following would confirm an active TB infection?
a. Sputum containing acid fast bacilli
b. Chest X-ray (it can be latent)
c. Quantiferon test
342. (3X) Acid fast bacilli?
a. TB Pneumonia
343. (1X) A nurse had a positive PPD results after being in contact with an active TB patient. She is
asymptomatic and her CXR is normal. How would you manage her?
a. INH (Isoniazid ) for 9 mo (always give B6 with it)
b. INH (Isoniazid) plus rifampicin
c. Give her the TB regimen
344. (1X) A nurse having her routine healthcare checkup was positive for PPD. CXR was done and was negative
for tuberculosis. Her previous PPD was negative. What is the management?
a. Isoniazid for 9 months
345. (3X) PPD positive?
a. Isoniazid 6 months
b. Isoniazid 9 months
346. (1X) PDD + ve?
a. Rule out active TB + prophylactic INH
347. (TO CHECK 1X)[C] What is NOT true about TB?
a. TB does not start at vertebral space
b. TB does start at vertebral space
348. (1X) What is the most common location of a TB infection to spread to?
a. Spine
UTI & Deep Space Infection
349. (4X) What is the most common cause of UTI?
a. E. Coli
350. (1X) Pregnant (I think 32 weeks) came with positive urine E-coli what to do for her Choices were:
a. Antibiotics (Nitrofurantoin)
b. Follow up , c) Others...
351. (2X) A 21 years old male after a recent cruise holiday presents with fever, flu like symptoms, inguinal lymph
nodes and painful genital blisters (picture) and sores. Most prudent treatment option is:
a. Famciclovir (herpes?)
b. Clindamycin gel
c. Metronidazole, doxycycline & ciprofloxacin
d. Azithromycin & rocephin
e. Diflucan
352. (2X) What is the most likely organism that causes, deep neck space infection:
a. Polymicrobial
SIRS (Systemic Inflammatory Response Syndrome)
353. (5X) A 50 y/o burn victim develops fever, tachycardia, tachypnoea while receiving treatment on burns
ward. You make a diagnosis of SIRS. Which of the following is NOT an essential element for diagnosis of SIRS?
a. High lactate of 2 mmol/L (imp in sepsis)
b. Heart rate > 90 beats/min
c. Temp < 36°C(96.8°F) or > 38°C(100.4°F)
d. Tachypnea > 20 breaths/min
e. Leukocytosis or leukopenia
354. (1X) Which of the following is NOT part of the SIRS?
a.
Back to TOC
Page 41 of 121
Lyme Disease (Deer Tick Bite)
355. (1X) History of deer tick bite, cardiac arrhythmias and facial nerve palsy. The most likely diagnosis is:
a. Lyme disease
356. (5X) Lyme disease after tick bite, presented with palsy?
a. Borrelia burgdorferi
357. (Again 1X) Patient presented with facial palsy after a tick bite, diagnosis:
a. Lyme disease/borrelia
EBV, Measles, Coxackie
358. (3X) Mononucleosis typical case; question about the virus?
a. EBV
359. (4X) A 17 y/o presented with pharyngitis symptoms, had splenomegaly and cervical lymphadenopathy.
What is the causative organism?
a. EBV
b. HIV, c) GBS
360. (2X) Picture of child with trunk maculopapular rash, with cough coryza and conjunctivitis preceding the
rash?
a. Measles
b. Mumps, c) Rubella
361. (1X) A child presents with a vesicular rash on his palms, soles & mouth, what is the most likely causative
agent?
a. Coxsackie A
b. HSV
362. (1X) A kid has a crusting yellow rash around his mouth also his brother has the same rash, which of the
following is TRUE regarding this?
a. Staph aureus is the most common cause
b. Treated with corticosteroids, c) It is viral, d) It is not contagious, e) It is mainly in adults
Cystic Fibrosis
363. (2X) Child with recurrent chest infections, malabsorption symptoms, chronic sinusitis, and nasal polyps.
What test would you do to confirm your suspected diagnosis?
a. Sweat chloride test [For Cystic Fibrosis]
RSV
364. (4X) Which of the following infections are spread from droplet route?
a. Respiratory Syncytial Virus RSV
b. Tuberculosis (airborne)
c. Herpes (airborne/ contact)
d. Varicella (airborne/ contact)
Scarlet Fever & Kawasaki
365. (1X) Strawberry tongue?
a. Scarlet fever but rule-out Kawasi symptoms first
366. (1X) Strawberry tongue and sandpaper rash. Diagnosis?
a. Scarlet fever
367. (1X) A 50 y/o male k/c of HIV presents with acute pain in his right hip. The ESR is 88, CRP 210. Radiographs
were normal, and ultrasound reveals fluid collection. Your next step is:
a. Ultrasound guided aspiration of the joint for a detailed analysis + Start a broad spectrum
antibiotic + Prepare the patient for debridement and washout of joint
b. Reassure the patient and send home on analgesics
c. Start IV antibiotics and admit
d. Ask for MRI scan of hip as well as WBC scan than admit the patient for aspiration, broad spectrum
antibiotics and debridement and washout of the joint
e. Give the patient steroids to reduce pain and swelling and broad spectrum antibiotics

Back to TOC
Page 42 of 121
368. (2X) Child with sore throat, fever, maculopapular rash all over, in hands and feet, conjunctival injection,
strawberry tongue. Dx?
a. Kawasaki [consider scarlet fever]
b. Coxsackie (hand foot mouth) disease
Scabies
369. (4X) A patient presents with an itchy rash on the webs of his fingers & waist also another member of his
family has the same compliant, what is the diagnosis?
a. Scabies
370. (2X) Picture with itching Dx?
a. Scabies

Back to TOC
Page 43 of 121
[R] Prenatal Visit vaccination:

371. (2X) Vaccination to a woman in a prenatal visit with no rubella antibodies should be?
a. AFTER delivery (right after)
372. (1X) Woman with no antibodies to rubella, comes for a prenatal visit, when do you give the rubella
vaccination?
a. Immediate postpartum period
373. (2X) 35 y/o woman, who is not sure of her childhood immunization. Which vaccine should you give her
now?
a. Tetanus
b. HPV
c. Polio
d. Zoster (Age >60)
374. (1X) A 9 y/o boy presents with a 3 cm clean hand laceration, he received a tetanus shot 2 weeks ago, what
is the best step?
a. No vaccination is needed
b. Give diphtheria
c. Give tetanus shot
d. Give tetanus immunoglobulins
375. (1X) BCG is?
a. Live attenuated

Back to TOC
Page 44 of 121
[R] HDL & LDL Management:

376. (1X) patient with dyslipidemia his tests show high total cholesterol, slightly decreased HDL, slightly
increased LDL (3.6 mmol/L) & a very high triglycerides (37mmol/L), what is the best drug to prescribe?
normal ranges were provided in the question.
a. Fenofibrate [This lower TAG more]
b. Atorvastatin [This lower LDL more]
c. Niacin (B3) [This lower LDL & increase HDL moderately]
377. (1X) A patient was started on a drug for his dyslipidemia and presents few weeks later with muscle pain
and cramping. Which of the following drugs would have caused it?
a. Statins
b. Niacin, c) Cholestyramine, d) Ezetimibe
378. (2X) Familial cholesterolemia exercise not helpful, LDL > 6 what to do?
a. Simvastatin
379. (1X) A 58 y/o male has history of hypertension and wants to reduce his risk of developing an MI. His HDL is 32
(normal is 40). What can increase his HDL?
a. Exercise (Mayo Clinic says: Get more physical activity. Within 2 months frequent aerobic exercise can increase HDL cholesterol by ~5% in otherwise healthy adults)

b. Fenofibrate
380. (3X) Best treatment of low HDL:
a. 1st exercise (best drug: niacin)
381. (1X) Exercise:
a. 5 times a week 30 mins (Recommendation is: 40-60 minutes 5 times a week)
382. (3X) Best treatment of high LDL:
a. Statin
383. (2X) Man, who is has prediabetes, pre-HTN, and High LDL, what will you do?
a. Lower cholesterol, carb and reduce weight & do exercise and F/U 3m
b. Lower cholesterol, carb and reduce exercise, and schedule F/U in 1 year

Back to TOC
Page 45 of 121
[R] Hepatitis:
384. (2X) HBV + Surface antibodies + Negative antigens?
a. Good immunity against HBV
385. (1X) Healthcare worker stuff, about vaccinations his hep B profile: negative surface antigens, High antibodies
,what's his status?
a. Has protection against hep B
386. (1X) Which of the following is seen in a chronic liver disease patient?
a. Palmar erythema
b. Tenosynovitis
387. (TO CHECK 2X) Breaking bad news about hepatitis?
a. Time for discussion tx choices, making sure to bringing a companion.
b. Help him prepare for the information by using a statement like, I m afraid I have some bad news for you.
c. Give enough time to discuss management options and plan
d. All the above
388. (1X) A case of the following labs: Hepatitis surface antibodies positive, HBsAg: negative, HBcAg: negative
a. Patient has adequate immunization against hep B
389. (TO CHECK 2X) Woman who had a RTA, had a splenectomy, got blood transfusions, 3 weeks later she
presents with high fever, chills, no GI, Resp or other Sx. Only abnormal was elevated AST/ALT?
a. Hep A
b. Hep C? (because of blood transfusion?)
c. Intraabdominal abscess
d. Ebola (no)
e. Brucella

Back to TOC
Page 46 of 121
[R] Musculoskeletal:
This case is for the next Four questions:
25 y/o man went to Thailand and came back, he is currently suffering from severe knee pain:
390. (3X) Which of the followings is an essential part of history in this case?
a. Having multiple sexual partners (gonococcal arthritis)
391. (3X) What is the next step management that would determine what is wrong with his knee?
a. Knee aspiration
392. (3X) What are you suspecting in this patient ?
a. Septic arthritis (N. Gonorrhea)
393. (3X) How do you confirm the diagnosis if you are suspecting gouty arthritis?
a. Joint aspiration
394. (1X) Diagnostic test for gout?
a. Joint aspiration
395. (2X) Patient w/ first toe finger joint pain (proximal phalanx), urine sample showed ve birefringent crystals Dx?
a. Gout
396. (1X) Negative birefringence:
a. Gout
397. (1X) Positive birefringence:
a. Pseudogout (P=P)
398. (1X) Which of the following is part of the treatment of neck or back pain?
a. Stretching
399. (1X) Which of the following can be given to a 62 y/o woman to decrease fracture risk?
a. (bisphosphonate)
b. Calcium only
c. Vitamin D only
400. (1X) Patient with weakness and numbness in lower back and buttocks worsens when he coughs or sneezes.
What s the most important investigation?
a. MRI of whole spine
b. CT scan
401. (TO CHECK 4X) Which has a proven benefit for chronic lower back pain?
a. Exercise
b. Massage
c. Acupuncture
d. Aroma therapy
402. (1X) A young health boy is having back pain with no red flags. How would you treat him?
a. Give analgesia and educate about proper posture and back care
b. Reassurance
c. Do a CT spine
403. (1X) Which examination is more appropriate in a patient aged 58 y/o with history of low back pain and
paresthesia in the leg?
a. MRI of the spine
b. EMG
c. Nerve conduction study
d. CT scan of the lumbar spine
e. Anal examination to exclude prostatic malignancy
404. (2X) A Porter (Similar to custodians & janitors job) presents with medial epicondyle pain and swelling. X-
rays are normal. What is the most likely diagnosis?
a. Golfer Elbow
b. Tennis elbow
c. Bursitis
405. (1X) A man complains of right elbow pain, the pain is along the lateral epicondyle interfering with his job to
Back to TOC
Page 47 of 121
lift boxes, what is the diagnosis?
a. Tennis elbow
b. Golfer s elbow
c. Osteoarthritis
406. (1X) X-ray finding of: (osteophytes formation, joint space narrowing, subcondral sclerosis)
a. Osteoarthritis
407. (1X) A patient complains of neck pain with a restricted rage of movement, what is the diagnosis?
a. Osteoarthritis
408. (1X) Ankylosing spondylitis presented with
a. Sacroiliac Stiffness
409. (1X) Young patient presented with sacroiliac stiffness and pain, diagnosis:
a. Ankylosing spondylitis
410. (1X) Lady feels fatigued for the past few years, swollen red PIP and DIP with nodules of elbow
a. RA
411. ( 1X) Woman with pain in finger joints for some time and nodules at elbows:
a. Rheumatoid arthritis
412. (3X) A 60 y/o lady has MCP joints and PIP nodules (and stiffness I think)?
a. Rheumatoid arthritis
b. Osteoarthritis.

413. (TO CHECK 1X) A patient with Rheumatoid Arthritis who has limited ROM. Which of the following statements
is CORRECT?
a. The rotator cuff is
b. Arthroplasty needs to be done (indication is joint destruction despite therapy & rehab) (PAIN)
c. MRI needs to be done and shows joint space narrowing (research is uncertain)
d. Diffuse inflammation of the articular cartilage
e. Patchy inflammation of the articular cartilage
414. (1X) A 40 y/o wants to screen for osteoporosis and have a X-ray done, what do you tell her?
a. BMD (Bone Marrow Density) screening start at age of 65 for osteoporosis
b. She can have her X-ray done right now
415. (2X) Woman who wants osteoporosis scan?
a. Scan after age 65
416. (1X) Woman who wants osteoporosis scan (DEXA)?
a. Scan after age 65
417. (TO CHECK 1X) Supraspinatus injury?
a. Can t abduct 90 degrees
418. (NEW) What is TRUE about hand examination?
a. Carpal tunnel causes thenar muscle wasting
b. Koilonychia is caused by liver disease (iron deficiency)
c. Osler and heberden nodes seen in patients with osteoarthritis
419. (TO CHECK 2X) Investigations of carpal tunnel? (~ nerve conduction)
a.
420. (1X) Trauma to 2nd finger and hand swelling, spread to forearm Tx?
a. Analgesia +tetanus +send to ortho for OR open and clean
421. (1X) Sciatica leg raise test degree?
a. 0-30
b. 10-60
c. 45-90
Back to TOC
Page 48 of 121
422. (1X) Exercise:
a. 5 times a week 30 mins
423. (3X) Man came with back pain, when would it be an emergency?
a. Saddle anesthesia
b. Weakness of dorsiflexion
c. Pain radiation to the big toe

424. (2X) There was a question about a man came with back pain and they said what can be considered as
emergency?
a. Numbness in the buttocks region (saddle anesthesia- wasn t written)
b. Pain radiation to the big toe
c. Weakness of group of muscles in the leg (it should not be localized)
425. (1X) Frozen shoulder presentation:
a.

426. (2X) What is initial management of a regular MSK complaint (I believe low back pain) in a normal patient?
a. Stretching exercise
427. (1X) A guy fell on his right shoulder and as unable to abduct his arm from 0 to 90 degrees. Which of the
following muscle is affected?
a. Supraspinatus
b. Teres minor
c. Subscapularis
d. Infraspinatus
428. (3X) Woman with pain at sole of the foot, worse in morning, better with long distance walking, what is the
Dx?
a. Plantar fasciitis
b. Osteomalacia
c. Calcaneal epiphysisitis

Back to TOC
Page 49 of 121
[R] Sexually Transmitted Disease (STD):
429. (3X) Which STD needs lifetime of suppression?
a. HSV
b. Gonorrhea
c. Chlamydia
430. (Again 1X) Which of the following STDs need lifelong treatment?
a. Trichomonas
b. Herpes simplex virus
c. Chlamydia
d. Gonorrhea
431. (And again 1X) HSV infection needs?
a. Treatment/suppression for life (acyclovir 400 mg PO bid, or famciclovir 250 mg bid, or valacyclovir 0.5-1 g PO
OD)
432. (2X) What investigation would you order for a patient with vaginal discharge suspected to have
trichomonas?
a. Saline mount microscopy
b. Whiff test
c. PCR
433. (TO CHECK 4X)[C] a patient presents with typical signs of PID, which of the following tests would you do?
a.
434. (Again 1X)[C] Which of the following is done to confirm a diagnosis for gonorrhea and chlamydia infection?
a. DNA probe test [NAAT]
b. PCR [according to Up-to-date, PCR is more sensitive and becoming more cost-competitive than NAAT]
435. (1X) Chlamydia & Gonorrhea testing?
a. DNA probe test
i. Also PCR is becoming more frequently used
436. (2X) Most common sexually transmitted Infection STD:
a. HPV (as per CDC fact sheet)
437. (2X) A 19 y/o sexually active female; most common vaginal infection?
a. Bacterial vaginosis
b. HPV
c. Candida
i. According to Dr Butt:
1. MC STD HPV warts
2. MC Vaginal Infection Bacterial Vaginosis
438. (1X) A lady came with abnormal vaginal discharge. It is most likely?
a. Bacterial vaginosis
439. (3X) Bacterial vaginosis; fishy odor of grayish white/yellow vaginal discharge; clue cells treatment?
a. Metronidazole
440. (7X) STD with PID (cervical motion tenderness, bilateral adnexal tenderness, and purulent discharge) and
Bac Vaginosis also for 1-week thin discharge with fishy odor, what management?
a. Metronidazole (MLA), Rocephin (Ceftriaxone), Azithromycin
b. Metronidazole (MLA), Rocephin (Ceftriaxone), Clindamycin
c. Metronidazole (MLA), Doxycycline, Ciprofloxacin [old batch answer, double check]
d. Other options that didn t have metronidazole

Back to TOC
Page 50 of 121
[R] Centor Criteria (for GAS)
441.
(1X)
Who
m
from
the
follow
ing
patie
nts
would
you
give
antibi
otics
accor
ding
to
Cento
r
criteri
a:
a. 1
0 y/o
boy (1) + Lymphadenopathy (1) + Fever (1) + no cough (1) + tonsil redness (1) = 5
442.
a. No cough + Fever + Tonsillar swelling + Lymphadenopathy + Age 3-14
443. (1X) Which of the following is a part of Centor criteria?
a. Absence of cough
444. (1X) Man presents with sore throat + fever + tender lymphadenopathy + no cough. What is your management?
a. Amoxicillin
b. Rapid strep test

Back to TOC
Page 51 of 121
[R] Obstetrics & Gynecology (Emergencies)
445. (1X) A 22 y/o who had acne, and you started her on Isotretinoin. What should you warn her about?
a. Teratogenicity
b. CAD risk factors
446. (2X) Which medication induces ovulation and restores the monthly period in a patient with PCOS?
a. Metformin
b. Misoprostol
447. (TO CHECK 1X) How do u help with the infertility in PCOS?
a. 1st line: Weight loss helps with spontaneous ovulation, then clomiphene citrate, metformin
448. (3X) Pregnant lady with cat exposure and lymphadenopathy
a. Toxoplasma. (Bartonella henselae if scratch)
449. (1X) DMPA (Depot medroxyprogesterone)?
a. Given every 3 months
450. (1X) A lady presented with bone pain, Abdominal striate+hirsutism. Which if the following drugs could have
caused this?
a. Steroids
451. (1X) Female with HTN and facial acne and hirsutism?
a. Cushings (if no HTN, I would think more of PCOS)
452. (1X) Eclampsia case was given mg sulfate and signs of toxicity:
a. Stop mg sulfate and start calcium gluconate
b. Continue Mg sulfate and add K
453. (1X) A patient with eclampsia is treated with magnesium sulfate started to have respiratory difficulties and
decreased deep tendon reflexes, hat is the management?
a. Attend to the patient s ABCs then stop Mg++ & start calcium gluconate
454. (TO CHECK 2X)[C] A lady with symptoms of generalized weakness, she had some weight gain, her cycles
have been irregular and sometime absent. What is the diagnosis?
a. Cushings
b. Hypothyroidism (?)
c. Anemia
455. (3X) Menopause criteria:
i. Age greater than 50 y/o +12 months of amenorrhea
ii. Laboratory criterion: FSH level > 35 miu/ml & Estrogen < 20pg/ml
456. (2X) A 51 y/o IUD + not having period for year?
a. Menopause
457. (1X) When to remove copper coil ->
a. 5 or 7 years?
i. I guess copper = 5-10, progesterone 3-5 yrs.
458. (1X) Postmenopausal vaginal bleeding investigation?
a. US
459. (1X) Postmenopausal women came with complaints of vaginal bleeding. What is your next step?
a. Transvaginal US and endometrial biopsy [Cancer until proven otherwise]
460. (1X) Post-menopausal female with bleeding. What s first step of management?
a. Pelvic US
b. Give OCP
461. (1X) Endometrial biopsy?
a. In a post-menopausal woman with PV bleeding
462. (2X) First line treatment for 1ry dysmenorrhea?
a. NSAIDs
463. (1X) Primary dysmenorrhea Tx?
a. NSAIDs
464. (1X) When is the cardiac output highest in the pregnant mother?
Back to TOC
Page 52 of 121
a. Right After birth
b. First trimester of pregnancy
c. Second trimester of pregnancy
465. (1X) Which of the following decreases in pregnancy
a. Systemic vascular resistance
b. CVP
c. Cardiac output
d. Stroke volume
466. (1X) Which of the following nicotine routes reaches the brain the fastest?
a. Cigarettes
b. Nicotine patch
c. Nicotine nasal spray
467. (1X) Which of the following should you NOT use in a pregnant woman trying to stop smoking?
a. Bupropion
b. Nicotine gum
c. Nicotine patch
d. Nicotine pills
468. (2X) Earliest time you can hear a fetal heartbeat?
a. 5 weeks
b. 6 weeks
c. 8 weeks
d. 10 weeks
469. (3X) Methotrexate indication
a. <4cm with no Fetal heart rate (<3.4cm)

Back to TOC
Page 53 of 121
[R] OCPs (Side effects) [Family Medicine]
470. (4X) Which of the following is CORRECT regarding Combined oral contraceptive pills:
a. Increases Risk of DVT
b. Increases Risk of endometrial cancer
c. Decreases incidence of cervical cancer
471. (1X) 35 y/o female visited the family medicine clinic for contraception counselling. What is your role as a GP?
a. Education
b. Review the contraception options with her
c. Tell her what is the best way of contraception for her
472. (1X) A contraindication to use OCP:
a. Previous history of DVT
b. Migraine without aura
473. (5X) Which of the following would you NOT prescribe combined oral contraceptives?
a. A 25 y/o female with migraine with aura (absolute C/I)
b. A 35 y/o female with family history of DVT
c. A 30 y/o female who smokes d. (should be >35 y/o)
474. (1X)[C] Patient with OCP?
a. Educate
b. Assist to look at options
475. (1X) What is a side effect of OCPs?
a. Increase risk of Venous thromboembolism
476. (3X) Ectopic pregnancy: what to do next if U/S is ve and urinary hCG is low/negative?
a. Do blood hCG
477. (2X) A 21 y/o married female presents to ED with 2 hours history of right LQ pain. She denies any urinary
symptoms or vaginal bleeding. You check a urine point of care pregnancy test and it is negative. The next
most appropriate step would be to:
a. Check serum bHCG quantitative, IVF and pain control
b. Check pelvic US, R/O
c. Order CT scan to R/O appendicitis
d. Treat her symptomatically, reassurance and discharge home
e. Acute abdominal screening x-rays to r/o bowel obstruction
478. (1X) What to do next if U/S -ve + urinary hCG is low/negative in a lady of childbearing age with SEVERE
lower abd pain?
a. Repeat hCG (ectopic pregnancy until proven otherwise)

Back to TOC
Page 54 of 121
Anesthesia:
[R] General
479. (1X) NOT correct about RSI (Rapid sequence induction)?
a. No bag mask-in (you pre-oxygenate) to prevent air going to esophagus (so this is correct!!)
b. It doesn t need a muscle relaxant (it does) (3: Propofol + fentanyl + muscle relaxant)
480. (2X) RSI not correct?
a. No bag mask ventilation
481. (TO CHECK 1X) NOT correct about RSI?
a. No bag masking
b. It doesn t need a muscle relaxant (This will be given after induction)
482. (1X) To terminate the effects of a muscle relaxant, atropine & neostigmine were given. How does it work?
a. Antimuscarinic
b. Antihistamine
c. Anti-nicotinic
483. (2X) Malignant hyperthermia what would you do?
a. Urgent admission (start IV dantrolene)
484. (1X) Which drug causes malignant hyperthermia?
a. Succinylcholine (also inhaled anesthetics)
485. (3X) A patient had decrease in O2 saturation post operatively, what is the next step in management?
a. Mask
b. Intubation
486. (1X) What is the most common complication of post op pain? (chronic pain?!)
a. Acute Confusion
b. Bradycardia
487. (4X) Why should we be cautious in the hx taking in pre-anesthesia of a pt with a history of coronary stent:
a. Because they stay on long term blood thinners
488. (3X) which is NOT associated with high oxygen therapy?
a. No answer, just read below:
i. Potential adverse clinical consequences of supplemental oxygen therapy include:
1. Extrapulmonary toxicity.
2. Injury of:
a. Airway
b. Lung Parenchyma
3. Absorptive Atelectasis
4. Accentuation of preexisting hypercapnia
489. (2X) Why do we give preoxygenation before intubation, possible answer is? To Inc. storage of O2
a. More time for apnea
490. (3X) What affect the oxygen transport more:
a. Cardiac Output
491. (2X) How frequently should HR and BP be measured in OR according ASA guidelines:
a. 2 minutes
b. 5 minutes
c. 7 minutes
d. 10 minutes
e. 15 minutes
492. (1X) Best method for Post op pain assessment:
a. Objective test using numerical values or sad/happy faces if it s a child patient.
493. (1X) Post laparotomy op pain management is important due?
a. Normally coughing will increase pain; therefore patients avoid coughing, decreasing O2 levels.
494. Few Important Notes on Post op pain:
Management best method:
Back to TOC
Page 55 of 121
PCA.
Pain best assessed:
objective test using numerical values, or
Sad/happy faces (if it s a pediatric patient)
Beneficial effects of effective pain management (PCA):
(a) relieves suffering
(b) Early mobilization
(c) Short hospital stay
(d) reduce hospital cost
(e) increased patient satisfaction.
Most common complication of post-op pain:
Developing chronic pain
Post laparotomy op pain management is important due?
Normally coughing will increase pain; therefore, patients avoid coughing, decreasing O2 levels.
What is the advantage of patient controlled analgesia?
Better patient satisfaction

Back to TOC
Page 56 of 121
[R] EtCO2 & Parkland Formula
495. (2X)[C] How much blood can a 4x4 sponge hold?
a. 5 ml
b. 10 ml
c. 20 ml
i. A towel can hold 100-150 ml
496. (2X) What is the blood volume for a 70 kg woman?
a. 2450 ml
b. 3550 ml
c. 4550 ml
d. 5250 ml
e. 52600 ml
i. Men kg x 75 ml/kg [So here if male it is 70 x 75 = 5250 ml] [Calculation Reference]
ii. Women kg x 65 ml/kg [So here it is 70 x 65 = 4550 ml
497. (2X) What is the maintenance fluid for a 70 kg woman?
a. 55 ml
b. 70 ml
c. 80 ml
d. 110 ml
e. 220 ml
4 (first 10 kg) by 2 (2nd 10 kg) by 1 (rest of kg) Rule [so 4(10 kg) + 2(10 kg) + 1(50 kg) = 110 ml]
498. (2X) What is the maximum allowed dose of 1% Lidocaine for a 50 kg patient?
a. 25 ml [5 mg/kg x 50 kg = 250 mg] and [250 mg = 25 ml]
499.

(2X
)

What is the normal value of end tidal CO2?(35-45)


a. 15 mmHg
b. 18 mmHg
c. 33 mmHg
d. 55 mmHg
e. 75 mmHg
500. (1X) A 20ml Normal EtCO2 is
a. 35-45 mmHg
501. (5X) What reduces the EtCO2?
a. Increase tidal volume
b. Decrease minute ventilation (Inc.)
c. Decrease RR (hypoventilation) (Inc.)
d. Malignant hyperthermia
502. (1X) Which of the following decreases EtCO2:
a. Hyperventilation
503. (1X) Which of the following is TRUE about EtCO2?
a. Hyperventilation decreases EtCO2
b. Hypoventilation decreases EtCO2
504. (1X) The function of soda lime is to:
a. Absorb CO2
b. Provide Oxygen
Back to TOC
Page 57 of 121
[R] Pain-Killer & Post-OP/Procedures
505. (1X) Post-operative pain assessment could:
a. Turn into chronic pain if poorly treated
b. Be disregarded if the anesthetist in charge is convinced that the patient had got sufficient pain killers
c. Be scored on a visual analogue scale in a semi-conscious patient
d. Be treated with high boluses of narcotic medications with no need for recovery nurse monitoring
506. (1X) All of the following are excitatory neurotransmitters that modulate pain EXCEPT:
a. Somatostatin
b. Substance P
c. Glutamate
d. Aspartate
e. ATP
f. Calcitonin gene related peptide
507. (TO CHECK 4X) Chemical arteritis can be caused by
a. Thiopental
508. (1X) Pain with Propofol?
a. Add lidocaine
509. (TO CHECK 4X) The optimal head position for intubating the patient under anesthesia is?
a. Extension of the neck
b. Flexion of the neck and extension at atlanto-occipital joint. (neck flexion at body and extension at head)
c. Flexion of the neck, extension at the atlanto-occipital joint and jaw thrust
d. Extension of the neck and jaw thrust
510. (1X) During intubation, the aim of jaw thrust is, which of the following?
a. Put the tongue at the posterior pharynx and put your fingers behind the angle of the jaw and push the
mandible forward
i. (The main aim of the jaw thrust is to bring the chin forwards without moving the neck. This lifts
the tongue from the back of the airway and relieves obstruction)
511. (1X) The preferred method to open the airway in the case of head & neck trauma is:
a. Jaw thrust
b. Head tilt chin lift
c. Head tilt neck lift
512. (NEW) What is the most potent painkiller
a. Su-Fentanil [ Su = Super ] (5-10 more potent than fentanyl)
b. Remifentanil
c. Fentanyl
d. Morphine
e. Alfentanil
513. (1X) Order the following opiates from most potent to least potent?
a. Remifentanil > Fentanyl > Morphine > Pethidine
b. Morphine > Fentanyl > Remifentanil > Pethidine
514. (TO CHECK 4X) Morphine which of the following isn t a side effect:
a. Hallucinations
515. (4X) PCA (Patient Controlled Analgesia) advantage:
a. Better Patient Satisfaction
516. (1X) Best method for post op pain management?
a. Patient Controlled Analgesia
517. (1X) Beneficial effects of effective pain management (PCA)?
a. Increased patient satisfaction.
b. Relieves suffering
c. Early mobilization?
d. Short hospital stay
Back to TOC
Page 58 of 121
e. Reduce hospital cost
518. (3X) In Pregnancy?
a. Sphincter tone decreases
519. (1X) All of the following increase during pregnancy EXCEPT?
a. Functional Residual Capacity
b. Tidal Volume
c. Respiratory Rate
520. (Again 1X) All of the following increase during pregnancy EXCEPT?
a. Functional Residual Capacity (FRC)
521. (1X)[C] Relative contraindication for spinal anesthesia:
a. Low back pain
b. Hypotension (absolute)
c. Coagulopathy (absolute)
d. Infection at site (absolute)
e. Severe valvular stenosis (?) severe absolute; mild relative
522. (1X) Multimodal approach to treat postoperative pain means:
a. Give different groups of medications that attack pain from different angles
b. Use the maximum tolerable dose of painkillers
c. Consult different specialties to reach the best possible treatment approach
d. Anesthetist has to handover all the names and doses of the intraoperative painkillers used to recovery nurses

Back to TOC
Page 59 of 121
[R] Spinal Anesthesia
523. (2X) Which one is a IV hypnotic?
a. Propofol
524. (2X) What is NOT CORRECT about spinal anesthesia?
a. give it thoracic
b. it s inserted at L3-4/L4-5 (true)
525. (NEW) Which of these layers are NOT pierced during spinal anesthesia?
a. Posterior longitudinal ligament
b. Supraspinous ligament
c. Interspinous ligament
d. Ligamentum Flavum
e. Dura Mater
526. (TO CHECK 2X) Which of the following is CORRECT?
a. Epidural needle is larger than spinal MPA
b. Only spinal causes hypotension
527. (1X) Which of the following statements is CORRECT?
a. T10 is at the level of the umbilicus
b. Thoracic vertebrae is similar to lumbar vertebrae
c. There are 21 spinal nerves
d. Caudal spine contains CSF
528. (TO CHECK 2X) Spinal anesthesia indication?
a. For surgeries below T11 [possible]
b. Below lumbar, C/S is typical
529. (TO CHECK 1X) Spinal anesthesia for?
a. Surgery below the umbilical
530. (1X) Which of the following is NOT a requirement to be monitored during general anesthesia?
a. CVP
b. ECG
c. End Tidal CO2
d. Non-invasive BP
e. Heart Rate
531. (TO CHECK 1X) Which of the following locations can you inject epinephrine with a local anesthetic?
a. Scalp (highly vascularized so epi won t cause ischemia here)
b. Earlobe
c. Fingers
d. Toes
e. Nose
532. (1X) What is NOT correct about spinal anesthesia?
a. Give it thoracic, Laparoscopic surgery, patients on Plavix, Agitated patients
b. it s inserted at L3-4/L4-5
c. Surgeries below T11/T10 (Umbilicus)
d. High risk patients
533. (TO CHECK 1X) Spinal anesthesia can be done for which of the following?
a. Not sure.???

Back to TOC
Page 60 of 121
[R] Drug Administration
534. (2X) Drug cause adrenal insufficiency?
a. Etomidate
535. (1X) Pain with Propofol?
a. Add lidocaine
536. (3X) Burns which med to use?
a. Ketamine
537. (NEW) Child in pain with difficult IV access. What can be given IM?
a. Ketamine
538. (1X) NMDA antagonist (N-methyl-D-aspartate an ionotropic glutamate receptor)?
a. Ketamine
539. (1X) Induction agent has analgesic function too?
a. Ketamine
540. (1X) Which IV anesthetic increases blood pressure:
a. Ketamine
b. Propofol
c. Thiopental
541. (1X) What is the most common site for arterial injection?
a. Radial artery
b. Brachial artery
c. Inguinal artery
d. Femoral artery
e. Popliteal artery
542. (2X) Post anesthesia care with high HR & BP & RR, and agitated what do we give?
a. Tx: Fentanyl, Propofol, Midazolam
543. (1X) Which of the following is TRUE about Dexmedetomidine?
a. It has analgesic properties
544. (1X) An anesthesiologist injected thiopental in the artery by mistake. What would happen as a result?
a. Chemical arteritis (treated with anti-coagulant)
b. Lactic acidosis
c. Venous embolism
545. (3X) Which one of the following is an intravenous hypnotic anesthetic?
a. Propofol
b. Sevoflurane
c. Fentanyl
d. Nitrous oxide
546. (1X) Which of the following is NOT a definitive airway?
a. LMA (for emergency)
b. Tracheostomy
c. The tip of the ET tube below the vocal cords in the trachea
d. The ET tube connected to a ventilator
547. (2X) At what level do you insert the needle for spinal anesthesia?
a. L1-L2
b. L3-L4
c. L5-L6
548. (1X) While giving an epidural which layer do you NOT cross?
a. Dura (you stop in the epidural space)
b. Ligamentum flavum
c. Infraspinatus ligament
d. Supraspinous ligament
549. (1X) Management for post epidural puncture headache:
Back to TOC
Page 61 of 121
a. Epidural Blood Patch
550. (1X) Which of the following is NOT included in the management of post puncture headache?
a. Prolonged standing
b. Epidural blood patch
c. Analgesics
d. IV fluids
e. Bed rest
551. (TO CHECK 1X) Properly applied pressure patch
a.
552. (1X) A very important way of assessing cardiorespiratory risk of anesthesia is:
a. Asking about patient s functional capacity
b. History of previous anesthetics
c. Knowing patient s occupation
d. Vital signs
553. (3X) Patient scheduled for an elective surgery, but he had a breakfast meal by mistake. How many hours should
the surgery be delayed?
a. no need to delay
b. 4 hrs
c. 2 hrs
d. 6 hours
554. (1X) A patient of 60 kg body weight scheduled for surgery and has been fasting for 8 hours, which
statement is TRUE for that patient: (4X10 + 2X10+ 1X40 = 100/hr = 800/8hrs) rule of 4X2X1
a. His fasting fluid replacement is around 850 ml
b. We need to give him a minimum of 1500 ml as a substitute for fasting hours [One Senior Answer]
c. He needs to fast a minimum of two more hours to avoid aspiration risk
d. Fasting fluid replacement is changing based on the type of surgery
555. (TO CHECK 1X)[C] You were allocated to the paranesthesia clinic where a 3 y/o child scheduled for
tonsillectomy in 2 days came in and his mother complained that he had got an upper respiratory tract
infection with fever and runny nose for the last 10 days. Currently, he is not febrile but still has a runny nose.
The best action is to:
a. Defer the surgery for at least another week after his full recovery? [more likely]
b. Be cautious with regards to the possible airway irritation during anesthesia?
c. Reassure the mother and proceed
d. Consult a chest physician
556. (1X) Using a high flow CPAP mask is helpful in:
a. Improving oxygenation in a spontaneously breathing patient
b. Improving oxygenation in a comatose patient
c. Decreasing the incidence of aspiration
d. Providing good ventilation support to a bleeding patient with airway trauma
557. (4X) Indication to use CPAP:
a. Obstructive sleep apnea
558. (1X) Morphine which of the following isn t a side effect:
a. Hallucinations
559. (3X) Induced by Propofol and was Bevin curonium then developed bilateral wheezing and hypotension
(Anaphylaxis secondary to Propofol), what to give?
a. IM Epinephrine 1:1000 0.5mg
560. (1X) Opioid side effect EXCEPT:
a. Mydriasis (dilated pupils)
b. Nausea vomiting
c. Bradycardia
d. Constipation
Back to TOC
Page 62 of 121
e. Respiratory depression
561. (1X) What is a symptom of opioid overdose?
a. Miosis (conristed pupils)
b. Mydriasis (dilated pupils)
c. Diarrhea
d. Respiratory Stimulation
e. Anaphylaxis
562. (2X) Narcotics?
a. The mu receptor is G protein coupled
b. Morphine is highly lipid soluble
c. Intrathecal morphine is more respiratory depressant than fentanyl
[R] Minimum Alveolar Concentration (MAC)
563. (3X) Which of the following is NOT true regarding MAC?
a. It doesn t change depending on the weight (true)
b. It is the same from males & females (true)
c. It is different for different types of inhaled anesthetics (true)
564. (Again 1X) MAC is:
a. Not affect by weight
565. (NEW) Definition of MAC
a. The concentration at which 50% of patients respond to a surgical stimulus.
i. Link: https://accessanesthesiology.mhmedical.com/Sol/SolQuestion?solQuizId=398448&solQuestionId=
566. (3X) MAC is a measure of:
a. Inhaled anesthetic potency
b. Induction speed
c. Recovery speed

Back to TOC
Page 63 of 121
[R] Resuscitation
567. (5X) Hypovolemic shock management:
a. IV crystalloid (3 liters of crystalloid per 1 liter of blood loss)
568. (1X) Hypovolemic shock:
a. ABC, IV fluids, identifying source of hypovolemia
569. (4X) CPR
a. Increases ventricular fibrillation time to shock
570. (3X) Way to monitor CPR effectiveness:
a. Capnography
b. Intra-arterial pressure
c. Systolic Blood pressure
571. (1X) Which of the following is the most effective method of knowing the CPR quality?
a. Capnography
b. Arterial oxygen
c. Systolic blood pressure
572. (1X) Which one of the following is right about CPR?
a. The rate of the chest compressions is 80/100/min
b. The rate of the chest compressions is 100-120/min
c. The rate of the chest compressions is 60-80/min
573. (1X) What is the correct compression to ventilation ratio in an 8 y/o attended by 2 rescuers?
a. 15 compressions to 2 breaths
b. 30 compressions to 2 breaths
c. 15 compressions to 1 breaths
574. (1X) You are preforming a CPR on a patient with multiple rescuers & the AED just arrived, what is the best step?
a. Turn on the AED
b. Continue CPR for 2 minutes
575. (1X) Unconscious patient not sure of pulse?
a. Start CPR and call for help
576. (1X) When to stop CPR?
a. Team leader tells you to stop for shocking, or pulse checking.

Back to TOC
Page 64 of 121
[R] Airway
577. (1X) Which one is the most serious airway scenario?
a. Patient is paralyzed, and you cannot Ventilate (hypoxemia)
b. Patient is paralyzed, and you cannot Intubate
c. Patient is still breathing on his own, but you cannot apply a proper face mask (no)
d. A tooth got broken whilst intubating a patient and you couldn t find it
578. (1X) A patient that couldn t get intubated what do you do?
a. Insert laryngeal mask airway (LMA) and call for help
579. (1X) Laryngeal mask airway (LMA) can be used in: (for short procedure)
a. Hand ganglion excision
b. Gastrectomy
c. C section
d. Cholecystectomy
e. Emergency appendectomy
580. (TO CHECK 1X) Which one of the following is a contraindication for inserting a NGT?
a. Penetrating neck trauma
b. C-spine fracture
c. Blood at meatus.
581. (1X) You have been called to recovery room for 3 y/o post tonsillectomy patient who had 140/90 blood
pressure, 130/min pulse rate and oxygen sat at 91%. He was quite agitated with working accessory muscles of
breathing. The best approach could be:
a. Check airway as he might have an obstructed airway
b. Do nothing as his vital signs are acceptable
c. Give him a dose of painkiller
d. Call the surgeon as it might be a surgical problem
582. (2X) Oropharyngeal airway, what is NOT TRUE?
a. It is only one size/comes in 1 color
b. It has rounded edges
c. It s a bite block
d. It s a central airway?
583. (1X) Oropharyngeal airway feature TRUE:
a. Bite block
b. All same colors
584. (2X) Sign of INCORRECT EET tube:
a. O2 going down
585. (1X) Which is NOT predictive of difficult intubation
a. Thyromental distance of >8
b. Mallampati 4
c. Obesity
d. Short bullneck
e. Bucktooth, protruding
586. (3X) Which of the following is an indication of a difficult intubation?
a. < 4 cm mentor to the thyroid
b. > 9 cm from angle of mandible to mentum
c. They can extend Atlanto-axial
d. Can open mouth fully
587. (2X) Which one of the following is NOT a risk factor for difficult bag mask ventilation?
a. Thyromental less than 6 cm
b. Thyro-angle more than 9 cm
c. Mallampati
i. Answers don t make sense
Back to TOC
Page 65 of 121
ii. 5 independent risk factors of difficult mask ventilation:
1. (1) Age > 55 years of age
2. (2) BMI > 26
3. (3) History of snoring (obstruction)
4. (4) Beard
5. (5) Edentulous
588. (5X) Causes of difficult airway:
a.
i. (I forgot the exact choices, but these are the correct answer).
589. (3X) What is TRUE about LMA?
a. Less traumatic than ETT
b. Had to be laryngoscope mediated
c. Used for long surgeries
d. Secure 100% airway
e. Non-fasting patient usage
590. (Again 1X) What is TRUE about the LMA?
a. LMA is less traumatic than ETT
591. (4X) which is NOT a definitive airway
a. LMA
592. (1X) Just before commencing surgery and while giving a dose of antibiotic, the patient started to have
increasing wheezes and his consciousness started to deteriorate. Best immediate action is:
a. Announce an emergency situation and call for help
b. Start inotropic support
c. Ask nurses to prepare antihistaminic
d. Reassure the team that the patient will be fine soon
593. (1X) The main target for the preanesthetic visit is to:
a. Optimize patients conditions if needed
b. Discuss the procedural risks
c. Take an informed consent
d. Check lemon score
594. (4X) Airway paranesthesia assessment:
a. Answer was no single method is conclusive, use multiple
595. (2X) Which of the following can be used for assessing difficult airway?
a. Mallampati score
596. (2X) Mallampati classification when you cannot see the soft palate:
a. Class IV
597. (5X) Ambu-Bag indications:
a. Acute emergency cases such as post-trauma.
b. Inadequate oxygenation or ventilation
c. As bridge to intubation
598. (1X) Which of the following devices deliver the maximum FiO2?
a. Non-rebreather mask
b. Venturi mask
c. Partial rebreather mask
599. (1X) A COPD patient presents with severe shortness of breath & cyanosis, his O2 sat is 86%, what is the best
step in the management?
a. Venturi mask to achieve an O2 sat of 90% (?)
b. Non-rebreather mask in preparation for intubation
c. Intubation
600. (1X) First line in the treatment of COPD: (smoking cessation)
a. Anticholinergic
Back to TOC
Page 66 of 121
b. Oxygen
601. (TO CHECK 1X) Respiratory depression post thyroidectomy, manage:
a. Tracheotomy (?)
b. MRI
602. (2X) The following are independent risk factors for bag mask ventilation EXCEPT:
a. BMI < 26
b. Edentulous
c. Beard
d. Snoring
e. Short neck
603. (2X) Gold standard for ETT confirmation:
a. Capnography [End-tidal carbon dioxide (ETCO2) is the gold standard for identifying Endotracheal tube]
b. Visualization of vocal chords
c. Chest X ray
d. Auscultation
e. Chest movement
604. (1X) For making sure the intubation is in the correct place:
a. Capnometry and Auscultation
605. (TO CHECK 2X) what is the best airway management device for respiratory failure?
a. BIPAP (Need to check)
[R] Epiglottitis:
606. (1X) Epiglottitis patient(drools)
a. You prepare for intubation in the ED
607. (1X) Epiglottitis patients has characteristic presentation of drooling
a. Intubation in ED
[R] Anesthesia ER (Mixed to Edit)
608. (3X) You always need to?
a. Check pulse oximetry during OR
609. (3X) Woman with appendectomy, what is her ASA score?
a. ASA 1 E
i. Note: E = For Emergency Operation.
610. (2X) A patient with uncomplicated diabetes is scheduled for elective surgery, what is her ASA class?
a. ASA 2
b. ASA 2 E
c. ASA 1
d. ASA 1 E
i. Check table for this
ii. Note: E = For Emergency Operation.
611. (2X) Patient with controlled DMII and HTN, has elective laparoscopic surgery for cholecystectomy. According to
ASA: (explanation in between brackets):
a. ASA 1 (Healthy person)
b. ASA 2 (Mild systemic illness)
c. ASA 3 (Severe systemic illness)
d. ASA 4 (Severe systemic illness cannot survive without the surgery)
e. ASA 5 (Brain dead patient doing surgery for organ transplantation)
612. (2X) According to ASA guidelines which one of the following is NOT used intraoperatively for monitoring:
a. Peripheral nerve stimulator (Toronto notes; Pg 6 - Monitoring)
b. Oxygen analyses
c. ECG continuous
d. BP
e. Capnometry
Back to TOC
Page 67 of 121
Emergency (ER):
[R] Primary Survey (ABCDE):
613. (4X) A question asking about ABCDE meaning, which of them was a WRONG abbreviation?
a. D for Danger (Note E is for Environment [Danger] or Exposure)
614. (1X Again) The ABCDE is used for the primary survey in ATLS, which one of the letters are WRONG?
a. D for Danger
615. (TO CHECK 1X) The ABCDE assessment in trauma in regard to a pregnant, pediatric, or elderly patient is:
a. Does not change
b. Depends on gender
c. Depends on age
d. Depends on complexity of disease
616. (3X) which you do in primary Survey?
a. You can only use board for 6 hr
617. (2X) which of the following is TRUE in primary survey?
a. (chin lift & jaw thrust?); Need scenario to decide
618. (1X) What is the first step in the management of 20 y/o brought
by a layman after an accident saying I can t feel my lower half?
a. Spinal immobilization

[R] Triage:
619. (1X) What is the TRUE about the triage systems?
a. The sickest patients are served first
b. It is a fool-proof system
620. (2X) Most appropriate statement regarding triage is:
a. Immediate action may be provided to the sickest
b. There is one universal system
c. It is a foul-proof system
d. First come, first serve basis is maintained (No)
e. CTAS/ESI/START are all the same (No)
621. (1X) What is true about START triage?
a. It has a Color coding system [Green (Minor), Black (Deceased), Red (Immediate), Yellow (Delayed)]
b. It is divided into five categories [Wrong 4]

Back to TOC
Page 68 of 121
[R] Trauma
622. (2X) Water skiing patient, near drowning experience, came unconscious, what to do next:
a. C-spine collar, CPR etc, then head tilt chin movement.
b. LLD position and Intranasal O2
c. Head-tilt chin lift, CPR (NO, assume cervical injury)
d. Jaw thrust, CPR, cervical precautions
Most probable answer as it have the option to do proper cervical protection measures and had CPR also.
623. (1X) Balancing fluids in trauma?
a. Initial fluid resuscitation for trauma patients in hemorrhagic shock consist of 2 L of isotonic saline
624. (1X) A 79 y/o male who is hypertensive and is an insulin dependent diabetic fell at home and presented to
the emergency department with right hip pain, the hip is flexed and externally rotated. After you stabilized
him, you sent him to get an X ray (or was it CT?? not sure) and noticed that he had a fracture of the
intertrochanteric line. What is the most appropriate management?
a. Immediately send him for open reduction internal fixation
b. Give him analgesia and send him home (No)
c. Give him steroids to lessen the inflammation (No)
d. Schedule a hip arthroplasty in 3 weeks
625. (1X) Blood transfusion inside the operating room?
a. Is better avoided in healthy ASA 1 patients if hemodynamically stable and hemoglobin is > 7 g/dL
b. Is indicated if hemoglobin level falls below 10g% in all patients categories
c. Has to be ready and cross matched for all patients coming for surgery (True?)
d. May require warning devices to be used
626. (TO CHECK 3X)[C] A 4 y/o boy suddenly develops a limp? No other symptoms what is your diagnosis?
a. Transient synovitis [other batch answer]
b. Trauma [More correct, if no trauma at all for sure could be then transient synovitis, but Trauma is MC

c. Growing pains
d. Avascular necrosis of femur
627. (3X) What is the most common cause of a sudden limp?
a. Trauma
628. (3X) patient presented with gunshot wound in his lung, the next step in management after primary survey is
a. Putting in a chest tube
629. (1X) A kid with a snake bite presented with diffuse redness and two puncture wounds and bruising across
body. How would you manage?
i. Calm + Reassure patient + Cleanse bite area + Splint (minimize movement) + Analgesia + Tetanus
ii. NO ICE or Constricting Bands + Nothing Oral
iii. Monitor Vital signs for ANY Changes (for example of envenomation give Antivenom)
iv. Can bring the snake to hospital for Identification and if venomous or not if killed (but not that
important as same anti-venom is used for all snakebites except few rare ones, so not a good idea to
bring it)
630. (3X)[C] Snake bite what is the most important thing?
a. Time is the most important thing (DIC risk greatest within 4hrs)
b. Bring the snake with you if possible to the ED (type of snake is important) [senior answer, this is wrong
i. Also patient can bring the snake to hospital for Identification and if venomous or not if killed (but not
that important as same anti-venom is used for all
snakebites except few rare ones, so not a good
idea to bring it)
631. (1X) Snake bite?
a. Venom poisoning
632. (1X) Snake bite:
a. Venom prolongs bleeding time
Back to TOC
Page 69 of 121
633. (1X) Which of the following is TRUE about bites?
a. The organism in cat bites is usually Pasteurella Multocida
b. Human bites are safe (most dangerous)
c. All dog bites need antibiotics (No)
d. There was something related to rabies
634. (1X) A man presents with suspicious cut on hand and claims he fell down. What is your management?
a. Suspected of fighting and cut is human bite
635. (3X) Trauma series X-rays include which of these:
a. Cervical Spine-AP + Chest-AP + Pelvis-AP
636. (1X) 20 y/o man presents to ER after a fall from a horse. He is confused on arrival to ER but localizes pain
on stimulation. You assess him to be a case of mild head injury with GCS of 13/15. Your next step is to:
a. C-ABC and CT brain [First C = Collar]
b. Consult neurosurgery
c. Intubation
d. ABCDE
e. Trauma code and admit for observation
637. (4X) A patient with certain neurological disabilities, what is his glasgow coma scale?
a. 13
638. (1X)[C] In MVA, when do you NOT do a cervical spine immobilization?
a. Paraspinal pain with no midline tenderness
b. Closed fracture of shaft of the femur [Femur Fracture is a Distracting fracture]
c. An alcoholic with a level of 200+ with severe injuries
d. Someone who has only internal bleeding and was thrown outside the car
e. Neck pain on turning the neck only
639. (1X) A near drowning case management?
a.
640. (1X) Patient in dashboard accident what's the most likely injury?
a. PCL and posterior hip dislocation
641. (2X) Patient in spinal shock after accident, which reflex is gone
a. Bulbocavernosus reflex

[R] Needle Stick injury


642. (3X) A nurse sustained a needle stick injury while assisting in a procedure at the clinic yesterday. The
patient s status is unknown for any blood-borne illnesses. The nurse is vaccinated against Hep B. Which of the
following is the most appropriate next step?
a. Investigate both the nurse and patient, for blood-borne illnesses
643. (1X) During surgery, the surgeon had a needle stick injury. Which of the following is TRUE?
a. Both the patient and the surgeon need to have their blood tested

[R] Wound
644. (3X) Adhesive material placed around wound edges needs how long to hold it before release?
a. 30 seconds;
i. At no time should the applicator tip be pressed into the wound;
ii. At least three layers should be applied to ensure optimal strength
645. (1X) Which of the following should NOT be sutured in the ER? (others: crossing the vermillion of lip, vital
structure like vessels, severe dental injury, deep/penetrating wound, compound Frx, bites)
a. Contaminated foot laceration with exposed tendons

Back to TOC
Page 70 of 121
[R] Toxicity
646. I
mport
ant
point
in
Cocai
ne
overd
ose to
keep
in
mind:

harac
teristi
c
sign:
Mydri
asis
(Dilat
ed
pupils
)

ntido
te: Benzodiazepine
iii. Never give: Beta-blockers
647. (2X) A Cocaine abuser came to ER with chest pain and tachycardia. What would you give this patient?
a. Benzodiazepine
b. Beta blocker
c. Calcium channel blocker
648. (Again 1X) Cocaine overdose, what do you give?
a. Benzo
b. B-blockers
c. Nitro
649. (TO CHECK 1X) Cocaine NOT SE: [incomplete??]
a. Mydriasis
650. (TO CHECK 1X) Cocaine NOT SE:
a. Miosis
651. (3X) First line in Beta-Blocker toxicity:
a. Glucagon
b. Calcium gluconate
652. (2X) How to differentiate between sympathomimetic toxicity vs. anticholinergic toxicity?
a. Sympathomimetic Diaphoresis vs. Anticholinergic Dry skin
653. (4X)[C] Anticholinergic Overdose (OD) antidote?
a. Benzo [One group answered it by 6th Batch] (the lec says it is 1st line for symptomatic care)
b. Physostigmine [Correct answer according to First Aid 2018] (if refractory to benzo/seizure/arrhythmia)
(the only antidote)
c. Glucose
d. Glucose and thiamine?
Back to TOC
Page 71 of 121
654. (1X) How would you differentiate between a sympathetic and anticholinergic toxicity?
a. Diaphoresis (in Sympathomimetic)
b. Mydriasis
c. Miosis
d. Tachycardia
655. (1X NEW) Which of the following poorly binds to charcoal? (I CCHAnAAAL)
a. Iron
b. Carbamazepine
c. Theophylline
656. (TO CHECK 2X) What is the drug of choice for convulsions?
a. Carbamazepine? [TO CHECK]
i. Notes that first line is lorazepam
1. Lorazepam (Benzo) first-line agents for an actively seizing patient
2. Phenytoin second line anticonvulsant in a continuously seizing patient
3. Phenobarbital third line anticonvulsant
657. (1X) Which of the following is the least likely to benefit from lavage:
a. Hydrocarbon
b. Aspirin
c. Acetaminophen
658. (1X) Patient asymptomatic had ingested acetaminophen tablets 4 hours ago, what s next step
management?
a. Check acetaminophen levels
b. Whole bowel irrigation
659. (1X) Acetaminophen tablets swallow, no symptoms?
a. Charcoal
b. Measure Acetaminophen levels

i. It Depends on the time (if > 4hour Activated Charcoal Won t be effective)
660. (1X) A patient ingested 30 pills of aspirin, what should you give?
a. NaHCO3
661. (TO CHECK 3X) A female presents to the ER with overdose of beta blocker and suicidal ideation, with
unstable heart block, which one of the following management is least likely to help her:
a. Glucagon (true)
b. Calcium gluconate
c. Hemodialysis MPA (maybe)
d. Cardiac pacing (patient is unstable which excludes using pacing at this (true)
e. Dopamine
f. All the above can be done in BB toxicity (maybe)
662. (1X) Which one of the following is it NOT a sign of unstable heart rhythm?
a. Hypertension
b. Chest pain
c. Dyspnea
d. Altered mental status
663. 1X) A patient is found unconscious outside the ER with needle marks in his arm and pinpoint pupils, vitals
were also given, antidote is?
a. Naloxone
Back to TOC
Page 72 of 121
b. Flumazenil
c. Atropine
d. Cocaine
664. (Again 3X) Morphine/opioid overdose treatment?
a. Naloxone
665. (And again 1X) What do you give for fentanyl overdose?
a. Naloxone
666. (8X) 40 y/o farmer presents with reduced GCS, salivation, lacrimation, urination, diarrhea, GI upset and
emesis. He has been using a new insecticide spray today. O/E: P:52/min, BP:110/60 mmHg, RR:36/min. Pupils
are small on examination. What is the diagnosis?
a. Organophosphate toxicity (cholinergic)
b. Opioid poisoning (mydriasis)
c. Salicylate poisoning
d. Cocaine toxicity (mydriasis)
e. Lead poisoning
667. (1X) Which of the following is the first line treatment for beta blocker toxicity?
a. Glucagon
b. Insulin
c. Potassium
668. (Again 1X) First line in treating of beta blocker toxicity?
a. Glucagon
669. (TO CHECK 1X) Beta blocker toxicity:
a. Give insulin and high dose glucagon?
670. (TO CHECK 1X) A patient presented with excessive sweating, dilated pupils and high blood pressure. What
toxidrome is it?
a. Sympathomimetic
b. Cholinergic
c. Anticholinergic
d. Opioid overdose
671. (TO CHECK 1X) Patient rescued from fire, had soot in her mouth, what was given to her, (this is a case of
cyanide poisoning)
a. Hydroxocobalamin was given to her as rescue (one seniors answer)

672. (2X) Paracetamol Overdose (OD) antidote?


a. N-Acetylcysteine
673. (Again 2X) What is the drug used to Antagonize paracetamol toxicity ?
a. N-Acetylcysteine
674. (2X) What is the antidote for iron toxicity?
a. Deferoxamine
b. N-acetylcysteine/parvolex
c. Atropine
d. Activated charcoal
Back to TOC
Page 73 of 121
e. Ethanol
675. (1X) Tricyclic Antidepressants (TCA) Overdose antidote:
a. Sodium bicarbonate
676. A 20 y/o male presents to ED with decreased mental status and agitation. He was found with empty bottle
of tricyclic antidepressants (TCA) taken 3-4 hours ago. After his initial resuscitation you have decided to
counteract the actions by:
a. Sodium bicarbonate
b. 10 gm activated charcoal
c. Atropine
d. Narcan and thiamine
e. N Acetylcysteine (NAC)
677. (4X) In cases of Carbon Monoxide (CO) poisoning which of the following is the first line mainstay
treatment?
a. High flow oxygen /HBO Therapy
678. (1X) A patient with Carbon Monoxide (CO) poisoning, What is the antidote?
a. Hyperbaric oxygen
679. (Again 1X) CO poisoning treatment:
a. High flow O2/HBO
680. (1X) Which of the following drugs has an antidote?
a. Diazepam
681. (TO CHECK 1X) Fire inhalation? (smoke think of Cyanide Nitrites/ hydroxycobolamin)
a. Mg, NaHco3 (look later)??

Back to TOC
Page 74 of 121
[R] Sepsis
682.

(1X)
An
onc
olog
y
pati
ent
dev
elop
s
pur
ple
blist
ers
on
his
lower limbs with crepitus on palpation also he developed fever & hypotension, what is the best step in the
management?
a. Call orthopedics & start treatment with broad-spectrum antibiotics as a case of sepsis
683. (1X) Immunocompromised pt admitted and given Ceftriaxone & blood cultures taken. What s the next step?
a. Wait for culture and consult ID for antibiotic regimen
b. Add antifungal Abs
c. Start on Vancomycin
684. (1X) Sepsis Care guidelines; Look below: (there is a new guidelines of 1 hour)
i. First 3 hours Measure lactate
ii. During 6 hours
685. (2X) As per Surviving Sepsis Campaign Care Bundles which one of the following is the first line choice for
inotropes?
a. Norepinephrine
686. (2X) The basic definition of shock is:
a. Impaired tissue oxygenation
b. Insufficient blood flow to organs
c. Hypotension and poor capillary perfusion
d. Depressed pumping function of the heart
687. (6X) 20 y/o old male, known leukemia with recent chemotherapy presented to ER with chest infection with
BP 70/40 and lactate of 5. You make a diagnosis of septic shock and start resuscitation. As per Surviving Sepsis
Campaign Care Bundles which one of the following is the recommended initial dose of fluids?
a. 30 mls/kg of Crystalloids
688. (Again 1X) A patient is diagnosed with septic shock and needs resuscitation. As per Surviving Sepsis
Campaign Care Bundles which one of the following is the recommended initial dose of fluids?
a. 20 ml/kg of Crystalloids
b. 30 ml/kg of Crystalloids
689. (TO CHECK 1X) Diagnostic requirement/criteria for sepsis at 6 hrs?
a. According to guidelines (written in Dr.pasha s lecture about sepsis)
690. (3X) Fever + Pancytopenia: [incomplete question]
a. Bone Marrow
691. (7X) According to the sepsis guidelines what is NOT a step you do in the first 3 hours
a. Remeasure lactate
Back to TOC
Page 75 of 121
b. Measure Lactate
c. Give crystalloid
d. Give antibiotic
e. Take cultures
692. (3X) A 20 y/o male, known leukemia with recent chemotherapy presented to ER with fever. He was
diagnosed to be in SEPTIC SHOCK secondary to chest infection. As per Surviving Sepsis Campaign Care Bundles
which one of the following is essential to deliver during first 3 hours of resuscitating a septic patient?
a. Measure lactate level
b. Central and arterial line placement
c. Perform chest x-ray
d. Administer inotropes
e. Administer 50 mL of 50% dextrose to prevent hypoglycemia
693. (4X) A 30 y/o female, known breast cancer patient with recent chemotherapy presented to ER with chest
infection and diagnosed to be in SEPTIC SHOCK. As per Surviving Sepsis Campaign Care Bundles which one of
the following is essential to deliver during first 6 hours of resuscitating a septic patient?
a.
b. Prevent hypocalcemia
c. Maintain temperature > 36°C
d. Keep Hb above 10 gm/dL
e. Start dopamine @ renal dose
694. (2X) A 20 y/o male, known leukemia with recent chemotherapy presented to ER with chest infection
diagnosed to be in SEPTIC SHOCK. Despite 2 liters of IV fluids he is still hypotensive BP 70/40. You think he has
fluid resistant shock and decide to use inotropes. As per Surviving Sepsis Campaign Care Bundles which one of
the following is the first line choice for inotropes?
a. Norepinephrine
695. (TO CHECK 1X) A known case of breast cancer patient comes with fever (do not remember the symptoms,
her vitals were not normal, but diagnosis was she is in sepsis). What would you for her?
a. Give IV fluids, take blood cultures and give antibiotics
b. Take blood sample and wait for results
c. Give her antibiotics and send her home
696. (1X)[C] An elderly patient presents with fever & a BP of 80/30, a chest X-ray reveals lobar opacification,
after 2 liters of crystalloids her BP is 80/50 what is the best next step?
a. Refer to ICU
b. Central line insertion & administration of vasopressors
c. CT to exclude PE
697. (1X) An elderly patient presents with fever & a BP of 80/30, a chest X-ray reveals lobar opacification, after 2
liters of crystalloids her BP is 80/50 what is the best next step?
a. Periphral IV dopamine
b. Central line and vasopressors.
c. [no ICU in the options this time]

Back to TOC
Page 76 of 121
[R] Environmental (Emergencies) [Heat Exhaustion/Stroke/Frostbite/Diving/
698. (3X) Signs of Severe hypothermia:
a. No shivering, coma/decreased consciousness signs
699. (Again 1X) Signs of Severe hypothermia:
a. No shivering, coma/decreased consciousness signs
700. (2X NEW) Guy who ran a marathon, while it was hot and humid outside, later started complaining of
abdominal and leg cramping and pain. He was vitally stable with no other symptoms. What is the most likely
diagnosis?
a. Heat Exhaustion (Most probable answer)
b. Heat Stroke
c. Rhabdomyolysis (some chose this answer because of exertional rhabdomyolysis)
701. (2X) Athlete running a marathon on a hot day, comes with 40 degrees temp, altered LOC, mumbling, and
his HR is high, SBP 135, he is sweating, what will you do next?
a. General cooling measure (because this is Heat Exhaustion)
b. Give IV hypertonic solution (No)
c. Give isotonic NS hydration (Yes?)
702. (3X) Worker in the sun, what to advise?
a. Cool down because Heat Stroke
703. (NEW) Deep sea diver who went for a dive and tried to show off or something. Later started having joint
pain
a. Decompression Sickness
b. Air Embolism
704. (1X) Scuba diver fainted after rising. What was the cause?
a. Air embolism
b. Decompression sickness C- The bends
705. (1X) A man post diving had pain in his hips & different joints. He had no issues while ascending. What is the
condition?
a. Decompression sickness
b. Air embolism
c. Carbon dioxide narcosis
706. (TO CHECK 1X) Mechanism of PE in diving? PE is caused by air embolism (which is seen in nitric narcotic)
a. Decompression Sickness: Occurs when gas bubbles obstruct blood vessels?
b. Air pressure in lungs remains high while external pressure decreases causing alveoli to rupture
707. (TO CHECK 1X) Mechanism of PE in diving?
a. Holding breath with pressure alveoli rupture? (barotruam)
b. Nitric oxide air bubbles in vessels with rapid ascending (this is a complication but causes decompression sickness not PE)

Back to TOC
Page 77 of 121
[R] Fluid Therapy / Blood transfusion
708. (1X) A 30 y/o male who is diagnosed case of Von Willebrand disease presents with acute hemarthrosis of
the right knee joint. No fever. What is the most important immediate management?
a. Immediate administration of factor VIII + Platelets
b. Immediate administration of steroids
c. Splint the knee and discharge home
d. Analgesia and admit
e. Aspirate the knee
709. (2X)[C] Hemophilia A pt presented with knee swelling/hemarthrosis, management:
a. Give factor viii immediately (only in severe cases joint aspiration is done, that too only after Coag factor
correction)
710. (3X) What is TRUE about the use of packed RBCs?
a. They are checked by 2 medical staff, reaction can be predicted in some donors.
711. (2X) Which is a feature of IV fluids:
a. Colloid stays intravascular unless there is capillary leak
712. (3X) Question about Colloid vs Crystalloid:
a. Colloid theoretically last longer in vessels due to high protein content
713. (4X) Normal Saline excess causes?
a. Hyperchloremic Acidosis
714. (1X) Using high volumes of normal saline 0.9% can lead to:
a. Respiratory acidosis
b. Metabolic alkalosis
c. Respiratory alkalosis
d. None of the above [old answer]
i. It causes Hyperchloremic Metabolic Acidosis
715. (2X) compensated respiratory acidosis values [Incomplete]?
716. (2X) Patient presented with symptoms of shock after losing blood following an accident, what is this condition?
a. Hypovolemic/hemorrhagic shock
717. (4X) Which of the following is TRUE regarding Balanced Resuscitation in a trauma patient?
a. Give blood products early during resuscitation to maintain a low target BP
b. Give copious amounts of IV Saline
c. Delay blood products as much as possible
718. (Again 1X) What is TRUE about balanced resuscitation?
a. Give blood products early to maintain blood pressure
b. Give copious amounts of crystalloids to maintain low BP 120/80
719. (1X NEW) Which of the following is best describes conservative fluid therapy?
a. Minimize blood products use early
720. (1X) Regarding blood transfusion in the emergency department, which of the following is TRUE?
a. Men and Post-menopausal females can be given O Rh-positive blood
b. Premenopausal females should be given O Rh-positive blood
721. (Again 1X) Blood transfusion group type for certain patients, which is TRUE?
a. O+ for Males & Post-menopausal females
722. (4X) In blood transfusion emergency, which of the following is WRONG?
a. Give males and post-menopausal women O Rh-negative blood
723. (1X) Which one regarding blood transfusion is CORRECT?
a. O- for Children & Women of childbearing age (to avoid immune reaction if we give O+ in females with O-
group of child bearing age).
i. Note: O+ can be given to Adult males & Post-menopausal females (Rh incompatibility)
724. (2X) What is the best way to measure body fluids intraoperatively?
a. Central Venous Pressure
725. (4X) FFP (Fresh Frozen Plasma) for?
Back to TOC
Page 78 of 121
a. Low platelets [old one senior group answer]
b. Low coagulation factors
726. (5X) Best way to monitor fluids intraoperatively:
a. Dynamic
b. Static.

Back to TOC
Page 79 of 121
[R] DKA
727. (TO CHECK 2X) 20 y/o male unconscious is brought to emergency department by medics. After ABC, the
next most appropriate intervention is:
a. Thiamine 100 mg IV, Narcan 1 mg IV, D50 1 amp IV (cocktail)
b. Normal saline bolus 30 ml/kg
c. Start CT brain
d. Intubation, activated charcoal
e. Drug tox screen, ETOH level and gastric lavage
728. (NEW) A weird deleted question: female with DKA symptoms asking about the diagnosis, but all of the
answers were about the management, which included:
a. Insulin IV push of 7-10 units (which could be true if the question was about the initial management)
b. Insulin drip of 0.1U/kg/hr (which could be true if the question was about controlling the patient)
729. (1X) Management of DKA:
a. Insulin IV drip (for controlling patient)
730. (2X) DKA you give insulin and IV-fluid, but you have low K, low Na, high glucose?
a. Hold insulin and give K
731. (1X) Most likely electrolyte abnormality:
a. Hypokalemia
732. (1X) Case of DKA, Patient had been given fluids and insulin, Glucose was still not in normal range, HCO3
was <6, also had hypokalemia. He was feeling tired and fatigued (metabolic acidosis). Next step of
management?
a. Give K and STOP insulin [answered by 6th batch]
b. Give HCO3
c. Give bolus fluids/normal saline
d. Give bolus of insulin
733. (5X) What guides management of DKA?
a. Anion gap
b. Serum glucose
c. Urinary ketones
d. pH
734. (1X) How to measure treatment response to DKA?
a. Monitor anion gap
735. (2X) A 76 y/o female with nausea, vomiting, and lower abdominal pain diagnosed with recent UTI presented
to emergency room with h/o being unwell for few days. O/E patient severely dehydrated, tachycardia and
tachypneic. Her HR: 136 BP: 110/70, RR: 40, SAO2: 98%, She has no drug allergies.
LABS:
(1) Na: 130 mmol/L (2) Cl: 95 mmol/L (3) K: 5.5 mmol/L (4) HCO3: 20 mmol/L (5) Glucose: 30 mmol/L
Urinalysis:
(1) Blood: -ve (2) Nitrogen: +ve (3) Leukocytes: +ve (4) Protein: -ve (5) Ketones: -ve
The next most appropriate management plan is to:
a. Ciprofloxacin IV and NaCl 0.9% 1 liter IV
b. Glucophage (Metformin) 500 mg BID
c. A & B
d. Insulin 10 units IV and NaCl 0.9% 2 liter IV with ciprofloxacin 400 mg IV
e. Insulin 10 units SQ and Glucophage 500 mg PO now
736. (1X) A patient presents with dehydration & abdominal pain, he had positive ketones & high glucose & a
high K, what is the best step in the management?
a. 10 unites 0.1 IV insulin drip
737. (1X)[C] Which of the following is causes an increased anion gap?
a. Salicylates
b. Ethanol
Back to TOC
Page 80 of 121
c. Paracetamol
738. (1X) A 6 y/o girl came to the ER with nausea and vomiting, abdominal pain, you smell a sweet odor from
her mouth what is the diagnosis?
a. Diabetic ketoacidosis
739. (1X) Which of the following does NOT cause high anion gap?
a. DKA
b. Ethanol
c. Renal Tubular acidosis
d. CO poisoning
740. (1X) Which of the following would you monitor while managing DKA?
a. Anion gap
b. Serum glucose
741. (2X) DKA:
a. Insulin Drip according to weight
742. (1X) A patient (only lab tests are shown), what is the diagnosis?
a. DKA (since there were positive ketones in urine ++++)
743. (1X)[C] A diabetic patient collapses in front of you what is the first step that you would do?
a. Shake his shoulder & ask if he is ok [according to BLS]
b. Shout for help
744. (3X) Person presented with abdominal pain, nausea/vomiting, lab showed ++++ urine ketone, diagnosis is:
a. DKA

Back to TOC
Page 81 of 121
[R] Genitourinary (Emergencies)
745. (2X) Case of Diabetic, with scrotal skin peeling:
a. Fournier s Gangrene
746. 3X) A patient with testicular torsion what to do in ER setting?
a. Examine and alert urology
747. (1X) A case of Testicular Torsion. Which is correct?
a. Early intervention is 90-100% successful
748. (1X) Testicular exam felt like a bag of worms. What is the diagnosis?
a. Varicocele
b. Hydrocele
749. (5X) A 70 y/o male had blunt trauma. On examination he has contusions on his
lower back. CT showed 1 kidney cyst on the right side measuring 2 cm. Both
kidneys are of normal size. RFTs all normal. Dx.?
a. Simple renal cyst
b. RCC
c. PCKD (Polycystic Kidney Disease)
d. Recurrent UTI
e. Prostatic nodular hyperplasia
i.
750. (2X) Painless hematuria, what will you do next?
a. Renal CT
b. Pelvic CT
c. Cystoscopy
751. (1X) A lady pass some drops of urine when she coughs, what is the diagnosis?
a. Stress incontinence
b. Overflow incontinence
[R] GIT (Emergencies)
752. (6X) An 80 y/o female from nursing home with history of chronic laxative use presents with sudden onset
of abdominal pain with distension. She has absolute constipation with no passage of flatus. Her acute
abdominal series X-ray reveals the following: What is the diagnosis?
a. Volvulus (Coffee Bean Appearance)
b. Large bowel obstruction
c. Bowel; perforation
d. Small bowel obstruction
e. Gallstones
753. (1X) An X-Ray with Air under diaphragm?
a. Perforated hollow viscus/viscera

Back to TOC
Page 82 of 121
[R] ENT (Otitis Media & Externa)
754. (TO CHECK 3X) DM, friable with granulation (ESR, CR, Glu, culture of the secretion)
a. Biopsy it (necrotizing otitis externa)
755. (TO CHECK 2X) A diabetic with middle ear discharge, pain especially at night, and facial nerve paralysis as
well as a friable tissue and asking you for the best step in management which included:
a. IV Antibiotics [correction by 6th batch] (1st, if not resolved you Bx)
b. Oral Antibiotics + Steroids [One senior option probably wrong]
c. Oral Antibiotic
d. Biopsy
756. (TO CHECK 1X) Herpes zoster/varicella of ear
a.
757. (2X) Next step in patient that has closed ear canal?
a. Clean it out
758. (4X) Malignant Otitis Media or Extrena is?
a. Common in diabetic
i. Malignant External otitis (MEO): Infection that affects the external auditory canal and temporal bone.
ii. The causative organism is usually Pseudomonas aeruginosa
iii. Disease commonly manifests in elderly patients with diabetes.
iv. The infection begins as an external otitis that progress into an osteomyelitis of the temporal bone.
v. It Spread outside the external auditory canal occurs through the fissures of Santorini and the
osseocartilaginous junction.
759. (3X) Otitis externa most common organism:
a. P. Aeruginosa
760. (2X) Mild otitis media management?
a. Analgesia and Amoxicillin (if >2 yrs then may be initial observation before ABx)
761. (TO CHECK 2X) Mild OM with temperature 37.2?
a. Antibiotics (either immediately or initial observation)
b. Analgesics (always)
762. (4X) Patient presents with acute otitis media, what is the drug of choice?
a. Amoxicillin
b. Augmentin
763. (TO CHECK 1X)[C] Bullous myringitis (with picture): small fluid-filled blisters on ear drum that causes pain
a. Incision and drainage (unnecessary)
b. Give Ceftriaxone
c. Paracetamol and discharge
i. Should be treated with Erythromycin or Clarithromycin to cover Mycoplasma species & S.penumonia.
ii. Incision & Drainage can be done depending on the picture and size.
treatment of BM comprises mainly analgesics, anti-inflammatory agents, and nasal decongestants. A combination of
systemic and topical antibiotics with topical steroid is shown to be effective
iii.
[R] Neurology (Emergencies)

Back to TOC
Page 83 of 121
1. (4X) A 33 y/o man suffered head injury after falling from a roof at work. His initial GCS
was 9/15 with unilateral dilated pupil. The CT brain revealed the following: What is your
Dx?
a. Epidural bleed
b. Subdural bleed
c. Intracerebral bleed
d. Cerebral contusion
e. Basal skull fracture
2. (1X) What is the etiology of the following finding on CT scan after sustaining a head
injury?
. Epidural bleed
a. Subdural bleed
b. Intracerebral bleed
c. Cerebral contusion

d. Basal skull fracture


3. (3X) Cincinnati (CPSS) is?
. FAST for stroke (it is a scale for stroke - Cincinnati Prehospital Stroke Scale)
4. (1X) Which of the following increases the probability of developing a stroke in 1-2 weeks after a TIA?
. Blood pressure >140 systolic & >90 diastolic at presentation (lacunar infarct)
a. Symptoms resolving in 1-2 minutes
b. Female gender
5. (2X) A 70 y/o male presents to ED with acute symptoms of stroke. He is a candidate for stroke thrombolysis (tPA) if?
. SBP is 180 mmHg and DBP 100
a. Head trauma 4 weeks ago [could be if minor head trauma not sever]
b. NIH stroke scale score > 25
c. He has an INR > 3.5
d. Blood glucose is <50 mg/dl (2.7 mmol/L)
6. (1X) A 70 y/o male presents to ED with acute symptoms of stroke. He is a candidate for stroke thrombolysis (tPA) if?
. Window of 6 hours
a. Head trauma 4 weeks ago [could be if minor head trauma not sever]
b. NIH stroke scale score >25
c. He has an INR > 3.5
d. Blood glucose is <50 mg/dl (2.7 mmol/L)
Though we were conflicted about the 4.5 hrs rule , all other options are clear contraindications from Dr.
Wagleys stroke lecture
7. (1X) Contraindication to tPA for stroke?
. INR of 3.5
8. (1X) Patient had a stroke what to do next?

Back to TOC
Page 84 of 121
. Stop Anti-hypertensive meds for the next 72 hors
9. ( 1X) Which one of the following is an indication for thrombolysis?
. Definite ischemic stroke with focal neurological symptoms
a. Systolic blood pressure of 200 mmHg
b. Symptoms for more than 8 hours
c. High serum glucose level
10. (TO CHECK 2X) When to worry and send the patient to the neurologist: (maybe hemorrhagic stroke)
. Weak plantar flexion. MRI
11. (1X) What is the drug of choice for convulsions?
. Carbamazepine
12. (1X) Man came in complaining of a sudden headache and a pain score of 10/10, neck stiffness, and photophobia.
What is your next step?
. Urgent CT to rule out Subarachnoid Hemorrhage
13. (1X) A patient develops headache + neck stiffness with NO fever
. Subarachnoid Hemorrhage
14. (3X) A 45 y/o male sustains a penetrating injury to the thoracic spine. There was loss of ipsilateral motor function and
contralateral pain and temperature sensation. What is the diagnosis?
. Anterior cord syndrome
a. Posterior cord syndrome
b. Brown sequard syndrome
c. Central cord syndrome, e) Cauda equina
15. (TO CHECK 1X) Old patient with back pain?
. IV steroids and STAT MRI
16. (TO CHECK 1X) Cauda equina (a. Old pt with back pain, b. full bladder + Hx of lung cancer)
. IV steroids and Urgent MRI.
17. (2X) Which cranial nerve closes eyes
. CN7, and CN3 Open eye
18. (TO CHECK 4X) A 25 y/o man presents with sudden onset of complete weakness of one side of the face. The clinical
examination reveals as follows. What is the most appropriate ER management?
. Reassure, start acyclovir/steroids/artificial tears
a. Immediate CT scan of brain [Seniors thought this was the answer, however COMPLETE weakness of one side of face =
LMN injury, hence possibly Bell s palsy which is treated with steroids]
b. Immediate MRI with referral to neurology
c. Stroke thrombolysis
d. Immediate referral to EN
19. (1X) Which side has the facial paralysis?
. His Left side
a. His Right side
20. (TO CHECK 1X) Low motor neuron bell s palsy:
. Prednisolone (a must)
a. Prednisone and acyclovir (if there are vesicles yea)
. If the patient can close his eyes, and use forehead frontalis muscle, it means its only upper motor neuron
lesion which is a stroke and you need to do CT
21. (2X) True about face exam?
. Bell s palsy cause ptosis by orbicularis oris paralysis
22. (1X) A middle-aged lady develops paralysis of half of her face acutely, she can t wrinkle her forehead, what is the Dx?
. Bell s palsy
a. Stroke
23. (2X) A man fell down, his CT is shown, what is the cause?
. Bleeding from the bridging veins
24. (2X) Acute vestibular neuritis Tx?
. Corticosteroids
25. (3X) Facial nerve palsy Tx?
. Steroids
26. (6X) Facial palsy + Vesicles treatment?
Back to TOC
Page 85 of 121
. Corticosteroids and antiviral/acyclovir (Ramsay-hunt Sx)
27. (2X) Facial nerve paralysis + Vesicles in ear, diagnosis + treatment?
. Ramsey hunter syndrome; Tx with antiviral + analgesia
28. (1X) Seizure, non-blanching red pinpoint rash? (meningitis)
. Brain CT
29. ( TO CHECK 3X) A patient started having symptoms of stroke 2 hours ago. What is the time frame to get a non-
enhanced CT scan for this patient?
. 10 minutes
a. 15 minutes
b. 25 minutes
c. 45 minutes
d. 60 minutes
30. (TO CHECK 1X) A patient presents with a stroke what is the optimal time frame to read his head CT?
. 25 minutes
a. 45 minutes
31. (TO CHECK 1X)[C] Stroke patient came 1 week later with ischemic anterior circulation. What to do:
. Stop antiplatelet
a. Carotid doppler
b. Thrombolysis
c. Stop aspirin?
d. Some other medication
32. (1X) A known epileptic patient was given IV lorazepam to stop his seizure in the ER and it successfully stopped the
seizure. 20 minutes later he started seizing again, what would be your management?
. IV phenytoin
a. IV phenobarbital
b. Rectal lorazepam
33. (1X) Tonic Clonic Seizure? (phenytoin; valproate)
. Benzodiazepine (wrong)
34. (TO CHECK 1X) A patient has a stroke affecting his left frontal lobe. What symptoms would you notice in him?
. Emotional and behavioural changes
a. Contralateral hemiplegia
b. Loss of vision in the left eye (if it is deviation of eyes toward the lesion then yes)
c. Ipsilateral paresthesia
35. (1X) More with delirium than dementia?
. Auditory hallucination
. Hallucination are more common with an acute cause, which delirium is.
36. (1X) Auditory hallucinations?
. Psychiatric (functional) Psychosis.
37. (1X) Visual hallucinations:
. Organic (medical, drugs, tumor) psychosis.

[R] Oncology (Emergencies)


1. (4X) A man complains of incomplete bowel emptying & blood in the stools, his wife noticed weight loss, what is the
diagnosis?
a. Rectal Cancer (rectal because of incomplete bowel emptying so rectum)
2. (TO CHECK 3X) Febrile neutropenia in a leukemic patient, who has a Hickman's catheter, comes with infection. They
took cultures, she is started IV Paracetamol/IV Cefepime. With Cefepime Tx?
. Add Vancomycin (the presence of cath is indication for vanco use)
a. Switch to Piperacillin + Tazobactam
b. Add some other abx
3. (TO CHECK 1X) Leukemia on chemo after 24 hours has fever and neutropenia?
. Consult ID
4. (TO CHECK 1X) Spinal cord compression due to metastatic tumor management?
Back to TOC
Page 86 of 121
. Identifying the level of sensory loss will help you in the management (to prevent irreversible damage? spine MRI)
a. Identifying the primary tumor is not the utmost priority for the management
b. Identifying the primary tumor will help you in the management
5. (1X) An oncology patient presents with cord compression due to metastasis, which of the following is the most
predictive neurological outcome in this patient?
. Neurological symptoms on presentation (motor and sensory and then autonomic)
a. Stage & Type of primary cancer
6. (1X) Which of the following is TRUE regarding SVC syndrome?
. Should be dealt with as an emergency treated with Thrombolytics & Stenting (ABCD)
a. The most important thing is to make a tissue diagnosis of the tumor (this is for definitive diagnosis only)

[VIP] Cardiology & ACLS & Electrolytes (Emergencies)


Pulmonary vs Cardiac
1. (2X) Patient with Cor pulmonale what is the cause?
a. Pulmonary fibrosis
b. Pneumonia
2. (3X) how can you differentiate between cardiac tamponade vs. tension pneumothorax?
. Breath sounds
AAA
3. (1X) AAA:
a. Do Bedside US / Oxygen / Large bore IV needles
4. (2X)[C] Abdominal Aortic Aneurysm (AAA) investigation?
. ABC [but of course you should first ensure the patient ABC are ok]
a. Bedside Abdomianl US
i. Investigation for AAA Include:
1. Blood work: CBC, electrolytes, urea, creatinine, PTT, INR, type and cross
2. Abdominal U/S (100% sensitive, up to ±0.6 cm accuracy in size determination)
3. CT with contrast (accurate visualization, size determination, EVAR planning)
4. aortogram (only for EVAR)
5. Doppler/duplex (rule out vascular tree aneurysms elsewhere, e.g. popliteal)
Hyper & Hypo Calcemia
5. (2X) A patient with mild hyper-calcemia:
a. Calculate corrected calcium with plasma protein
b. Give fluids, steroids, alendronic acid
c. Treat with hemodialysis
6. (1X) Which electrolyte abnormality CAN T cause cardiac arrest:
. Hypocalcemia
7. (4X) Which electrolyte abnormality CANT cause cardiac arrest:
. Hypocalcemia
8. (TO CHECK 1X)[C] A patient s labs showed hypercalcemia (calcium was 13). What is your next step?
. Give IV normal saline and bisphosphonates. [Until JVP is raised, also diuretic should be given after with IV]
a.
9. (1X)[C] Patient with mild hypercalcemia:
. Treat with hemodialysis
a. Calculate corrected calcium with plasma protein
b. Give fluids, steroids, alendronic acid
Hyper & Hypo Kalemia
10. (TO CHECK 3X) ESRD cause of disease : [incomplete?]
a. Hyperkalemia
11. (3X) Which of the following is the MC electrolyte abnormality associated with cardiac arrest in ESRD patients?
. Hypocalcemia
a. Hypernatremia
b. Hyperkalemia

Back to TOC
Page 87 of 121
c. Hyponatremia
12. (1X) Cause of cardiac arrest in renal failure patient
. Hyperkalemia
Hyper & Hypo Natremia
13. (1X) A patient with non-small cell lung cancer is dehydrated with hypernatremia & a K+ of 4.9, what is the initial
best step in the management?
a. Normal saline
b. Dextrose & insulin

PE / DVT
1. (1X) A 36 y/o male after recent flight from Australia to Saudi Arabia presented to ER with sudden onset of SOB. His vitals
were as follows: Afebrile, P: 123/min, RR: 40/min, BP: 90/60 mmHg, SaO2: 90%. Labs: Hb 14, WCC: 9.0. D-dimer: 600 (high).
Which of the following will confirm your diagnosis?
a. Chest X-ray AP/Lateral & EKG
b. Chest CTPA
c. Ultrasound venous Doppler lower limb
d. ABG and checking A-a gradient
e. Blood culture and sputum gram stain
2. (4X) Pulmonary embolism best diagnostic test?
. Chest CTPA
3. (TO CHECK 4X)[C] Pregnant woman with signs of PE/DVT
. Send for CTPA OR DOPPLER
i. Ideally would go with CT but pregnancy makes that a problem, so Doppler more likely here? Maybe V/Q scan?
4. (TO CHECK 2X) Most common ECG finding in PE?
. Sinus tachycardia
a. S1Q3T3
5. (TO CHECK 1X) PE ECG?
. S1Q3T3 (Probably wrong, or they rephrased the question)
6. (TO CHECK 1X) Which of the following is the most common ECG finding in PE?
. Sinus tachycardia
a. S1T3Q3
b. ST elevation
c. PR elevation
7. (2X) Obese woman presents with unilateral calf pain. She is also on OCPs. Her history is unremarkable. General
Physical Examination is normal. What do you do?
. Do a Doppler of her calf since DVT can t be ruled out (and OCP & Obesity are risk factors)
MI & Angina & Heart Failure
8. (1X NEW) Earliest sign of myocardial ischemia on ECG
a. Peaked T wave
b. O wave
c. ST elevation
d. ST depression
e. Synchronized Cardioversion 200J
9. (2X) Which of the following is the first sign on a ECG in MI (myocardial infraction)?
. Peaked T wave
a. ST elevation
b. ST depression
c. O wave
10. (5X) Which of the following would present with atypical MI?
. Diabetics
a. Young
11. (TO CHECK 3X) Risk of MI and troponin?
. 85 % PPV
12. (1X) A 50 y/o with 2 hours history of Chest pain has been given Aspirin his blood pressure was 155/85mmHg what is
the most appropriate drug that should be given next?
Back to TOC
Page 88 of 121
. Morphine
a. Carvedilol
b. An ACEI
c. Furosemide
13. (4X) What drug is proven to reduce second MI risk (Reduce mortality)/ Recommended for all MI s?
. Beta blocker
14. (Again 1X) Which medication improves M&M (morbidity/mortality) Post MI?
. Beta Blocker
15. (4X) A patient with MI was given aspirin. What medication should you add for him?
. Beta blockers
16. (1X)[C] Drug that every HF patient should be on?
a. ACE Inhibitor [HF]
b. Beta Blocker [MI]
3. (5X) Which one of the following is the Beck s triad?
. Muffled heart sounds + Increased JVP + Hypotension
a. Hypercoagulability + Stasis + Vessel injury
b. Low SBP + Absent breath sounds + Raised JVP
c. Fever + Jaundice + RUQ pain
d. Recurrent oral ulcers + Genital ulcers + Iridocyclitis
4. (1X) A patient present with shortness of breath & lower limb edema which of the following would you give?
. Furosemide
a. Spironolactone
5. (TO CHECK 2X) 1 drug in MI?
st

. Aspirin
6. (TO CHECK 1X) MI? [incomplete]
. 2 mm 2 leads
7. (TO CHECK 2X) MI:
. 90 mins for FMC to angio for accredited institutions
8. (TO CHECK 2X) In cases of acute MI, the target door to balloon time in a chest pain accredited centre is?
. 30 mins
a. 60 mins
b. 90 mins
c. 120 mins
d. 3.5 hours
9. (1X) A patient presents with STEMI, what is the door to needle time?
. 30 minutes
10. (1X) man presents with a heavy chest pain he had this pain 7 times in the past period with excursion, what is the
diagnosis?
. Unstable angina
11. (4X) Which of the following would you give for a patient with low output HF?
. ACE inhibitors
a. Beta blockers
b. CCB
c. Aspirin
Arrhythmias & Tachy & Bradycardia
11. (1X) ECG of patient tachycardia, normotensive what do you do?
a. Supportive measures
12. (1X NEW) Guy came into ER with hypotension and bradycardia. ECG showed Mobitz type II AV-block. How would
you manage acutely? (he is unstable? Atropine?)
. Transcutaneous Pacing
a. IV epinephrine
b. Defibrillation 200J
13. (1X) A patient presents to you with bradycardia, ECG was provided, and it showed Mobitz type 2. How would you
manage?
. Insert pacemaker
Back to TOC
Page 89 of 121
14. (TO CHECK 2X) When can nitrates NOT be given?
. CHF, Inferior, Last 24hr Viagra
15. (TO CHECK 1X) Patient had Vfib was shocked, on third attempt showed rhythm: bradycardia. What do you do next?
. Check if he has pulse
a. Give atropine
16. (TO CHECK 1X) V.fib had CPR+ Defib ECG changed to brady?
. Atropine [??]
17. (TO CHECK 1X)[C] Sinus brady but no pulse?
. Atropine? [No pulse is usually PEA or Vfib so CPR + shock (200,300J) + Epinephrine + Amiodarone]
18. (TO CHECK 1X) What medication can help in bradycardia?
a. Atropine
2. (TO CHECK 1X) An unstable, bradycardia patient s ECG is given below . What would you do? [They did not provide
an ECG for this question during the exam, so mostly was cancelled for us]
a. Give epinephrine
b. Cardioversion
c. Transcutaneous pacing
3. (1X) A patient with inferior STEMI developed bradycardia & hypotension what is the management?
a. Atropine & Prepare for transcutaneous pacing
4. (1X) A patient has an inferior STEMI & is hypotensive, which medication you Shouldn t use?
. Nitroglycerin
5. (TO CHECK 1X) In which case do you do transcutaneous pacing?
.
6. (2X) Most common complication of atrial fibrillation?
. Stroke
7. (3X) Patient with Cardiac abnormalities had stroke, which of the following is the likely cause?
. Atrial Fibrillation
8. (3X) Which patient to do synchronized cardiovert :
. A fib
9. (1X) For which of the following would performing synchronized cardioversion be appropriate?
. Unstable atrial fibrillation
10. (1X) What is the mechanism behind sudden cardiac death?
a. The heart goes into arrhythmia then stops beating
11. (2X) Pulseless electrical activity (PEA) management:
. IV epinephrine [P E A E for Epinephrine also many e
a. IV atropine
b. IV amiodarone
c. Defibrillation (It s not non-shockable rhythm)
12. (2X) PEA is treated with?
. IV epinephrine
a. IV atropine
b. IV amiodarone
c. Defibrillation
i. Note: PEA is a non-shockable rhythm.
13. (2X) You are the team leader in a cardiopulmonary resuscitation team attending an arrest in ER. The patient is in
the following rhythm on cardiac monitor. You decided this is a shockable rhythm and you start chest compression with
ventilation. Which of the following is most appropriate next step?

a. Defibrillation @ 100 Joules


b. Defibrillation @ 200 Joules
c. Synchronized cardioversion @ 100 Joules
d. Synchronized cardioversion @ 300 Joules
e. Periodic chest thump
15. (3X) 62 y/o male attended ER in Cardio-Respiratory
Arrest. Following rhythm was seen on Cardiac Monitor. Which of
the following Drug should be given during ACLS Resuscitation?
Back to TOC
Page 90 of 121
a. Adrenaline (Epinephrine)
3. (Again 1X) Picture of flat ECG:
. Epinephrine
4. (3X) Question with an ECG strip, identify the pattern:
. Supraventricular tachycardia (SVT)
4. (TO CHECK 1X) Prompt defibrillation is indicated in patients with?
a. Hemodynamically unstable torsades de pointes
5. (3X) 20 y/o male with recent history of alcohol use presented to emergency room with sudden loss of
consciousness. On the monitor you note the following rhythm. What medication may resolve this rhythm?

a. Magnesium Sulphate ( if stable; if unstable defib)


b. Atropine
c. Adenosine
d. Calcium chloride
e. Epinephrine
7. (6X) A picture of an ECG with torsade, what is the management?
. Magnesium Sulphate
8. (3X) Shocking a normal rhythm will lead to?
. Ventricular fibrillation (Vfib)
9. (1X) Which of the following is NOT a sign of unstable rhythm?
. Tachycardia
a. Hypotension
b. Chest pain
c. Shortness of breath
d. Altered mental status
10. (7X) In Shockable Adult Cardio Respiratory Resuscitation which of the following is the correct ratio between chest
compression and ventilation plus type of defibrillation?
. 30:2/Asynchronous Defibrillation
i. Prompt defibrillation is indicated in patients with hemodynamically unstable torsades de pointes.
11. (1X) A patient develops ventricular fibrillation after an MI, what is the best step in the management?
. Defibrillation
12. (2X) Patient presents without pulse and with an ECG showing Ventricular Fibrillation, how would you manage?
. 30:2 and Unsynchronised shocking
13. (2X) the drug combination mostly being used in shockable rhythms is:
. Epinephrine + Amiodarone
14. (3X) Ventricular fibrillation after defibrillation what are the medications to use:
. Epinephrine + Amiodarone
15. (6X) An 80 y/o man with chest pain collapsed in CCU. The cardiac monitor showed the following rhythm. In
shockable cardiorespiratory arrest resuscitation, which of the following drug combinations are used:

Back to TOC
Page 91 of 121
a. Adrenaline (epinephrine) & Amiodarone
b. Adrenaline (epinephrine) & Bicarb
c. Adrenaline (epinephrine) & Atropine
d. Atropine & Amiodarone
e. Vasopressin & Bicarb
3. (1X) Picture of ECG:
. Amiodarone and Adrenaline
4. (1X) Which combination of medications is used in ventricular tachycardia or fibrillation?
. Adrenaline and Amiodarone
5. (1X)[C] An 18 y/o healthy young man presented to ER with c/o palpitation and chest pain. His chest is clinically
positive for bilateral crepitations and observations are as follows: Pulse 170/min, BP: 70/50 mmHg RR: 24/min. ECG showed
following. What is the next management step?

a. Vagal maneuver
b. Adenosine 6 mg [One Senior Group, also can be used for regular narrow complex, but CI in Asthma Bilateral
Crepitations can indicate asthma or interstitial lung disease in this young patient so Adenosine is CI]
c. Adenosine 12 mg
d. Synchronized cardioversion [More likely]
6. (1X) SupraVentricular Tachycardia (ECG identify) & stable:
. IV Adenosine
7. (1X) Ventricular fibrillation + No pulse (Unstable):
. 30:2 and defibrillation
8. (2X) patient presents with signs of instability (HR:190, BP:70/40,etc) with an ECG picture showing ventricular
tachycardia, how would you manage this patient ?
. Synchronised cardioversion
9. (1X) Ventricular tachycardia (ECG) + Unstable:
. Cardioversion [?]
10. (5X) SVT & unstable:
. Cardioversion
11. (2X) Stable SVT ECG, Tx?
. IV adenosine
12. (1X) Which of the following rhythm would you give a synchronized cardioversion?
. Unstable atrial fibrillation
a. Ventricular fibrillation
b. Pulseless ventricular tachycardia
c. Sinus tachycardia

Back to TOC
Page 92 of 121
[R] Hypertension (Emergencies)
2. (2X) Case (scenario) of malignant hypertension, what is the treatment?
a. Sodium Nitroprusside
3. (1X) A patient with DM & HTN presents with headache & a BP of 220/120, retinal exam reveals flame hemorrhage,
what is the correct statement regarding the diagnosis?
. Cannot make the diagnosis without a head CT (correct)
a. Hypertensive encephalopathy
b. Intracranial hemorrhage
c. Central retinal artery occlusion
i. Funny Note: (correct)
4. (1X) An old man presented to the walk in clinic with HTN 210/110 and symptoms of papilledema. What is your
next step of management?
. Refer to ER
a. Give oxygen and treat with IV
5. (2X) To what do you correct blood pressure :

Back to TOC
Page 93 of 121
. < 140/90
. Goal depends on patient age; check table
6. (TO CHECK 2X) Multi organ dysfunction
.
7. (TO CHECK 2X) An Old fat lady with hypertension +

[R] Ophthalmology (Emergencies)


Glaucoma
1. (1X) 63 y/o man comes in with severe painful red eye + mid dilated pupil. Has nausea &
vomiting. What is the Dx? Seeing halos & lights DON T give epinephrine
a. Acute angle closure glaucoma
2. (3X) Acute closed angle glaucoma sign & symptoms?
. Sudden Pain + Loss of vision
3. (1X) On his way back from watching a movie, 33 y/o male presented with sudden onset of
painful red eye (as shown in picture) with blurred vision associated with headache, nausea
and vomiting. The most appropriate next step is?
. Pilocarpine intraophthalmic drops STAT (to lower IOP)
a. Gentamicin ophthalmic drops every 4 hours
b. To keep in a dark room
c. Ketorolac 30 mg IV stat
d. Oral Acetazolamide/Zofran stat
4. (2X) Angle closure glaucoma; Loss of vision + pain what is initial Tx in ER setting before surgery?
. Pilocarpine
5. (1X) picture of patient who lost his vision while being in the cinema, he wears glasses (Glaucoma) Tx?
. Pilocarpine
6. (1X) A case of acute glaucoma. What would you do?
. Give topical beta blocker urgently and referral to ophthalmology
a. Follow up appointment with ophthalmology
7. (1X) What causes primary open angle glaucoma? (1ry cause is unclear; 2ry meshwork obst.). There is cupping
. Pupil obstructing flow
a. Increased production
b. Decreased Absorption
c. A + C (i think the answer but in some question versions was different question, mine was correct)
Subconjunctival Hemorrhage
8. (2X) 46 y/o male on warfarin presented to ER with sudden onset of painless left eye bleeding. There is no problem
with vision. You examine the eye and note the following: (Subconjunctival hemorrhage)
a. Urgent referral to ophthalmology
b. Apply eye patch
c. Check PT and INR
d. Sleeping with head end elevation is recommended
e. Immediately needs FFP
9. (1X) A patient present with redness in his eye after repeated bouts of smokers cough, what is the diagnosis?
. Subconjunctival hemorrhage
a. Conjunctivitis
b. Pterygium
Orbital Cellulitis & fractures
10. (2X) Child presented with fever, eye pain, diplopia. Skin red around the eye and eye appears to be bulging out.
What is your diagnosis
a. Orbital Cellulitis
b. Periorbital cellulitis
c. Open angle glaucoma
11. (1X) Presentation of eye trauma and bruising and limited eye movement:
. CT for orbital floor fracture
Eye Viral Infections
Back to TOC
Page 94 of 121
12. (TO CHECK 1X)[C] An 8 w/o baby, mom complains of watery right eye, last week he had a viral infection and now his
symptoms are worse. What is the next step in management?
a. Educate mother on nasolacrimal massage
b. Tell mom to clean the upper eyelid and apply warm compress
c. Urgent referral to ophthalmology
d. Routine referral to ophthalmology
13. (1X) Girl presents with eye pain and vesicular lesions around her eye? What does she have?
. Herpes zoster ophthalmicus
Miscellaneous
14. (2X) Light receptor cells found in?
a. Cone & rods
15. (1X) Trauma patient (eye), what stain should we use?
. Fluorescein stain (as it gets stuck on the Abnormal epithelium/damaged one)
16. (Again 2X) Eye trauma diagnostic assessment:
. Fluorescein stain
17. (5X) A fundoscopic picture was shown what is the Dx?
. Retinal detachment (crinkling & change in vessels direction)
18. (2X) Clinical scenario (sudden painless loss of vision) with the following fundus picture (below) asking for the Dx?
. Central retinal artery occlusion (CRAO) (acute, monocular vision loss & cherry spot at fovea)
a. Central retinal venous occlusion (CRVO)
b. Vitreous hemorrhage
c. Glaucoma
d. Retinal detachment
19. (2X) Instrument NOT used to measure the IOP?
. No options Note: Tonometry is the instrument used to measure IOP
20. (2X) Slit lamp shows pus accumulating in the ant chamber
. Hypopyon
21. (2X) Patient presented with blood in the anterior chamber/cornea, management:
. Raise head of bed & Analgesia
22. (3X) Hyphemia presentation (Picture from lecture)
. Raise head + Analgesia
23. (1X) person got bleach in his eye calls office what do you tell him?
. Irrigate 15 mins then come to office
24. (2X) Patient presents with ptosis. If you suspect sympathetic damage as a cause
(Horner s Syndrome), what another sign this patient should have?
. Miotic pupil (also anhydrosis)

[R] Anaphylaxis
1. (1X) 14 year old presented with swollen lips and other allergy symptoms after eating peanuts, Anaphylaxis:
a. IM epinephrine I:1000 0.5mg IM big muscle (Quadriceps/vastus lateralis muscle)
2. (1X) A 16 y/o lady got shortness of breath after taking penicillin. Answer?
. Intramuscular epinephrine
3. (3X) A 15 y/o lady was outside collecting flowers when she was stung by a bee. She reports being stung once before
without any complications, but now she is having generalized urticarial rash, swelling and dyspnea. Which one of the
following is the next most appropriate stat treatment?
. Hydrocortisone 100mg IV
a. Diphenhydramine 25 mg IV
b. Ranitidine 50mg IV
c. Epinephrine 1:1000 0.5mg IM
d. Epinephrine 1:10,000 1mg IV
4. (1X) Anaphylaxis question, 1 line of treatment?
st

. IM epinephrine
5. (1X) You prescribed amoxicillin to a patient and she had an allergic reaction, which was managed. What is your next
step?
Back to TOC
Page 95 of 121
. Document it in her patient file
a. Report it to the ministry of health

[R] Palliative Care


1. (3X) In a patient where allow natural death is used, what of the following can be given
a. Feeding tube
2. (1X) What interventions are allowed in DNR patients:
. Feeding tube & Pain control (Analgesia)
3. (2X) During end of life care in a palliative DNAR patient, an emergency Physician may?
. Keep foley cath (urinary) as there Is no long term risk of infection
a. Initiate inotropes and pressors (No)
b. Initiate tube feeding and pain control
c. Refer to surgery and transplant (No)
d. Intubate and mechanical ventilation (No)

[R] Procedures
1. (2X) What is wrong about arterial line?
a. It is used to deliver emergency medications (central line is used intra-op)
b. It s used to measure BP
c. infection can happen
d. Seldinger technique is used to insert it
2. (2X) Central IV line:
. Decreasing viscosity + length
3. (1X) What is correct about paracentesis:
. SAAG > 250 WBC/mm3 with >50 % PMNs reflects SBP
a. SAAG <1.1 = portal HTN
b. 3 liters could be removed
4. (1X) A man with hepatitis B develops ascites, jaundice, fever & chills, he looks ill, what is your management?
. Paracentesis, cultures & 3rd generation Cephalosporins

Others:
1. (2X) After performing a procedure in the emergency room department- the next most important step is to?
a. Checking if the procedure is done right
b. Documentation and taking pictures
c. Inform the family
d. Teaching the staff
e. Taking a quick break before attending next patient
2. (TO CHECK 1X) Methotrexate : [incomplete]
. sign or ?
a. ruptured tube ?
3. (1X) Lung cancer Hx, severe back pain?
. Analgesia + CT + Consult (MRI?)
4. (TO CHECK 1X)[C] A 38 y/o male presents with loss of sexual interest, physical examination reveals no abnormities,
which of the following is the initial test of choice?
. Serum testosterone
a. Serum prolactin
5. (1X) A lady suprapubic pain, dysuria & positive nitrites, which antibiotic to choose?
. Trimethoprim-sulfamethoxazole [UTI]
a. Ciprofloxacin
6. (TO CHECK 1X) IV LINE?
. Decreasing viscosity + Length

Back to TOC
Page 96 of 121
7. (1X) A 55 y/o male, non-smoker presented to you at the clinic for his regular follow up. Which one of the following
would you do for him?
. Fecal occult blood test (FOBT)
a. Abdominal US
b. Low dose CT-scan
8. (2X) Which of the following is an appropriate suture size?
. Face 6-0

(2X) Subdural hematoma: CT: hyperdense, crescent-shaped lesions

(4X) Epidural hematoma: CT: hematomas appear lenticular or biconvex

(1X) Subarachnoid

(5X) X-ray Volvulus

(5X) X-ray Small bowel obstruction: multiple air fluid levels

(1X) Retinal detachment

Back to TOC
Page 97 of 121
[R] Respiratory (Emergencies)
1. (2X) Lobar pneumonia case, management?
a. IV fluids 30 ml/kg
2. (2X) What antibiotic do you administer for community acquired MRSA?
. Doxycycline
a. Metronidazole
b. Cephalexin
3. (2X) Hemothorax X-ray shown what is the management?
. Chest tube pointed downwards
4. (1X)[C] Hemothorax X-ray + Hx of GSW (Gun Shot Wound) in chest what is management?
. Chest tube pointed downwards
5. (4X) A patient with Hemothorax?
. Do chest drain/tube.
6. (2X) Patient with signs and symptoms tension pneumothorax + X-ray is shown. What is the Immediate initial Tx?
. Needle thoracocentesis (since the X-ray shows tension pneumothorax).

Back to TOC
Page 98 of 121
[R] Neurology (Emergencies)

Back to TOC
Page 99 of 121
8th Batch Group D AMB 5X2 MCQ Final

green Highlight by Group B 8th Batch

1- Snake bite:

a. Immobilization and splinting

b. Rare to cause coagulopathy

2- MgSo4 is not useful in

A. Acute diarrhea
B. Asthma
C. Alcoholism
D. Preeclampsia

3- Lobar pneumonia with sepsis and hypotension initial management

a. Maintain MAP > or = to 65 mmHg

4- Ultrasound in early pregnancy

a. Double ring

5- MAC is affected by the following except:

a. Hypokalemia

b. Hypothermia

c. Hyperthermia

d. Pregnancy

6- [C] 2 y/o, temp. 39 C, vomiting , purpuric rash on thigh

a. Give antibiotics

b. Lumbar puncture

7- Rhinosinusitis treatment in adult and children

a. Augmentin (amoxicillin/clavulonic acid)


8- Number needed to treat calculation. UTI prophylaxis new treatment vs. placebo. UTI
prophylaxis (n=222 and UTI affected 14). Placebo (n=226 and UTI affected 17)

a. 63 (Wrong)
b. it should be 82.30
check the calculation

9- Hypotensive and hypernatremia. What to give

a. 0.9% NaCl
we need to treat the
hypotension 1st

10- Ovid better than pubmed. Why?

a. Access full text

11- True about IPSG

a. Decrease hospital associated infections

12- Bitemporal hemianopia

a. Pituitary tumor

13- Medication that you can give in pregnancy

a. Heparin

b. Coumadin

c. Ibuprofen in 3 trimester
rd

d. Tetracycline

14- Soft tissue abscess management

a. I&D and local wound care

b. Systemic antibiotic

c. Topical antibiotic

15- Suggestive of c-spine fracture

a. Prevertebral space 22 mm

b. Prevertebral space 6 mm

c. Predental space > 2mm

they must give you which space or level it's 6 mm at C2 and 2 cm at C6 but
most likely here A is the answer
16- Focal seizure management

a. Lorazepam

b. Phenytoin

c. Phenobarbital

17- Tonic clonic seizure scenario in which doctor enters room and nurse is trying to place an
IV line. What is your management?

a. Assist 2nd nurse in placing patient on lateral decubitus, give


oxygen and prepare lorazepam

b. Go out of the room and call for more help

18- DKA scenario, patient weighs 50 kg. How to manage?

a. Insulin drip 5 units/hr

b. Insulin drip 5 units/kg/hr

c. Humalin 70/30 insulin 5 units bolus

- to calculate it (0.1 unit/kg/hr * Weight kg) = unit/hr

19- [Check] PE symptoms and she had a doppler ultrasound that was positive. What to do
next?

a. No more tests need and treat

b. CT non-contrast to confirm PE

20- PE question

a. CT angio

21- PE question

a. CT angio

22- Bacterial vaginosis treatment

a. Metronidazole

23- Tall, thin man with symptoms of pneumothorax. What next?

a. Chest x-ray

24- [Check] What is true about ectopic pregnancy

a. Vaginal bleeding is most common symptom (it's only in 30%)

b. Ectopic pregnancy can happen in almost all cases


25- Primigravida 20 weeks pregnant with UTI E. Coli, asymptomatic. What to do?

a. Start antibiotics

b. Reassure

c. Repeat urinalysis in 3rd trimester

d. Treat after delivery

26- 58 y/o male with hypertension and no comorbidities. Management?

a. ACEi

b. Loop diuretic

c. CCB

27- Obese with BMI 33, no-comorbidities

a. Simple behavioral modification therapy

b. Strenuous exercise

c. Anorexic medications

it's either A or B, ask the Family physicians

28- Back pain red flag need urgent MRI and neurological referral

a. Buttock numbness S3, S4, S5 this is Saddle anesthesia

b. Difficulty standing on tip toes because of group muscle weakness

c. Radiculopathy

d. Paresthesia of anterior thigh

e. Parasternal pain

29- Decompression sickness

a. 100% oxygen

30- [Review] Mountain hiker scenario that develops ataxia and dyspnea (?) after elevation of
15,000 ft

a. Gammo bag

b. Lasix

c. Acetazolomide

d. Viagra

e. Dexamethasone
31- ECG showing ventricular tachycardia (wide complex tachycardia) and BP of 86/60 with
HR 116

a. Synchronized cardioversion with 100 J

32- Midazolam antidote

a. Flumazenil

33- Facial abnormality which is true

a. Malar flush is in aortic stenosis

b.

c. paralysis of orbicularis oculi

d.

34- Aspirin antidote

a. Sodium bicarbonate it's used to Alkalanize the urine

35- Patient with ascites with SAAG values gives (kno w SAAG values & causes of each)
check the picture

a. Cirrhosis

b. Congestive heart failure

c. Malignant ascites

d. Nephrotic syndrome

36- Psychosis organic cause

a. All the above

b. >40 with no previous psychiatric history

c. Abnormal vitals

d. Confusion

e. Recent loss of memory

37- DMPA (type of contraception injection)

a. Every 3 months

38- Sympathomimetic toxidrome

39- Fentanyl antidote

a. Naloxone
40- GABA agonist (2 answers were right)

a. Propofol

b. Etomidate

41- [Review] Side effect of propofol (repeated)

a. Decrease in CO , BP , CMR

42- [Review]

a. Disease oriented evidence (DOE)

b. Patient oriented evidence

c. Evidence based medicine

43- Post-menopausal abdomen with PV bleed

a. Pelvic US

44- Severe hypokalemia clinical manifestation

a. Fatigue and weakness

46- Hypothermia

a. Osborne waves

47- Tertiary prevention example

a. Urinalysis in diabetic patient

48- Diabetic medication that does not cause weight gain

a. GLP1 agonist (liraglutide)

49- What is not correct about RSI

a. Bag mask ventilation

50- Difficult intubation and desatting

a. LMA

51- Difficult face mask. Next step

a. Oral airway

52- Contraindication to LMA

a. Recent meal
53- Recent meal, what is contraindicated?

a. LMA

54- To confirm intubation

a. Change color of EtCo2

55- diarrhea prophylaxis

a. Invest cooked food and boiled water only

56- IBS

a. Mucus in stool

57- IBS first-line

a. Mebeverin

58- Difficult airway preparation

a. Plan ahead

59- Atrial fibrillation complication

a. Stroke

60- Testicular torsion

a. Bilateral orchidopexy

61- Asthma question

a. LABA

62- Asthma question

a. Low dose inhaled + LABA

63- Hyperglycemia and hypernatremia (repeated) correction calculation for sodium

a. 132
b. 146
c. 150

64- GAD treatment

a. Fluoxetine

65- Asthma exacerbation

a. Nebulized beta agonist with anticholinergic


66- Microcytic hypochromic anemia with sister with chronic anemia. Normal ferritin and iron.
Next step

a. Electrophoresis

67- Combined OCP

a. Increased risk of VTE

68- Macrocytic anemia

a. Vitamin B12

69- Vitamin D in breastfeeding

a. 400

70- HPV

a. Common warts

71- 54 y.o patient came for FM visit

a. Colon cancer screen

72- Patient coming with STEMI symptoms that started one hour ago, what would you give

a. Aspirin

b. Heparin

73- Most common cause of fatigue

a. Depression

b. Hypothyroidism

c. Anemia

74- [Review] GCS question ( eye opens with pain , withdrawal from pain , and speak with
inappropriate words )

a. 9

75- Door to balloon time

a. Less than 1 hour, 30 minutes

76- [Review] Cluster headache (repeated)

77- [Review] Posterior and anterior fat pad

a. Supracondylar fracture

78- [Review] EBV scenario


79- [Review] Non-maleficence

80- [Review] Confidentiality

81-

a. Pain on proximal sheath

82- Framingham risk is 22, what to give?

a. Statin

83- V. fib

a. CPR and defibrillation ASAP

84- A question about someone that fell (no CT image), presenting with unilateral mydriasis

85- Proper sign-out

a. SBAR

86- 35, healthy, what vaccine to discuss (depends on scenario but we think tetanus is the
better choice here)

a. Tetanus

b. HPV

87- New employee PPD positive. Next step? Depends on the scenario but A most likely
here

a. Exclude active TB and give isoniazid prophylaxis

b. Step two PPD

c. Chest x-ray if fine ressaure nothing else needed

88- Breast cancer risk

a. Family history in sister

b. Breastfeeding

c. Late menarche

d. Early first pregnancy

e. Multiparity

89- Centor criteria. Which would benefit most from antibiotic treatment (gave different
scenarios)

a. 10 y/o with fever,Tonsillitis, lymphadenopathy and absent cough

90- ECG finding in PE


a. Tachycardia with Non-specific ST changes

91- 15 y.o. girl stung by a bee and having anaphylaxis symptoms

a. Epinephrine 1:1000 0.3 mg IM

b. Epinephrine 1:10,000 1g IV

92- Diabetic + Hypertensive. Which antihypertensive will you give?

a. ACEi

93- Recommended in post-dural headache except

a. Fluid restriction

b. Caffeine

94- Carpal tunnel causes thenar muscle wasting (repeated)

95- [Review] everything to help her except intubation and mechanical ventilation (repeated)

96- Patient coming in wanting prostate cancer screening using PSA

a. Discuss pros and cons of PSA

97- Sickle cell patient with osteomyelitis. What is the most common organism?

a. Salmonella

98- When to stop insulin in DKA?

a. When potassium is 2.8

99- Which of the following types of insulin has the longest duration of action

a. Insulin Glargine

b. Novomix

c. Insulin aspart
AMB MCQ Group B 8th Batch
1. 20-Year-old female with lower abdominal pain is diagnosed with ectopic
pregnancy. Which one of the following is an indication to use methotrexate to
terminate the pregnancy?

a. HCG of greater than 10,000


b. Patient who cannot follow up
c. Active vaginal bleeding with hypotension
d. Gestational sac 4 cm with cardiac activity
e. No evidence of tubal rupture

2. The 16 years old presented with stab injury to the chest. On arrival in ER you
suspect the patient might be suffering from cardiac tamponade. The patient
examination reveals

a. Hypercoagulability, stasis, vessel injury (Virchow triad)


b. Muffled heart sounds, decrease BP, increase JVP
c. Recurrent oral ulcers, genital ulcers, iridocyclitis (Behcet Triad)
d. Jaundice, fever, Right upper abdominal pain (Charcot Triad)
e. Low blood pressure, absent breath sounds, raised JVP (Pneumothorax)

3. A 66 years old man diagnosed with CHF. Which of the following medications
should be used in a patient with heart failure with reduced ejection fraction
(HFREF) unless contraindicated?

a. CCB
b. Amiodarone
c. Anticoagulant
d. ACEi
e. Alpha blockers

4. Which of the following tests would you do for a 60 years old male, nonsmoker,
with no medical history of note?
a. BMD
b. Mammogram
c. PSA
d. FOBT
e. US abdomen
5. The most common cause of anesthesia related complication is:
a. Bleeding events
b. Respiratory events
c. Human error
d. Cardiac events
e. Anesthetic agent overdose
6. In an unresponsive victim of trauma suspected head and neck injury, what is the
best method of opening airway?

a. Head tilt neck lift


b. Chin lift - head lift
c. Jaw thrust
d. Head tilt - chin lift
e. Do not attempt to open the airway

7. 40 years old female presented to your emergency department in daytime


complaining of right lower limb pain and swelling for the 3 days. She had a recent
hysterectomy 3 weeks ago for uterine cancer and no history of trauma. Clinically
vital signs were within normal limits and you noticed a swollen right leg with
tender calf muscle. Which of the following is most appropriate in your
manageme

a. X-ray of lower limb


b. Ask for D-Dimer
c. MgSO4 4gm IV
d. Request for Ultrasound Doppler of lower limb
e. CTA-PE Study

8. 46-year-old female presents to the walk-in clinic with severe epigastric pain for 36
hours, radiating to her back. Clinically, she has upper abdominal tenderness and
guarding. Her serum amylase is markedly raised. Which of the following is the
most likely diagnosis?

a. Acute gastritis
b. Oesophagitis
c. Acute Cholecystitis
d. PUD
e. Acute Pancreatitis

9. 65-year-old female develops hypertension, hyperglycemia, decreased bone density


and epigastric pain. Which of the following agents associated with these adverse
effects?

a. Hydrochlorothiazide
b. Simvastatin
c. Lasix
d. Alendronate
e. Prednisolone

10. You know that the sensitivity of the D-Dimer test for the diagnosis of DVT is 95%
and its specificity is 90%, PPV is 85%, and NPV 80%. If the test is positive for patient
A, this means:

a. Patient A has 95% chance of having DVT


b. Patient A has 85% chance of having DVT
c. Patient A has 80% chance of having DVT
d. Patient A has 90% chance of having DVT
e. Patient A has 10% chance of having DVT

11. Which of the following is not part of definitive airway definition?

a. Inflated cuffed endotracheal tube attached to the ventilator


b. Tracheostomy tube anchored with a tape
c. Endotracheal tube connected to oxygen
d. Endotracheal tube below the vocal cords
e. LMA tube anchored at the lips

12. 18-year-old female presents to the ED with suicide ideation and overdose with
beta blockers. Her heart rate is 44/min, regular pulse, BP 88/50 mmHg, saturation
94 on room air. Which one of the following is indicated as the first line of
treatment?

a. Calcium Gluconate
b. Glucagon
c. Start hemodialysis
d. Dobutamine
e. Trans venous cardiac pacemaker

13. 55-year-old male ex-smoker presents with progressive dyspnea, facial plethora
and distended chest and neck veins for 2 months. Which of the following is most
accurate regarding superior vena cava (SCV) syndrome?
a. The most important aspect in the management of SVC syndrome is to
establish etiology and tissue diagnosis
b. Diagnosis is usually established by chest X-ray, then venography if needed
c. The commonest causes are rapidly enlarging aortic aneurysms, syphilis and
fibrosing mediastinitis
d. It should be treated as an immediate emergency with IV thrombolytic and
stenting of the vena cava

14. 20 years old man presented with palpitations. EKG reveals as follows. His vitals are
pulse: 190/min, BP: 72/40 mmHg, RR: 40/min, SaO2 94%. Clinical examination:
chest is bilateral crepitus, what is the next most appropriate management? Case of
SVT

a. Amiodarone 300mg
b. Synchronized Cardioversion
c. Defibrillation
d. Vasovagal Maneuvers
e. Adenosine 6 mg

15. A 48 years old healthy lady comes to your office asking for her screening test for
osteoporosis, you should:

a. CT is indicated for her age


b. Order bone scan as it is indicated for her age
c. Order a lumbar x-ray to screen for osteoporosis
d. Order bone profile to screen her osteoporosis
e. BMD screening should be done at the age of 65

16. 82-year-old woman presented with fever, cough and shortness of breath. Vital
signs BP: 80/30 mmHg P: 130/min RR: 24/min SAT 89% ON RA. P/E show sick
looking lady, tachypneic and confused. She received oxygen supplement and 2 L of
crystalloid fluid. Chest x ray shows pneumonia. Her blood work show high WBC
and lactate. The repeat vital signs are BP: 82/50 mmHg P: 120/min RR: 24/min O2
SAT: 95 on 100% non-rebreather mask. What is the next step in management?

a. Insert central venous catheter and start vasopressors


b. Perform abdominal ultrasound
c. Discharge patient home
d. Perform needle decompression

17. A 27-year-old female patient married, presents to the clinic complaining of


abnormal vaginal discharge. Which of the following is most likely diagnosis?

a. Group B Streptococcus
b. Trichomonas Vaginalis
c. Candida Albicans
d. Bacterial Vaginosis
e. Neisseria Gonorrhea

18. One of the following medications is triggering factor for malignant hyperthermia?
a. Ketamine
b. Propofol
c. Sevoflurane
d. Midazolam
e. Fentanyl

19. A 36-year-old female was eating lunch in a restaurant when she started choking.
All are the signs of severe airway obstruction except:

a. Coughing forcefully
b. Unable to speak or cry
c. Clutching the throat with thumb and fingers
d. Poor or no air exchange
e. High pitched noise while inhaling

20. 23 years old unrestrained driver involved in RTA sustained a closed head injury.
His CT scan is as follows; most likely diagnosis is:

a. diffuse axonal injury


b. subdural hematoma
c. subarachnoid bleed
d. intra-cerebral hematoma
e. epidural hematoma

21. Post-dural puncture headache treatment includes all of the following except:

a. caffeine
b. analgesia
c. fluid restriction
d. epidural blood patch
e. bed rest

22. correct statement about emergency department physician documentation is:

a. documentation of events during ED stay should only done by nurses


b. there is no need to document a procedure if there was no problem
c. documentation is done to improve quality of life
d. it's important to document well so lawsuits can be avoided
i. Either C or D, we are not sure

23. 6-month old baby going for elective procedure under GA and prior shifting the
baby to the operation room the mother fed him formula milk by mistake. How
many hours the procedure should be delayed?
a. 2 hours
b. 3 hours
c. 4 hours
d. 6 hours
i. 6 hours for solid food, formula milk and non-human milk
ii. 4 hours for Breast fed
iii. 2 hours for liquids and oral medications

24. PubMed platform of Midline contains:


Either A or B
a. primary and secondary sources of evidence
b. primary sources of evidence
c. mainly RCT
d. secondary sources of evidence
e. mainly systematic reviews
i. make sure from Dr.Alkhnizan

25. you have been called to see a patient with pulsatile mass in the abdomen. The
emergency US shows as follows; he is pale and vitals are as follows P: 140/min BP:
80/40 mmHg RR: 20/min, SaO2: 99%. What should be the next step after
addressing the airway and breathing?

I think A is the correct one from Surgery Block


a. Stat vascular surgery consult and activate OR team
b. Wait till patient is stable then proceed with CT Scan
c. Needs angiogram
d. Needs contrast enhanced CT Abdomen STAT
e. MRI and MRA

26. When facing a difficult airway, the most critical action would be:

a. Expedite the attempts to intubate the tracheal


b. Insertion of arterial line
c. Maintain ventilation and oxygenation
d. Search for alternative airway devices
e. Find an IV access

27. Contraindications to nasogastric tube insertion in trauma patients include:

a. Penetrating neck trauma


b. C spine fracture
c. Blunt chest trauma
d. Rectal bleeding
e. Blood at the meatus
28. 29-year-old man presents with new onset severe perianal pain and a history of
chronic constipation. No weight loss, no foreign travel. On examination, he has
tenderness along the posterior anal margin, but no other findings of note. What is
likely diagnosis?

a. External hemorrhoids
b. Perianal abscess
c. Internal hemorrhoids
d. anal fistula
e. anal fissure

29. of the following drugs acts as antagonizing the NMDA Receptor?

a. Propofol (GABA)
b. Dexmedetomidine (alpha 2 agonist)
c. Ketamine
d. Midazolam (GABA)
e. Etomidate (GABA)

30. One of the first line treatment for smoking cessation therapy is:

a. Quetiapine
b. Benzodiazepines
c. Nicotine replacement therapy
d. Amitriptyline
e. Fluoxetine (Prozac)
i. Champix (Varenicline) > NRT > Bupropion

31. Which of the following statements is true regarding GAD?

a. It means that the patient becomes anxious in crowded places


b. Treatment with benzodiazepines for long term is essential
c. CBT has been shown to be effective in treating GAD
d. It is not associated with depression
e. It usually starts at old age, over 60 years old.

32. A 73-year-old man presents with SOB and productive cough. He Is a long term
smoker and smokes 20 cigarettes per day. Chest examination shows
hyperinflation, no tracheal deviation. What is the most likely diagnosis?

a. COPD
b. Chest infection
c. Pneumothorax
d. Lung cancer
e. Asthma
33. Which of the following is correct regarding end tidal CO2?

a. 50 mmHg considered to be normal


b. the value increases with hypotension
c. with hypoventilation the value decreases
d. the sample analyzed taken from the blood
e. with hyperventilation the value decreases

34. a 69-year-old man presents with recurrent rectal bleeding with passing stool with
feeling of incomplete and straining. His wife has also noticed significant weight
loss. Which of the following is the most likely diagnosis?

a. Diverticulosis
b. IBS
c. Hemorrhoids
d. Colorectal cancer
e. Sigmoid volvulus

35. A study demonstrates that drug X for the treatment of hypertension has a greater
reduction of stroke than drug Y. the authors conclude that this should make drug X
a better choice with hypertension than drug Y. this study is a good example of
which of the following?

a. Observational analysis
b. EBM
c. Biased analysis
d. Patient oriented evidence
e. Disease oriented evidence

36. Hypotension during spinal anesthesia is due to:

a. Multiple trials of insertion


b. Cerebrospinal fluid leakage
c. Gauge of spinal needle
d. Dural puncture
e. Sympathetic Block

37. Choose the most accurate statement regarding the diagnosis and management of
meningitis in infants and children:

a. Bulging fontanelle is an early sign of meningitis


b. Head CT should be obtained on a routine basis prior LP
c. In an infant with lethargy and irritability when being handled, a normal
spinal tap is ground for discharge
d. Empiric antibiotics should never be delayed in suspected cases of
meningitis
e. A positive gram stain in the absence of other CSF abnormalities can be
reasonably be judged to be a skin containment
i. We think it's D, so just make sure

38. 50 years old female patient controlled DM, HTN going for elective laparoscopic
cholecystectomy here ASA Physical status is:
a. ASA 3
b. ASA 1
c. ASA 5
d. ASA 2
e. ASA 4
39. In a patient who is having a grand mal (tonic clonic) seizure, the most appropriate
first line IV drug of choice after attending to the airway and breathing is:

a. Flumazenil
b. Keppra (Levetiracetam)
c. Tegretol
d. Phenytoin sodium
e. Benzodiazepines

40. a 30-year-old man known to have asthma, on regular low dose inhaled
corticosteroid twice daily. He has been using his salbutamol inhaler every night for
his night symptoms of coughing and wheeze. What is the most appropriate next
step in management?

a. Increase the dose of the CS inhaler


b. Add combined LABA + CS
c. Add oral Theophylline
d. Add oral Montelukast (Leukotriene antagonist)
e. Add oral CS

41. The most appropriate first line management for low back pain in a young and
healthy individual with no red flags is:

a. Bed rest
b. Analgesia with back care advice
c. Reassurance
d. Analgesia with lumbar spine X ray
e. Analgesia with MRI Scan

42. During IV insulin treatment in a patient with DKA, the most likely electrolyte
imbalance is:

a. hypokalemia (stop infusion if K < 2.8)


b. hypocalcemia
c. hypophosphatemia
d. hyponatremia
e. hypomagnesemia

43. 53-year-old lady with hyperlipidemia and HTN complains of diffuse muscle pain
and weakness. Her serum CK is increased. Which of the following drugs is most
likely to have caused this?

a. Atorvastatin
b. Atenolol
c. Nicotinic acid
d. Amlodipine
e. Lisinopril
44. At the minimum a physician or a health care provider should not do anything to
harm a patient. This is referred to:

a. Beneficence
b. Non Maleficence
c. Non beneficence
d. Maleficence
e. Standard of care

45. A 34-year-old obese man (BMI of 43 kg/m2), presents with poorly controlled HTN,
low back pain and DM. he tried different ways to reduce his weight over the last
two years without success. Which of the following would be the most appropriate
intervention?

a. Referral to Bariatric surgery


b. Hypnotherapy
c. 6 months of intensive dietary program
d. pharmacologic therapy
e. 6 months of intensive dietary and exercise program

46. a 28-year-old nurse with a previously negative tuberculin skin test was found to
have positive tuberculin skin test 3 months after having been exposed to a patient
with active TB in the ward. The nurse is asymptomatic and CXR was normal. Which
of the following is the best management?

a. Isoniazid for 6 months


b. Isoniazid and rifampicin until sputum smear is negative
c. Rifampicin and ethambutol for 12 months
d. No treatment is needed
e. Initiative treatment if patient is symptomatic

47. A factor decreasing the risk of having post dural puncture is:

a. Multiple trials of insertion


b. Pencil point spinal needle
c. Large CSF leakage
d. Large gauge spinal needle
e. Quincke spinal needle

48. In an obese patient with type 2 DM. which of the following drugs most increases
the insulin sensitivity?

a. Sitagliptin
b. Short acting insulin
c. Gliclazide (Diamicron)
d. Empagliflozin
e. Metformin (Glucophage)
49. Which of the following aspects of the family medicine consultation include
advising the patient to return to the walk in clinic or ER if symptoms persist or get
worse?

a. Clarification
b. Safety netting
c. Addressing ideas, concerns and expectations (ICE)
d. Reflection
e. Checking understanding

50. Pregnant lady has more risk for developing hypoxia during induction of anesthesia
due to:

a. Increase plasma volume


b. Decreased functional residual capacity
c. Increase cardiac output
d. Increase Respiratory rate
e. Increase stroke volume

51. In regards to PE, the most common finding in ECG is:

a. S1 Q3 T3 Sign (this is the most specific but not the most common)
b. Right bundle branch block
c. Sinus tachycardia
d. STEMI
e. Atrial fibrillation

52. Which investigation is the gold standard for diagnosis of COPD?

a. Spirometry
b. Peak flow diary
c. CXR
d. Urea breath test
e. Oxygen saturation
53. Which of the following best describes the family physician s role when counselling
a 32-year-old female about contraception?

a. Prevention
b. Selecting the method with the lowest cost
c. Helping her in reviewing her options
d. Education
e. Explaining which method is best for her

54. Asthma avoidance measures would include which of the following?

a. Upholstered furniture is preferable to leather and wood


b. It's preferable not to wash the bedding
c. Cover mattress and all pillows in allergen proof covers
d. Increase indoor relative humidity with humidifier
e. Living rooms below ground level are preferable

55. 31-year-old lady presents with generalized abdominal discomfort for 6 months.
The pain improves with defection. Abdominal examinations are normal except for
some mild distension. What is the most likely diagnosis?

a. Hemorrhoids
b. Crohns disease
c. GERD
d. Irritable bowel syndrome (IBS)
e. Inguinal hernia

56. The 80 years old female from nursing home with history of chronic laxative use
presents with sudden onset of abdominal pain with distension. History of absolute
constipation with no passage of flatus. Her acute abdominal series X-rays reveals
the following. What is the diagnosis?

a. Gall stones
b. Bowel perforation
c. Large bowel obstruction
d. Small bowel obstruction
e. Volvulus

57. A 73-year-old female with HTN and DM presents to the ER with epigastric pain,
nausea and vomiting. She is tachypneic and diaphoretic. A 12 lead EKG showed ST
segment elevation MI in anterior leads. You activated cardiac catheterization team
for emergent percutaneous coronary intervention. A few minutes later patient
became bradycardic and hypotensive. What is the next action?
a. Give thrombolytic
b. Start doing CPR
c. Do synchronized cardioversion
d. Give atropine and attach the pads for possible pacing
e. Give acetaminophen

58. A 56-year-old male with tobacco smoking, hyperlipidemia and HTN presents to ED
with chest pain. All of the following are appropriate management for patient
except:

a. Take history and do physical examination


b. Give thrombolytic
c. Insert IV lines and draw blood for laboratory tests
d. Obtain EKG
e. Give analgesic if still in pain

59. A 32-yea-old man has been having headaches for 1 year. The pain is unilateral and
severe, associated with blocked nose and watery eyes. Attacks last for few hours,
and might occur daily for the period of a week or so. Then he would be
asymptomatic for 3-4 months, before the next bout of attacks. What is most likely
the diagnosis?

a. Recurrent tension type headaches


b. Recurrent acute closed angle glaucoma
c. Cluster headache
d. Trigeminal neuralgia
e. Migraine with aura

60. A left frontal lobe stroke (Cerebrovascular Accident) in a 57-year-old lady would
likely lead to which of the following?

a. Anosmia
b. Cranial nerve palsy
c. Visual impairment
d. Hearing deficits
e. Changes in emotion and behavior

61. An elderly patient from a local home presents to your ED with hypernatremia. The
next most appropriate step in patient management is?

a. Give bolus of D5W to rapidly correct electrolytes


b. Order D50 and insulin stat
c. Measure anion gap
d. Calculate fluid requirement with parkland formula
e. Calculate water deficits

62. In patients presenting with ST elevation MI and going for emergent PCI, the word

a. house
b. Door of ambulance which is bringing the patient
c. Time since the EKG is done
d. Time since onset of symptoms
e. Time since a patient arrives in the ER
63. One of the following interventions increasing the possibility of aspiration during
induction of general anesthesia for pregnant lady:

a. Using EET as airway device


b. Bag mask ventilation
c. Applying cricoid pressure
d. 3-5 minute pre-oxygenation
e. rapid sequence technique for induction

64. which one of the following is an ABSOLUTE CONTRAINDICATION to flying on a


commercial airline flight?

a. Pneumothorax
b. Schizophrenia
c. Pregnancy 30 weeks
d. Primary pulmonary HTN
e. History of PE

65. A 59-year-old female with strong family history of premature coronary artery
disease suddenly collapses. You found that she is unresponsive so you activated
the emergency response system. On further examination, she is not breathing and
there is no pulse. What is your best next action?

a. Wait for AED to arrive to check rhythm


b. Give rescue breath every 5-6 seconds
c. Start CPR by doing chest compressions at a rate of 80-100 compressions
d. Start CPR by doing chest compressions at a rate of 100-120 compressions
e. Start CPR by doing chest compressions at a rate of 120-140 compressions

66. Which of the following symptoms would suggest a diagnosis of asthma?

a. Diurnal variation of degree of cough and SOB


b. Chest pain on exertion
c. Chronic productive cough in absence of wheeze or breathlessness
d. SOB when lying flat
e. Croaky voice
67. DNAR code status patient may not receive CPR and:

a. G-Tube feeding
b. NG Feeding
c. Intubation
d. IV Fluids
e. IV Antibiotics

68. The four major principles of medical ethics are:


a. Autonomy, beneficence, non-maleficence and morality
b. Privacy, autonomy, beneficence and justice
c. Autonomy, beneficence, universality and justice
d. Autonomy, beneficence, non-maleficence and justice

69. 45 year-old taxi driver has recently been diagnosed with T2DM. he is keen to
adopt to a healthy lifestyle, and attends seeking advice on exercise. What is the
minimum amount of moderate intensity exercise you should recommend?

a. 100 min per week


b. 60 min per week
c. 1 hour daily
d. 30 min per day, 5 days per week
e. 45 min per day, 5 days per week
i. it has to be 150 minutes per week

70. which of the following tests would you order to confirm active pulmonary TB
infection?

a. Quantiferon blood test


b. Sputum culture for acid fast bacilli
c. PPD
d. Chest X-ray
e. ESR

71. A 75-year-old man attends the clinic following a recent total gastrectomy for
stomach cancer. Which one of the following supplements does the patient need
following surgery?

a. Vitamin B12
b. Thiamine
c. Vitamin K
d. Vitamin D
e. Vitamin C

72. Balanced resuscitation in trauma patient include:


a. Not using blood early in the patient management
b. Early use of blood and products to maintain lower target BP
c. Maintaining normal BP during resuscitation
d. Giving copious amounts of crystalloids to maintain BP
e. Using packed RBCs only

73. Regarding the treatment of IBS, which one of the following is true?

a. Gluten should always be excluded


b. Peppermint oil is indicated
c. A low protein diet is supported by a strong evidence
d. There is no evidence that high fiber is beneficial in IBS
e. Lactose free diet has strong evidence of benefit

74. A 70 years old female with history of multiple abdominal surgeries for chrons
colitis presented with severe abdominal pain, nausea, vomiting and abdominal
distension. You ordered basic labs and acute abdominal series x-ray, the x-ray
revealed. What is your diagnosis?

a. Intussusception
b. Volvulus
c. Small bowel obstruction
d. Bowel perforation
e. Large bowel obstruction

75. A surgeon sustained a needle stick injury while a procedure in the operating room
yesterday. Which of the following is the most appropriate next step?

a. Investigate the surgeon and the patient, for blood borne disease
b. No further investigation
c. Start antiviral therapy
d. Initiate immunoglobulin therapy
e. Investigate the surgeon after 6 weeks

76. A 30 years old female, known breast cancer with recent chemotherapy presented
to the ER with chest infection diagnosed to be in septic shock. As per surviving
sepsis campaign care bundles. Which one of the following is essential to deliver
during first 6 hours of resuscitating a septic patient?

a. Keep Hb above 14 mg/dl


b.
c. Maintain temperature > 36 C
d. Start dopamine/dobutamine
e. Prevent hypothermia
77. 70-year-old male presents to the ED with weakness, BP 100/80 mmHg. You
diagnose hyponatremia (Na 120 mg/dl) on the initial labs. Which one of the
following may exac
Either A or D
a. furosemide (Lasix)
b. 3% Saline
c. 0.9 Normal Saline
d. dextrose 5% in water
e. fluid restriction
78. which of the following muscle relaxant preferable to use in rapid sequence
induction technique of general anesthesia?

a. Cistracurium
b. Atracirium
c. Vecuronium
d. Succinylcholine
e. Rocronium

79. Absolute contraindication for spinal anesthesia include all except:

a. Patient refusal
b. Infection at the site of injection
c. Coagulopathy
d. Increased ICP
e. Scoliosis

80. Among the anatomic features suggestive of difficult airway is:

a. Long neck
b. Limited range of motion of the neck
c. Mallampati class of 1
d. Wide mouth opening
e. Lack of teeth

81. A reasonable approach when suspecting a difficult airway is:

a. To prepare a difficult cart in the room


b. All of the above
c. To have different plans in case of failure
d. To focus on maintaining adequate oxygenation all the time
e. To have extra help available

82. 32 year old female presents to you complaining g right sided face weakness for 1
day, her past medical history was unremarkable. On examination, you note right
sided face weakness involving her forehead. Which of the following is the best
treatment which has shown the best evidence?
a. Systemic steroids
b. Electric stimulation
c. Immune modulators
d. Observation
e. Antiviral agents

83. A 67-year-old male developed chest pain and his EKG showed ST elevation MI. you
decided to give thrombolytic as there is no immediately available PCI in your area.
What is standard door to needle time for thrombolytic?

a. Door to needle time of 3 hours


b. Door to needle time of 90 minutes
c. Door to needle time of 4.5 hours
d. Door to needle time of 30 minutes
e. Door to needle time of 60 minutes

84. A 47-year-old lady falls on her right shoulder and now restricted abduction
between 0 and 90 degrees. Which tendon is most likely affected?

a. Subscapularis
b. Teres minor
c. Triceps
d. Supraspinatus
e. Trapezius

85. Sickle 70 years old male with history of heart disease attended to the ER after
sudden collapse and was found to be in Cardiopulmonary arrest. CPR was promptly
started in ER. Following rhythm was seen on the cardiac monitor. Which of the
following drug should be given during ACLS resuscitation? Case of Asystole

a. Atropine
b. Vasopressors
c. Amiodarone
d. Adrenaline (Epinephrine)
e. Amitriptyline

86. The following infections spread via droplet route:

a. TB
b. Measles
c. Chicken pox
d. Zoster
e. RSV
87. In a patient presenting to the ED with vaginal discharge. The diagnosis of
gonococcal and chlamydial infection is confirmed by:

a. DNA probe testing


b. Whiff test
c. Wet prep microscopy
d. Blood culture
e. Vaginal fluid culture

88. Which of the following patients should not be prescribed Combined OCP?

a. 33-year-old lady who is a smoker


b. 31-year-old with lady a BMI of 34
c. 32-year-old lady with family history of DVT
d. 27-year-old lady with migraine with aura
e. 35-year-old lady with Diabetes

89. a 59-year-old female with HTN, hyperlipidemia and Diabetes suddenly collapses.
You are the first person to arrive at the scene and you found her lying on the floor.
You have verified that the scene is safe. What Is your best next action?

a. Start CPR by doing chest compressions


b. Activate Emergency response system and ask for AED
c. Shout for nearby help
d.

e. Give rescue breaths

90. Which of the following disorders of the hand is the most commonly seen in chronic
liver disease?

a. Palmar erythema
b. Nail pitting
c. Carpal tunnel syndrome
d. Oslers nodes
e. Tesnosynovitis

91. In a patient with acute anaphylactic reaction, the most appropriate treatment is:

a. Methylprednisolone IV
b. Epinephrine 1:1000 0.5 mg IM in a large muscle
c. Antihistamine IV
d. Attend breathing and intubation
e. Epinephrine IV
92. You are team leader in CPR team attending an adult cardiac arrest in ER. The
patient is in the following rhythm on cardiac monitor and has no pulse. Which of
the following is the most appropriate compression to ventilation ration and further
care? Case of Torsades de pointes

a. 15:2 synchronized cardioversion


b. 30:2 synchronized cardioversion
c. CPR with chest compression only
d. 30:2 defibrillation
e. 15:2 defibrillation
93. a study was conducted to evaluate the accuracy
of Abdominal US compared to CT scan in the diagnosis of Acute Appendicitis. This
study is an example of which study design?

a. RCT
b. A cross sectional study
c. A case control study
d. A cohort study
e. A case series

94. Side effects of opioids include all of the following except:

a. Constipation
b. Dizziness
c. Mydriasis
d. Respiratory depression
e. Nausea

95. A 40 years old man presented with painful swollen knee joint of sudden onset.
Gram stain was negative and rhomboid shaped crystals were identified which were
birefringence positive. The diagnosis is:

a. Pseudogout
b. Gout
c. Nongonococcal septic arthritis
d. Gonococcal septic arthritis
e. Tuberculosis arthritis

96. A diabetic and known HTN patient presents with sudden onset of confusion and
blurriness in vision. His BP on arrival is 260/130 mmHg. On fundal examination he
has flame shaped hemorrhage. What is likely reason for his presentation?

a. Sub Arachnoid Hemorrhage


b. Hypertensive encephalopathy
c. The diagnosis cannot be made until after CT brain
d. Central artery occlusion
97. While a patient with eclampsia is being treated with magnesium sulfate, the nurse
notices poor reflexes and shallow breathing efforts. The next appropriate
management decision is:

a. Give calcium gluconate


b. Start CPR
c. Prepare to intubate the patient
d. Start dose of narcan (Naloxone)
e. Epinephrine

98. Difficult airway is defined as difficulty with:

a. Intubation and ventilation


b. Intubating the trachea
c. Airway obstruction after extubation
d. Insertion of oral airway device
e. Mask ventilation

99. 82-year-old female complains of headache and left painful red eye suddenly after
watching a movie in a theater. Next most appropriate step is:

a. gentamicin ophthalmic drops every 4 hours


b. ketorolac 30 mg IV Stat
c. pilocarpine intra-ophthalmic drops stat
d. acyclovir ointment stat

100. 22-year-old male presents with abdominal pain associated with vomiting,
now the pain is localizing to the right lower quadrant. On examination there is
guarding in the right iliac fossa. What is the most likely diagnosis?

a. Acute appendicitis
b. Testicular torsion
c. Ulcerative colitis
d. Diverticulitis
e. Incarcerated hernia
AMB MCQs 9th Batch Group C

1. PubMed is considered a:
a. Primary source
b. Secondary source
c. Primary and secondary source
Ans : A

2. A child that fell on his hip and now is unable to bear weight on it. What is
the most likely diagnosis?
a. Slipped capital femoral epiphysis
b. Leg Calve Perthes disease
Ans : A

3. A man presents with a pulsatile abdominal mass, he has hypotension and


does not respond to IV fluids. What would be the best line of management?
a. Send him to get a CT with contrast
b. Send him to the vascular OR STAT
Ans : B

4. A man brings his child to the ER after he saw him ingest something. The
child coughed a bit but doing fine now. Which of the following is the
most appropriate management?
a. Retrieval of foreign body using Magill
Forceps
b. Endoscopy
c. Rigid Bronchoscope
d. Assure the parent and tell him to come
back 3-5 days later
Ans : B (Button battery necessitates immediate
removal due to the risk of perforation)

5. A COPD patient presents with worsening cough and dyspnea. Which of the
following is the most appropriate management?
a. Anticholinergic inhaler
b. Prednisone
Ans : A
6. Which of the following is considered a risk factor for suicide?
a. Recent bereavement
b. Female gender
c. Living in a supportive society
Ans : A

7. A patient presents with worsening shortness of breath and lower limb


edema. Which of the following is the most appropriate initial treatment?
a. ACE inhibitors
b. Furosemide
Ans : B

8. A patient presents complaining of a painful big toe. Which of the following


is the most appropriate management?
wording. If the question asks for the management of an acute episode then
Indomethacin (NSAID) would be correct, however; if it was about long term
management then allopurinol would be correct instead)
a. Allopurinol
b. Indomethacin
Ans : B

9. Which of the following is considered a secondary prevention?


a. Pap smear
b. Screening for hypertension
Ans : A

10. A patient has a copper IUD. After how many years should she remove it?
a. 5 years
b. 7 years
c. 10 years
Ans : A

11. In which of the following conditions can dialysis be considered?


a. Hypervolemia
b. High serum ammonia
c. Alkalosis
Ans : A
12. A patient with an established ischemic heart disease recently did blood
tests that showed dyslipidemia. Labs showed high total cholesterol and LDL.
Which of the following is the most appropriate management?
a. Assess him using the ASCVD risk assessment score
b. Start him on statins directly
Ans : B

13. A pregnant patient recently developed hypertension. What would you start

a. Beta blocker (LABETELOL)


b. Spironolactone
Ans : A

14. A patient k/c of Crohn colitis presented with a complaint of abdominal


distension, vomiting, and not being able to pass stool. An abdominal X-ray
was given. Which of the following is the most likely diagnosis?
a. Volvulus
b. Small bowel obstruction
c. Large bowel obstruction
Ans : B or C

15. A patient presented with epistaxis that wont stop. He also has severe
hypertension 180/105. Which of the following is the most appropriate
management?
a. Anterior nasal packing
b. Posterior nasal packing
c. IV beta blockers
Ans : C

16. Which of the following is the most common artery that causes posterior
epistaxis?
a. Greater palatine artery
b. Sphenopalatine artery
c.
d. Anterior ethmoidal artery
e. Posterior ethmoidal artery
Ans : B

17. Which of the following is not a side effect of opioids?


a. Increased intracranial pressure
b. Bradycardia
c. Constipation
Ans : A

18. A woman presented with worsening cough and shortness of breath. She

for it. How would you manage her case?


a. Start her on daily use of the inhaler
b. Start her on inhaled corticosteroids
Ans : B

19. A woman presented with pain of her foot. It is relieved by walking. Which
of the following is the most likely diagnosis?
a. Plantar fasciitis

20. A woman was sitting on a barstool when she fell and bumped her head.
Which of the following would be the most appropriate investigation? (exact
image presented)
a. X-ray of the facial bones
b. Cervical spine X-ray
c. Non-contrast CT
Ans : C

21. A woman presented with history of migraines. Which of the following


would help prevent these headaches (prophylaxis)?
a. Triptans
b. Amitriptyline
Ans : B

22. A child presents with a rash over his abdomen. He recently had an episode
of coryza and cough. Which of the following is the most likely diagnosis?
(image was presented)
a. Measles
b. Rubella
Ans : A

23. A patient presented with a fluid filled lesion that was up to 0.5cm. What is
this lesion?
a. Vesicle
b. Pustule
Ans : A

24. A man presented with skin lesions on the dorsum of his hand. Image was
presented.
a. Scabies
b. Impetigo
c. Pityriasis rosea
d. Pityriasis versicolor
Ans : A

25. A 65 y/o man presented with hemoptysis and fatigue for some time.
Temperature was normal. What is the most likely diagnosis? (CXR was
provided)
a. Lung cancer
b. Pneumonia

Ans : maybe A?

26. An HIV patient that is taking Indinavir presents with flank pain. He denies
any dysuria or fever. What is the most appropriate investigation?
a. X-ray
b. CT with no contrast
c. CT with contrast
Ans : B

27. A study that showed CI of 0.6 to 0.7. What is your conclusion?


a. Statistically significant
28. Patient presented with complete loss of movement on one side of the face.
How to treat?
a. Systemic steroids
b. Antivirals
Ans : A

29. A case of an adult with peritonsillar abscess with an anteriorly displaced soft
palate and uvula. How to manage?
a. US of the neck
b. Antibiotics
c. Large bore needle aspiration
Ans : C

30. What would the CSF of a bacterial meningitis show?


a. High protein, low glucose, high WBC (neutrophils)

We had a total of 113 MCQs. Most other questions were repeated from 8th batch
or were easy. These were the most confusing questions we remembered.

Good luck!
9th batch group D MCQs

1. A 55 year old lady post menopausal came in with


vaginal bleeding. What do you want to order?
A. Pelvic Ultrasound

2. Dose of paracetamol for a child?


A. 15mg/kg

3. 11 month old with runny nose wheezing and bilateral


basal lung crackles. How to manage?
A. Observe and supportive care
B. Chest X ray
C. racemic epinephrine

4. What medication can be given via ETT?


A. Epinephrine
B. NaHCO3
C. Amiodarone

5. Lady went to honeymoon and came back with diarrhea


for 3 days. Most common cause?
A. Enterotoxigenic E.coli

6. Male riding a bike had an accident and got a splenic


infarction. When to take him for OR?
A. Grade 4 infarction
B. hemodynamic instability

7. Old lady has constipation for 3 years, goes to the


bathroom on average about once a week. She came in with
severe pain on defecation and posterior anal tenderness.
What is the diagnosis?
A. Anal fissure

8. Patient came in with STEMI. What medication will


improve his mortality?
A. Aspirin
B. O2
C. morphine
D. nitroglycerin

9. Most common cause for acute pancreatitis?


A. Gallstones
B. Alcohol
C. idiopathic

10. Cholecystitis case. Sign on Physical exam?


A. Pain on inspiration while palpating RUQ

11. Patient came in with an on and off headache for a few


months, nasal congestion and watery eyes. Headache is
unilateral and worse at night. What's the diagnosis ?
A. Cluster headache

12. Patient with DM and HTN. Hypertension not


controlled what medication to add?
A. ACE inhibitor

13. Patient who is obese with a BMI of 41. Has multiple


comorbidities. Back pain etc.. tried to lose weight with diet
and exercise for the past 2 years but failed. Next option is?
A. Bariatric surgery

14. Drug that causes the most increase in insulin


sensitivity?
A. Metformin

15. Fatima is a Pregnant lady 8 weeks of gestation, came in


with HTN for the first time. What medication will you start
her on?
A. Labetalol
16. Newly married female. wants to start a family.What
medication is the most beneficial?
A. Folic acid supplements

17. Lady came in with weakness, fatigue, pallor, and pica


(eating ice) what is the diagnosis?
A. Iron deficiency anemia

18. A patient came in with signs of anemia with loss of


sensation in limbs and Labs showed increased MCV. What
do you give him?
A. vitamin B12

19. A patient came wanting to have prostate cancer


screening. What would you tell him?
A. Prostate cancer screening is not recommended
B. Screen using PSA

20. Patient came in with jaundice, abdominal pain n/v.


Took some pills yesterday. What's the management?
A. N-acetylcysteine

21. When to scan for osteoporosis?


A. More than 65 years, using BMD

22. Case of adult with otitis media. Ear pain and discharge.
What medication do you give?
A. Amoxi-Clav

23. A female received chemotherapy, she presented with


fever, mild fatigue?
A. Cefepime
B. Ciprofloxacin
C. Amoxi-clav
D. Nafcillin

24. High altitude cerebral edema case, what's the most


sensitive sign?
A. Cerebellar Ataxia
B. Seizures
C. Nerve VI palsy

25. B1 agonist was used by a patient multiple times a day,


What electrolyte abnormality would it cause?
A. Hypokalemia
B. Hypoglycemia

26. A 47 year old male whom his father had a colon cancer
at the age of 72. When should he screen?
A. 40
B. 50 https://www.aafp.org/afp/2018/0115/p111.html
If the patient had a family history of colon cancer
<60 years old, then we screen at 40 or 10 years
prior, whichever comes first. if the patient had 2
members with colon cancer at any age we screen
at 40.
However, this patient doesn't fit the criteria

27. A 58 year old smoker - years not mentioned - who


denied any SOB, Hemoptysis, or any abnormalities came to
you in your primary care clinic. Do you screen him for Lung
cancer?
A. Chest x-ray
B. Chest CT
C. Screening not recommended

28. A typical Cholelithiasis case, What is the sign that


would be present?
A.
B.
C. Tenderness upon palpation

29. A patient presented with right upper quadrant pain,


jaundice and fever. What's the diagnosis?
A. Acute cholecystitis
30. What is the abnormality in the picture -not the same
one that came in the exam but similar-?

A. Retroperitoneal bleeding

31. What do you use for general surgery induction? RSI

32. What is a sign of difficult airway management?


A. Limited neck range of motion
B. long neck

33. A test that has sensitivity of 95%, specificity of 80%,


NPV of 80% and a PPV of 85%, which of the following is
true?
A. The probability that if the test is positive the patient
has the disease is 85%
the probability that the test is positive when the patient has
the disease is PPV. No need for calculation here.

34. Patient hurt his ankle. What would indicate doing an X


Ray?
A. Tenderness over posterior medial malleolus
B. Tenderness over the anterior medial malleolus
C. Tenderness over the base of the 5th metatarsal

35. A 26 year old male patient k\c of HIV is presented with


the rash in the picture below, how to treat him?
A. Acyclovir

36. Patient came in the Ambulance, Loss of consciousness,


CPR was done and ECG showed Asystole. What do you do
next?
A. Epinephrine
B. Defibrillation

37. Patient came in an ambulance, losing consciousness,


very low blood pressure and bradycardic with a complete
heart block. What to do next?
A. Percutaneous pacing
B. Dopamine
C. Epinephrine

38. Patient came in with palpitations, ECG showing below.


What to do next?
A. Give 6mg Adenosine (no option to start with vagal
maneuvers)

39. Difficult airway refers to?


A. Inability to ventilate/ intubate
B. Inability to intubate only

40. Obese patient with DM. His labs showed high blood
sugar levels and high hBA1c. What medication can you add
to aid in his weight loss and control his diabetes?
A. GLP-1 alone
B. GLP1 with DPP4

41. Patient came in worried about breast cancer. At what


age do you start breast cancer screening?
A. 40
B. 50

42. Patient who is a smoker is willing to stop smoking.


What medication do you give him?
A. Fluoxetine
B. Quetiapine
C. Varenicline

43. Which of the following is not done in post dural


headache?
A. Fluid restriction
B. Blood patch
C. Analgesia
D. Caffeine, rest

44. A 43 year old female presented to the clinic with 167\X


blood pressure and headache, past medical history is
unremarkable. Family history is positive for her mother
having DM2. What lab test will you order?
A. Fasting lipid profile
B. Renin/angiotensin
C. Urine metanephrines

45. You find someone unconscious, you make sure the


scene is safe, call for help and then you check the pulse and
find it absent. What will you do next?
A. Start chest compressions at a rate of 100 - 120 per
minute

46. Which of the following is demonstrated in the CT


below?

A. Epidural hematoma
B. Subdural hematoma

47. Question with an ECG attached, what is it?

A. Complete heart block


B. First degree heart block
C. Second degree Mobitz 1
D. Second degree Mobitz 2

48. What muscle relaxant do you use in rapid sequence


induction?
A. Succinylcholine
B. Rocuronium
49. Pregnant lady going for general anesthesia. How will
you manage the airway?
A. RSI
B. LMA
C. Bag mask ventilation

50. All the following are absolute contraindications for


spinal anesthesia except?
A. Scoliosis
B. Coagulopathy
C. Patient refusal,
D. Skin infection over the area
E. Increased ICP

51. What is a contraindication to inserting a nasogastric


tube?
A. Chest trauma
B. Middle facial trauma
C. Rectal bleeding

52. Which of the following medications can cause


Malignant Hyperthermia?
A. Sevoflurane

53. A patient came with right lower quad pain. She had
similar episodes twice before and it was relieved within an
hour. This time the episode lasted 6 hours. Physical
examination revealed right adnexal tenderness. Which of the
following is the diagnosis?
A. Appendicitis
B. Ovarian torsion
C. Hernia
D. PID

54. Which enzyme is specific for pancreatitis?


A. LIPASE
55. Which of the following is true about IBS?
A. Peppermint oil is proven to be good

56. Which of the following means doing no harm to the


patient?
A. non-maleficence

57.
crowded elevator?
A. breaking confidentiality

58. A patient with high LDL and previous MI, what


medication to add?
A.
B. Calculate the ASCVD risk and if less than 5%
prescribe statin
C. Give Statin as he already had previous MI

59. Which of the following decreases in pregnancy?


A. Functional residual capacity

60. Which of the following decreases ETC02?


A. HYPERVENTILATION

61. Opiate overdose?


A. Naloxone

62. What is beck triad?


A. Muffled heart sounds + hypotension + jugular venous
distention

63. Most common adverse events of anesthesia?


A. Respiratory events

64. NMDA antagonist?


A. Ketamine
65. An old lady, 39 pack smoker, with shortness of breath
and cough, her chest x-ray showed hyperinflation,
diagnosis?
A. PE
B. COPD

66. Hypotension in spinal anesthesia is caused by?


A. Puncturing the dura
B. Sympathetic block

67. High ck and muscle pain are side effects of which


medication?
A. Statins

68. 40 year old female with HTN, screen her for?


A. Fasting glucose

69. A patient on ICS and SABA not controlled (repeated


twice in the exam)?
70. Add LABA

71. A female was complaining of shortness of breath,


paramedics arrived at the scene where they found her
unconscious. They sent her to the hospital and kept doing
CPR in the way. Her ECG showed pulseless electrical
activity, She was given Naloxone but with no response.
While they were doing the CPR a doctor noticed a cast on
her leg due to a fracture 3 weeks ago. At the scene there

cause?
A. PE
B. Opioid abuse

72. A patient went rock climbing where he fell landing on his


feet with his knees extended. An x-ray was done and it
showed no fractures. Which of the following is the most
serious immediate complication?
A. Compartment syndrome
B. DVT
C. Anterior cruciate ligament injury
D. Popliteal artery injury

73. A patient came with epidural hemorrhage (image


attached). Blood was sent to cross-match. His blood type was
AB. Which blood type is present at the ED is always ready
and can be given to him immediately?
A. O
B. AB

74. A patient has severe headache, watery eyes and other


symptoms of cluster headache. Which of the following will
help?
A. 100% oxygen
B. Sumatriptan

75. A 26 y\o female presented with high temperature, weight


sis?
A. Toxic goiter

76. 2 groups of patients who presented with appendicitis


were sent to either a US or CT scan to compare between the
2 modalities, which study type is this?
A. Randomised control trial
B. Cohort

77. A study was done to see the link between smoking and
cancer. 90% of those who had lung cancer were smokers,
but 30% of people who were smokers did not have lung

A. 90%
B. 30%
C. 70%
78. A female with travel history 2 days ago developed
diarrhea, which of the following is the causative organism?
A. Giardiasis
B. Enterotoxic E. coli

79. In a study, out of 600 people who had the disease, 300
were exposed and 300 were not. And out of 600 who did not
s
the positive predictive value?
A. 85%
B. 60%

80. In a study about breast cancer, 5% of those who had

sensitivity?
A. 5%
B. 2%
C. 93%
D. 95%

81. In ICE used in history taking, what does E stand for?


A. E stands for what the patient expects from the doctor.

82. A 47 yr HTN female with no fam hx of malignancies


what would you test her for?
A. Mammogram
B. Fasting blood sugar

83. When to screen for colon cancer?


A. 50

84. An old female, constipation, fatigue, bloody stools, when


she empties she feels residual. What's the diagnosis?
A. Colorectal cancer
86. Patient with travel history, prolonged sitting, swelling
unilateral leg. What to do next?
A. Doppler
B. D-dimer
C. CTA

87. Most common disease in international travellers?


A. Diarrhea

best action?
A.
as you
B. Consult ethics board
C. Respect the patient's decision and try to know the
reason behind their refusal.

90. Married female with vaginal discharge, the husband

A. Candida
B. Bacterial vaginosis
C. Trichomonas
D. Chlamydia

91. A patient who has hemoptysis and the x-ray showed


hyperventilation?
A. Exacerbation of COPD
B. Lung cancer

92. In type 1 DM patient the HBA1c is?


A. Less than 5
B. Less than 7

You will be doing great in shaa allah! All the


best
Note: This is the AMB concepts which has combined ER/Anesth/ FM from
the separate blocks that our seniors had all compiled in one document. This
document has been corrected to the best of our knowledge. Things in red
are answers we think are correct, purple is additional notes/most probable
answers. Anything highlighted in grey is sth we aren’t sure of at all.

5th Batch Girls: AMB Feb-April

1. What dose MAC indicate? Potency of anesthetic

2. Which of the following is correct?


a. Epidural needle is larger than spinal MPA
b. Only spinal causes hypotension

3. Crohns colitis + multiple surgeries who developed small bowel obstruction with photo – X
ray done-diagnosis
a. Bowel perforation
b. Small intestine obstruction
c. Large intestinal obstruction
4. Signs of severe hypothermia:
a. Shivering, rapid pulse & respirations
b. No shivering, coma/decreased consciousness signs (can’t recall
exact wording but this is the most likely answer)

5. Breaking bad news about hepatitis: time for discussion tx choices, making sure to
bringing a companion
6. Mechanism of PE in diving –
a. nitric oxide air bubbles in vessels with rapid ascending – this is a
complication but causes decompression sickness not PE
b. Holding breath with pressure alveoli rupture
7. A near drowning case management – same approach to trauma

1. Picture of child with trunk maculopapular rash, with cough coryza and
conjunctivitis preceding the rash.
a. Measles
b. Mumps
c. Rubella

2. Which of the following is more in favor of organic psychosis over


psychiatric?
a. Auditory hallucinations
b. visual hallucination

3. Question describing a study with 2 groups of children with Pb exposure and


following them up to see who develops anemia, what type of study?
a. Cohort study
b. Cross sesctional
c. Case Control

4. Water skiing patient, near drowning experience, came unconscious, what do


you do next?
a. jaw thrust, CPR, cervical precautions most probable answer bc only
option that had proper cervical protection measures and had CPR
b. C-spine collar, CPR etc, then head tilt chin movement
c. LLD position and Intranasal O2
d. head-tilt chin lift, CPR

5. Athlete running a marathon on a hot day, comes with 40 degrees temp,


altered LOC, mumbling, and his HR is high, SBP 135, he is sweating, what
will you do next?
a. Give IV hyper tonic solution
b. Give istonic NS hydration
c. General cooling measure (bc this is heat exhaustion)

worker in the sun : cool down coz heatscroke

6. Anaphylaxis question, 1st line of treatment?


IM epinephrine.
7. Woman with pain at sole of the foot, worse in morning, better with long
distance walking, what is the Dx?
a. Osteomalacia
b. Calcaneal epiphysisitis
c. Plantar fasciitis - most probable answer (MPA)
8. Child with sore throat, fever, maculopapular rash all over, in hands and
feet, conjunctival injection, strawberry tongue. Dx?
a. Coxsackie (hand foot mouth) disease
b. Kawasaki – most possibly
9. Pt scheduled for an elective surgery, but he had a breakfast meal by
mistake. How many hours should the surgery be delayed?
a. no need to delay
b. 4hrs
c. 2 hrs
d. 6 hours MPA
10. Breast cancer hx treated, came with back pain symptoms 
a. Lumbar Xray
b. CT Spine (MPA)
c. MRI spine
11. Stretching exercise as initial management of a regular MSK complaint (I
believe low back pain) in a normal patient
12. Old pt with back pain,  IV steroids and STAT MRI
13. Pt with well controlled diabetes, for elective surgery
ASA 2
14. Oropharyngeal airway, what is not true? 
a. It has rounded edges
b. It is only one size/comes in 1 color
c. it’s a bite block
d. it’s a central airway?
15. What is wrong about arterial line?
a. It’s used to measure BP
b. infection can happen
c. Seldinger technique is used to insert it
d. It is used to deliver emergency medications (central line is used intra-op)
16. DKA
Insulin Drip according to weight
17. What is not correct about spinal anesthesia? 
give it thoracic, it’s inserted at L3-4/L4-5 all other answers were correct
18. PEA is treated with?
a. IV epinephrine
b. IV atropine
c. Defibrillation (it’s a non shockable rhythm)
d. IV amiodarone
19. Stable SVT ECG, Tx?
IV adenosine
20. 35 year old woman, who is not sure of her childhood immunization. Which
vaccine should you give her now?
a. HPV
b. Tetanus MPA, but please check
c. Polio
d. Zoster
21. Case of Diabetic, with scrotal skin peeling:
Fourneirs gangrene
22. Female who lost her husband 4 months ago, and is sad, hallucinates about
him. Mx?
a. SSRi
b. Bereavement clinic for 1 year
c. antipsychotics
23. Suicide: what is true?
a. men widowed has highest risk MPA
b. suicide is more common in winters
c. women are more successful
d. men attempt more
24. Primary dysmenorrhea, Tx?
– NSAIDs
25. Woman who had a RTA, had a splenectomy, got blood transfusions, 3
weeks later she present with high fever, chills, no GI, Resp or other Sx.
Only abnormal was elevated AST/ALT?
a. Hep A
b. Hep C possibly
c. Intraabdominal abcess
d. Ebola
e. brucella
26. Painless hematuria, what will you do next?
a. renal CT
b. pelvic CT
c. Cytoscopy
27. Woman with classic appendicitis with no rebound tenderness. What will you
do next? Don’t know
a. Do more labs and stool culture
b. Consult surgery
c. Give IV paracetamol
d. ask her to go home and come back in 24hrs
28. Man who is has prediabetes, pre HTn, and High LDL, what will you do?
a. lower cholesterol, carb and reduce exercise and F/U 3m
b. lower cholesterol, carb and reduce exercise schedule F/U in 1 year
29. Facial palsy –
Steroids
30. Difficult Intubation what is true:
a. They can extend atlanto axial
b. can open mouth fully
c. >9 cm from angle of mandible to mentum
c. <4cm mentum to the thyroid
31. Scabies picture with itching Hx–
Dx scabies
32. Which one is an IV hypnotic?
Propofol
33. STD with PID (cervical motion tenderness, bilateral adnexal tenderness, and
purulent discharge) and Bac Vaginosis also for 1 week thin discharge with
fishy odor, what management?
a. Metronidazol, Clinda, Rocephin
b. Azithromycin, Rocephin (Ceftriaxone) , Metronidazole MPA
c. Doxycycline, Cipro, Metronidazole
d. Other options that didn’t have metronidazole
34. Malignant hyperthermia, what do you dp?
urgent admission
35. Cocaine overdose, what do you give?
a benzo
b. nitro
c. b blockers
36. Pic of Subdural
37. Pic of Epidural
38. Pic of Volvulus

39. Pic of Small bowel obstruction – multiple air fluid levels

40. Methotrexate indication?


<4cm with no Fetal heart rate
41. Snake bite, to confirm its better you bring the snake with you if possible to
the ED? Don’t know
a. True
b. False
c. All of the above
a. None of the above XD

42. Febrile neutropenia in a leukemic patient, who has a hickmans catheter,


comes with infection. They took cultures, she is started IV paracetamol/Iv
cefipime. with Ce Tx?
a. Add vanco: ans bc she has a catheter MPA
b. switch to piperacillin/tazobactom/
c. add some other abx
d.

43. Pt with mild hypercalcemia: (confused) Don’t know


a. Treat with hemodialysis
b. Calculate corrected calcium with plasma protein
c. Give fluids, steroids, aldronic acid MPA

44. Not correct about RSI?


It doesn’t need a muscle relaxant
No bag masking

45. Way to monitor CPR effectiveness:


a. Capnography
b. intraarterial pressure
c. systolic Blood pressure

46. What reduces the EtCO2 ?


a. Decrease minute ventilation
b. Increase tidal volume
c. Decrease RR
d. malignant hyperthermia

Woman who wants osteoporosis scan > scan after age 65

1. which is not a definitive airway > LMA


2. What you do at sign out > SBAR
3. what you do to avoid framing > SBAR
4. woman with appendectomy : ASA 1 E
5. atypical MI : Diabetic patient
6. NNT calculate with given table of numbers > 48
7. appendicitis (RLQ pain periumbilical, localized to RLQ with N,V, woman)
8. testicular torsion : examine and alert urology
9. rectal cancer (incomplete emptying symptoms)
10. bacterial vaginosis / candida (19 y/o sexually active female; most common vaginal
infection) Acc to Dr Butt MC STD - HPV warts, MC infection is BV
11. dna probe test (chalmydia gonorrha)
12. HSV infection  treat for life
13. PPD positive : isoniazid 6 months or 9months??
14. acid fast bacillli (TB Pneumonia)
15. cross sectional study (diagnostic)
16. disease oriented (med x and y; one has better results in terms of disease)
17. Pregnant woman with signs of PE/DVT > send for CTPE OR DOPPLER
18. IV LINE : decreasing viscosity + length
19. 55 yr old man for folllow up : fobt (55yr old )
20. man of prostate cancer :
21. old fat lady with hypertension + tried dieting
22. asthma patient with no control on ICS > LABA
23. guy with full bladder and previous lung cancer : urgent MRI
24. balancing fluids in trauma==> early blood transfusion to maintain lowest possible
bp
25. ferritin for IDA
26. fever + pancytopenia : Bone Marrow Biopsy
27. Dysphagia old : esophageal cancer vs achalasia
28. burns which med : ketamine
29. asthma : decrease allergens
30. dm pt to increase insulin sensitivity > tzd-MPA vs metformin
31. which patient do u not give combined COCP > migraine w. aura
32. patient with OCP : educate vs assist to look at options
33. rsi : no bag mask ventilation
34. anal fissure vs fistula (loose stools)
35. mc chest pain in fam med : gerd vs msk ?
36. pancreatitis
37. snake bite : time is imp (DIC risk within 4 hours! MPA), type of snake is
important
38. cpr : increase v fib time to shock
39. shocking normal rythm > vfib
40. smoking cessation : NRTS
41. tobacco clinics saudi : 70 in saudia
42. dm woman : ace i
43. acr ratio to monitor dm
44. risk of MI and troponin : 85 % PPV
45. patient to remove second mi risk : Beta Blocker
46. low out HF > ace i ?
47. first line in bb toxicity: calcium or glucagon
48. cocaine not se : mydriasis
49. pregnancy : sphincter tone decreases
50. exercise : 5 times a week 30 mins
51. methotrexate : sign or ruptured tube ?
52. DKA: measure with anion gap
53. esrd cause of disease : hyperkalemia
54. ibs : CBC Normal to say its ok
55. RSV by droplet
56. sepsis : remeasure lactate
57. AAA : Do Bedside US / oxygeb/ large bore iV needles
58. acute angle glaucoma (sudden pain and loss of vision)
59. cluster headache (stays for < 3 hr on daily basis)

Contraindication to fly on airplanes : pneumothorax

Travel by air is normally contraindicated in the following cases:

Infants less than 48 h old.


Women after the 36th week of pregnancy (32nd week for
multiple pregnancies).
Those suffering from:
a angina pectoris or chest pain at rest;
b any active communicable disease;
c decompression sickness after diving;
d increased intracranial pressure due to haemorrhage, trauma or
infection;
e infections of the sinuses or of the ear and nose, particularly if the
Eustachian tube is blocked;
f recent myocardial infarction and stroke (elapsed time since the
event depending on severity of illness and duration of travel);
g recent surgery or injury where trapped air or gas may be present,
especially abdominal trauma and gastrointestinal surgery,
craniofacial and ocular injuries, brain operations, and eye
operations involving penetration of the eyeball;
h severe chronic respiratory disease, breathlessness at rest, or
unresolved pneumothorax;
i sickle-cell anaemia;

60. psychotic illness, except when fully controlled.

61. pancreatitis high amylase


62. whats the most common mistakes : wrong medication vs wrong patient ? Dr butt
mentioned that said wrong diagnosis is most common :p
63. MAC not affect by weight
64. chemical arteritis (use of thiopental)
65. to what do you correct blood pressure : 140/90
66. epiglottitis patient(droools) you prepare for intubation in the ED
67. you always check pulse oximetry during OR
68. which u do in primary Suvery : u can only use board for 6 hr
69. pregnant lady with cat exposure and lymphadenopathy > toxoplasma
70. NON MALEFICIENCES
71. Cincinatti – FAST for stroke
72. which patient to do synchronized cardiovert : a fib
73. copd diagnosis spirometry
74. vit b12 deficiency in gastrectomy
75. 20 years old diabetic patient brought to ER suffering from hypoglycemic coma, the treatment
of choice is:
- MLA : 50 milligram glucose IV
76. 63. What is the most common cause of UTI?
- MLA : E.Coli (was repeated twice)
77. Which of the followings not present as an oral preparation for HTN?
- MLA : ?? (Atenolol, Nifedipine, Thiazide, Furosemide, Captopril)
78. Which antihypertensive should be avoided in bronchial asthma patients?
- MLA : Labetolol
79. Adhesive material placed around wound edges needs how long to hold it before release ?
- MLA : 30 seconds; At no time should the applicator tip be pressed into the wound;At least
three layers should be applied to ensure optimal strength t
80. Case (scenario) of malignant hypertension, what is the treatment?
- MLA : Sodium Nitroprusside
81. Case of bronchial Asthma and hypertensive patient, which anti-hypertensive best avoided in
this case?
- MLA : Labetalol
82. What is the drug of choice for convulsions ?
- MLA : Carbamazepine
83. (This case is for the next two question) A 35 year old female patient presenting with a
complaint of dizziness, she describes her dizziness as “like she is out of this world”, all results
of labs and investigations were negative except, however she have accepted a new job which
is very competitive and stressful:

72. Which is the least next action to be taken in this condition ?


- MLA : Lab work + imaging
73. What is the most probable diagnosis ?
- MLA : Non specific dizziness
84. (This case is for the next three questions) 25 year old man went to Thailand and came back,
he is currently suffering from severe knee pain:

74: Which of the followings is an essential part of history in this case?


- MLA : Having multiple sexual partners

75. What is the next step management that would determine what is wrong with his knee?
- MLA : Knee aspiration

76. What are you suspecting in this patient ?


- MLA : Septic arthritis (N. Gonorrhea)
85. IPSG 1 (International Patient Safety Goals) & IPSG 2 are?
- MLA : Define patient correctly & improve communication

Goal 1 - Identify Patients Correctly


Goal 2 - Improve Effective Communication
Goal 3 - Improve the safety of high-Alert Medications
Goal 4 Ensure correct Site, Correct Procedure, Correct Patient Surgery
Goal 5 - Reduce Risk of Health Care-Associates Infections
Goal 6 - Reduce the Risk of Patient Harm resulting from Fall

86. Medical Trainee JC’s AMC Orientation Includes:


- MLA : Any Ongoing Required Education, International Safety Goals, Infection Control
Program, Medication Safety Program

87. Which of the followings best define Never Events:


- MLA : "adverse events that are serious, largely preventable, and of concern to both the
public and health care providers for the purpose of public accountability."
Are 29 Particularly Shocking Medical Errors Causing Death or Significant Injury of Patient or
Staff, classified to 7 categories. IPSG 4 is geared toward Never Event

88. What cause Infectious Mononucleosis?


- MLA : Virus
89. Obese woman presents with unilateral calf pain. She is also on OCPs. Her history is
unremarkable. General Physical Examination is normal. What do you do?
- MLA : Do a Doppler of her calf since DVT can’t be ruled out
90. White male comes in with asthma, you gave short acting beta 2 agonist, however he is not
improving. What do you give him as an outpatient?
- MLA : Inhaled corticosteroids
91. Lady with HTN got pregnant. The physician changed her drug which has teratogenic effect to
another safer drug?
- MLA : Captopril changed to Methyl-Dopa
92. Which of the following is an appropriate suture size?
- MLA : Face 6-0
93. Most common cause of chronic cough is?
- MLA : Post-nasal drip
94. 86. Which of the following statements is correct?
- MLA : Acute Pancreatitis is an emergency
95. 87. Which of the following statements is correct?
- MLA : Duodenal Ulcer cause episodic epigastric pain
96. (This case is for next two questions) Patient with DM, he developed dizziness.
97. Which history question is essential in the start of history taking?
- MLA : The dizziness happens in which circumstances (what do you mean by dizziness)??
98. What physical examination you do to narrow you differential diagnosis?
- MLA : Measure BP supine, sitting, standing
3- smoker patient failed nicotine patches treatment, next step?
Counselling
23) smoker failed to quit using nicotine patches, next step? Bupropion, council

4- patient on rescue salbutamol and max dose inhaled steroids, still not controlled, next step?
Add LABA
28) Asmatic patient on inhaled CS (high dose already) & short acting b agonist, not managed well.
What to do? LABA, oral steroids

5- diabetic with high BP on 3 settings and +ve microalbuminurea, drug of choice?


ACE-i
Agoraphobia means:
- fear of insects such as spiders
- fear of closed places
- fear of crowds

A woman is anxious and she had tried psychotherapy sessions, but she's still anxious. You will give
her medical therapy. The first line would be?
- SSRI

What's true about depression?


- Maximum treatment is reached at 2-8 weeks of therapy
- No need for a follow up appointment after taking medications
- Treatment should be given for 2 years after a single episode of depression

60 year old lady has MCP joints and PIP nodules (and stiffness I think)?
- Rheumatoid arthritis

2. Which of these diabetic drugs increases weight:


⎬ Metphormin
⎬ Acarbose
⎬ Sitagliptin
⎬ Glyclazide

4. When to worry and send the patient to the neurologist:


⎬ Weak planter flexion
25) trauma with bruise to lower back, on CT abdomen there is a kidney cyst, labs are normal.
Diagnosis: simple renal cyst
26) NNT: 50, 63
27) child can't sleep well due to anal itching, it is due to: ascariasis, pinworms, giardiasis
29) routinely for newly diagnosed hypertension with no previous investigations: EKG, renin
30) cause of weight gain: SU
31) uncontrolled severe asthma: FEV1/FVC ration less than 70%, FEV1/FVC ration is normal
32) asthmatic, should decrease exposure to dust & other indoor allergens by: increase humidity, not
washing beds
33) IBS treatment: mebeverin
34) IBS will show: mucus stool, hard to flush/ float stool, pale/ clary stool
36) 28 yo female not married, want to do cervical screening. What do you tell her: do HPV test, she
can't do it (out of the screening ppl)
�Otitis externa most common organism, s.aruginosa
�Malignant otitis media common in diabetic
�Instrument not used to measure the iop
�First line medication for ibs
�What is true about depression "medication need 6-��8wk to get the maximum effect"
�Menopause criteria
�Secondary screening: cervical cancer vs hypertension?
�HZV of the ear?
�Which side has the facial paralysis? Right?
�Mild OM what to do?
�Malingering 15 yrs old girl claims she cant see her mobile? Or hyperopia?
�Definition of malingering?
�next step in pts that has closed ear canal?

Which of the following Diabetic medications causes weight gain:


Acarbose
Metformin
Sitagliptin
Glyburide
- answer was Glyburide

Which of the following is correct regarding Combined oral contraceptive pills:


Increases risk of endometrial cancer
Decreases incidence of cervical cancer
Increases Risk of DVT
- Right answer was Increases Risk of DVT

Which medication induces ovulation and restores the monthly period in a patient with Polystic ovary
syndrome?
Metformin
Misoprostol
- Answer: Metformin

Male patient diagnosed with uncomplicated HTN what is the 1st line treatment?
Carvedilol
Amlodipine
Hydralazine
(not sure about the options, Can’t remember the options clearly

- 70 year old male had blunt trauma. On examination he has contusions on his lower back. CT
showed 1 kidney cyst on the right side measuring 2 cm. Both kidneys are of normal size. RFTs all
normal. Dx.? PCKD, simple renal cyst, RCC, recurrent UTI, prostatic nodular hyperplasia

Light receptor cells?


Recurrent epistaxis Pic of the mouth? Talengectasia
Young man with nasopharyngeal tumor + epistaxis what is the next step? Imaging
15 yo with Problem seeing mobile? Malingering

Angel closure glaucoma symptoms?


Number needed to treat ? 63

Properly applied pressure patch?


Cervical cancer in 28 yo not sexually active?
51 year old IUD + not having period for year?

1. CURB-65->used for assessment, abx and decision for admission/mortality


Confusion of new onset (defined as an AMTS of 8 or less)
Blood Urea nitrogen greater than 7 mmol/l (19 mg/dL)
Respiratory rate of 30 breaths per minute or greater
Blood pressure less than 90 mmHg systolic or diastolic blood pressure 60 mmHg or less
Age 65 or older
2. Becks triad: JVD, muffled heart sounds, low blood pressure
Again, simply asking what beck’s triad is
3. Cardiac arrest not caused by: hypocalcemia
Cardiac arrest can be caused by the following except…
4. Blood transfusion group type for certain patients: O+ for males and post-meno females
This is straight from one of the ppts. I think it should be O+ for males and post-meno
females, not O-
5. DKA presentation: diagnose from labs
The Q was a clinical picture of DKA with ketones in urine + other DKA lab findings and
asking you for the diagnosis.
6. SVT ecg identification
The picture they show will be clear, no worries. You just need to answer what type of
arrhythmia is the pic showing
7. Tx for unstable SVT: cardioversion
8. Hypovolemic shock
I can’t remember what this Q was about to be honest. It was probably about management
(ABC, IV fluids, identifying source of hypovolemia, etc)
9. Sepsis requirement at 6hrs, according to guidelines
This is exactly according to the lecture Dr. Pasha gave about sepsis. You must memorize the
requirements by heart, as the MCQs will directly ask about them.
10. CO poisoning: tx, high flow O2/HBO
11. Hemothorax X-ray: management, chest tube pointed downwards
The x-ray will be very clear and the clinical scenario made it obvious (I think it was an old guy
with a stab to the chest). Management is chest tube
12. Epidural hematoma ct: identify its epi
13. Trauma series: X-rays included
• Cervical Spine - AP
• Cervical Spine - AP Open Mouth (Odontoid)
• Cervical Spine - Lateral - Horizontal Ray
• Cervical Spine - Swimmers if C7-T1 not visualised
• Chest - AP
• Pelvis - AP

“Which of the following x-rays should be performed as a part of trauma series x-rays?” I
honestly don’t know the right answer for this one.
14. MI: 90mins for FMC to angio for accredited institutions
This Q was a case of MI and asking you about the ideal time for angio since first medical contact.
Answer is 90min
15. ABCDE wrong abbreviation: D for Danger
Which of the following is not correct about what ABCDE abbreviation stands for?
16. Facial palsy: management
Can’t remember this Q
17. Retinal detachment on fundoscope: diagnosis
They used the same pic in the ppt he gave about ophtha emergencies
18. Lyme disease after tick bite: presented with palsy, Borrelia burgdorferi
19. Colloid vs crystalliid
Can’t remember the Q
20. Post op pain assessment: best method
I think the Q was actually about best post op pain management method. The answer should be
patient controlled analgesia and that it results in best satisfaction.
21. Post op pain effects: common effects of pain/as a result of pain management
I can’t remember this Q
 Relieves suffering
 Early mobilisation
 Short hospital stay
 Reduced hospital cost
 Increased patient satisfaction

22. Airway preanesthesia assessment: answer was no single method is conclusive, use multiple
23. Difficult airway
Which of the following can be used for assessing difficult airway? Mallampati score
24. NS excess causes: hyperchlremic acidosis
25. FFP for: low platelets
26. Best way to monitor fluids: dynamic
27. CPAP: indication spontanuos breathing
Can’t remember this Q
28. Morphine which of the following isn’t a side effect: hallucinations
29. Stent patient preanesthesia: its worrying because they stay on long term blood thinners
30. Hypovolemic shock: management
31. Ectopic pregnancy: what to do next if u/s –ve and urinary hCG is low/negative
Do blood hCG
32. Ambubag: indication
I don’t remember this Q
Inadequate oxygenation or ventilation, as bridge to intubation
33. PCA advantage: better patients satisfaction
34. Hyphema presentation: management raise head+analgesia
They will show you a pic of hyphema. Same one from PPT
35. Spinal anesthesia indication: surgeries below T11
36. Do no harm: Non-maleficence
Which principle does “Do no harm” refer to?
37. Xray of volvulus
Coffee bean appearance. Same as ppt
38. Xray of Small Bowel Obstruction
39. Xray: Perforated hollow viscus/viscera
Air under diaphragm
40. V. fib, after defib: what are the medications to use, epi+amio

41. What is the best way to measure body fluids intra-operatively?I forgot its name but it was the
one where u monitor it using a central cath and measure variations. It’s from an SGT with dr.
Geddawi
42. which of the following is true in primary survey ? (chin life & jaw thrust ?)
43. patient presents with signs of instability (HR:190, BP:70/40,etc) with an ECG picture showing
ventricular tachycardia, how would you manage this patient ? synchronised cardioversion
44. what is true about the use of packed RBCs ? (they are checked by 2 medical staff, reaction can
be predicted in some donors, etc ..)
45. patient presents without pulse and with an ECG showing Ventricular Fibrillation, how would you
manage ? 30:2 and synchronised shocking
46. the drug combination mostly being used in shockable rhythms is : amiodarone& epinephrine
47. what is the best airway management device for respiratory failure ? Bi-PAP/ Intubation
48. What is true about the 4:2:1 formula ? (it is not used for pediatrics, i don’t remember the other
answers)
49. What is the step that has to be performed within the 1st 3 hours in suspicion of sepsis ? lactate
measurement
50. patient presented with gun shoot wound in his lung, the next step in management after
primary survey is ? putting in a chest tube
51. picture of patient who lost his vision while being in the cinema, he wears glasses (Glaucoma)
==>pilocarpine
52. 25 year old with sudden onset left sided face weakness, what is your next step in management ?
(acyclovir, CT, etc )
53. what is the most common complication of post op pain ? (bradyarrythmias, etc ??)
54. picture of an ECG with torsade, what is the management ? Magnesium Sulphate
55. in a patient where “allow natural death” is used, what of the following can he be given? (feeding
tube)
56. female patient presents with symptoms of vaginosis ==> (Metronidazole, doxicycline and
Ciprofloxacin are the combination of choice)
57. Patient with facial nerve paralysis. You look in his ear you find vesicles. What is the diagnosis
and treatment?

MLA: Ramsey Hunter Syndrome; Tx with antiviral and steroids


58. A drug that every HF patient should be on:

MLA: ACE-I
59. What is a drug recommended for every MI patient:

MLA: cannot recall (maybe nitroglycerin or morphine)


60. What is true about bariatric surgeries?

MLA: Nutritional deficiencies can be replaced with supplements


61. 60 yr old lady presents with changing bowel habits and some cramps with bleeding and
stuff?

MLA: Advice colonoscopy as the colon CA risk increases above 60 yrs of age. (there is
another answer stating ‘reassure as this likely IBS’ but age was worrisome for me)
62. Young male with bloating, cramps and changing bowel habits:

MLA: IBS
63. Patient is diabetic and develops hypertension. Tx is:

MLA: ACE-I
64. Africo-caribean patient develops HTN. Tx is:

MLA: Calcium Channel Blocker


65. A case of torsion. Which is correct?

MLA: Early intervention is 90-100% successful


66. A case of headache (description is that of tension) for which patient is now using around 200
pills a month:

MLA: Chronic daily HA (medication overuse headache)


67. Question about prevention

MLA: cannot recall the choices but know what Primary, Secondary and Tertiary prevention
stand for.
68. A question about vaginal discharge with fishy odor:

MLA: Bacterial vaginosis. Tx metronidazole


69. Patient with mild asthma that only requires rescue salbutamol presents now with worsening
of symptoms?

MLA: add inhaled steroid


70. Mononucleosis typical case; question about the virus  EBV
71. Pregnant ( I think 32 weeks) came with positive urine E-coli what to do for her 
Antibiotics.
Choices were:
1. Follow up
2. Antibiotics
3. Others

72. What is the most likely organism that causes deep neck space infection?

MLA: Polymicrobial

73. Patient with Cardiac abnormalities had stroke, which of the following is the likely cause?

MLA: Atrial Fibrillation

74. What is the most common cause of a sudden limp?

MLA: Trauma

75. Which of the following agents decreases weight?

MLA: GLP1

76. 55 year old patient with heart burn after meals, which is your first line investigation?
MLA: The majority of us put Endoscopy, some put Urea breathe test. We are still not sure
who is right.
99. 1- Glargine; longest duration available in KFSH, not sure.
100. 2- Back pain; evidence-based treatment? massage?/exercise?
101. 3- Red flag for back pain; weak leg dorsiflexion (I put this)? or saddle anesthesia?
102. 5- BCG; live attenuated
103. 7- which of the following is most effective regarding smoking cessation? advise the patient
every visit, not sure.
104. 9- IBS: caffeine
105. 10- IBS in > 50yo, rule out ovarian pathology
106. 11- Tertiary prevention; easy example.
107. 12- Cervical cancer screening question
108. 13- If a drug is proven to lower LDL but does not affect patient’s life, this is an
109. example of? Disease oriented study (not patient oriented)
FM:

10) epistaxis with pic of lip & tongue have dots: hereditary hemmoragic telangectasia, SLE
11) tongue: kaposi sarcoma, hairy leukoplakia, SCC
12) Treatment of choice for the previous patient? Surgery, surgery with neck dissection
13) Acute vestibular neuritis: corticosteroids
14) mild OM with temperature 37.2: antibiotics, analgesics
19) respiratory depression post thyroidectomy, manage: tracheotomy, MRI
20) malignant otitis media (or was it externa?): MC in diabetics
23) smoker failed to quit using nicotine patches, next step? Bupropion, council
29) routinely for newly diagnosed hypertension with no previous investigations: EKG, renin

Family Medicine: (AMB qs doc)


1. Young male with bloating, cramps and changing bowel habits:

MLA: IBS
2. Africo-caribean patient develops HTN. Tx is:

MLA: Calcium Channel Blocker


3. A case of headache (description is that of tension) for which patient is now using around 200
pills a month:

MLA: Chronic daily HA (medication overuse headache)


4. Question about prevention

MLA: cannot recall the choices but know what primary, secondary and tertiary prevention
stand for
5. A question about vaginal discharge with fishy odor:

MLA: Bacterial vaginosis. Tx metronidazole


6. Patient with mild asthma that only requires rescue salbutamol presents now with worsening
of symptoms?

MLA: add inhaled steroid

13- If a drug is proven to lower LDL but does not affect patient’s life, this is an
example of? Disease oriented study (not patient oriented)

FM: (Last Year Questions modi doc)


1. Female with HTN and facial acne and hirsutism  cushings (if no HTN, PCOS)
2. Vaccination to a woman in a prenatal visit with no rubella antibodies should be AFTER
delivery (right after)
3. Best treatment of low HDL: 1st exercise (best drug: niacin)
4. Best treatment of high LDL: statin
5. Megakaryocytic anemia: folic acid def
6. Pancreatitis presentation
7. HBV + surface antibodies and negative antigens  good immunity against HBV
8. Stretching as a treatment to neck or back pain
9. Suspect low PTH in post-thyroidectomy pt with hypocalc
10. EBV (mono) in 16 yo pt with splenomegaly and lymphadenopathy
11. OCP inc risk of DVT
12. Asthma treatment (stepwise)
13. Treatment of depression and treatment of anxiety (know first lines)
14. Agoraphobia: fear of crowds
15. anti depression meds will start to be effective in 2-8 weeks
16. know how to calculate number needed to treat
17. know the tables in the preventive medicine lecture (they asked 3 or 4 questions about that
in MCQs and in OSCE).
18. RA presentation
19. DMPA Given every 3 months
20. How do u help with the infertility in PCOS (I don’t know, I answered metformin)
21. Frozen shoulder presentation
22. Chronically constipated, anal pain and itching (either hemorrhoids or fissures, they were
both options)

Girls 2015
Family medicine:

1- 21 lady just came from Africa and diagnosed with malaria what is the most specific test to make
the diagnosis of malaria: thick and thin blood film
2- 71 old man diabetic , feel dizziness when he wakes up form the bed for the last 7 days he had
syncope , he is on aspirin , frusemide and beta blocker what is the cause : orthostatic
hypotension (I'm not sure)
3- 6 yr old girl came to the ER with nausea and vomitting, abdominal pain, you smell a sweet odour
from her mouth what is the diagnosis ?
Diabetic ketoacidosis
4- Patient with cor polmnale what is the cause : pulmonary fibrosis , pneumonia
5- Diabetic pt put on lifestyle and exercise < didn't work wt to do next > metfor
6- Diabetic patient ii, hyperlipidemia, HTN and microalbuminuria > ACEI *lisinopril
8- 16 yo with 6 day history of fatigue develops cervical lymphadenopathy and later splenomegaly
what is the most likely causative organism
Adenovirus
CMV
EBV
9- 16 year old lady got shortness of breath after taking penicillin.
Answer: intramuscular epinephrine
10- 58 year old male has history of hypertension and wants to reduce his risk of developing an MI.
His HDL is 32 (normal is 40). What can increase his HDL?
Answer: Exercise
Mayoclinic says: Get more physical activity. Within two months of starting, frequent aerobic exercise
can increase HDL cholesterol by about 5 percent in otherwise healthy sedentary adults.
11- 50 yo with 2 hour history of chest pain has been given aspirin his blood pressure was 155/85
what is the most appropriate drug that should be given next
Carvidelol
An ACEI
Morphine
Furosemide
12-familial cholesterolemia exercise not helpful, LDL more than 6 what to do?
Simvastatin
13- healthcare worker stuff, about vaccinations his hep B profile: negative surface antigens ,High
antibodies ,what's his status?
Has protection against hep B
14- man with fatigue and shortness of breath Hb is 7.5 macrocytosis what type of deficiency?
Folic acid
15-Drug which reduces FBG and HBA1c. Is it disease or patient oriented?
Disease oriented

16-Diabetic patient with 3 readings of 160/100, what to do next?


Start ACEI

17-Woman with no antibodies to rubella, comes for a prenatal visit, when do you give the rubella
vaccination?
Immediate postpartum period
18-Child with recurrent chest infections, malabsorption symptoms, chronic sinusitis, and nasal
polyps. What test would you do to confirm your suspected diagnosis?
Sweat chloride test

19-Trauma patient. What stain should we use?


Flourescein stain

20-How can you be cost effective in family practice?


Options included: generic prescription, drug formulations with multiple drug.

110. is itchy --> Scabies


111. When to remove copper coil -> 5 or 7 years? I guess copper = 10, progesterone – 3-5 yrs
https://www.verywell.com/what-to-expect-during-an-iud-removal-906771

112. Risk for breast cancer -> alcohol consumption


113. Which of the following is true about mammograms -> low radiation, painful???
114. Supplementation of which vitamin has known health benefits -> Vitamin D
115. Which of the following can be given to a 62 y/o woman to decrease fracture risk ->
alendronate ( just to point out that the guidline is for >65yr old women)
116. CVS risk assessment in a low risk individual and some tests were already done and asking
what other tests to do? -> do nothing
117. Diabetic develops HTN, first line drug to treat his HTN -> ACEi
118. Man with HTN is on ACE i, develops chronic cough so doctor changes his medication. Which
medication did he change it to -> Losartan (ARB)
119. 1st line drug for the treatment of newly diagnosed DM -> Metformin
120. First line treatment for 1ry dysmenorrhea -> NSAIDs
121. Screening for AAA with US -> 67 y/o male smoker
122. Black male develops HTN, what do you prescribe -> CCB
123. Pubmed is an example of -> 1ry source
124. Calculate Number Needed to Treat (NNT)
125. First step in the management of OSA -> weight loss
126. Complication of OSA -> CHF
127. Having difficulty communicating with patient, what should you do? Prevent pt from going off
topic / open ended questions
128. Single blood test that would be expected to be the most sensitive for determining whether
the patient is euthyroid, hypothyroid or hyperthyroid? -> TSH
129. A nurse sustained a needle stick injury while assisting in a procedure at the clinic yesterday.
The patient’s status is unknown for any blood-borne illnesses. The nurse is vaccinated
against Hep B. Which of the following is the most appropriate next step? Investigate both the
nurse and patient, for blood-borne illnesses
130. Lady feels fatigued for the past few years, swollen red PIP and DIP with nodules of elbow -->
RA
131. Diagnostic test for gout -> joint aspiration
132. Lady with amenorrhea, weight gain, etc. -> Hypothyroidism vs Cushing’s Bc with
hypothyroidism its more of mennorhagia not amennorhea
133. Asthmatic on ventolin, has exacerbations at night, next step? -> add low dose inhaled CS
134. Patient presents with right lower quadrant pain that started off as periumbilical pain,
anorexia, N&V (acute appendicitis), next step? -> immediate admission and surgical consult
135. Which of the following would indicate poor DM control -> HbA1c of 10%
136. Kid has 3 months hx of abdominal pain, now he can’t attend school, which of the following
would hint towards an organic rather than a functional cause -> headache, anxiety,
suprapubic pain, family hx of abdominal pain?? IBS
137. Facial Nerve palsy -> treat with steroids
138. A patient develops headache + neck stiffness with NO fever -> subarachnoid hemorrhage
139. Hx of man with rash on hands, arms, buttocks, son has similar presentation, rash is itchy -->
Scabies

1. During end of life care in a palliative DNAR patient, an emergency Physician may:
a. Keep foleycath (urinary) as there Is no long term risk of infection
b. Initiate inotropes and pressors
c. Initiate tube feeding and pain control
d. Refer to surgery and transplant
e. Intubate and mechanical ventilation

2. The loss of a loved one results in grief as universal human behavior, The following Is the most
appropriate statement regarding the stages (denial, anger, bargaining, depression, and
acceptance):
a. Everyone must go through the stages in order
b. Medications should be used to shorten the grieving process
c. One may go through grieving process without passing through all stages of grieving
d. The stages of grief have been validated in randomized control trials
e. Normally, grief should be over by 1 month

3. In a patient with pneumonia, the CURB 65 score will assist you with:
a. Choice of antibiotics drug therapy
b. Need for hospitalization and mortality risk
c. c. Follow up appointment with primary service
d. d. Need for lab orders and ABG
e. e. Intubation decision and early goal directed therapy

4. The Beck’s triad includes the following elements:


a. Low SBP, absent breath sounds and raised JVD
b. Muffled heart tones, decrease BP, increase JVD
c. c. Hypercoagulability, stasis, vessel injury
d. d. Jaundice, fever, R upper abdominal pain
e. e. Recurrent oral ulcers, genital ulcers, iridocyclitis

5. 20 year old man presents to ER after a fall from a horse. He is confused on arrival to ER but
localizes pain on stimulation. You assess him to be a case of mild head injury with GCS of 13/15.
Your next step is to:
a. Consult neurosurgery
b. Intubation
c. ABCDE
d. C-ABC and CT brain
e. Trauma code and admit for observation

6. A 15 year-old female was outside collecting flowers when she was stung by a bee. She reports
being stung once before without any complications, but now she is having generalized urticarial
rash, swelling and dyspnea. Which one of the following is the next most appropriate stat
treatment?
a. Hydrocortisone 100mg IV
b. Diphenhydramine 25mg IV
c. Ranitidine 50mg IV
d. Epinephrine 1:1000 0.5mg IM
e. Epinephrine 1:10,000 1mg IV

7. 80 yrs old man with chest pain suddenly collapsed in CCU. The cardiac monitor showed the
following rhythm

In shockable cardiorespiratory arrest resuscitation, which of the following drug combinations


are used:
a. Atropine & Adrenaline (epinephrine)
b. Atropine & Amiodarone
c. Vasopressin & bicarb
d. Adrenaline(epinephrine) & amiodarone
e. Adrenaline(epinephrine) & bicarb

8. You are the team leader in a cardiopulmonary resuscitation team attending an arrest in ER. The
patient is in the following rhythm on cardiac monitor

You decided this is a shockable rhythm and you start chest compression with ventilation. Which
of the following is most appropriate next step?
a. Defibrillation @ 100 Joules
b. Defibrillation @ 200 Joules
c. Synchronized cardioversion @ 100 Joules
d. Synchronized cardioversion @ 300 Joules
e. Periodic chest thump

9. 20 yrs old male, known leukemia with recent chemotherapy presented to ER with fever. He was
diagnosed to be in SEPTIC SHOCK secondary to chest infection.
As per Surviving Sepsis Campaign Care Bundles which one of the following is essential to deliver
during first 3 hours of resuscitating a septic patient?
a. Measure lactate level
b. Central and arterial line placement
c. Perform chest x-ray
d. Administer inotropes
e. Administer 50 mL of 50% dextrose to prevent hypoglycemia

10. 30 yrs old female, known breast cancer patient with recent chemotherapy presented to ER with
chest infection and diagnosed to be in SEPTIC SHOCK

As per Surviving Sepsis Campaign Care Bundles which one of the following is essential to deliver
during first 6 hours of resuscitating a septic patient?
a. Prevent hypocalcemia
b. Maintain temperature > 36°C
c. Mean arterial pressure (MAP) ≥ 65mmHg
d. Keep Hb above 10 gm/dL
e. Start dopamine @ renal dose

11. After performing a proceducre in the emergency room department- the next most important
step is to:
a. Documentation and taking pictures
b. Inform the family
c. Teaching the staff
d. Checking if the procedure is done right
e. Taking a quick break before attending next patient

12. 80 yrs old female from nursing home with history of chronic laxative use presents with sudden
onset of abdominal pain with distension. She has absolute constipation with no passage of
flatus. Her acute abdominal series x-ray reveals the following:

What is the diagnosis?


a. Volvulus
b. Large bowel obstruction
c. Bowe; perforation
d. Small bowel obstruction
e. Gall stones

13. Most appropriate statement regarding triage is:


a. There is one universal system
b. It is a foul-proof system
c. First come, first serve basis is maintained
d. Immediate action may be provided to the sickest
e. CTAS/ESI/START are all the same

14. 76 yrs old female with nausea, vomiting, and lower abdominal pain diagnosed with recent UTI
presented to emergency room with h/o being unwell for few days. O/E patient severely
dehydrated, tachycardic and tachpnoeic. Her HR: 136 BP: 110/70, R/R: 40, SAO2: 98%, She has no
drug allergies.

LABS:
Na: 130 mmol/L Cl: 95 mmol/L
K: 5.5 mmol/L HCO3: 20 mmol/L
Glucose: 30 mmol/L
Urinalysis:
Blood: -ve Nitrogen: +ve Leukocytes: +ve
Protein: -ve Ketones: -ve

The next most appropriate management plan is to:


a. Ciprofloxacin IV and NaCl 0.9% 1 liter IV
b. Glucophage (Metformin) 500 mg BID
c. A & B
d. Insulin 10 units IV and NaCl 0.9% 2 liter IV with ciprofloxacin 400 mg IV
e. Insulin 10 unties SQ and Glucophage 500 mg PO now

15. 36 yrs old female after recent thyroid surgery presented with h/o fever, nausea, vomiting,
restlessness, agitation and extreme confusion. Her observations revealed
Temp: 40°C Pulse: 140/min
BP: 190/110 mmHg R/R: 36/min O2 saturation: 94%
Which of the following is used for acute treatment of above condition?
a. Aspirin
b. IV propranolol
c. IV thyroxine
d. IV hydrocortisone
e. IV acetaminophen

16. 20 yr old male unconscious is brought to emergency department by medics. After ABC, the
next most appropriate intervention is:

a. Normal saline bolus 30 ml/kg


b. Thiamine 100 mg IV, Narcan 1 mg IV, D50 1 amp IV
c. Start CT brain
d. Intubation, activated charcoal
e. Drug toxscreen, ETOH level and gastric lavage

17. 50 yrs old burn victim develops fever, tachycardia, tachypnoea while receiving treatment on
burns ward. You make a diagnosis of SIRS. Which of the following is NOT an essential element
for diagnosis of SIRS (Systemic inflammatory response syndrome)?

a. Temp less than 36°C(96.8°F) or greater than 38°C(100.4°F)


b. Heart rate greater than 90 beats per minute
c. High lactate of 2 mmol/L
d. Tachypnea greater than 20 breaths per minute
e. Leukocytosis or leukopenia
18. A 33-year-old obese man comes to the emergency department because of abdominal pain
nausea and vomiting. O/E rebound tenderness and guarding in Rt iliac fossa. You provisionally
make a diagnosis of acute appendicitis. Which of the following will be essential before a patient goes
to OR?
a. Labs and overnight NPO
b. Labs and CT scan of abdomen and pelvis with contrast
c. Antibiotics, US abdomen, IVFs
d. SAMPLE history, exam, labs surgical consent and surgical consult
e. SAMPLE history, exam, CT scan and surgical consent

19. A 33-year-old man suffered head injury after falling from a roof at work. His initial GCS was
9/15 with unilateral dilated pupil. The CT brain revealed the following:
What is your diagnosis?
a. Epidural bleed
b. Subdural bleed
c. Intracerebral bleed
d. Cerebral contusion
e. Basal skull fracture

20. What is the etiology of the following finding on CT scan after sustaining a head injury

a. Epidural bleed
b. Subdural bleed
c. Intracerebral bleed
d. Cerebral contusion
e. Basal skull fracture

21. 36 years old male after recent flight from Australia to Saudi Arabia presented to ER with sudden
onset of SOB. His obs were as follows: Afebrile, P: 123/min, RR: 40/min, BP: 90/60mmHg, SaO2:
90%. Labs: Hb 14, WCC: 9.0. D-dimer: 600 (high). Which of the following will confirm your
diagnosis?
a. Chest x-ray AP/lat & EKG
b. Ultrasound venous Doppler lower limb
c. ABG and checking A-a gradient
d. Blood culture and sputum gram stain
e. Chest CTPA

22. 40 years old farmer presents with reduced GCS, salivation, lacrimation, urination, diarrhea, GI
upset and emesis. He has been using a new insecticide spray today. O/E: P: 52/min, BP:
110/60mmHg, RR: 36/min. Pupils are small on examination. What is the diagnosis?
a. Opioid poisoning
b. Organophosphate toxicity
c. Cocaine toxicity
d. Salicylate poisoning
e. Lead poisoning

23. What is the antidote for iron toxicity?


a. N-acetylcysteine/parvolex
b. Atropine
c. Activated charcoal
d. Ethanol
e. Deferoxamine

24. In cases of (AMI) acute myocardial infarction, the target door to balloon time in a chest pain
accredited centre is
a. 30 mins
b. 60 mins
c. 90 mins
d. 120 mins
e. 3.5 hours

25. 25 year old young man presents with sudden onset of complete weakness of one side of the
face. The clinical examination reveals as follows

What is the most appropriate ER management?


a. Reassure, start acyclovir/steroids/artificial tears
b. Immediate CT scan of brain/ seniors thought this was the answer, however
COMPLETE weakness of one side of face = LMN injury, hence possibly Bells palsy
which is treated with steroids
c. Immediate MRI with referral to neurology
d. Stroke thrombolysis
e. Immediate referral to ENT

26. 30 yrs old obese female presented with sudden onset of fever, severe RUQ abdominal pain,
nausea, and vomiting. You ordered basic labs
Labs:
Hb: 12 g/dL WBC: 17.5 x 109/L
Platelets: 110 x 109/L Lipase: 46 U/L (N) AST: 90 U/L (H)

What is the investigation of choice?


a. CT scan abdomen without contrast
b. Acute abdominal series x-ray
c. USS abdomen
d. CT scan abdomen with contrast – Note CT is diagnostic for appendicitis!
e. Barium swallow

27. The optimal head position for intubating the patient under anesthesia is
a. Extension of the neck
b. Flexion of the neck and extension at atlanto-occipital joint.
c. Flexion of the neck, extension at the atlanto-occipital joint and jaw thrust
d. Extension of the neck and jaw thrust
28. Spinal anesthesia can be considered for:
a. High risk patients
b. Surgery above the umbilicus
c. Laparoscopic surgery
d. Patients on Plavix
e. Agitated patients

29. Multimodal approach to treat postoperative pain means:


a. Give different groups of medications that attack pain from different angles
b. Use the maximum tolerable dose of pain killers
c. Consult different specialties to reach the best possible treatment approach
d. Anesthetist has to handover all the names and doses of the intraoperative painkillers
used to recovery nurses

30. Using high volumes of normal saline 0.9% can lead to:
a. Respiratory acidosis
b. Metabolic alkalosis
c. Respiratory alkalosis
d. None of the above
Actually causes hyperchloremic metabolic acidosis

31. As a health care provider, we must at least act and not hurt any patient. This is referred as:
a. Beneficence
b. Maleficence
c. Non beneficence
d. Non maleficence
e. Standard of care

32. 70 years old male presents to ED with acute symptoms of stroke. The patient is a candidate for
stroke thrombolysis (tPA) if:
a. Window of 6 hours
b. NIH stroke scale score >25
c. He has an INR > 3.5
d. Blood glucose is <50 mg/dl (2.7 mmol/L)
e. Head trauma 4 weeks ago
Though we were conflicted about the “4.5 hrs rule”, all other options are clear
contraindications from Dr. Wagleys stroke lecture

33. 20 years old male presents to ED with decreased mental status and agitation. He was found with
empty bottle of tricyclic antidepressants taken 3-4 hours ago. After his initial resuscitation you
have decided to counteract the actions by:
a. 10 gm activated charcoal
b. Atropine
c. Sodium bicarbonate
d. Narcaine and thiamine
e. N Acetylcysteine (NAC)

34. 20 years old male with recent history of alcohol use presented to emergency room with sudden
loss of consciousness. On the monitor you note the following rhythm
A medication that may resolve this rhythm is:
a. Atropine
b. Adenosine
c. Magnesium sulphate
d. Calcium chloride
e. Ephinephrine

35. 21 years old married female presents to ED with 2 hours history of right LQ pain. She denies any
urinary symptoms or vaginal bleeding. You check a urine point of care pregnancy test and it is
negative. The next most appropriate step would be to:
a. Check serum BHCG quantitative, IVF and pain control
b. Check pelvic US, R/O
c. Order CT scan to R/O appendicitis
d. Treat her symptomatically, reassurance and discharge home
e. Acute abdominal screening x-rays to r/o bowel obstruction

36. A 21 years old male after a recent cruise holiday presents with fever, flu like symptoms, inguinal
lymph nodes and painful genital blisters and sores. Most prudent treatment option is
a. Metronidazole, doxycycline & ciprofloxacin
b. Diflucan
c. Azithromycin & rocephine
d. Famcyclovir
e. Clindamycin gel

37. All of the following are excited to read neurotransmitters that modulate pain except:
A. Substance P
B. ATP
C. Somatostatin
D. Glutamate
E. Aspartate

38. A female presents to the ER with overdose of beta blocker and suicidal ideation, with unstable
heart block, which one of the following managements is least likely to help her:
A. Glucagon
B. Calcium gluconate
C. Hemodialysis MPA
D. Cardiac pacing (patient is unstable which excludes using pacing at this stage :/)
E. Dopamine
All the above can be done in BB toxicity apparently
39. According to ASA guidelines which one of the following is not used intraoperatively for
monitoring:
A. Oxygen analyses
B. ECG continuous
C. BP
D. Peripheral nerve stimulator (Toronto notes; Pg 6 - Monitoring)
E. Capnometry

40. Mallampati classification when you cannot see the soft palate:
Ans: class IV

41. Which one of the following is not a risk factor for difficult bag mask ventilation:
A. Thyromental less than 6 cm
B. Thyro angle more than 9 cm
C. Mallampati 1
Answers don’t make sense

5 independent risk factors of difficult mask ventilation:


– Age > 55 years of age
– BMI > 26
– History of snoring (obstruction)
– Beard
– Edentulous

42. The following are independent risk factors for bag mask ventilation except:
A. BMI < 26
B. Edentulous
C. Beard
D. Snoring
E. Short neck
43. Oropharyngeal airway feature:
A. Bite block
B. All same colors
44. What is true about LMA:
A. Secure 100% airway
B. Had to be laryngyscope mediated
C. Less traumatic than ett
D. Non fasting patient usage
45. A patient is found unconscious outside the ER with needle marks in his arm and pinpoint pupils:
antidote? Vitals
A. Flumazenil
B. Naloxone
C. Atropine
D. Cocaine
46. Gold standard for ETT confirmation:
A. Capnohraphy
B. Visualization of vocal chords
C. Chest X ray
D. Auscultation
E. Chest movement

47. Narcotics:

A. Morphine is highly lipid soluble


B. Intrathecal morphine is more respiratory depressant than fentanyl
C. The mu receptor is G protein coupled
48. Opioid side effect except:
A. Nausea vomiting
B. Bradycardia
C. Constipation
D. Resp depression
E. Midriasis

49. How frequently should Hr and BP be measured in OR according ASA guidelines:


A. 2 minutes
B. 5 minutes
D. 7 minutes
C. 10 minutes
D. 15 minutes

50. Which is a feature of IV fluids:


A. Colloid stays intravascular unless capillary leak

51. A patient started having symptoms of stroke 2 hours ago. What is the time frame to get a non
enhanced CT scan for this patient? Dr Wagleys lecture, there is a picture with the times)
a. 10 minutes
b. 15 minutes
c. 25 minutes
d. 45 minutes
e. 60 minutes
140. anticholenergic overdose antidote: glucose and thiamin
141. chest tube for hemothorax
142. post anesthesia care with high HR & BP & RR, and agitated what do we give
143. why do we give pre oxygenation before intubation, possible answer is more time for apnea
144. GCS = 13
145. compensated respiratory acidosis values
146. not essential for diagnosis of SIRS: lactate
147. what affect the oxygen transport more: cardiac output
148. induced by propafol and was Bevin curonium then developed bilateral wheezing and
hypotension
149. documentation to communicate with other doctors
150. cause of cardiac arrest in renal failure patient <-- hyperkalemia
151. which is not associated with high oxygen therapy
MCQs
1) X-ray picture. What is the diagnosis? Volvulus.
2) X-ray picture. What is the diagnosis? Small intestinal obstruction.
3) X-ray picture. What is the diagnosis? Perforated viscus.
4) Patient with signs and symptoms. An X-ray is shown. Immediate initial treatment is needle
thoracocentesis (since the X-ray shows tension pneumothorax).
5) A patient (only lab tests are shown), what is the diagnosis? DKA (since there were positive
ketones in urine ++++).
6) Which combination of medications is used in ventricular tachycardia or fibrillation?
Adrenaline and amiodarone.
7) A patient was bitten by a deer tick, what is the likely organism? Burgdorferi.
8) A patient with COPD. Check previous culture.
9) A patient with thyroid storm, what is the most important immediate treatment? IV
propranolol.
10) A patient with certain neurological disabilities, what is his glassgow coma scale? 13.
11) A patient with hemothorax. Do chest drain.
Which blood type can be given for elderly women and -----------? O+ blood.
Toronto ER4: O –ive blood for women of child bearing age, O + men (maybe for
postmenopaudal women too – this isn’t from toronto)

152. Which one regarding blood transfusion is correct: O- for children and women of child-
bearing age (to avoid immune reaction if we give O+ in females with O- group of child bearing
age). O+ can be given to adult males and post-meno females (Rh incompatibility)

12) Which of the following can be done to a DNAR (Do not attempt resuscitation) patient?
Morphine
13) 62 years old male attended ER in CardioRespiratory Arrest . Following Rhytm was seen on
Cardiac Monitor Which of the following Drug should be given during ACLS Resuscitation?
Adrenaline(Epinephrine)

14) In Shockable Adult CardioRespiratory Resuscitation which of the following is the correct
ratio between chest compression and ventilation plus type of defibrillation?
30:2/ Asynchronous Defibrillation

153. All of the choices can cause Cardiac arrest except: hypocalcemia
154. Person presented with abdominal pain, nausea/vomiting, lab showed ++++ urine ketone,
diagnosis: DKA
155. Question with an ECG strip, identify the pattern: Supraventricular tachycardia (SVT)
156. Tx for unstable SVT: cardioversion
157. Patient presented with symptoms of shock after losing blood following an accident, what is
this condition: Hypovolemic/hemorrhagic shock
158. Trauma series: X-rays of cervical spine, chest and pelvis included
159. ABCDE is used for the primary survery in ATLS, which one of the letters are wrong: D for
Danger
160. Facial palsy treatment: Corticosteroids and antiviral/acyclovir (ramsay-hunt Sx)
161. Picture of fundoscopy, diagnosis: Retinal detachment
162. Patient with first toe finger joint pain (proximal phalanx), urine sample showed –ve
birefringent crystals, diagnosis: gout
163. Hemophilia A pt presented with knee swelling/hemarthoses, management: give factor viii
immediately (only in severe cases joint aspiration is done, that too only after coag factor
correction)
164. Question about Colloid vs crystalloid: colloid theoretically last longer in vessels due to high
protein content
165. Best method for Post op pain assessment: objective test using numerical values or
sad/happy faces if it’s a child patient.
166. Post laparotomy op pain management is important due: normally coughing will increase
pain; therefore patients avoid coughing, decreasing O2 levels.
167. Best method for Airway preanesthesia assessment: no single method is conclusive, use
multiple
168. Causes of difficult airway: arched palate, micrognathia, no teeth, denture wearing, beard,
ect… (I forgot the exact choices, but these are the correct answer)
169. NS excess causes: hyperchloremic acidosis
170. FFP is used for: low platelets (shouldn’t it be low coagulation factors?)
171. Best way to monitor fluids: dynamic monitoring rather than static
172. Indication to use CPAP: Obstructive sleep apnea
173. Which of the following isn’t a side effect of morphine: hallucinations?
174. Why should we be cautious in the history taking in preanesthesia of a patient with a history
of coronary stent: because they stay on long term blood thinners
175. Hypovolemic shock management: IV crystalloid (3 liters of crystalloid per 1 liter of blood
loss)
176. What to do next if u/s –ve and urinary hCG is low/negative in a lady of child bearing age with
SEVERE lower abd pain: repeat hCG (ectopic pregnancy until proven otherwise).
177. What is the indication to use an ambubag: acute emergency cases, such as post-trauma.
178. What is the advantage of patient controlled analgesia: better patients satisfaction
179. Patient presented with blood in the anterior chamber/cornea, management: raise head of
bed+analgesia
180. Spinal anesthesia indication: below lumbar, C/S is typical
181. What is the best way to measure body fluids intra-operatively ? Dynamic Vs. Static
182. which of the following is true in primary survey ? (chin lift & jaw thrust ?)
183. a patient had decrease in O2 saturation post operatively, what is the next step in
management ? (mask, intubation, .. )
how can you differentiate between cardiac temponade and tension pneumothorax ? breath
sounds
184. patient presents with acute otitis media, what is the drug of choice ? Amoxicillin, Augmentin
185. patient presents with signs of instability (HR:190, BP:70/40,etc) with an ECG picture showing
ventricular tachycardia, how would you manage this patient ?
synchronised cardioversion
186. what is true about the use of packed RBCs ? (they are checked by 2 medical staff, reaction
can be predicted in some donors, etc ..)
187. patient presents without pulse and with an ECG showing Ventricular Fibrillation, how would
you manage ? 30:2 and asynchronised shocking
188. what is the best airway management device for respiratory failure ? Bi-PAP
189. What is true about the 4:2:1 formula ? (it is not used for pediatrics, i don’t remember the
other answers)
190. patient presented with gun shoot wound in his lung, the next step in management after
primary survey is ? putting in a chest tube
191. What is the drug used to antagonise paracetamol toxicity ? N Acetylcystine
192. picture of patient who lost his vision while being in the cinema, he wears glasses (Glaucoma)
==> pilocarpine
193. 25 year old with sudden onset left sided face weakness, what is your next step in
management ? (acyclovir, Steroids! CT, etc )
194. what is the most common complication of post op pain ? Acute Confusion
195. picture of an ECG with torsade, what is the management ? Magnesium Sulphate
196. in a patient where “allow natural death” is used, what of the following can he be given ?

(feeding tube)

197. female patient presents with symptoms of vaginosis ==> (Metronidazole, doxicycline and
Ciprofloxacin are the combination of choice)
FM: (Last Year Questions modi doc)
1. Female with HTN and facial acne and hirsutism  cushings (if no HTN, PCOS)
2. Vaccination to a woman in a prenatal visit with no rubella antibodies should be AFTER
delivery (right after)
3. Best treatment to lower HDL: 1st exercise (best drug: niacin)
4. Best treatment of high LDL: statin
5. Megaloblastic anemia: folic acid def
6. HBV + surface antibodies and negative antigens 
good immunity against HBV
7. neck or back pain Stretching as an initial managament
8. Suspect low PTH in post-thyroidectomy pt with hypocalcemia
9. EBV (mono) in 16 yo pt with splenomegaly and lymphadenopathy
10. OCP inc risk of
DVT
11. Treatment of depression and treatment of anxiety (know first lines) SSRIs for all
12. Agoraphobia:
fear of crowds
13. anti depression meds will start to be effective in 2-8 weeks
14. know how to calculate number needed to treat
15. know the tables in the preventive medicine lecture (they asked 3 or 4 questions about that
in MCQs and in OSCE).
16. RA presentation
17. Depot medroxy progresterone? Given every 3 months
18. How do u help with the infertility in PCOS : 1st line: Wt loss helps with spontaneous
ovulation, then clomiphene citrate, metformin?
19. Chronically constipated, anal pain and itching (either hemrhoids or fissures, they were both
options)

1) In Shockable Adult CardioRespiratory Resuscitation which of the following is the correct


ratio between chest compression and ventilation plus type of defibrillation?
30:2/ Asynchronous Defibrillation

2) 20 yrs old male , known leukemia with recent chemotherapy presented to ER with chest
infection diagnosed to be in SEPTIC SHOCK.Despıte 2 litres of iv fluids he is still hypotensive
BP70/40. You think he has fluid resistant shock and decide to use inotropes. As per Surviving
Sepsis Campaign Care Bundles which one of the following is the first line choice for
inotropes? Norepinephrine

3) 20 yrs old male , known leukemia with recent chemotherapy presented to ER with chest
infection with BP70/40 and lactate of 5 . You make a diagnosis of septic shock and start
resuscitation . As per Surviving Sepsis Campaign Care Bundles which one of the following is
the recommended initial dose of fluids? 30 mls/kg of Crystalloids (repeated!)

4) In cases of Carbonmonoxide poisoning which of the following is the firstline mainstay


treatment? High flow oxygen /HBO Therapy
5) In Shockable CardioRespiratory Arrest (VT/VFib) Resuscitation which of the following drug
combinations are used? Adrenaline(Epinephrine)&Amiodarone

6) 20 yrs old male , known leukemia with recent chemotherapy presented to ER with chest
infection diagnosed to be in SEPTIC SHOCK. As per Surviving Sepsis Campaign Care Bundles
which one of the following is essential to deliver during First 6 hours of resuscitating a septic
patient? Mean arterial pressure (MAP) ≥65mm Hg

7) 20 yrs old male , known leukemia with recent chemotherapy presented to ER with chest
infection diagnosed to be in SEPTIC SHOCK. As per Surviving Sepsis Campaign Care Bundles
which one of the following is essential to deliver during First 3 hours of resuscitating a
septic patient? Measure lactate level
ER Concepts
May 2017 (Males)

1) Trauma Station (patient stabbed in chest)


- Approach
- C-ABCDE (check each)
- Peripheral line was difficult to insert, what to do?  central line
- Auscultate chest and answer questions (patient had hemothorax)
- How to manage wound?
- How to replenish fluid? How much volume to give? 3:1
- Complications? Shock

2) Anasthesia
- Intubation (same as lab skills_
- MLA, Breathing mask & ET intubation
- Name instruments and contraindications for MLA

3) DKA Station
- Patient presented with classical DKA symptomology (same as lecture with Dr.
Butt)
- Approach patient
- ABCDE + Vitals + History + PEx
- What to order? ABG, Electrolytes, urinalysis, ECG, Glucose
- What is your diagnosis? DKA
- Interpret results (high sugar, high anion gap, Sinus Tach, hyperglycemia)
- What to monitor? Vitals, sugar, watch out for confusion

4) Trauma Station
- RTA
- Same Approach as above
- Answer Examiner’s questions regarding how to check ABC
- Stabilize patient (fluids, pain medication, withdraw blood & cross match)
- On examination patient had no active bleeding  suspect retroperitoneal bleed
- Send for imaging? NO! patient unstable
- Management? Send to OR  explorative laparotomy
1. Woman who wants osteoporosis scan > scan after age 65
2. which you do transcutaenous pacing > C n E
3. which is not a definitive airway > Laryngeal Mask Airway
4. What you do at signout > SBAR (Situation, Background, Assessment,
Recommendation)
5. which is not a definitive airway > LMA
6. what you do to avoid framing > SBAR
7. woman with appendectomy: ASA 1 E
8. atypic MI : Diabetic patient
9. NNT calculate with given table of numbers > 48
10. appendicitis (RLQ pain periumbilical, localized to RLQ with N,V, woman)
11. testicular torsion : examine and alert urology
12. IBS
13. rectal cancer (incomplete emptying symptoms)
14. bacterial vaginosis / candida (19 y/o sexually active female; most common
vaginal infection)
15. dna prode test (chalmydia gonorrha)
16. HSV treat for life
17. PPD positive: isoniazid 6 months
18. acid fast bacillli (TB Pneumonia)
19. cross sectional study (diagnostic)
20. disease oriented (med x and y; one has better results in terms of disease)
21. Pregnant woman with signs of PE/DVT > send for CTPE OR DOPPLER
22. IV LINE : decreasing viscosity + length
23. man for folllow up : fobt (55yr old )
24. man of prostate cancer :
25. old fat lady with hypertension + tried dieting
26. asthma patient with no control > LABA
27. guy with full bladder and previous lung cancer : urgent MRI
28. balancing fluids in trauma==> early blood transfusion to maintain lowest
possible BP
29. ferritin for IDA
30. fever + pancytopenia : BMB
31. Dysphagia old : esophageal cancer vs achalasia
32. burns which med : ketamine
33. asthma : decrease allergens
34. cocaine overdose: benzo ?
35. dm pt to increase insulin sensitivity > tzd vs metformin
36. which patient do u not give combined COCP > migraine
37. calculate NNT ? 48 ?
38. prediabetes: lose weight to not become diabetic/ or rescreen within one year
39. patient with OCP: educate vs assist to look at options
40. osetoarthritis + obese > bariatric >
41. rsi : no bag mask ventilation
42. anal fissure vs fistula (loose stools)
43. mc chest pain in fam med : gerd vs msk ?
44. pancreatitis
45. snake bite : time is important, type of snake is important
46. CPR : increase v fib time to shock
47. shock normal rythm > might lead to vfib
48. smoking cessation : NRTS
49. tobacco clinics saudi : 70 in saudia
50. dm woman : ace i
51. acr ratio to monitor dm
52. risk of MI and troponin : 85 % PPV
53. patient to remove second mi risk : BB
54. low out HF > ace
55. first line in bb toxicity : calcium or glucagon ?

Prehospital administration of charcoal is indicated when there are no


contraindications and the patient is alert and cooperative. Then glucagon

56. cocaine not se : miosis


57. pregnancy : sphincert tone decreases
58. exercise : 5 times a week 30 mins
59. worker in the sun : cool down coz heatstroke
60. methotrexate : sign or ruptured tube (<4 cm and no heart activity)
61. DKA measure with anion gap
62. esrd cause of disease : hyperkalemia
63. ibs : CBC Normal to say its ok
64. RSV by droplet
65. sepsis : remeasure lactate
66. AAA : Do Bedside US / oxygeb/ large bore iV needles
67. acute angle glaucoma (sudden pain and loss of vision)
68. cluster headache (stays for < 3 hr on daily basis)
69. Contraindication to fly on airplanes : pneumothorax
70. pancreatitis high amylase
71. whats the most common mistakes : wrong medication vs wrong patient ?
72. MAC not affect by weight
73. chemical arteritis (use of thiopental)
74. to what do you correct blood pressure : 140/90
75. epiglottitis patient (droools) you prepare for intubation in the ED
76. SIGNS OF incorrect EET tube : O2% going down
77. you always check pulse oximetry during OR
78. which u do in primary Suvery : u can only use board for 6 hours
79. pregnant lady with cat exposure and lymphadenopathy > toxoplasma
80. NON-MALEFICIENCES
81. Cincinatti – FAST for stroke (Facial drooping, Arm weakness, Speech difficulties
and Time to call emergency service)
82. which patient to do synchronized cardiovert : a fib
83. COPD diagnosis spirometry
84. vitamin b12 deficiency in gastrectomy
85. 20 years old diabetic patient brought to ER suffering from hypoglycemic coma, the
treatment of choice is:
- MLA : 50 milligram glucose IV
86. 63. What is the most common cause of UTI?
- MLA : E.Coli (was repeated twice)
87. Which of the followings not present as an oral preparation for HTN?
- MLA : ?? (Atenolol, Nifedipine, Thiazide, Furosemide, Captopril)
88. Which antihypertensive should be avoided in bronchial asthma patients?
- MLA : Labetolol
89. Adhesive material placed around wound edges needs how long to hold it before
release ?
- MLA : 30 seconds
90. Case (scenario) of malignant hypertension, what is the treatment ?
- MLA : Sodium Nitroprusside
91. Case of bronchial Asthma and hypertensive patient, which anti-hypertensive best
avoided in this case?
- MLA : Labetalol
92. What is the drug of choice for convulsions ?
- MLA : Carbamazepine
93. (This case is for the next two question) A 35 year old female patient presenting with
a complaint of dizziness, she describes her dizziness as “like she is out of this world”,
all results of labs and investigations were negative except, however she have
accepted a new job which is very competitive and stressful:

72. Which is the least next action to be taken in this condition ?


- MLA : Lab work + imaging

73. What is the most probable diagnosis ?


- MLA : Non specific dizziness
94. (This case is for the next three questions) 25 year old man went to Thailand and
came back, he is currently suffering from severe knee pain:

74: Which of the followings is an essential part of history in this case?


- MLA : Having multiple sexual partners

75. What is the next step management that would determine what is wrong with his
knee?
- MLA : Knee aspiration

76. What are you suspecting in this patient ?


- MLA : Septic arthritis (N. Gonorrhea)
95. IPSG 1 (International Patient Safety Goals) & IPSG 2 are?
- MLA : Define patient correctly & improve communication
96. Medical Trainee JC’s AMC Orientation Includes:
- MLA : Any Ongoing Required Education, International Safety Goals, Infection
Control Program, Medication Safety Program
97. Which of the followings best define Never Events:
- MLA : Are 29 Particularly Shocking Medical Errors Causing Death or Significant
Injury of Patient or Staff, classified to 7 categories. IPSG 4 is not geared toward
Never Event
98. What cause Infectious Mononucleosis?
- MLA : Virus
99. Obese woman presents with unilateral calf pain. She is also on OCPs. Her history is
unremarkable. General Physical Examination is normal. What do you do?
- MLA : Do a Doppler of her calf since DVT can’t be ruled out
100. White male comes in with asthma, you gave short acting beta 2 agonist,
however he is not improving. What do you give him as an outpatient?
- MLA : Inhaled corticosteroids
101. Lady with HTN got pregnant. The physician changed her drug which has
teratogenic effect to another safer drug?
- MLA : Captopril changed to Methyl-Dopa
102. Which of the following is an appropriate suture size?
- MLA : Face 6-0
103. Most common cause of chronic cough is?
- MLA : Post-nasal drip
104. 86. Which of the following statements is correct?
- MLA : Acute Pancreatitis is an emergency
105. 87. Which of the following statements is correct?
- MLA : Duodenal Ulcer cause episodic epigastric pain
106. (This case is for next two questions) Patient with DM, he developed
dizziness.
107. Which history question is essential in the start of history taking?
- MLA : The dizziness happens in which circumstances (what do you mean by
dizziness)??
108. What physical examination you do to narrow you differential diagnosis?
- MLA : Measure BP supine, sitting, standing
3- smoker patient failed nicotine patches treatment, next step?
Counselling
4- patient on rescue salbutamol and max dose inhaled steroids, still not controlled,
next step?
Add LABA
5- diabetic with high BP on 3 settings and +ve microalbuminurea, drug of choice?
ACE-i
Agoraphobia means:
- fear of insects such as spiders
- fear of closed places
- fear of crowds

A woman is anxious and she had tried psychotherapy sessions, but she's still
anxious. You will give her medical therapy. The first line would be?
- SSRI

What's true about depression?


- Maximum treatment is reached at 2-8 weeks of therapy
- No need for a follow up appointment after taking medications
- Treatment should be given for 2 years after a single episode of depression

60 year old lady has MCP joints and PIP nodules (and stiffness I think)?
- Rheumatoid arthritis
2. Which of these diabetic drugs increases weight:
 Metphormin
 Acarbose
 Sitagliptin
 Glyclazide

3. Sciatica leg raise test:


 0-30
 10-60
 45-90
4. When to worry and send the patient to the neurologist:
 Weak planter flexion
23) smoker failed to quit using nicotine patches, next step? Bupropion, council
24) medial elbow pain, worse when flex, no swelling. Diagnosis: Tennis elbow, golfer
elbow, olecranion bursitis
25) trauma with bruise to lower back, on CT abdomen there is a kidney cyst, labs are
normal. Diagnosis: simple renal cyst
26) NNT: 50, 63
27) child can't sleep well due to anal itching, it is due to: ascariasis, pinworms,
giardiasis
28) Asthmatic patient on inhaled CS (high dose already) & short acting b agonist, not
managed well. What to do? LABA, oral steroids
29) routinely for newly diagnosed hypertension with no previous investigations: EKG,
renin
30) cause weight gain: SU
31) uncontrolled sever asthma: FEV1/FVC ration less than 70%, FEV1/FVC ration is
normal
32) asthmatic, should decrease exposure to dust & other indoor allergens by:
increase humidity, not washing beds
33) IBS treatment: mebeverin
34) IBS will show: mucus stool, hard to flush/ float stool, pale/ clary stool
36) 28 yo female not married, want to do cervical screening. What do you tell her: do
HPV test, she can't do it (out of the screening ppl)

�Otitis externa most common organism, P.aruginosa


�Malignant otitis media common in diabetic
�Instrument not used to measure the iop
�Diabetic Medication causes wight gain
�First line medication fir ibs
�Anxiety first line medication
�What is true about depression "medication need 6-��8wk to get the maximum
effect"
�Menopause criteria
�Secondary screening cervical cancer vs hypertension?
�HZv of the ear?
�Which side has the facial paralysis? Right?
�Mild OM what to do?
�Malingering 15 yrs old girl claims she cant see her mobile? Or hyperopia?
�Definition of malingering?
�To restore menstruation and induce ovulation in pcos? Metaformin ?
�next step in pts that has closed ear canal?

Which of the following Diabetic medications causes weight gain:


Acarbose
Metformin
Sitagliptin
Glyburide
- answer was Glyburide

Which of the following is correct regarding Combined oral


contraceptive pills:
Increases risk of endometrial cancer
Decreases incidence of cervical cancer
Increases Risk of DVT
- Right answer was Increases Risk of DVT

A Porter presents with medial epicondyle pain and swelling. Xrays


are normal. What is the most likely diagnosis::
Tennis elbow
Bursitis
Golfer elbow
- Answer : Golfer elbow

Which medication induces ovulation and restores the monthly


period in a patient with Polystic ovary syndrome?
Metformin
Misoprostol
- Answer: Metformin

Male patient diagnosed with uncomplicated HTN what is the 1st line
treatment?
Carvedilol
Amlodipine
Hydralazine
(not sure about the options, Can’t remember the options clearly

- 70 year old male had blunt trauma. On examination he has contusions on his lower
back. CT showed 1 kidney cyst on the right side measuring 2 cm. Both kidneys are
of normal size. RFTs all normal. Dx.? PCKD, simple renal cyst, RCC, recurrent UTI,
prostatic nodular hyperplasia

Light receptor cells?


Recurrent epistaxis Pic of the mouth? Talengectasia
Young man with nasopharyngeal tumor +epistaxis what is the next step? Imaging
15 yo with Problem seeing mobile? Malinger

Angel closure glaucoma symptoms?


Number needed to treat ? 63

Properly applied pressure patch?


Cervical cancer in 28 yo not sexually active?
51 year old IUD + not having period for year?

1. CURB-65->used for assessment, abx and decision for admission


This q was simply asking what CURB65 is
2. Becks triad: JVD, muffled heart sounds, low blood pressure
Again, simply asking what beck’s triad is
3. Cardiac arrest not caused by: hypocalcemia
Cardiac arrest can be caused by the following except…
4. Blood transfusion group type for certain patients: O- for males and post-meno
females
This is straight from one of the ppts. I think it should be O+ for males and post-meno
females, not O-
5. DKA presentation: diagnose from labs
The Q was a clinical picture of DKA with ketones in urine + other DKA lab
findings and asking you for the diagnosis.
6. SVT ecg identification
The picture they show will be clear, no worries. You just need to answer
what type of arrhythmia is the pic showing
7. Tx for unstable SVT: cardioversion
8. Hypovolemic shzock
I can’t remember what this Q was about to be honest. It was probably about
management (ABC, IV fluids, identifying source of hypovolemia, etc)
9. Sepsis requirement at 6hrs, according to guidelines
This is exactly according to the lecture Dr. Pasha gave about sepsis. You
must memorize the requirements by heart, as the MCQs will directly ask
about them.
10. CO poisoning: tx, high flow O2/HBO
I don’t remember this Q
11. Hemothorax X-ray: management, chest tube pointed downwards
The x-ray will be very clear and the clinical scenario made it obvious (I think
it was an old guy with a stab to the chest). Management is chest tube
12. Epidural hematoma ct: identify its epi
13. Trauma series: X-rays included
• Cervical Spine - AP
• Cervical Spine - AP Open Mouth (Odontoid)
• Cervical Spine - Lateral - Horizontal Ray
• Cervical Spine - Swimmers if C7-T1 not visualised
• Chest - AP
• Pelvis - AP

“Which of the following x-rays should be performed as a part of trauma series


x-rays?” I honestly don’t know the right answer for this one.
14. MI: 90mins for FMC to angio for accredited institutions
This Q was a case of MI and asking you about the ideal time for angio since first
medical contact. Answer is 90min
15. ABCDE wrong abbreviation: D for Danger
Which of the following is not correct about what ABCDE abbreviation stands
for?
16. Facial palsy: management
Can’t remember this Q
17. Retinal detachment on fundoscope: diagnosis
They used the same pic in the ppt he gave about ophtha emergencies
18. Lyme disease after tick bite: presented with palsy, Borreliaburgdorferi
19. Colloid vs crystalliid
Can’t remember the Q
20. Post op pain assessment: best method
I think the Q was actually about best post op pain management method. The
answer should be patient controlled analgesia and that it results in best
satisfaction.
21. Post op pain effects: common effects of pain/as a result of pain management
I can’t remember this Q
 Relieves suffering
 Early mobilisation
 Short hospital stay
 Reduced hospital cost
 Increased patient satisfaction

22. Airway preanesthesia assessment: answer was no single method is conclusive,


use multiple
“Which method is best?” Must use multiple analgesia
23. Difficult airway
Which of the following can be used for assessing difficult airway? Mallampati
score
24. NS excess causes: hypochloremic acidosis
25. FFP for: low platelets
26. Best way to monitor fluids: dynamic
27. CPAP: indication spontanuos breathing
Can’t remember this Q
28. Morphine which of the following isn’t a side effect: hallucinations
29. Stent patient preanesthesia: its worrying because they stay on long term blood
thinners
30. Hypovolemic shock: management
31. Ectopic pregnancy: what to do next if u/s –ve and urinary hCG is low/negative
Do blood hCG
32. Ambubag: indication
I don’t remember this Q
Inadequate oxygenation or ventilation, as bridge to intubation
33. PCA advantage: better patients satisfaction
34. Hyphema presentation: management raise head+analgesia
They will show you a pic of hyphema. Same one from PPT
35. Spinal anesthesia indication: surgeries below T11
36. Do no harm: Non-maleficence
Which principle does “Do no harm” refer to?
37. Thyrotoxicosis: Propranolol
38. Farmer exposed to insecticide: organophosphate overdose
39. Xray of volvulus
Coffee bean appearance. Same as ppt
40. Xray of Small Bowel Obstruction
41. PE suspected: do Chest CTPA
42. Xray: Perforated hollow viscus/viscera
Air under diaphragm
43. V. fib, after defib: what are the medications to use, epi+amio

44. What is not true about TB ? (TB does not start at vertebral space, TB does not
start at vertebral space)
45. What is the best way to measure body fluids intra-operatively?I forgot its name
but it was the one where u monitor it using a central cath and measure
variations. It’s from an SGT with dr. Geddawi
46. which of the following is true in primary survey ? (chin life & jaw thrust ?)
47. which is true about the greif process ?
48. a patient had decrease in O2 saturation post operatively, what is the next step
in management ? (mask, intubation, .. )
49. a spinal shock patient will lose all reflexes except for ?
50. 14 year old presented with swollen lips and other allergy symptoms aftereating
peanuts ==> inject IM epinephrine 1:1000
51. how can you differentiate between cardiac temponade and tension
pneumothorax ? breath sounds
52. patient presents with acute otitis media, what is the drug of choice ?
Amoxicillin, Augmentin tonsillitis penicillin V
53. patient presents with signs of instability (HR:190, BP:70/40,etc) with an ECG
picture showing ventricular tachycardia, how would you manage this patient ?
synchronised cardioversion
54. what is true about the use of packed RBCs ? (they are checked by 2 medical
staff, reaction can be predicted in some donors, etc ..)
55. patient presents without pulse and with an ECG showing Ventricular
Fibrillation, how would you manage ? 30:2 and synchronised shocking
56. the drug combination mostly being used in shockable rhythms is : amiodarone&
epinephrine
57. what is the best airway management device for respiratory failure ? Bi-PAP/
Intubation
58. What is true about the 4:2:1 formula ? (it is not used for pediatrics, i don’t
remember the other answers)
59. What is the step that has to be performed within the 1st 3 hours in suspicion of
sepsis ? lactate measurement
60. patient presented with gun shoot wound in his lung, the next step in
management after primary survey is ? putting in a chest tube
61. What is the drug used to antagoniseparacetamoltoxicity ? N Acetylcystine
62. picture of patient who lost his vision while being in the cinema, he wears
glasses (Glaucoma) ==>pilocarpine
63. 25 year old with sudden onset left sided face weakness, what is your next step
in management ? (acyclovir, CT, etc )
64. what is the most common complication of post op pain ? (bradyarrythmias, etc
??)
65. picture of an ECG with torsade, what is the management ? Magnesium Sulphate
66. in a patient where “allow natural death”is used, what of the following can he
be given ? (feeding tube)
67. female patient presents with symptoms of vaginosis ==> (Metronidazole,
doxicycline and Ciprofloxacin are the combination of choice)
68. Patient with facial nerve paralysis. You look in his ear you find vesicles. What is the
diagnosis and treatment?

MLA: Ramsey Hunter Syndrome; Tx with antiviral and steroids


69. A drug that every HF patient should be on:
MLA: ACE-I
70. What is a drug recommended for every MI patient:

MLA: cannot recall (maybe nitroglycerin or morphine)


71. What is true about bariatric surgeries?

MLA: Nutritional deficiencies can be replaced with supplements


72. 60 yr old lady presents with changing bowel habits and some cramps with bleeding
and stuff?

MLA: Advice colonoscopy as the colon CA risk increases above 60 yrs of age. (there is
another answer stating ‘reassure as this likely IBS’ but age was worrisome for me)
73. Young male with bloating, cramps and changing bowel habits:

MLA: IBS
74. Patient is diabetic and develops hypertension. Tx is:

MLA: ACE-I
75. Africo-caribean patient develops HTN. Tx is:

MLA: Calcium Channel Blocker


76. A case of torsion. Which is correct?

MLA: Early intervention is 90-100% successful


77. A case of headache (description is that of tension) for which patient is now using
around 200 pills a month:

MLA: Chronic daily HA (medication overuse headache)


78. Question about prevention

MLA: cannot recall the choices but know what Primary, Secondary and Tertiary
prevention stand for.
79. A question about vaginal discharge with fishy odor:

MLA: Bacterial vaginosis. Tx metronidazole


80. Patient with mild asthma that only requires rescue salbutamol presents now with
worsening of symptoms?

MLA: add inhaled steroid


81. Mononucleosis typical case; question about the virus  EBV

82. Pregnant ( I think 32 weeks) came with positive urine E-coli what to do for her 
Antibiotics.
Choices were:
1. Follow up
2. Antibiotics
3. Others

83. What is the most likely organism that causes deep neck space infection?

MLA: Polymicrobial
84. Patient with Cardiac abnormalities had stroke, which of the following is the likely
cause?

MLA: Atrial Fibrillation

85. What is the most common cause of a sudden limp?

MLA: Trauma

86. Which of the following agents decreases weight?

MLA: GLP1

87. 55 year old patient with heart burn after meals, which is your first line investigation?

MLA: The majority of us put Endoscopy, some put Urea breathe test. We are still not
sure who is right.
109. 1- Glargine; longest duration available in KFSH, not sure.
110. 2- Back pain; evidence-based treatment? massage?/exercise?
111. 3- Red flag for back pain; weak leg dorsiflexion (I put this)? or
saddle anesthesia?
112. 5- BCG; live attenuated
113. 7- which of the following is most effective regarding smoking
cessation? advise the patient every visit, not sure.
114. 9- IBS: caffeine
115. 10- IBS in > 50yo, rule out ovarian pathology
116. 11- Tertiary prevention; easy example.
117. 12- Cervical cancer screening question
118. 13- If a drug is proven to lower LDL but does not affect patient’s
life, this is an example of? Disease oriented study (not patient oriented)
FM:

10) epistaxis with pic of lip & tongue have dots: hereditary hemmoragic
telangectasia, SLE
11) tongue: kaposi sarcoma, hairy leukoplakia,SCC
12) Treatment of choice for the previous patient? Surgery, surgery with neck
dissection
13) Acute vestibular neuritis: corticosteroids
14) mild OM with temperature 37.2: antibiotics, analgesics
19) respiratory depression post thyroidectomy, manage: tracheotomy, MRI
20) malignant otitis media (or was it externa?): MC in diabetics

23) smoker failed to quit using nicotine patches, next step? Bupropion, council
29) routinely for newly diagnosed hypertension with no previous investigations: EKG,
renin
30)

Family Medicine: (AMB qs doc)


1. Young male with bloating, cramps and changing bowel habits:

MLA: IBS
2. Africo-caribean patient develops HTN. Tx is:

MLA: Calcium Channel Blocker


3. A case of headache (description is that of tension) for which patient is now using
around 200 pills a month:

MLA: Chronic daily HA (medication overuse headache)


4. Question about prevention

MLA: cannot recall the choices but know what primary, secondary and tertiary
prevention stand for
5. A question about vaginal discharge with fishy odor:

MLA: Bacterial vaginosis. Tx metronidazole


6. Patient with mild asthma that only requires rescue salbutamol presents now with
worsening of symptoms?

MLA: add inhaled steroid

13- If a drug is proven to lower LDL but does not affect patient’s life, this is an
example of? Disease oriented study (not patient oriented)

FM: (Last Year Questions modi doc)


1. Female with HTN and facial acne and hirsutism  cushings (if no HTN, PCOS)
2. Vaccination to a woman in a prenatal visit with no rubella antibodies should be
AFTER delivery (right after)
3. Best treatment of low HDL: 1st exercise (best drug: niacin)
4. Best treatment of high LDL: statin
5. Megakaryocytic anemia: folic acid def
6. Pancreatitis presentation
7. HBV + surface antibodies and negative antigens  good immunity against HBV
8. Stretching as a treatment to neck or back pain
9. Suspect low PTH in post-thyroidectomy pt with hypocalc
10. EBV (mono) in 16 yo pt with splenomegaly and lymphadenopathy
11. OCP inc risk of DVT
12. Asthma treatment (stepwise)
13. Treatment of depression and treatment of anxiety (know first lines)
14. Agoraphobia: fear of crowds
15. anti depression meds will start to be effective in 2-8 weeks
16. know how to calculate number needed to treat
17. know the tables in the preventive medicine lecture (they asked 3 or 4 questions
about that in MCQs and in OSCE).
18. RA presentation
19. DMPA Given every 3 months
20. How do u help with the infertility in PCOS (I don’t know, I answered metformin)
21. Frozen shoulder presentation
22. Chronically constipated, anal pain and itching (either hemrhoids or fissures, they
were both options)

Family Medicine (AMB Concepts)


1. Hx of man with rash on hands, arms, buttocks, son has similar presentation, rash is
itchy --> Scabies
2. When to remove copper coil -> 5 or 7 years?
3. Risk for breast cancer -> alcohol consumption
4. Which of the following is true about mammograms -> low radiation, painful?
5. Supplementation of which vitamin has known health benefits -> Vitamin D
6. Which of the following can be given to a 62 y/o woman to decrease fracture risk ->
alendronate
7. CVS risk assessment in a low risk individual and some tests were already done and
asking what other tests to do? -> do nothing
8. Diabetic develops HTN, first line drug to treat his HTN -> ACEi
9. Man with HTN is on ACE-i, develops chronic cough so doctor changes his medication.
Which medication did he change it to -> Losartan (ARB)
10. 1st line drug for the treatment of newly diagnosed DM -> Metformin
11. Lady's husband died 4 months ago, she has stopped eating, 2% weight loss, can hear
her husband's voice, how would you manage? --> Bereavement clinic for 1 year
12. First line treatment for 1ry dysmenorrhea -> NSAIDs
13. Screening for AAA with US -> 67 y/o male smoker
14. Black male develops HTN, what do you prescribe -> CCB
15. Pubmed is an example of -> 1ry source
16. Calculate Number Needed to Treat (NNT)
17. First step in the management of OSA -> weight loss
18. Complication of OSA -> CHF
19. Having difficulty communicating with patient, what should you do? Prevent pt from
going off topic / open ended questions
20. Single blood test that would be expected to be the most sensitive for determining
whether the patient is euthyroid, hypothyroid or hyperthyroid? -> TSH
21. A nurse sustained a needle stick injury while assisting in a procedure at the clinic
yesterday. The patient’s status is unknown for any blood-borne illnesses. The nurse
is vaccinated against Hep B. Which of the following is the most appropriate next
step? Investigate both the nurse and patient, for blood-borne illnesses
22. Lady feels fatigued for the past few years, swollen red PIP and DIP with nodules of
elbow --> RA
23. Diagnostic test for gout -> joint aspiration
24. Lady with amenorrhea, weight gain, etc. -> Hypothyroidism vs Cushing’s (?)
25. Asthmatic on ventolin, has exacerbations at night, next step? -> add low dose
inhaled CS
26. Patient presents with right lower quadrant pain that started off as periumbilical pain,
anorexia, N&V (acute appendicitis), next step? -> immediate admission and surgical
consult
27. Which of the following would indicate poor DM control -> HbA1c of 10%
28. Kid has 3 months hx of abdominal pain, now he can’t attend school, which of the
following would hint towards an organic rather than a functional cause -> headache,
anxiety, suprapubic pain, family hx of abdominal pain??
29. Facial Nerve palsy -> treat with steroids
30. A patient develops headache + neck stiffness with NO fever -> subarachnoid
hemorrhage

Girls 2015
Family medicine:

1- 21 lady just came from Africa and diagnosed with malaria what is the most
specific test to make the diagnosis of malaria: thick and thin blood film
2- 71 old man diabetic , feel dizziness when he wakes up form the bed for the
last 7 days he had syncope , he is on aspirin , frusemide and beta blocker what
is the cause : orthostatic hypotension (I'm not sure)
3- 6 yr old girl came to the ER with nausea and vomitting, abdominal pain, you
smell a sweet odour from her mouth what is the diagnosis ?
Diabetic ketoacidosis
4- Patient with cor polmnale what is the cause : pulmonary fibrosis , pneumonia
5- Diabetic pt put on lifestyle and exercise < didn't work wt to do next > metfor
6- Diabetic patient ii, hyperlipidemia, HTN and microalbuminuria > ACEI
*lisinopril
7- Dx of gout
8- 16 yo with 6 day history of fatigue develops cervical lymphadenopathy and
later splenomegaly what is the most likely causative organism
Adenovirus
CMV
EBV
9- 16 year old lady got shortness of breath after taking penicillin.
Answer: intramuscular epinephrine
10- 58 year old male has history of hypertension and wants to reduce his risk of
developing an MI. His HDL is 32 (normal is 40). What can increase his HDL?
Answer: Exercise
Mayoclinic says: Get more physical activity. Within two months of starting,
frequent aerobic exercise can increase HDL cholesterol by about 5 percent in
otherwise healthy sedentary adults.
11- 50 yo with 2 hour history of chest pain has been given aspirin his blood
pressure was 155/85 what is the most appropriate drug that should be given
next
Carvidelol
An ACEI
Morphine
Furosemide
12-familial cholesterolemia exercise not helpful, LDL more than 6 what to do?
Simvastatin
13- healthcare worker stuff, about vaccinations his hep B profile: negative
surface antigens ,High antibodies ,what's his status?
Has protection against hep B
14- man with fatigue and shortness of breath Hb is 7.5 macrocytosis what type
of deficiency?
Folic acid
15-Drug which reduces FBG and HBA1c. Is it disease or patient oriented?
Disease oriented

16-Diabetic patient with 3 readings of 160/100, what to do next?


Start ACEI

17-Woman with no antibodies to rubella, comes for a prenatal visit, when do


you give the rubella vaccination?
Immediate postpartum period

18-Child with recurrent chest infections, malabsorption symptoms, chronic


sinusitis, and nasal polyps. What test would you do to confirm your suspected
diagnosis?
Sweat chloride test

19-Trauma patient. What stain should we use?


Flourescein stain
20-How can you be cost effective in family practice?
Options included: generic prescription, drug formulations with multiple drug.

21-Malaria test > thick and thin film.

FM: (Last Year Questions modi doc)


23. Female with HTN and facial acne and hirsutism  cushings (if no HTN, PCOS)
24. Vaccination to a woman in a prenatal visit with no rubella antibodies should be
AFTER delivery (right after)
25. Best treatment of low HDL: 1st exercise (best drug: niacin)
26. Best treatment of high LDL: statin
27. Megakaryocytic anemia: folic acid def
28. Pancreatitis presentation
29. HBV + surface antibodies and negative antigens  good immunity against HBV
30. Stretching as a treatment to neck or back pain
31. Suspect low PTH in post-thyroidectomy pt with hypocalc
32. EBV (mono) in 16 yo pt with splenomegaly and lymphadenopathy
33. OCP inc risk of DVT
34. Asthma treatment (stepwise)
35. Treatment of depression and treatment of anxiety (know first lines)
36. Agoraphobia: fear of crowds
37. anti depression meds will start to be effective in 2-8 weeks
38. know how to calculate number needed to treat
39. know the tables in the preventive medicine lecture (they asked 3 or 4 questions
about that in MCQs and in OSCE).
40. RA presentation
41. DMPA Given every 3 months
42. How do u help with the infertility in PCOS (I don’t know, I answered metformin)
43. Frozen shoulder presentation
44. Chronically constipated, anal pain and itching (either hemrhoids or fissures, they
were both options)
1. During end of life care in a palliative DNAR patient, an emergency Physician may:
a. Keep foleycath (urinary) as there Is no long term risk of infection
b. Initiate inotropes and pressors
c. Initiate tube feeding and pain control
d. Refer to surgery and transplant
e. Intubate and mechanical ventilation

2. The loss of a loved one results in grief as universal human behavior, The following Is the
most appropriate statement regarding the stages (denial, anger, bargaining, depression,
and acceptance):
a. Everyone must go through the stages in order
b. Medications should be used to shorten the grieving process
c. One may go through grieving process without passing through all stages of grieving
d. The stages of grief have been validated in randomized control trials
e. Normally, grief should be over by 1 month

3. In a patient with pneumonia, the CURB 65 score will assist you with:
a. Choice of antibiotics drug therapy
b. b. Need for hospitalization and mortality risk
c. c. Follow up appointment with primary service
d. d. Need for lab orders and ABG
e. e. Intubation decision and early goal directed therapy

4. The Beck’s triad includes the following elements:


a. Low SBP, absent breath sounds and raised JVD
b. b. Muffled heart tones, decrease BP, increase JVD
c. c. Hypercoagulability, stasis, vessel injury
d. d. Jaundice, fever, R upper abdominal pain
e. e. Recurrent oral ulcers, genital ulcers, iridocyclitis

5. 20 year old man presents to ER after a fall froma horse. He is confused on arrival to ER
but
localizes pain on stimulation. You assess him to be a case of mild head injury with GCS of
13/15.
Your next step is to:
a. Consult neurosurgery
b. Intubation
c. ABCDE
d. C-ABC and CT brain
e. Trauma code and admit for observation

6. A 15 year-old female was outside collecting flowers when she was stung by a bee. She
reports being stung once before without any complications, but now she is having
generalized urticarial rash, swelling and dyspnea. Which one of the following is the next
most appropriate stat treatment?
a. Hydrocortisone 100mg IV
b. Diphenhydramine 25mg IV
c. Ranitidine 50mg IV
d. Epinephrine 1:1000 0.5mg IM
e. Epinephrine 1:10,000 1mg IV

7. 80 yrs old man with chest pain suddenly collapsed in CCU. The cardiac monitor showed
the following rhythm

In shockable cardiorespiratory arrest resuscitation, which of the following drug


combinations
are used:
a. Atropine & Adrenaline (epinephrine)
b. Atropine & Amiodarone
c. Vasopressin & bicarb
d. Adrenaline(epinephrine) & amiodarone
e. Adrenaline(epinephrine) & bicarb

8. You are the team leader in a cardiopulmonary resuscitation team attending an arrest in
ER. The patient is in the following rhythm on cardiac monitor

You decided this is a shockable rhythm and you start chest compression with ventilation.
Which
of the following is most appropriate next step?
a. Defibrillation @ 100 Joules
b. Defibrillation @ 200 Joules
c. Synchronized cardioversion @ 100 Joules
d. Synchronized cardioversion @ 300 Joules
e. Periodic chest thump

9. 20 yrs old male, known leukemia with recent chemotherapy presented to ER with fever.
He was diagnosed to be in SEPTIC SHOCK secondary to chest infection.

As per Surviving Sepsis Campaign Care Bundles which one of the following is essential to
deliver
during first 3 hours of resuscitating a septic patient?
a. Measure lactate level
b. Central and arterial line placement
c. Perform chest x-ray
d. Administer inotropes
e. Administer 50 mL of 50% dextrose to prevent hypoglycemia

10. 30 yrs old female, known breast cancer patient with recent chemotherapy presented to
ER with chest infection and diagnosed to be in SEPTIC SHOCK
As per Surviving Sepsis Campaign Care Bundles which one of the following is essential to
deliver
during first 6 hours of resuscitating a septic patient?
a. Prevent hypocalcemia
b. Maintain temperature > 36°C
c. Mean arterial pressure (MAP) ≥ 65mmHg
d. Keep Hb above 10 gm/dL
e. Start dopamine @ renal dose

11. After performing a proceducre in the emergency room department- the next most
important step is to:
a. Documentation and taking pictures
b. Inform the family
c. Teaching the staff
d. Checking if the procedure is done right
e. Taking a quick break before attending next patient

12. 80 yrs old female from nursing home with history of chronic laxative use presents with
sudden onset of abdominal pain with distension. She has absolute constipation with no
passage of flatus. Her acute abdominal series x-ray reveals the following:

What is the diagnosis?


a. Volvulus
b. Large bowel obstruction
c. Bowe; perforation
d. Small bowel obstruction
e. Gall stones

13. Most appropriate statement regarding triage is:


a. There is one universal system
b. It is a foul-proof system
c. First come, first serve basis is maintained
d. Immediate action may be provided to the sickest
e. CTAS/ESI/START are all the same

14. 76 yrs old female with nausea, vomiting, and lower abdominal pain diagnosed with
recent UTI presented to emergency room with h/o being unwell for few days. O/E
patient severely
dehydrated, tachycardic and tachpnoeic. Her HR: 136 BP: 110/70, R/R: 40, SAO2: 98%, She
has no drug allergies.

LABS:
Na: 130 mmol/L Cl: 95 mmol/L
K: 5.5 mmol/L HCO3: 20 mmol/L
Glucose: 30 mmol/L
Urinalysis:
Blood: -ve Nitrogen: +ve Leukocytes: +ve
Protein: -ve Ketones: -ve

The next most appropriate management plan is to:


a. Ciprofloxacin IV and NaCl 0.9% 1 liter IV
b. Glucophage (Metformin) 500 mg BID
c. A & B
d. Insulin 10 units IV and NaCl 0.9% 2 liter IV with ciprofloxacin 400 mg IV
e. Insulin 10 unties SQ and Glucophage 500 mg PO now

15. 36 yrs old female after recent thyroid surgery presented with h/o fever, nausea,
vomiting,
restlessness, agitation and extreme confusion. Her observations revealed
Temp: 40°C Pulse: 140/min
BP: 190/110 mmHg R/R: 36/min O2 saturation: 94%

Which of the following is used for acute treatment of above condition?


a. Aspirin
b. IV propranolol
c. IV thyroxine
d. IV hydrocortisone
e. IV acetaminophen

16. 20 yr old male unconscious is brought to emergency department by medics. After ABC,
the next most appropriate intervention is:

a. Normal saline bolus 30 ml/kg


b. Thiamine 100 mg IV, Narcan 1 mg IV, D50 1 amp IV
c. Start CT brain
d. Intubation, activated charcoal
e. Drug toxscreen, ETOH level and gastric lavage

17. 50 yrs old burn victim develops fever, tachycardia, tachypnoea while receiving
treatment on
burns ward. You make a diagnosis of SIRS. Which of the following is NOT an essential
element
for diagnosis of SIRS (Systemic inflammatory response syndrome)?

a. Temp less than 36°C(96.8°F) or greater than 38°C(100.4°F)


b. Heart rate greater than 90 beats per minute
c. High lactate of 2 mmol/L
d. Tachypnea greater than 20 breaths per minute
e. Leukocytosis or leukopenia
18. A 33-year-old obese man comes to the emergency department because of abdominal
pain
nausea and vomiting. O/E rebound tenderness and guarding in Rt iliac fossa. You
provisionally
make a diagnosis of acute appendicitis. Which of the following will be essential before a
patient goes to OR?
a. Labs and overnight NPO
b. Labs and CT scan of abdomen and pelvis with contrast
c. Antibiotics, US abdomen, IVFs
d. SAMPLE history, exam, labs surgical consent and surgical consult
e. SAMPLE history, exam, CT scan and surgical consent

19. A 33-year-old man suffered head injury after falling from a roof at work. His initial GCS
was 9/15 with unilateral dilated pupil. The CT brain revealed the following:
What is your diagnosis?
a. Epidural bleed
b. Subdural bleed
c. Intracerebral bleed
d. Cerebral contusion
e. Basal skull fracture

20. What is the etiology of the following finding on CT scan after sustaining a head injury

a. Epidural bleed
b. Subdural bleed
c. Intracerebral bleed
d. Cerebral contusion
e. Basal skull fracture

21. 36 years old male after recent flight from Australia to Saudi Arabia presented to ER with
sudden onset of SOB. His obs were as follows: Afebrile, P: 123/min, RR: 40/min, BP:
90/60mmHg, SaO2:
90%. Labs: Hb 14, WCC: 9.0. D-dimer: 600 (high). Which of the following will confirm your
diagnosis?

a. Chest x-ray AP/lat & EKG


b. Ultrasound venous Doppler lower limb
c. ABG and checking A-a gradient
d. Blood culture and sputum gram stain
e. Chest CTPA

22. 40 years old farmer presents with reduced GCS, salivation, lacrimation, urination,
diarrhea, GI upset and emesis. He has been using a new insecticide spray today. O/E: P:
52/min, BP: 110/60mmHg, RR: 36/min. Pupils are small on examination. What is the
diagnosis?
a. Opioid poisoning
b. Organophosphate toxicity
c. Cocaine toxicity
d. Salicylate poisoning
e. Lead poisoning

23. What is the antidote for iron toxicity?


a. N-acetylcysteine/parvolex
b. Atropine
c. Activated charcoal
d. Ethanol
e. Deferoxamine

24. In cases of (AMI) acute myocardial infarction, the target door to balloon time in a chest
pain
accredited centre is
a. 30 mins
b. 60 mins
c. 90 mins
d. 120 mins
e. 3.5 hours

25. 25 year old young man presents with sudden onset of complete weakness of one side of
the
face. The clinical examination reveals as follows
What is the most appropriate ER management?
a. Reassure, start acyclovir/steroids/artificial tears
b. Immediate CT scan of brain
c. Immediate MRI with referral to neurology
d. Stroke thrombolysis
e. Immediate referral to ENT

26. 30 yrs old obese female presented with sudden onset of fever, severe RUQ abdominal
pain,
nausea, and vomiting. You ordered basic labs
Labs:
Hb: 12 g/dL WBC: 17.5 x 109/L
Platelets: 110 x 109/L Lipase: 46 U/L (N) AST: 90 U/L (H)

What is the investigation of choice?


a. CT scan abdomen without contrast
b. Acute abdominal series x-ray
c. USS abdomen
d. CT scan abdomen with contrast
e. Barium swallow

27. A 45 years old male sustains a penetrating injury to the thoracic spine. There was loss of
ipsilateral motor function and contralateral pain and temperature sensation. What is the
diagnosis?
a. Anterior cord syndrome
b. Posterior cord syndrome
c. Brown sequard syndrome
d. Central cord syndrome
e. Cauda equine

28. The optimal head position for intubating the patient under anesthesia is
a. Extension of the neck
b. Flexion of the neck and extension at atlanto-occipital joint.
c. Flexion of the neck, extension at the atlanto-occipital joint and jaw thrust
d. Extension of the neck and jaw thrust

29. Spinal anesthesia can be considered for:


a. High risk patients
b. Surgery above the umbilicus
c. Laparoscopic surgery
d. Patients on Plavix
e. Agitated patients

30. Multimodal approach to treat postoperative pain means:


a. Give different groups of medications that attack pain from different angles
b. Use the maximum tolerable dose of pain killers
c. Consult different specialties to reach the best possible treatment approach
d. Anesthetist has to handover all the names and doses of the intraoperative
painkillers
used to recovery nurses

31. Using high volumes of normal saline 0.9% can lead to:
a. Respiratory acidosis
b. Metabolic alkalosis
c. Respiratory alkalosis
d. Non of the above

32. As a health care provider, we must at least act and not hurt any patient. This is referred
as:
a. Beneficence
b. Maleficence
c. Non beneficence
d. Non maleficence
e. Standard of care

33. 70 years old male presents to ED with acute symptoms of stroke. The patient is a
candidate for stroke thrombolysis (tPA) if:

a. Window of 6 hours
b. NIH stroke scale score >25
c. He has an INR > 3.5
d. Blood glucose is <50 mg/dl (2.7 mmol/L)
e. Head trauma 4 weeks ago
34. 20 years old male presents to ED with decreased mental status and agitation. He was
found with empty bottle of tricyclic antidepressants taken 3-4 hours ago. After his initial
resuscitation you have decided to counteract the actions by:
a. 10 gm activated charcoal
b. Atropine
c. Sodium bicarbonate
d. Narcaine and thiamine
e. N Acetylcysteine (NAC)

35. 20 years old male with recent history of alcohol use presented to emergency room with
sudden loss of consciousness. On the monitor you note the following rhythm

A medication that may resolve this rhythm is:


a. Atropine
b. Adenosine
c. Magnesium sulphate
d. Calcium chloride
e. Ephinephrine

36. 21 years old married female presents to ED with 2 hours history of right LQ pain. She
denies any urinary symptoms or vaginal bleeding. You check a urine point of care
pregnancy test and it is negative. The next most appropriate step would be to:
a. Check serum BHCG quantitative, IVF and pain control
b. Check pelvic US, R/O
c. Order CT scan to R/O appendicitis
d. Treat her symptomatically, reassurance and discharge home
e. Acute abdominal screening x-rays to r/o bowel obstruction

37. A 21 years old male after a recent cruise holiday presents with fever, flu like symptoms,
inguinal lymph nodes and painful genital blisters and sores. Most prudent treatment
option is

a. Metronidazole, doxycycline & ciprofloxacin


b. Diflucan
c. Azithromycin & rocephine
d. Famcyclovir
e. Clindamycin gel
38. All of the following are excited to read neurotransmitters that modulate pain except:

A. Substance P
B. ATP
C. Somatostatin
D. Glutamate
E. Aspartate

39. A female presents to the ER with overdose of beta blocker and suicidal ideation which
one of the following managements is least likely to help her:
A. Glucagon
B. Calcium gluconate
C. Hemodialysis
D. Cardiac pacing
E. Dopamine

40. According to ASA guidelines which one of the following is not used intraoperatively for
monitoring:
A. Oxygen analyses
B. ECG continuous
C. BP
D. Peripheral nerve stimulator
E. Capnometry

41. Mallampati classification when you cannot see the soft palate:
Ans: class IV

42. Which one of the following is not a risk factor for difficult bag mask ventilation:
A. Thyromental less than 6 cm
B. Thyro angle more than 9 cm
C. Mallampati 1

43. The following are independent risk factors for bag mask ventilation except:
A. BMI < 26
B. Edentulous
C. Beard
D. Snoring
E. Short neck

44. Oropharyngeal airway feature:


A. Bite block
B. All same colors

45. What is true about LMA:


A. Secure 100% airway
B. Had to be laryngy scope mediated
C. Less traumatic than endo trachyal t
D. Non fasting patient usage

46. A patient is found unconscious outside the ER with needle marks in his arm and pinpoint
pupils: antidote? Vitals

A. Flumazenil
B. Naloxone
C. Atropine
D. Cocaine

47. Gold standard for ETT confirmation:

A. Capnohraphy
B. Visualization of vocal chords
C. Chest X ray
D. Auscultation
E. Chest movement

48. Narcotics:

A. Morphine is highly lipid soluble


B. Intrathecal morphine is more respiratory depressant than fentanyl
C. The mu receptor is G protein coupled

49. Opioid side effect except:

A. Nausea vomiting
B. Bradycardia
C. Constipation
D. Resp depression
E. Mydriasis

50. How frequently should Hr and BP be measured in OR according ASA guidelines:


A. 2 minutes
B. 5 minutes
D. 7 minutes
C. 10 minutes
D. 15 minutes

51. Which is a feature of IV fluids:


A. Colloid stays intravascular unless capillary leak

52. A patient started having symptoms of stroke 2 hours ago. What is the time frame to get
a non enhanced CT scan for this patient?
a. 10 minutes
b. 15 minutes
c. 25 minutes
d. 45 minutes
e. 60 minutes

119. SSS MCQs


120. 1
121. 3non melefesence
122. 4carpal tunnel
123. 5investigation of carpal tunnel
124. 6dka
125. 7thyrotoxicosos give propranolol
126. 8torsade a de pointes, give mgsulfate
127. 9X-ray finding of osteoarthritis
128. 10naloxone for opioid overdose
129. 11anticholenergic overdose antidote: glucose and thiamine
130. 13needle thiracocentesis for tension
131. 14chest tube for hemothorax
132. 15small bowel obstruction X-ray
133. 16sigmoid volvulus X-ray
134. 17epidural ct
135. 18spinal anesthesia for surgery below the umbilical
136. 19post anesthesia care with high HR & BP & RR, and agitated
what do we give
137. 20why do we give pre oxygenation before intubation, possible
answer is more time for apnea
138. 21GCS = 13
139. 22for intubation optimal position is neck extension: Flexion of the
neck and extention of the atlanto-occepital.
140. 23compensated respiratory acidosis values
141. 34not essential for diagnosis of SIRS: lactate
142. 39PE suspected do CT Angio
143. 45what affect the oxygen transport more: cardiac output
144. 48metronidazole, doxycycline, ceprofloxacin is the answer
145. 49induced by propafol and was Bevin curonium then developed
bilateral wheezing and hypotension
146. 55DNAR with intervention you can give feeding tube
147. 56documention to communicate with other doctors
148. 57answer is not all people will pass all stages of grief
149. 58capnometary and auscultion for making sure the intubation is in
the correct place
150. 59answer is do beta HCG and give IVF and analgesics
151. 62cause of cardiac arrest in renal failure patient <-- hyperkalemia
152. 63anaphylaxis what to give <-- epi 1:1000 IM
153. 64which is not associated with high oxygen therapy
154. 65contraindication to tPA for stroke <-- INR3.5 <
155. 67organophosphate poisoning is the answer
156. 68appendicitis, what test will you order to confirm (radiology(
MCQs
1) X-ray picture. What is the diagnosis? Volvulus.
2) X-ray picture. What is the diagnosis? Small intestinal obstruction.
3) X-ray picture. What is the diagnosis? Perforated viscus.
4) Patient with signs and symptoms. An X-ray is shown. Immediate initial treatment is
needle thoracocentesis (since the X-ray shows tension pneumothorax).
5) A patient (only lab tests are shown), what is the diagnosis? DKA (since there were
positive ketones in urine ++++).
6) Spinal anesthesia can be done for which of the following? Not sure.
7) Which combination of medications is used in ventricular tachycardia or fibrillation?
Adrenaline and amiodarone.
8) A patient was bitten by a deer tick, what is the likely organism?
Burgdorferi.
9) Documentation question. To communicate with other doctors
10) Pneumonia CURB-65 question? Admit if it was 2-3
11) A patient with Paracetamol toxicity, what is the antidote?
N-acetylecystein.
12) A patient with Carbon Monoxide poisoning, What is the antidote? Hyperbaric
oxygen.
13) A patient with COPD. Check previous culture.
14) A CT scan is shown, what is the diagnosis? Epidural hematoma.
15) A patient with thyroid storm, what is the most important immediate treatment? IV
propranolol.
16) A female patient that has right or left lower quadrant pain and a negative pregnancy
test, which investigation you want to do? (not sure).
17) A patient with certain neurological disabilities, what is his glassgow coma scale? 13.
18) A patient with hemothorax. Do chest drain.
19) Which blood type can be given for elderly women and -----------?
O+ blood.
20) Check lactate for sepsis.
21) Which of the following is not part of the systemic inflammatory response syndrome
(SIRS)? (forgot).
22) End of life care/Life support question. The answer is that NOT all patients pass
through all the stages of grief.
23) Which of the following can be done to a DNAR (Do not attempt resuscitation)
patient? Morphine (not sure).
24) Patient with appendicitis, which investigation you want to do? CT or laparoscopy
(not sure but maybe CT).
25) 62 years old male attended ER in CardioRespiratory Arrest . Following Rhytm was
seen on Cardiac Monitor Which of the following Drug should be given during ACLS
Resuscitation? Adrenaline(Epinephrine)

26) In Shockable Adult CardioRespiratory Resuscitation which of the following is the


correct ratio between chest compression and ventilation plus type of defibrillation?
30:2/ Asynchronous Defibrillation

27) 20 yrs old male , known leukemia with recent chemotherapy presented to ER with
chest infection diagnosed to be in SEPTIC SHOCK. Despıte 2 litres of iv fluids he is still
hypotensive BP70/40. You think he has fluid resistant shock and decide to use
inotropes. As per Surviving Sepsis Campaign Care Bundles which one of the following
is the first line choice for inotropes? Norepinephrine

28) 20 yrs old male , known leukemia with recent chemotherapy presented to ER with
chest infection with BP70/40 and lactate of 5 . You make a diagnosis of septic shock
and start resuscitation . As per Surviving Sepsis Campaign Care Bundles which one of
the following is the recommended initial dose of fluids? 30 mls/kg of Crystalloids

29) In cases of Carbonmonoxide poisoning which of the following is the firstline


mainstay treatment? High flow oxygen /HBO Therapy

30) In Shockable CardioRespiratory Arrest (VT/VFib) Resuscitation which of the following


drug combinations are used? Adrenaline(Epinephrine)&Amiodarone

31) 20 yrs old male , known leukemia with recent chemotherapy presented to ER with
chest infection diagnosed to be in SEPTIC SHOCK. As per Surviving Sepsis Campaign
Care Bundles which one of the following is essential to deliver during First 6 hours of
resuscitating a septic patient? Mean arterial pressure (MAP) ≥65mm Hg

32) 20 yrs old male , known leukemia with recent chemotherapy presented to ER with
chest infection diagnosed to be in SEPTIC SHOCK. As per Surviving Sepsis Campaign
Care Bundles which one of the following is essential to deliver during First 3 hours of
resuscitating a septic patient? Measure lactate level
157. What is CURB-65? used for assessment, abx and decision for admission
158. What is Becks triad? JVD, muffled heart sounds, low blood pressure
159. All of the choices can cause Cardiac arrest except: hypocalcemia
160. Which one regarding blood transfusion is correct: O- for children and
women of child-bearing age (to avoid immune reaction if we give O+ in
females with O- group of child bearing age). O+ can be given to adult males
and post-meno females (Rh incompatibility)
161. Person presented with abdominal pain, nausea/vomiting, lab showed
++++ urine ketone, diagnosis: DKA
162. Question with an ECG strip, identify the pattern: Supraventricular
tachycardia (SVT)
163. Tx for unstable SVT: cardioversion
164. Patient presented with symptoms of shock after losing blood
following an accident, what is this condition: Hypovolemic/hemorrhagic
shock
165. Diagnostic requirement/criteria for sepsis at 6hrs, according to
guidelines (written in Dr.pasha’s lecture about sepsis)
166. CO poisoning treatment: high flow O2/HBO
167. Patient with a Hemothorax on X-ray, management: chest tube
pointed downwards
168. CT showing lens-shaped hematoma, what is it: Epidural hematoma
169. Trauma series: X-rays of cervical spine, chest and pelvis included
170. Question asking about the MI protocol once patient arrives to ER:
90mins for FMC to angio for accredited institutions
171. ABCDE is used for the primary survery in ATLS, which one of the
letters are wrong: D for Danger
172. Facial palsy treatment: Corticosteroids and antiviral/acyclovir
(ramsay-hunt Sx)
173. Picture of fundoscopy, diagnosis: Retinal detachment
174. Patient presented with facial palsy after a tick bite, diagnosis: lyme
disease/borrelia
175. Young patient presented with sacroiliac stiffness and pain, diagnosis:
ankylosing spondylitis
176. Patient with first toe finger joint pain (proximal phalanx), urine
sample showed –ve birefringent crystals, diagnosis: gout
177. Question about Colloid vs crystalloid: colloid theoretically last longer
in vessels due to high protein content
178. Best method for Post op pain assessment: objective test using
numerical values or sad/happy faces if it’s a child patient.
179. Post laparotomy op pain management is important due: normally
coughing will increase pain; therefore patients avoid coughing, decreasing
O2 levels.
180. Best method for Airway preanesthesia assessment: no single method
is conclusive, use multiple
181. Causes of difficult airway: arched palate, micrognathia, no teeth,
denture wearing, beard, ect… (I forgot the exact choices, but these are the
correct answer)
182. NS excess causes: hyperchlremic acidosis
183. FFP is used for: low platelets
184. Best way to monitor fluids: dynamic monitoring rather than static
185. Indication to use CPAP: Obstructive sleep apnea
186. Which of the following isn’t a side effect of morphine: hallucinations
187. Why should we be cautious in the history taking in preanesthesia of a
patient with a history of coronary stent: because they stay on long term
blood thinners
188. Hypovolemic shock management: IV crystalloid (3 liters of crystalloid
per 1 liter of blood loss)
189. What to do next if u/s –ve and urinary hCG is low/negative in a lady
of child bearing age with SEVERE lower abd pain: repeat hCG (ectopic
pregnancy until proven otherwise).
190. What is the indication to use an ambubag: acute emergency cases,
such as post-trauma.
191. What is the advantage of patient controlled analgesia: better patients
satisfaction
192. Patient presented with blood in the anterior chamber/cornea,
management: raise head of bed+ analgesia
193. Spinal anesthesia indication: I forgot the answer to this question
194. What is the meaning of Non-maleficence: Do No Harm
195. Patient presented with symptoms of Thyrotoxicosis, management in
acute cases: Propranolol
196. Farmer exposed to insecticide, symptoms of cholinergic excess,
diagnosis: organophosphate overdose
197. Xray abdomen in patient with severe abd pain, diagnosis: volvulus
198. Xray abdomen, diagnosis: Small Bowel Obstruction
199. Patient with tachypnea, increased heart rate, and chest pain (suspect
PE), next step: do Chest CTPA
200. Xray abdomen showing air/hyperlucency beneath the diaphragm,
diagnosis: Perforated hollow viscus/viscera
201. What are the medications to use in a ventricular fib after
shock/synchronized cardioversion: epi+amiodarone
202. What is not true about TB ? (TB does not start at vertebral space, TB
does not start at vertebral space)
203. What is the best way to measure body fluids intra-operatively ?
204. which of the following is true in primary survey ? (chin life & jaw
thrust ?)
205. which is true about the grief process ? Forgot the choices
206. a patient had decrease in O2 saturation post operatively, what is the
next step in management ? (mask, intubation, .. )
207. 14 year old presented with swollen lips and other allergy symptoms
after eating peanuts ==> inject IM epinephrine 1:1000
208. how can you differentiate between cardiac temponade and tension
pneumothorax ? breath sounds
209. patient presents with acute otitis media, what is the drug of choice ?
Amoxicillin, Augmentin
210. patient presents with signs of instability (HR:190, BP:70/40,etc) with
an ECG picture showing ventricular tachycardia, how would you manage this
patient ? synchronised cardioversion
211. what is true about the use of packed RBCs ? (they are checked by 2
medical staff, reaction can be predicted in some donors, etc ..)
212. patient presents without pulse and with an ECG showing Ventricular
Fibrillation, how would you manage ? 30:2 and synchronised shocking
213. the drug combination mostly being used in shockable rhythms is :
amiodarone & epinephrine
214. what is the best airway management device for respiratory failure ?
Bi-PAP
215. What is true about the 4:2:1 formula ? (it is not used for pediatrics, i
don’t remember the other answers)
216. What is the step that has to be performed within the 1st 3 hours in
suspicion of sepsis ? lactate measurement
217. patient presented with gun shoot wound in his lung, the next step in
management after primary survey is ? putting in a chest tube
218. What is the drug used to antagonise paracetamol toxicity ? N
Acetylcystine
219. picture of patient who lost his vision while being in the cinema, he
wears glasses (Glaucoma) ==> pilocarpine
220. 25 year old with sudden onset left sided face weakness, what is your
next step in management ? (acyclovir, CT, etc )
221. what is the most common complication of post op pain ?
(bradyarrythmias, etc ??)
222. heamarthrosis management ==> give factor 8, aspirate, etc??
223. picture of an ECG with torsade, what is the management ? Magnesium
Sulphate
224. in a patient where “allow natural death” is used, what of the
following can he be given ? (feeding tube)
225. female patient presents with symptoms of vaginosis ==>
(Metronidazole, doxicycline and Ciprofloxacin are the combination of
choice)
1. Female with HTN and facial acne and hirsutism  cushings (if no HTN, PCOS)
2. Vaccination to a woman in a prenatal visit with no rubella antibodies should be
AFTER delivery (right after)
3. Best treatment of low HDL: 1st exercise (best drug: niacin)
4. Best treatment of high LDL: statin
5. Megakaryocytic anemia: folic acid def
6. Pancreatitis presentation
7. HBV + surface antibodies and negative antigens  good immunity against HBV
8. Stretching as a treatment to neck or back pain
9. Suspect low PTH in post-thyroidectomy pt with hypocalc
10. EBV (mono) in 16 yo pt with splenomegaly and lymphadenopathy
11. OCP inc risk of DVT
12. Asthma treatment (stepwise)
13. Treatment of depression and treatment of anxiety (know first lines)
14. Agoraphobia: fear of crowds
15. anti depression meds will start to be effective in 2-8 weeks
16. know how to calculate number needed to treat
17. know the tables in the preventive medicine lecture (they asked 3 or 4 questions
about that in MCQs and in OSCE).
18. RA presentation
19. DMPA Given every 3 months
20. How do u help with the infertility in PCOS (I don’t know, I answered metformin)
21. Frozen shoulder presentation
22. Chronically constipated, anal pain and itching (either hemrhoids or fissures, they
were both options)

1) In Shockable Adult CardioRespiratory Resuscitation which of the following is the


correct ratio between chest compression and ventilation plus type of defibrillation?
30:2/ Asynchronous Defibrillation

2) 20 yrs old male , known leukemia with recent chemotherapy presented to ER with
chest infection diagnosed to be in SEPTIC SHOCK.Despıte 2 litres of iv fluids he is still
hypotensive BP70/40. You think he has fluid resistant shock and decide to use
inotropes. As per Surviving Sepsis Campaign Care Bundles which one of the following
is the first line choice for inotropes? Norepinephrine

3) 20 yrs old male , known leukemia with recent chemotherapy presented to ER with
chest infection with BP70/40 and lactate of 5 . You make a diagnosis of septic shock
and start resuscitation . As per Surviving Sepsis Campaign Care Bundles which one of
the following is the recommended initial dose of fluids? 30 mls/kg of Crystalloids

4) In cases of Carbonmonoxide poisoning which of the following is the firstline


mainstay treatment? High flow oxygen /HBO Therapy

5) In Shockable CardioRespiratory Arrest (VT/VFib) Resuscitation which of the following


drug combinations are used? Adrenaline(Epinephrine)&Amiodarone

6) 20 yrs old male , known leukemia with recent chemotherapy presented to ER with
chest infection diagnosed to be in SEPTIC SHOCK. As per Surviving Sepsis Campaign
Care Bundles which one of the following is essential to deliver during First 6 hours of
resuscitating a septic patient? Mean arterial pressure (MAP) ≥65mm Hg

7) 20 yrs old male , known leukemia with recent chemotherapy presented to ER with
chest infection diagnosed to be in SEPTIC SHOCK. As per Surviving Sepsis Campaign
Care Bundles which one of the following is essential to deliver during First 3 hours of
resuscitating a septic patient? Measure lactate level
6th batch- first group- Males
Ambulatory block (ER, FM, Anesthesia)

We had 34 new questions, and the rest were repeated.


These are the new ones:
6th batch- first group- Males
Ambulatory block (ER, FM, Anesthesia)

FM:
Vitamin D
1- (NEW) Vitamin D recommended dosage for infant that is being breast-fed?
a. 400
b. 800
c. 1000
d. 600
e. 900000
Traveler’s Diarrhea
2- (NEW) Best way to prevent Traveler’s diarrhea
a. Only eat cooked food and drink boiled water
b. Metronidazole
c. Ciprofloxacin
Confidentiality (Healthcare Ethics)
3- (NEW) Scenario about a resident in an elevator who told his junior to “Tell Mr.Ibrahim
to continue on his usual HIV medications”. Junior knew there were drugs that could
have better response. There were other healthcare professionals in the elevator.
Which rule was violated?
a. Confidentiality
b. Knowledge
Breast Cancer
4- (NEW) 42-year-old woman who comes for first time and wants to be screened for
breast cancer, what could be a risk factor?
a. Family history in sister
b. Late menarche
c. Multiparity
d. Breastfeeding
e. Early first pregnancy
Carpel Tunnel
5- (NEW) What is TRUE about hand examination?
a. Carpal tunnel causes thenar muscle wasting
b. Koilonychia is caused by liver disease
c. Osler and heberden nodes seen in patients with osteoarthritis
Cluster Headache
6- (NEW) Man with headache behind the eye that was worse at night and woke him up
from sleep. Had history of these headaches that would go into remission and relapse
later
a. Cluster Headache
6th batch- first group- Males
Ambulatory block (ER, FM, Anesthesia)

b. Tension Headache
c. Migraine Headache
Asthma
7- (NEW) What would be suggestive of a diagnosis of asthma
a. Triggered by aspirin and beta-blockers
8- (NEW) What is the most effective medication for long term control in asthma?
a. LABA
b. SABA
c. ICS
Medical Errors
9- (NEW) Most common error that is cause of morbidity and mortality worldwide
a. Wrong diagnosis
b. Wrong patient
c. Wrong treatment
Dysphagia
10- (NEW) 26-year-old man with 10 pack smoking history comes in for dysphagia. What is
the appropriate management
a. Discuss Ideas, Expectations, Concerns
b. Consult him on a trial of smoking cessation and see response
Prostate Screening
11- (NEW) A patient comes to you and wants you to check his prostate by measuring PSA
levels, what would you do?
a. Discuss pros and cons of PSA
b. Do PSA
c. Suggest doing TRUS
Depression
12- (NEW) Which of the following is the most common cause of chronic fatigue?
a. Anemia
b. Depression
c. hypothyroidism
6th batch- first group- Males
Ambulatory block (ER, FM, Anesthesia)

ER:
Toxicity
13- (NEW) How to differentiate between sympathomimetic toxicity and anticholinergic
toxicity?
a. Diaphoresis in sympathomimetic while anticholinergic has dry skin
Fetal Heartbeat
14- (NEW) Earliest time you can hear a fetal heartbeat?
a. 5 weeks
b. 10 weeks
c. 8 weeks
d. 6 weeks
Orbital Cellulitis
15- (NEW) Child presented with fever, eye pain, diplopia. Skin red around the eye and eye
appears to be bulging out. What is your diagnosis
a. Orbital Cellulitis
b. Peri-orbital cellulitis
c. Open angle glaucoma
Heat Exhaustion
16- (NEW) Guy who ran a marathon, while it was hot and humid outside, later started
complaining of abdominal and leg cramping and pain. He was vitally stable with no
other symptoms. What is the most likely diagnosis?
a. Heat Exhaustion (Most probable answer)
b. Heat Stroke
c. Rhabdomyolysis (some chose this answer because of exertional rhabdomyolysis)
Diabetes
17- (NEW) A girl who ate her grandmother’s diabetes pills (sulfonylureas) and became
hypoglycemic. Was given glucose but still hypoglycemic. What medication would
inhibit insulin release?
a. Octreotide
b. Somatostatin?
c. Glucagon
Evidence based Medicine & Statistics
18- (NEW) Drug X was found to improve some lab values compared to drug Y. What type of
evidence is this?
a. Disease oriented evidence
b. Person oriented evidence
6th batch- first group- Males
Ambulatory block (ER, FM, Anesthesia)

19- (NEW) 5000 people were enrolled in study to do colonoscopy for screening of colon
cancer. What does this study tell you?
a. Prevalance
b. Incidence
c. Sensitivity
d. Specificity
e. The secret of life
ECG AND (PE + Myocardial Ischemia)
20- (NEW) Most common ECG finding in PE
a. Sinus tachycardia
b. S1Q3T3
21- (NEW) Earliest sign of myocardial ischemia on ECG
a. Peaked T wave
b. O wave
c. ST elevation
d. ST depression
Heart Block
22- (NEW) Guy came into ER with hypotension and bradycardia. ECG showed Mobitz type II
AV-block. How would you manage acutely?
a. Transcutaneous Pacing
b. IV epinephrine
c. Defibrillation 200J
d. Synchronized Cardioversion 200J
Sepsis
23- (NEW) According to the sepsis guidelines what is NOT a step you do in the first 3 hours
a. Remeasure lactate
b. Measure Lactate
c. Give crystalloid
d. Give antibiotic
e. Take culture
6th batch- first group- Males
Ambulatory block (ER, FM, Anesthesia)

(X) Decompression Sickness?


24- (NEW) Deep sea diver who went for a dive and tried to show off or something. Later
started having joint pain
a. Decompression Sickness
b. Air Embolus
Fluid Therapy
25- (NEW) Which of the following is best describes conservative fluid therapy?
a. Minimize blood products use early.

Medico-legal Aspects in ER
26- (NEW) Framing is one of the problems that can happen in ER. Which of the following
will help prevent this problem?
a. Taking consent
b. Documentation about everything you do in ER
Toxicity
27- (NEW) Which of the following poorly binds to charcoal?
a. Iron
b. Carbamazepine
c. Theophylline
28- (NEW) First line in treating of beta blocker toxicity?
a. Glucagon

29- (NEW) A weird deleted question: female with DKA symptoms asking about the
diagnosis, but all of the answers were about the management, which included:
a- Insulin IV push of 7-10 units (which could be true if the question was about the initial
management)
b- insulin drip of 0.1U/kg/hr (which could be true if the question was about controlling
the patient)
6th batch- first group- Males
Ambulatory block (ER, FM, Anesthesia)

Anesthesia:
Painkiller
30- (NEW) What is the most potent painkiller
a. Sufentanil
b. Fentanyl
c. Alfentanyl
d. Morphine
Pregnancy Physiology
31- (NEW) All of the following increase during pregnancy except
a. Functional Residual Capacity
b. Tidal Volume
c. Respiratory Rate
Spinal Anesthesia
32- (NEW) Which of these layers are not pierced during spinal anesthesia?
a. Posterior longitudinal ligament
b. Ligamentum Flavum
c. Supraspinous ligament
d. Interspinous ligament
e. DuraMater
Drug Administration
33- (NEW) Child in pain with difficult IV access. What can be given IM
a. Ketamine
Minimum Alveolar Concentration (MAC)
34- (NEW) Definition of MAC
a. The conce ntration at which 50% of patients respond to a surgical stimulus
Link:https://accessanesthesiology.mhmedical.com/Sol/SolQuestion?solQuizId=398448&solQuestionId=3
6th batch- first group- Males
Ambulatory block (ER, FM, Anesthesia)

These are the repeated concepts:

ER:
DKA
1. (1X) Case of DKA, Patient had been given fluids and insulin, Glucose was still not in
normal range, HCO3 was <6, also had hypokalemia. He was feeling tired and fatigued
(metabolic acidosis). Next step of management?
a. Give HCO3
b. Give bolus fluids/normal saline
c. Give K and STOP insulin
d. Give bolus of insulin
2. (1X) What guides management of DKA?
a. Anion gap
b. Serum glucose
c. Urinary ketones
d. pH
Medico-legal Aspects
3. (1X) To prevent framing/being framed should?
a. ISBAR
b. SOAP
Evidence Based Medicine (EBM)
4. (1X) Definition of Evidence Based Medicine
Preventative Medicine
5. (1X) What is an example of tertiary prevention?
a. Screening fundoscopy
b. Diabetic control
Toxicity
6. (1X) Morphine overdose treatment?
a. Naloxone
STD
7. (1X) Which STD needs lifetime of suppression?
a. Gonorrhea
b. Chlamydia
c. HSV
Asthma
8. (1X) What is the most-potent and proven effective long-term treatment of Asthma?
a. Inhaled corticosterioids
6th batch- first group- Males
Ambulatory block (ER, FM, Anesthesia)

b. SABA
c. LABA
Post-menopausal bleeding
9. (1X) Post-menopausal female with bleeding. What’s first step of management?
a. Give OCP
b. Pelvic US
c. Endometrial biopsy
OCPs (Side effects)
10. (1X) What is a side effect of OCPs?
a. Increase risk of Venous thromboembolism
Limping
11. (1X) A 4-year-old boy suddenly develops a limp? No other symptoms what is your
diagnosis?
a. Transient synovitis
b. Growing pains
c. Avascular necrosis of femur
d. Trauma
HTN & DM
12. (1X) Patient with HTN, DM what medication can help reduce mortality?
a. Simvastatin
Rotavirus
13. (1X) Patient from india trip came back with diarrhea for last 4 days. Most likely
organism?
a. Rotavirus
b. Shigella
c. E coli
Beck’s Triad
14. (1X) What is beck’s triad?
a. JVD, Muffled heart sounds, hypotension
Blood Transfusion
15. (1X) In blood transfusion emergency, which of the following is correct?
a. Give males and post-menopausal women O-
Pneumo/Hemo-thorax
16. (1X) Hemothorax X-ray: management?
a. chest tube pointed downwards
6th batch- first group- Males
Ambulatory block (ER, FM, Anesthesia)

ABCDE
17. (1X) ABCDE wrong abbreviation?
a. D for Danger

Lyme disease
18. (1X) Lyme disease after tick bite: presented with palsy?
a. Borrelia burgdorferi

Ectopic Pregnancy
19. (1X) Ectopic pregnancy: what to do next if u/s –ve and urinary hCG is low/negative
a. Do blood hCG

PCA
20. (1X) PCA advantage:
a. better patients satisfaction

Medical Ethics/Principles
21. (1X) Which principle does “Do no harm” refer to?
a. Non-maleficence
Toxicity (insecticide)
22. (1X) Farmer exposed to insecticide:
a. organophosphate overdose

Otitis Media
23. (1X) patient presents with acute otitis media, what is the drug of choice?
a. Amoxicillin, Augmentin
Shockable Rhythms
24. (1X) the drug combination mostly being used in shockable rhythms is:
a. amiodarone& epinephrine

ECG Torsade de points


25. (1X) picture of an ECG with torsade, what is the management?
a. Magnesium Sulphate

PEA

26. (1X) PEA is treated with?


a. IV epinephrine
b. IV atropine
c. Defibrillation (it’s a non shockable rhythm)
d. IV amiodarone
6th batch- first group- Males
Ambulatory block (ER, FM, Anesthesia)

ECG

27. (1X) Stable SVT ECG, Tx?


IV adenosine

Toxicity & Overdose

28. (1X) Cocaine overdose, what do you give?


a benzo
b. nitro
c. b blockers

Medico-legal Aspect

29. (1X) What you do at sign out?


a. SBAR

ECG (Heart block)


30. (1X) A patient presents to you with bradycardia, ECG was provided and it showed
Mobitz type 2. How would you manage?
a. Insert pacemaker.
6th batch- first group- Males
Ambulatory block (ER, FM, Anesthesia)

FM:
31. (1X) Risk for breast cancer ->
a. alcohol consumption
32. (1X) Diabetic develops HTN, first line drug to treat his HTN ->
a. ACEi
33. (1X) Man with HTN is on ACE i, develops chronic cough so doctor changes his
medication. Which medication did he change it to ->
a. Losartan (ARB)
34. (1X) 1st line drug for the treatment of newly diagnosed DM ->
a. Metformin
35. (1X) Lady's husband died 4 months ago, she has stopped eating, 2% weight loss, can
hear her husband's voice, how would you manage? -->
a. Bereavement clinic for 1 year
36. (1X) First line treatment for 1ry dysmenorrhea ->
a. NSAIDs
37. (1X) Screening for AAA with US ->
a. 67 y/o male smoker
38. (1X) Pubmed is an example of ->
a. 1ry source
39. (1X) First step in the management of obstructive sleep apnea ->
a. weight loss
40. (1X) Single blood test that would be expected to be the most sensitive for determining
whether the patient is euthyroid, hypothyroid or hyperthyroid? ->
a. TSH
41. (1X) A nurse sustained a needle stick injury while assisting in a procedure at the clinic
yesterday. The patient’s status is unknown for any blood-borne illnesses. The nurse is
vaccinated against Hep B. Which of the following is the most appropriate next step?
a. Investigate both the nurse and patient, for blood-borne illnesses
42. (1X) Asthmatic on ventolin, has exacerbations at night, next step? ->
a. add low dose inhaled CS
43. (1X) What is true about bariatric surgeries?
a. Nutritional deficiencies can be replaced with supplements
44. (1X) A case of headache (description is that of tension) for which patient is now using
around 200 pills a month:
a. Chronic daily HA (medication overuse headache)
45. (1X) Mononucleosis typical case; question about the virus 
a. EBV
46. (1X) Patient with Cardiac abnormalities had stroke, which of the following is the likely
cause?
a. Atrial Fibrillation
6th batch- first group- Males
Ambulatory block (ER, FM, Anesthesia)

47. (1X) A question about the differences between primary, secondary, and tertiary
prevention.
48. (1X) Vaccination to a woman in a prenatal visit with no rubella antibodies should be?
a. AFTER delivery (right after)
49. (1X) Best treatment of low HDL:
a. 1st exercise (best drug: niacin)
50. (1X) Best treatment of high LDL:
a. statin
51. (1X) HBV + surface antibodies and negative antigens 
a. good immunity against HBV
52. (1X) Woman with pain at sole of the foot, worse in morning, better with long distance
walking, what is the Dx?
b. Osteomalacia
c. Calcaneal epiphysisitis
d. Plantar fasciitis -
53. (1X) STD with PID (cervical motion tenderness, bilateral adnexal tenderness, and
purulent discharge) and Bac Vaginosis also for 1 week thin discharge with fishy odor,
what management?
a. Metronidazol, Clinda, Rocephin
b. Azithromycin, Rocephin (Ceftriaxone) , Metronidazole (MLA)
c. Doxycycline, Cipro, Metronidazole
d. Other options that didn’t have metronidazole
54. (1X) There was a question about a man came with back pain I and they said what can be
considered as emergency
a. Pain radiation to the big toe
b. Weakness of group of muscles in the leg
c. Numbness in the buttocks region (saddle anesthesia- wasn’t written)
55. (1X) Obese patient with BMI of 32, without comorbidities. What would you advise him?
a. Lifestyle modification
b. Medications to lose weight
c. Surgery
56. (1X) Whom from the following patients would you give antibiotics according to Centor
criteria:
a. 10 year old boy with lymphadenopathy, fever, no cough, tonsilar redness
57. (1X) atypical MI :
a. Diabetic patient
58. (1X) a patient presents with typical signs of PID, which of the following tests would you
do?
6th batch- first group- Males
Ambulatory block (ER, FM, Anesthesia)

a. DNA PROBE TEST  for chlamydia


59. (1X) What can help you in diagnosing COPD?
a. Spirometry
60. (1X) patient on rescue salbutamol and max dose inhaled steroids, still not controlled,
next step?
a. Add LABA
61. (1X) Agoraphobia means:
a. fear of insects such as spiders
b. - fear of closed places
c. - fear of crowds
62. (1X) African American patient diagnosed with uncomplicated HTN what is the 1st line
treatment?
a. Carvedilol
b. Amlodipine
c. Hydralazine
6th batch- first group- Males
Ambulatory block (ER, FM, Anesthesia)

Anesthesia
63. (1X) What is not correct about spinal anesthesia?
a. give it thoracic
b. it’s inserted at L3-4/L4-5
64. (1X) Way to monitor CPR effectiveness:
a. Capnography
b. intraarterial pressure
c. systolic Blood pressure
65. (1X) What reduces the EtCO2?
a. Decrease minute ventilation
b. Increase tidal volume
c. Decrease RR
d. malignant hyperthermia
66. (1X) All of the following increase during pregnancy except
a. Functional residual capacity
Anesthesia
1. What is the most common site for arterial injection?
a. Radial artery
b. Femoral artery
c. Brachial artery
d. Inguinal artery
e. Popliteal artery

2. An anesthesiologist injected thiopental in the artery by mistake. What would happen as a


result?
a. Chemical arteritis
b. Lactic acidosis
c. Venous embolism

3. Which one of the following is an intravenous hypnotic anesthetic?


a. Sevoflurane
b. Fentanyl
c. Propofol
d. Nitrous oxide

4. Which of the following is not a definitive airway?


a. The tip of the ET tube below the vocal cords in the trachea
b. Tracheostomy
c. LMA
d. The ET tube connected to a ventilator

5. At what level do you insert the needle for spinal anesthesia?


a. L3-L4
b. L1-L2
c. L5-L6

6. Which of the following is true about EtCO2?


a. Hyperventilation decreases EtCO2
b. Hypoventilation decreases EtCO2

7. What is true about the LMA?


a. LMA is less traumatic
b. Used for long surgeries

8. At what level does the spinal cord end?


a. Lower border of L1
b. Upper border of T12
c. Lower border of L3
d. Lower border of L4
9. While giving an epidural which layer do you not cross?
a. Ligamentum flavum
b. Dura
c. Infraspinous ligament
d. Supraspinous ligament

Emergency Medicine
10. A cocaine abuser came to ER with chest pain and tachycardia. What would you give this
patient?
a. Beta blocker
b. Calcium channel blocker
c. Benzodiazepine

11. An unstable, bradycardiac patient’s “ECG is given below”. What would you do? [They did
not provide an ECG for this question]
a. Give epinephrine
b. Cardioversion
c. Transcutaneous pacing

12. Which of the following is seen in a chronic liver disease patient?


a. Tenosynovitis
b. Palmar erythema

13. Which one of the following is right about CPR?


a. The rate of the chest compressions is 80/100/min
b. The rate of the chest compressions is 100-120/min
c. The rate of the chest compressions is 60-80/min

14. A guy fell on his right shoulder and as unable to abduct is arm from 0 to 90 degrees.
Which of the following muscle is affected?
a. Supraspinatus
b. Teres minor
c. Subscapularis
d. Infraspinatus

15. A known epileptic patient was given IV lorazepam to stop his seizure in the ER and it
successfully stopped the seizure. 20 minutes later he started seizing again, what would
be your management?
a. IV phenobarbital
b. IV phenytoin
c. Rectal lorazepam
16. Which of the following rhythm would you give a synchronized cardioversion?
a. Ventricular fibrillation
b. Pulseless ventricular tachycardia
c. Sinus tachycardia
d. Unstable atrial fibrillation

17. A 35 year old male (BMI 42) with poorly controlled diabetes, hypertension and low back
pain presents to you in the clinic. He has tried all possible ways to reduce his weight for
the past 2 years and nothing seemed to work. What would you advise him?
a. Intensive diet therapy and exercise for 6 months
b. Referral to bariatric surgery
c. Intensive diet therapy for 6 months

18. A positive D dimer value of >500, sensitivity 95%, specificity of 80%, PPV of 85%. Which
of the following statement is correct?
a. 85% of the patients with positive test will have the disease
a. 95% of the patients with positive test will have the disease
b. 90% of the patients with a positive test will have the disease

19. A patient presented wiîh excessive sweating, dilated pupils and high blood pressure.
What toxidrome is it?
a. Cholinergic
b. Anticholinergic
c. Sympathomimetic
d. Opioid overdose

20. How would you differentiate between a sympathetic and anticholinergic toxicity?
a. Diaphoresis
b. Mydriasis
c. Miosis
d. Tachycardia

21. A patient’s labs showed hypercalcemia (calcium was 13). What is your next step?
a. Give IV normal saline and bisphosphonates.
b. Calculate corrected calcium using total serum proteins

22. Regarding blood transfusion in the emergency department, which of the following is
true?
a. premenopausal females should be given O Rh-positive blood
b. men and postmenopausal females can be given O Rh-positive blood

23. A man post diving had pain in his hips and different joints. He had no issues while
ascending. What is the condition?
a. Decompression sickness
b. Air embolism
c. Carbon dioxide narcosis
24. A diver post ascent, lost his consciousness. What could be the reason?
a. Air embolism
b. Carbon dioxide narcosis

25. Which of the following will help in the diagnosis of iron deficiency anemia?
a. Low iron
b. Low hemoglobin
c. Low ferritin

26. Which of the following is causes an increased anion gap?


a. Salicylates
b. Ethanol
c. Paracetamol

27. Which of the following would you monitor while managing DKA?
a. Serum glucose
b. Anion gap

28. Which one of the following is a contraindication for inserting a NGT?


a. Penetrating neck trauma
b. C-spine fracture
c. Blood at meatus

29. A young pain presents to the ER with sudden anal pain. He is known to have chronic
constipation. On examination, there is tenderness on the posterior anal region. What is
the most likely diagnosis?
a. Anal fissure
b. External hemorrhoids
c. Diverticulitis

30. Which one of the following is the Beck’s triad?


a. Muffled heart sounds, increased JVP and hypotension
b. Hypercoagulability, stasis, vessel injury
c. Fever, jaundice and RUQ pain
d. Recurrent oral ulcers, genital ulcers, iridocyclitis

31. Which one of the following is an absolute contraindication for flying in a commercial
flight?
a. Pneumothorax
b. Pregnant 30 months

32. A heart failure patient on aspirin and ACE inhibitor. Which of the following can be added
for secondary prevention?
a. Beta blocker
b. Nitroglycerin

33. Which of the following is the most effective method of knowing the CPR quality?
a. Arterial oxygen
b. Capnography
c. Systolic blood pressure

34. According to the Sepsis care bundle, which of the following do you not to do in the first 3
hours?
a. Measure Lactate
b. Give antibiotics
c. Take blood cultures
d. Remeasure Lactate

35. A known case of breast cancer patient comes with fever ( do not remember the
symptoms, her vitals were not normal but diagnosis was she is in sepsis). What would
you for her?
a. Give IV fluids, take blood cultures and give antibiotics
b. Take blood sample and wait for results
c. Give her antibiotics and send her home

36. Which of the following is the most common ECG finding in PE?
a. Sinus tachycardia
b. S1T3Q3
c. ST elevation
d. PR elevation

37. The principle of do not harm is called?


a. Non- Maleficence
b. Beneficence
c. Autonomy
d. Maleficence

38. Which one of the following is an indication for thrombolysis?


a. Symptoms for more than 8 hours
b. Definite ischemic stroke with focal neurological symptoms
c. High serum glucose level
d. Systolic blood pressure of 200mmHg

39. Which of the following is the most common electrolyte abnormality associated with
cardiac arrest in ESRD patients?
a. Hypocalcemia
b. Hypernatremia
c. Hyperkalemia
d. Hyponatremia

40. Which of the following STDs need lifelong treatment?


a. Trichomonas
b. Herpes simplex virus
c. Chlamydia
d. Gonorrhea

41. A lady came with abnormal vaginal discharge. It is most likely?


a. Bacterial vaginosis

42. Which of the following is done to confirm a diagnosis for gonorrhea and chlamydia
infection?
a. DNA probe test
b. PCR

43. A 25 year old female overdosed herself with her grandmother’s sulfonylureas. Her blood
glucose does not get corrected even after giving her glucose and water. Which of the
following will help in treating her?
a. Octreotide

44. Which of the following medical errors is most common cause to increase the mortality
and morbidity?
a. Wrong diagnosis
b. Wrong treatment
c. Wrong medication
d. Wrong dose

45. A patient has a stroke affecting his left frontal lobe. What symptoms would you notice in
him?
a. Contralateral hemiplegia
b. Loss of vision in the left eye
c. Emotional and behavioural changes
d. Ipsilateral paresthesia

46. A case of acute glaucoma. What would you do?


a. Give topical beta blocker urgently and referral to ophthalmology
b. Follow up appointment with ophthalmology

47. Which one of the following is it not a sign of unstable heart rhythm?
a. Hypertension
b. Chest pain
c. Dyspnea
d. Altered mental status

48. Which of the following is the first sign on a ECG in MI?


a. Peaked T wave
b. ST elevation
c. ST depression

49. Which of the following is the first line treatment for beta blocker toxicity?
a. Glucagon
b. Insulin
c. Potassium

50. Which of the following can prevent framing?


a. Proper sign out
51. To avoid framing the method of communication used is?
a. SBAR
b. ICE
c. SAMPLE

52. A patient started having symptoms of stroke 2 hours ago. What is the time frame to get a
non enhanced CT scan for this patient?
a. 10 minutes
b. 15 minutes
c. 25 minutes
d. 45 minutes
e. 60 minutes

53. A guy after running a marathon while it was hot and humid outside, later started
complaining of abdominal and leg cramping and pain. He was vitally stable with no other
symptoms. What is the most likely diagnosis?
a. Heat exhaustion
b. Rhabdomyolysis

54. Child presents with fever, eye pain, diplopia. Skin red around the eye and eye appears
to be bulging out. What is your diagnosis?
a. Orbital cellulitis
b. Periorbital cellulitis
c. Open angle glaucoma

55. A patient is diagnosed with septic shock and needs resuscitation. As per Surviving
Sepsis Campaign Care Bundles which one of the following is the recommended initial
dose of fluids?
a. 20 ml/kg of Crystalloids
b. 30 ml/kg of Crystalloids
c. 40 ml/kg of Crystalloids

56. Which of the following is true regarding Balanced Resuscitation in a trauma patient?
a. Give copious amounts of IV Saline
b. Delay blood products as much as possible
c. Give blood products early during resuscitation to maintain a blood pressure of 120/80

57. When is the earliest you can listen to the fetal heart sounds?
a. 6 weeks
b. 8 weeks
c. 10 weeks

58. A classic scenario of acute appendicitis (right lower quadrant pain with rebound
tenderness), what is the diagnosis?

59. A classic scenario of acute pancreatitis, amylase was elevated, what is the diagnosis?

Family Medicine
60. A study was designed to compare the diagnostic accuracy of US vs CT in diagnosing
appendicitis. What is the study type?
a. A case control study
b. A case report
c. A randomised control trial
d. A cohort study
e. A case series

61. Drugs X and Y were compared to study the incidence of stroke in hypertensive patients.
Based on the results, the physicians chose to give drug X to their patients. What is this
practice called?
a. Disease based
b. Evidence based medicine

62. Counselling a patient to come to ER in case of alarming symptoms is called?


a. Safety netting
b. ICE
c. SBAR

63. Which of the following would confirm an active TB infection?


a. Sputum containing acid fast bacilli
b. Chest x ray
c. Quantiferon test

64. A patient is hypertensive and diabetic, what drug would you give?
a. ACE inhibitors
b. CCB
c. Aspirin
d. Beta blockers

65. Which of the following would you give for a patient with low output HF?
a. ACE inhibitors
b. Beta blockers
c. CCB
d. Aspirin

66. Which one of the following drugs increase insulin sensitivity?


a. Sitagliptin
b. Insulin
c. Metformin
d. Glyburide

67. A patient was started on a drug for his dyslipidemia and presents few weeks later with
muscle pain and cramping. Which of the following drugs would have caused it?
a. Statins
b. Niacin
c. Cholestyramine
d. Ezetimibe

68. A lady presented with bone pain, striae on abdomen and hirsutism. Which if the following
drugs could have caused this?
a. Steroids

69. Which of the following is true regarding the evidence to treat IBS?
a. Peppermint oil is clinically proven to benefit
b. Low FODMAP diet has strong clinical evidence
c. Gluten free diet
d. Lactose free diet has strong clinical evidence

70. A nurse had a positive PPD results after being in contact with an active TB patient. She
is asymptomatic and her CXR is normal. How would you manage her?
a. INH for 6 mo
b. INH plus rifampicin
c. Give her the TB regimen

71. A 35 year old female visited the family medicine clinic for contraception counselling.
What is your role as a family physician?
a. Review the contraception options with her
b. Education
c. Tell her what is the best way of contraception for her
72. Which of the following would you not prescribe combined oral contraceptives?
a. A 25 year old female with migraine with aura
b. A 35 year old female with family history of DVT
c. A 30 year old female who smokes
d.
73. A patient with BMI 28 with serum blood glucose of 6.5mmol/L (Normal: Diabetic: ) .
What would you advise him?
a. You are obese and need to lose weight
b. You need to lose weight in order to prevent diabetes mellitus
c. You are obese and have diabetes mellitus

74. Which of the following is the most common cause for chest pain?
a. MSK pain
b. GERD
c. Heart attack

75. What is the first line treatment for generalised anxiety disorder?
a. SSRI
b. SNRI
c. Cognitive behavioural therapy
d. Antipsychotics

76. Which of the following is the first line treatment for smoking cessation?
a. Nicotine replacement therapy
b. Fluoxetine
c. Venlafaxine
d. Mirtazapine

77. A patient initially had difficulty swallowing solids but can’t swallow liquids now. He also
has been expericine weight loss. What is it most likely? [do not remember the exact
details but it was classic scenario for esophageal cancer]
a. Esophageal Ca
b. CRC cancer
c. IBS

78. A classic scenario for CRC cancer asking for diagnosis (blood per rectum, weight loss
etc)

79. During surgery, the surgeon had a needle stick injury. Which of the following is true?
a. Both the patient and the surgeon need to have their blood tested

80. Which one of the following vitamins need to be replaced post gastrectomy?
a. Thiamine
b. Vitamin K
c. Vitamin C
d. Vitamin B12

81. Which of the following is a gold standard for diagnosing COPD?


a. Chest X Ray
b. Spirometry

82. A 55 year old male, non smoker presented to you at the clinic for his regular follow up.
Which one of the following would you do for him?
a. Fecal occult blood
b. Abdominal US
c.
83. Young patient with bloating, abdominal pain relieved by defecation, no red flags. What is
the diagnosis?
a. Irritable bowel syndrome
b. CRC

84. A smoker for the past 20 years has chronic cough. His CXR shows hyperinflation. What
is the most likely diagnosis?
a. COPD
b. Asthma
c. Pneumonia

85. A 40 year old wants to screen for osteoporosis and have a xray done, what do you tell
her?
a. She can have her xray done right now
b. BMD at 65 years for osteoporosis

86. Which of the following goes with the diagnosis of asthma?


a. Diurnal variation of symptoms
b. Prominent light headedness, tingling sensation in fingers and fainting episodes

87. An asthmatic patient is already taking his regular medication (SABA) but his symptoms
are still not improving. Which of the following medication should you add?
a. LABA + Steroid
b. Oral steroid
c.
88. A young health boy is having back pain with no red flags. How would you treat him?
a. Reassurance
b. Give analgesia and educate about proper posture and back care
c. Do a CT spine
89. A patient presented with a complaint of repeated episodes of headache which are very
severe and associated with rhinorrhea and lacrimation. It is not associated with any
aura. What is the diagnosis?
a. Cluster Headache
b. Tension type headache
c. Migraine

90. Which of the following would present with atypical MI?


a. Diabetics
b. Young

91. Which of the following is a primary source of evidence?


a. Pubmed

92. Which of the following should be recommended to asthmatic patients for secondary
prevention of asthma? [We don’t remember the other options]
a. Use hypoallergenic pillow covers
b. Use air humidifier

93. Which of the following is the most sensitive test for diagnosing thyroid disorders?
a. TSH
b. T4
c. Ferritin
d. T3

94. A study to see the effect of statin post MI. What is the number needed to treat?
[Calculate using the formula, calculators weren’t provided]
Simvastatin(N= 2222) Control (N=2220)

151 (6.8%) 197 (8.9%)

a. 56
b. 48
c. 100
d. 82

95. Which of the following is the most sensitive test for diabetic nephropathy
a. Albumin creatinine ratio
b. Serum creatinine
MCQ

1- What is correct about paracentesis:


A- SAAG > 250 WBC/mm3 with >50 % PMNs reflects SBP
B- SAAG <1.1 = portal HTN
C- 3 liters could be removed

2- Presentation about itchiness and vesicles in a child. His father has the same
symptoms.
A- Scabies

3- A mother brings her child complaining of chronic eczema. What do you do?
A- Give emollients and topical steroids for flare ups
B- Consult derma for allergy test
C- Topical emollients only
D- Topical steroids only

4- NMDA antagonist:
A- Ketamine

5- Bullous myringitis (with picture):


A- Incision and drainage
B- Give ceftriaxone
C- Paracetamol and discharge

6- Picture of flat ECG:


A- Epinephrine

7- A girl swallowed a lot of her grandmother’s Sulphonylurea drugs. What can be


given that will inhibit insulin completely:
A- octreotide

8- X-ray picture “after multiple surgeries”:


A- Small bowel obstruction
B- Bowel perforation

9- Presentation of eye trauma and bruising and limited eye movement:


A- CT for orbital floor fracture

10- Diagnosis of trichomonas vaginalis:

1
A- Saline wet mount microscopy
B. Whiff test
C. DNA probe test

11- CT picture of brain:


A- Subdural bleeding (bridging veins)

12- Low motor neuron bell’s palsy:


A- Prednisolone
B- Prednisone and acyclovir

13- Picture of ECG:


A- Amiodarone and adrenaline

14- 15 year old girl was stung by a bee while picking flowers and is having an
anaphylactic reaction. What do you give her?
A- Epinephrine IM 1:1000

15- Study statistical significance losartan with CI 95% (0.65-0.95):


A- The study shows statistical significance
B- The study showes clinical significance
C- The study shows no statistical significance
D- There is insufficient information to know the statistical significance

16- 60 year old man came to the clinic asking to get screened for prostate cancer,
what would you tell him?
A- PSA screening is NOT recommended
B- Do PSA for him
C- PSA is recommended according to many guidelines
D- PSA high means prostate cancer

17- Snake bite:


A- Venom prolongs bleeding time

18- When is the cardiac output highest in the pregnant mother?


A- Right after birth
B- First trimester of pregnancy
C- Second trimester of pregnancy

19- Which of the following decreases in pregnancy?

2
A- Systemic vascular resistance
B- CVP
C- Cardiac output
D- Stroke volume

20- Eight weeks old baby, mom complains of watery right eye, last week he had a
viral infection and now his symptoms are worse. What is the next step in
management?
A- Educate mother on nasolacrimal massage
B- Tell mom to clean the upper eyelid and apply warm compress
C- Urgent referral to ophthalmology
D- Routine referral to ophthalmology

21- Patient on warfarin presented with subconjunctival hemorrhage. What is your


next step?
A- Check PT and INR

22- Which of the following is NOT included on premarital screening in KSA?


A- Illicit drug use
B- HIV
C- Hep C
D- Hep B
E- Sickle cell anemia

23- A case of the following labs:


Hepatitis surface antibodies: positive
HBsAg: negative
HBcAg: negative
A- Patient has adequate immunization against hep B

24- A nurse having her routine healthcare checkup was positive for PPD. CXR
was done and was negative for tuberculosis. Her previous PPD was negative.
What is the management?
A- Isoniazid for 6 months

25- Which of the following drugs has an antidote?


A- Diazepam

26- Which drug causes malignant hyperthermia?


A- Succinylcholine

3
27- At what age do you start screening for breast cancer with mammogram
according to NICE and Task Force?
A- 30
B- 40
C- 45
D- 50
E- 65

28- Relative contraindication for spinal anesthesia:


A- Low back pain
B- Hypotension
C- Coagulopathy
D- Infection at site
E- Severe valvular stenosis

29- Which of the following should you not use in a pregnant woman trying to stop
smoking?
A- Nicotine gum
B- Nicotine patch
C- Nicotine pills
D- Bupropion

30- How long should you wait for doing a surgery on a baby that has just been
breastfed?
A- 4 hours
B- 2 hours
C- 6 hours

31- Breastfeeding lady complains of breast engorgement and discomfort. What


do you advise her?
A- Continue feeding and warm compress
B- Topical Abx
C- Stop breastfeeding

32- Man presents with sore throat + fever + tender lymphadenopathy + no cough.
What is your management?
A- Amoxicillin
B- Rapid strep test

4
33- Which of the following is a part of Centor criteria?
A- Absence of cough

34- 17 year old presented with pharyngitis symptoms, had splenomegaly and
cervical lymphadenopathy. What is the causative organism?
A- EBV
B- HIV
C- GBS

35- Which of the following is true about Dexmedetomidine?


A- It has analgesic properties

36- A 22 year old had acne, and you started her on Isotretinoin. What should you
warn her about?
A- Teratogenicity
B- CAD risk factors

37- Cochrane library is an example of:


A-Primary source
B-Secondary source
C- Electronic library
D- Both primary and secondary source

38- Beta blocker toxicity:


A- Give insulin and glucagon

39- Diverticulosis:
A- Increase fiber intake
B- Reassurance

40- Findings from a randomised control study showed you that the follow up rate
was 60%. What does that tell you about the study?
A- Low validity
B- High validity
C- Low reliability
D- High reliability

41- What differentiates between sympathomimetics and anticholinergics?


A- Diaphoresis

5
42- What is a symptom of opioid overdose?
A- Miosis
B- Diarrhea
C- Resp stimulation
D- Anaphylaxis

43- A patient with MI was given aspirin. What medication should you add for him?
A- Beta blockers

44- In MVA, when do you not do a cervical spine immobilization?


A- Closed fracture of shaft of the femur
B- An alcoholic with a level of 200+ with severe injuries
C- Someone who has only internal bleeding and was thrown outside the car
D- Paraspinal pain with no midline tenderness
E- Neck pain on turning the neck only

45-Woman with pain in finger joints for some time and nodules at elbows:
A- Rheumatoid arthritis

46- An old man presented to the walk in clinic with HTN 210/110 and symptoms of
papilledema. What is your next step of management?
A- refer to ER
B- Give oxygen and treat with IV

47- Which of the following does not cause high anion gap?
A- DKA
B- Ethanol
C- Renal Tubular acidosis
D- CO poisoning

48- Management of DKA:


A- Insulin IV drip

49- What do you NOT do in the first three hours in a case of sepsis?
A- Remeasure lactate

50- How to measure treatment response to DKA?


A- Monitor anion gap

51- First sign of myocardial infarction on ECG:

6
A- Peaked t wave
B- ST elevation
C- O wave

52- Patient with controlled DMII and HTN, has elective laparoscopic surgery for
cholecystectomy. According to ASA: (explanation in between brackets)
A- ASA 2 (Mild systemic illness)
B- ASA 1 (Healthy person)
C- ASA 3 (Severe systemic illness)
D- ASA 4 (Severe systemic illness cannot survive without the surgery)
E- ASA 5 (Brain dead patient doing surgery for organ transplantation)

53- You prescribed amoxicillin to a patient and she had an allergic reaction, which
was managed. What is your next step?
A- Report it to the ministry of health
B- Document it in her patient file

54- Stroke patient came 1 week later with ischemic anterior circulation. What to
do:
A- Stop antiplatelet
B- Carotid doppler
C- Thrombolysis
D- Stop aspirin? Some other medication

55- Stroke patient, what’s the time frame for CT:


A- 25 minute
B- 10 minutes
C- 15 minutes
D- 30 minutes

56- Which IV anesthetic increases blood pressure:


A- Ketamine
B- Propofol
C- Thiopental

57- Child with itching around anal region, cotton-like stuff in stool:
A- Pinworm
B- Tapeworm
C- Hookworm

7
58- Which of the following is true about bites?
A- The organism in cat bites is usually Pasteurella Multocida
B- Human bites are safe
C- All dog bites need antibiotics
D- there was something related to rabies

59- What is the true about triage?


A- It is a fool-proof system
B- The sickest patients are served first

60- What is true about START triage?


A- It is divided into five categories
B- It has a colour coding system

61- What is true about diabetic neuropathy?


A- It's associated with neuropathic pain
B- Neuro stimulatory test diabetic patients will have decrease in the amplitude
C- It can be acute and subacute
D- It happens due to demyelination of the nerves

62- Which of the following is not a sign of unstable rhythm?


A- Chest pain
B- Shortness of breath
C- Tachycardia
D- Hypotension
E- Altered mental status

63- With which of the following drugs do you have to monitor kidney function and
electrolytes?
A- ACE inhibitor
B- Clonidine
C- B blocker

64- A man taking warfarin 4 mg, his INR 3.5. He currently does not have any
bleeding or any problems. What would you do:
A- Decrease warfarin from 4 to 2
B- Stop warfarin and start aspirin
C- Stop warfarin and check INR next day
D- Continue Warfarin and check INR next day

8
65- What is true about mild depression?
A- It is best to wait and watch
B- Increase the doses of the drug

66- Patient with migraine asks for prophylaxis to prevent more episodes. What do
you give?
A- Sumatriptan
B- Atenolol

67- Testicular exam felt like a bag of worms. What is the diagnosis?
A- Varicocele
B- Hydrocele

68- Which one is a symptom in panic attacks?


A- Chest tightness

69- Patient with GAD is on CBT but is not managed. What do you do next?
A- Prescribe fluoxetine (SSRI)

70- Eclampsia case was given mg sulfate and signs of toxicity:


A- Stop mg sulfate and start calcium gluconate
B- Continue Mg sulfate and add K

71- What is true about IBS?


A- Peppermint oil is proven to be beneficial

72- Strawberry tongue and sandpaper rash. Diagnosis?


A- Scarlet fever

73- Symptoms of tiredness, weight gain, fatigue, menorrhagia. What is the


diagnosis?
A- Hypothyroidism

74- A patient who has hypothyroidism is being treated with levothyroxine. She
came in for a follow up. Which of the following will inform you about the
effectiveness of the treatment?
A- T3
B- TSH
C- FT4

9
75- What do you give for fentanyl overdose?
A- Naloxone

76- Postmenopausal women came with complaints of vaginal bleeding. What is


your next step?
A- Transvaginal US and endometrial biopsy

77- Tertiary prevention for cerebrovascular disease:


A- Antiplatelets
B- Speech and verbal therapy therapy

78- A nurse got a needle stick injury while drawing blood from a patient. What
should be done?
A- Check both the nurse and the patient for any blood-Borne diseases

79- Man came in complaining of a sudden headache and a pain score of 10/10,
neck stiffness, and photophobia. What is your next step?
A- Urgent CT to rule out subarachnoid hemorrhage

80- Management for post epidural puncture headache:


A- E Blood patch

81- Scuba diver fainted after rising. What was the cause?
A- Air embolism
B- Decompression sickness
C- The ‘bends’

82- Why should you always document in ER?


A. To prevent any lawsuits

83- Diabetic woman who is taking only metformin with no major complications
came in for follow up. Her HbC was high. Which of the following is true?
A- Adding sitagliptin to her medication regimen is a reasonable choice
B- Prescribe insulin

84- Filipino lady came in with diabetes mellitus and hypertension. What is your
choice of medication?
A- Lisinopril

85- Which of the following decreases etCO2:

10
A- Hyperventilation

86- For which of the following would performing synchronized cardioversion be


appropriate?
A- Unstable atrial fibrillation

87- Asthma patient who takes ventolin prn has to take his medications at least 2
times a week, and multiple times at night. What should you do for him?
A- Add inhaled fluticasone
B- Add LABA
C- Add low dose ICS + LABA

88- What is true about balanced resuscitation:


A- Give copious amounts of crystalloids to maintain low BP 120/80
B- Give blood products early to maintain blood pressure

89- Hypertension patient with risk for coronary artery disease (Probably also had
high LDL). What do you give?
A- Simvastatin

90- Patient with stroke, what makes him a candidate for tPA?
A- INR>3.5?
B- SBP>180 and DBP 110

91- In septic shock case, what is the amount of fluid you administer?
A- 30 crystalloid

92- Diabetic nephropathy how do you follow?


A- Albumin creatinine ratio

93- Patient who had inconsistent BP readings in 6 visits some of them were high,
how to proceed?
A- 24 hour monitor and lifestyle modification

94- What should you do at sign out?


A- SBAR

95- Spinal cord compression due to metastatic tumor management?


A- Identifying the primary tumor is not the utmost priority for the management
B- identifying the level of sensory loss will help you in the management

11
C- identifying the primary tumor will help you in the management

96- Patient had a stroke what to do next


A- stop antihypertensive meds for the next 72 hrs

97- Breast cancer ptx with fever


A- send for MRI

OSCE
- Osama’s awesome notes were enough for everything below.
Group 1:
1) Fatigue patient (either depressed or stressed)
2) Asthma exacerbation case
3) Chief complaint of the pt was (dizziness) Benign paroxysmal positional vertigo
case

Group 2:
1) Epigastric pain and blood in vomit - PUD.
2) 15 year old presented with abdominal pain, fatigue and dizziness - DKA. Take
history, do physical exam, order appropriate labs, manage the patient, and
monitor. (Don’t forget potassium here)

12
3) Patient presented with STEMI. Take history, do physical exam, order appropriate
tests and give appropriate medication. Eventually the nurse tells you that the
patient’s condition is worsening. You have to ask here for the ECG again. It
showed bradycardia, which you have to identify and manage.
4) Anesthesia - know everything about oxygenation, ventilation, spinal and epidural
anesthesia

13
A patient of 60 kg body weight scheduled for surgery and has been fasting for 8 hours, which
statement is true for that patient

a. He needs to fast a minimum of two more hours to avoid aspiration risk


b. His fasting fluid replacement is around 850 mls
c. We need to give him a minimum of 1500 mls as a substitute for fasting hours
d. Fasting fluid replacement is changing based on the type of surgery

Fasting fluid deficit: No.of fasting hours x maintenance fluid requirement (8 x (40 + 20 + 40))

A diabetic with middle ear discharge, pain especially at night, and facial nerve paralysis as well as a
friable tissue and asking you for the best step in management which included?

a. Biopsy
b. oral steroids + antibiotics
c. iv antibiotics
d. oral antibiotic

Toronto: debridement, IV antibiotics and hyperbaric oxygen

Case of DKA, patient had been given fluids and insulin. Glucose was still not normal range. HCO3: <6
and potassium was low. He was feeling tired and fatigued (metabolic acidosis). What is the next step
of management?

a. Give HCO3
b. Give bolus fluids/normal saline
c. Give K and stop insulin
d. Give insulin bolus

Which of the following is not present as an oral preparation for HTN?

a. Atenolol
b. Nifedipine
c. Thiazide
d. Furosemide
e. Captopril

Sodium nitroprusside taken orally is not absorbed from the GI tract, so that would be the most likely
answer. From the above options, not sure what to pick

Kid has 3 months hx of abdominal pain, now he can’t attend school, which of the following would hint
towards an organic rather than a functional cause?

a. Headache
b. Anxiety
c. Suprapubic pain
d. Family hx of abdominal pain

Patient from India trip came back with diarrhea for last 4 days. Most likely organism?

a. Rotavirus
b. Shigella
c. E.Coli

Anticholinergic overdose treatment: Benzodiazepines

Which electrolyte abnormality can’t cause cardiac arrest?

A 25 year old young man presents with sudden onset of complete weakness of one side of the face.
The clinical examination reveals as follows. What is the most appropriate ER management?

a. Reassure, start acyclovir/steroids/artificial tears


b. Immediate CT scan of brain
c. Immediate MRI with referral to neurology
d. Stroke thrombolysis
e. Immediate referral to ENT

Post anesthesia care with high HR, BP and RR an is agitated. What do we give?

Breast cancer history treated, came with back pain symptoms

a. Lumbar X-ray
b. CT spine
c. MRI spine
Anesthesia Questions
1. What does MAC indicate? Potency of anesthetic
2. MAC not affects by weight
3. Epidural needle is larger than spinal MPA
4. SIGNS OF incorrect EET tube: o2 going down
5. Pt scheduled for an elective surgery, but he had a breakfast meal by mistake. How many hours
should the surgery be delayed? 6 hours.
6. Pt with well controlled diabetes, for elective surgery  ASA 2. Check table for this
7. woman with appendectomy: ASA 1 E. Check table for this

8. Oropharyngeal airway, what is not true?


A. It has rounded edges
B. It is only one size/comes in 1 color
C. it’s a bite block
D. it’s a central airway?
9. What is wrong about arterial line?
A. It’s used to measure BP
B. infection can happen
C. Seldinger technique is used to insert it
D. It is used to deliver emergency medications (central line is used intra-op)
10. What is not correct about spinal anesthesia?
A. give it thoracic, Laparoscopic surgery, patients on Plavix, Agitated patients.
B. it’s inserted at L3-4/L4-5
C. Surgeries below T11/T10 (Umbilicus).
D. High risk patients

11. Difficult Intubation what is true:


A. They can extend atlanto axial
B. can open mouth fully
C. > 9 cm from angle of mandible to mentum
D. < 4cm mentor to the thyroid
12. Which one is a IV hypnotic?  Propofol
13. Not correct about RSI? Need to verify this answer
A. It doesn’t need a muscle relaxant (This will be given after induction)
B. No bag masking
14. Way to monitor CPR effectiveness:
A. Capnography
B. intraarterial pressure
C. systolic Blood pressure
15. What reduces the EtCO2?
A. Decrease minute ventilation
B. Increase tidal volume
C. Decrease RR (Increased EtCO2)
D. malignant hyperthermia
16. which is not a definitive airway > LMA
17. Fire inhalation? Mg, NaHco3 (look later)
18. Central IV line: decreasing viscosity + length
19. What analgesic should be given to burn patients  Ketamine.
20. Chemical arteritis can be caused by thiopental
21. Pain with Propofol? Add lidocaine
22. Induction agent has analgesic function too? Ketamine
23. you always check pulse oximetry during OR
24. Airway preanesthesia assessment: answer was no single method is conclusive, use multiple
25. Which of the following can be used for assessing difficult airway? Mallampati score
26. What is the best way to measure body fluids intra-operatively: Central venous pressure.
27. Best way to monitor fluids: (a) dynamic (b) static.

28. Indication to use CPAP: Obstructive sleep apnea


29. Morphine which of the following isn’t a side effect: hallucinations
30. Stent patient preanesthesia: its worrying because they stay on long term blood thinners
31. Ambubag indications: (1) Inadequate oxygenation or ventilation (2) as bridge to intubation (3)
acute emergency cases such as post-trauma.
32. Causes of difficult airway: arched palate, micrognathia, no teeth, denture wearing, beard, ect…
(I forgot the exact choices, but these are the correct answer).
33. Mallampati classification when you cannot see the soft palate: class IV
34. Spinal anesthesia indication: below lumbar, C/S is typical
35. a patient had decrease in O2 saturation post operatively, what is the next step in management?
(a) mask (b) intubation
36. Post op pain
• management best method: PCA.
• Pain best assessed: objective test using numerical values or sad/happy faces (if it’s a
pediatric patient.
• Beneficial effects effective pain management (PCA): (a) relieves suffering (b) Early
mobilization (c) Short hospital stay (d) reduce hospital cost (e) increased patient
satisfaction.
• Most common complication of post-op pain: Developing chronic pain
• Post laparotomy op pain management is important due: normally coughing will
increase pain; therefore, patients avoid coughing, decreasing O2 levels.
• What is the advantage of patient controlled analgesia: better patient’s satisfaction
37. According to ASA guidelines which one of the following is not used intraoperatively for
monitoring:
A. Oxygen analyses
B. ECG continuous
C. BP
D. Peripheral nerve stimulator (Toronto notes; Pg 6 - Monitoring)
E. Capnometry
38. Which one of the following is not a risk factor for difficult bag mask ventilation?
A. Thyromental less than 6 cm
B. Thyro-angle more than 9 cm
C. Mallampati 1
Answers don’t make sense
5 independent risk factors of difficult mask ventilation: (1) Age > 55 years of age (2) BMI > 26 (3)
History of snoring (obstruction) (4) Beard (5) Edentulous
39. The following are independent risk factors for bag mask ventilation except:
A. BMI < 26
B. Edentulous
C. Beard
D. Snoring
E. Short neck
40. What is true about LMA:
A. Secure 100% airway
B. Had to be laryngoscope mediated
C. Less traumatic than ETT
D. Non fasting patient usage
41. Gold standard for ETT confirmation:
A. Capnohraphy
B. Visualization of vocal chords
C. Chest X ray
D. Auscultation
E. Chest movement
42. Narcotics:
A. Morphine is highly lipid soluble
B. Intrathecal morphine is more respiratory depressant than fentanyl
C. The mu receptor is G protein coupled
43. Opioid side effect except:
A. Nausea vomiting
B. Bradycardia
C. Constipation
D. Resp depression
E. Mydriasis (dilated pupils)
44. How frequently should Hr and BP be measured in OR according ASA guidelines:
A. 2 minutes
B. 5 minutes
C. 7 minutes
D. 10 minutes
E. 15 minutes
45. why do we give pre oxygenation before intubation, possible answer is more time for apnea
46. what affect the oxygen transport more: cardiac output
47. which is not associated with high oxygen therapy: No answer, just look up
• Potential adverse clinical consequences of supplemental oxygen therapy include
absorptive atelectasis, accentuation of preexisting hypercapnia, airway injury,
parenchymal lung injury, and extrapulmonary toxicity.
48. what is the best airway management device for respiratory failure? Bi-PAP. Need to check
49. induced by propafol and was Bevin curonium then developed bilateral wheezing and
hypotension (Anaphylaxis secondary to propafol): IM Epinephrine 1:1000 0.5mg

20ml

Normal EtCO2 is 35-45mmHg

A towel can hold 100-150ml


70ml/kg

Succinylcholine only
depolarizing and short acting.
B is better
choice. Would cancel if child was less than 1yr or had systemic symptoms.

D
ER questions
1. Crohns colitis + multiple surgeries who developed small bowel obstruction with photo – X ray
done-diagnosis  Small intestine obstruction
2. Fever, GI symptoms? Ulcerative colitis/Gastroenteritis
3. Signs of severe hypothermia: No shivering, coma/decreased consciousness signs
4. Breaking bad news about hepatitis: time for discussion tx choices, making sure to bringing a
companion.
• Help him prepare for the information by using a statement like, “I’m afraid I have some
bad news for you.”
5. Mechanism of PE in diving:
• Air pressure in lungs remains high while external pressure decreases causing alveoli to
rupture
• Decompression Sickness: Occurs when gas bubbles obstruct blood vessels
6. Tonic clonic seizure? Benzodiazepine.
7. 60 years old fell at home and was brought with hip fracture, what is your next step in
management? take to OR (high risk of AVN)
8. 2nd finger and hand swelling, spread to forearm? Analgesia +tetanus +send to ortho for OR open
and clean
9. Lung cancer Hx, severe back pain? Analgesia + CT, consult
10. what you do to avoid framing > SBAR

11. A near drowning case management – same approach to trauma.


12. Picture of child with trunk maculopapular rash, with cough coryza and conjunctivitis preceding
the rash  Measles
13. More with delirium than dementia? Auditory hallucination
• Hallucination are more common with an acute cause, which delirium is.
14. Auditory hallucinations: Psychiatric (functional) Psychosis.
15. Visual hallucinations: Organic (medical, drugs, tumor) psychosis.
16. Unconscious patient not sure of pulse? Start CPR and call for help
17. When to stop CPR? Team leader tells you to stop for shocking.
18. True about face exam? Bell’s palsy cause ptosis by orbicularis oris paralysis.
19. Multi organ dysfunction score (or sth like that not sure what was it) composed of?

20. PE ECG? S1Q3T3


21. Water skiing patient, near drowning experience, came unconscious, what to do next:
A. jaw thrust, CPR, cervical precautions most probable answer bc only option that had
proper cervical protection measures and had CPR
B. C-spine collar, CPR etc, then head tilt chin movement.
C. LLD position and Intranasal O2
D. head-tilt chin lift, CPR
22. Athlete running a marathon on a hot day, comes with 40 degrees temp, altered LOC, mumblind,
and his HR is high, SBP 135, he is sweating, what will you do next? General cooling measure (bc
this is heat exhaustion)
23. worker in the sun: cool down coz heatscroke
24. Anaphylaxis: IM epinephrine I:1000 0.5mg IM big muscle (Quadriceps/vastus lateralis muscle)
25. Becks triad: JVD, muffled heart sounds, low blood pressure
26. Cauda equina (a. Old pt with back pain, b. full bladder + Hx of lung cancer)  IV steroids and
Urgent MRI.
27. DKA: Insulin Drip according to weight
28. Pulseless electrical activity management:
A. IV epinephrine
B. IV atropine
C. Defibrillation (It’s not non-shockable rhythm)
D. IV amiodarone
29. Case of Diabetic, with scrotal skin peeling: Fournier’s gangrene
30. Woman who had a RTA, had a splenectomy, got blood transfusions, 3 weeks later she presents
with high fever, chills, no GI, Resp or other Sx. Only abnormal was elevated AST/ALT?
A. Hep A
B. Hep C
C. Intraabdominal abscess
D. Ebola
E. Brucella
31. Woman with classic appendicitis with no rebound tenderness. What will you do next 
Consult surgery/OR
32. Facial never palsy  Steroids.
33. Facial palsy treatment: Corticosteroids and antiviral/acyclovir (ramsay-hunt Sx)
34. STD with PID (cervical motion tenderness, bilateral adnexal tenderness, and purulent
discharge) and Bac Vaginosis also for 1-week thin discharge with fishy odor, what
management?
A. Metronidazole, Clindamycin, Rocephin (Ceftriaxone)
B. Azithromycin, Rocephin (Ceftriaxone), Metronidazole
C. Doxycycline, Ciprofloxacin, Metronidazole
35. Malignant hyperthermia  Urgent admission
36. Cocaine overdose
• Characteristic sign: Mydriasis (Dilated pupils)
• Antidote: Benzodiazepine
• Never give Beta-blockers
37. Methotrexate indication  <4cm with no Fetal heart rate
38. Snake bite what is the most important thing?
A. Bring the snake with you if possible to the ED
B. Time is the most important thing (DIC risk greatest within 4hrs)
39. Febrile neutropenia in a leukemic patient, who has a hickmans catheter, comes with infection.
They took cultures, she is started IV paracetamol/Iv cefipime. with Ce Tx? Need to confirm this
A. Add Vancomycin
B. Switch to piperacillin + tazobactom
C. add some other abx
40. Pt with mild hypercalcemia:
A. Treat with hemodialysis
B. Calculate corrected calcium with plasma protein
C. Give fluids, steroids, aldronic acid
41. What you do at signout > ISBAR
42. what you do to avoid framing > ISBAR
43. atypical MI: Diabetic patient.
44. testicular torsion: examine and alert urology
45. rectal cancer (incomplete emptying symptoms)
46. Most common sexually transmitted Infection: HPV (as per CDC fact sheet)
47. Bacterial vaginosis; fishy odor of grayish white/yellow vaginal discharge; clue cells treatment:
Metronidazole
48. Chlamydia & Gonorrhea testing: DNA probe test
49. Pregnant woman with signs of PE/DVT > send for CTPE OR DOPPLER. Ideally would go with CT
but pregnancy makes that a problem
50. Balancing fluids in trauma  initial fluid resuscitation for trauma patients in hemorrhagic
shock consist of 2 L of isotonic saline
51. NS excess causes: hypercholemic acidosis
52. FFP for: (a) low platelets (b) Low coagulation factors.
53. Pancreatitis: High lipase (more specific) & Amylase, Emergency.
54. First line in Beta-Blocker toxicity: (a) Calcium gluconate (b) glucagon.
55. ESRD common develop Hyperkalemia
56. Abdominal aortic aneurysm: (1) ABC (2) Bedside US
57. Which u do in primary Suvery: u can only use board for 6 hr.
58. Non-maleficence: Do no harm
59. Cincinatti – FAST for stroke. Look more into this

60. What is the drug of choice for convulsions? Carbamazepine. First line is lorazepam. Need to
check this
• Lorazepam (Benzo) first-line agents for an actively seizing patient
• Phenytoin second line anticonvulsant in a continuously seizing patient
• Phenobarbital third line anticonvulsant

61. Obese woman presents with unilateral calf pain. She is also on OCPs. Her history is
unremarkable. General Physical Examination is normal. What do you do? Do a Doppler of her
calf since DVT can’t be ruled out.
62. Which of the following is an appropriate suture size? Face 6-0

63. Blood transfusion group type for certain patients: O+ for males and post-meno females
64. Hypovolemic shock: ABC, IV fluids, identifying source of hypovolemia
65. Retinal detachment on fundoscopy: diagnosis

66. Snake bite? Venom poisoning


67. Acetaminophen tablets swallow, no symptoms? Charcoal, measure Acetaminophen levels
68. Hyphema presentation (Picture from lecture): Raise head+analgesia

69. CO poisoning: tx, high flow O2/HBO


70. Angle closure glaucoma; Loss of vision + pain  Pilocarpine
71. Instrument not used to measure the IOP:
• Tonometry is the instrument used
72. Hemothorax X-ray + Hx of GW in chest: management, chest tube pointed downwards
73. ABCDE wrong abbreviation: D for Danger
74. which of the following is true in primary survey? (chin lift & jaw thrust?); Need scenario to
decide
75. what is true about the use of packed RBCs? (they are checked by 2 medical staff, reaction can
be predicted in some donors.
76. What is true about the 4:2:1 formula? (it is not used for pediatrics, i don’t remember the other
answers). It is used in peds, need to look up.
77. 25-year old with sudden onset left sided face weakness, what is your next step in
management?  (a) CT (b) Acyclovir. Need better scenario
78. What interventions are allowed in DNR patients: Feeding tube & Pain control (Analgesia)
79. A case of torsion. Which is correct? Early intervention is 90-100% successful
80. What is the most common cause of a sudden limp? Trauma
81. Eye trauma diagnostic assessment: Flurescein stain
82. Patient presents with right lower quadrant pain that started off as periumbilical pain,
anorexia, N&V (acute appendicitis), next step? -> immediate admission and surgical
consult
83. A patient develops headache + neck stiffness with NO fever -> subarachnoid hemorrhage
84. Thyroid storm: IV propranolol
85. Pulmonary embolism best diagnostic test: Chest CTPA
86. Paracetamol OD antidote: N Acetylcystine
87. Iron toxicity antidote: Deferoxamine
88. TCA OD antidote: Sodium bicarbonate
89. In cases of Carbonmonoxide poisoning which of the following is the firstline mainstay
treatment? High flow oxygen /HBO Therapy
90. Anticholenergic OD antidote: Physotigmine, glucose, benzo
91. Which is a feature of IV fluids: Colloid stays intravascular unless capillary leak
92. cause of cardiac arrest in renal failure patient <-- hyperkalemia
93. Which electrolyte abnormality can’t cause cardiac arrest: hypocalcemia
94. Slit lamp showes pus accumulating in the ant chamber >> hypopyon

95. DM, friable with granulation >> Biopsy it (necrotizing otitis externa)

96. A diabetic with middle ear discharge, pain especially at night, and facial nerve paralysis as well
as a friable tissue and asking you for the best step in management which included: biopsy, oral
steroids + antibiotics, iv antibiotics, oral antibiotic
97. which cranial nerve Close eyes >> CN7
98. patient in dashboard accident what's the most likely injury > PCL and posterior hip dislocation
99. patient in spinal shock after accident, which reflex is gone > Bulbocavernous reflex
100. person got bleach in his eye calls office what do you tell him > Irrigate 15 mins then come to
office
101. what causes primary open angle glaucoma > options included pupil obstructing flow, increased
production, decreased absorption, A + C (i think the answer but what different in some versions,
mine was correct)
102. Girl presents with eye pain and vesicular lesions around her eye? What does she have?
- MLA: Herpes zoster ophthalmicus
103. Patient presents with ptosis. If you suspect sympathetic damage as a cause (Horner’s
Syndrome), what another sign this patient should have? MLA: Miotic pupil
104. 63 year old man comes in with severe painful red eye and mid dilated pupil. He has nausea and
vomiting. What is the diagnosis? MLA: Acute angle closure glaucoma
105. Man had sudden painless loss of vision in one eye. Which of the followings can be the diagnosis?
MLA : Central Retinal Artery Occlusion (CRAO)
106. Leukemia on chemo after 24 hours has fever and neutropenia? Consult ID
107. Drug cause adrenal insufficiency? Etomidate
108. how can you differentiate between cardiac tamponade and tension pneumothorax? breath sounds
109. Hemophilia A pt presented with knee swelling/hemarthoses, management: give factor viii
immediately (only in severe cases joint aspiration is done, that too only after Coag factor
correction)
110. Hypovolemic shock management: IV crystalloid (3 liters of crystalloid per 1 liter of blood loss)
111. Contraindications to fly on airplanes:
• Pneumothorax
• Infants < 48hr old
• Pregnant women >36th wk & Multiple pregnancy > 32nd wk
• Unstable Angina, Recent MI or Stroke, Decompression Sickness, Raised ICP, Eustachian
tube blockage, Severe COPD
• Any communicable disease
• Sickle cell disease;
112. Sepsis Care guidelines; Look at table
• First 3 hours: Measure lactate
• During 6hrs: Mean arterial pressure (MAP) ≥ 65mmHg
• As per Surviving Sepsis Campaign Care Bundles which one of the following is the first
line choice for inotropes? Norepinephrine
• As per Surviving Sepsis Campaign Care Bundles which one of the following is the
recommended initial dose of fluids? 30 mls/kg of Crystalloids (repeated!)
113. Trauma series X-rays include which of these: Cervical Spine – AP, Chest – AP, Pelvis - AP
114. Diabetic ketoacidosis
• Follow progress with anion gap
• DKA you give insulin and IV-fluid, but you have low K, low Na, high glucose? Hold insulin
and give K
• Most likely electrolyte abnormality: Hypokalemia
115. Acute coronary syndrome
• Risk of MI and troponin: 85 % PPV
• What drug is proven to reduce second MI risk (Reduce mortality)/ Recommended for
all MIs: Beta blocker
• Low output heart failure: ACE inhibitor
• MI: 90mins for FMC to angio for accredited institutions
• MI? 2 mm 2 leads
• 1st dug in MI? aspirin

116. Arrhythmia related questions


• Supraventricular tachycardia (ECG identify) & stable: IV adenosine.
• SVT & unstable: Cardioversion.
• Ventricular tachycardia (ECG) + Unstable: Cardioversion.
• Vfib had CPR+ Defib ECG changed to brady? Atropine
• Most common complication of atrial fibrillation? stroke
• CPR Increases ventricular fibrillation time to shock
• Shocking a normal rhythm will lead to ventricular fibrillation.
• which patient to do synchronized cardiovert: a fib
• Ventricular fibrillation after defibrillation what are the medications to use: Epinephrine
+ Amiodarone.
• Ventricular fibrillation + No pulse (Unstable): 30:2 and defibrillation.
• Torsade (ECG)  Magnesium sulphate.
• Patient with Cardiac abnormalities had stroke, which of the following is the likely
cause? Atrial Fibrillation
• In Shockable Adult CardioRespiratory Resuscitation which of the following is the
correct ratio between chest compression and ventilation plus type of defibrillation?
30:2/ Asynchronous Defibrillation
• Sinus brady but no pulse? Atropine
• ECG of patient tachycardia, normotensive what do you do? Supportive measures
• What is the most common finding on an ecg for Pulmonary Embolism? Sinus
tachycardia
117. 20 year old man presents to ER after a fall from a horse. He is confused on arrival to ER but
localizes pain on stimulation. You assess him to be a case of mild head injury with GCS of 13/15.
Your next step is to:
A. Consult neurosurgery
B. Intubation
C. ABCDE
D. C-ABC and CT brain
E. Trauma code and admit for observation
118. After performing a procedure in the emergency room department- the next most important
step is to:
A. Documentation and taking pictures
B. Inform the family
C. Teaching the staff
D. Checking if the procedure is done right
E. Taking a quick break before attending next patient
119. Most appropriate statement regarding triage is:
A. There is one universal system
B. It is a foul-proof system
C. First come, first serve basis is maintained
D. Immediate action may be provided to the sickest
E. CTAS/ESI/START are all the same
120. 76 yrs old female with nausea, vomiting, and lower abdominal pain diagnosed with recent UTI
presented to emergency room with h/o being unwell for few days. O/E patient severely
dehydrated, tachycardia and tachypneic. Her HR: 136 BP: 110/70, R/R: 40, SAO2: 98%, She has
no drug allergies.
LABS: (1) Na: 130 mmol/L (2) Cl: 95 mmol/L (3) K: 5.5 mmol/L (4) HCO3: 20 mmol/L (5) Glucose:
30 mmol/L
Urinalysis: (1) Blood: -ve (2) Nitrogen: +ve (3) Leukocytes: +ve (4) Protein: -ve (5) Ketones: -ve
The next most appropriate management plan is to:
a) Ciprofloxacin IV and NaCl 0.9% 1 liter IV
b) Glucophage (Metformin) 500 mg BID
c) A & B
d) Insulin 10 units IV and NaCl 0.9% 2 liter IV with ciprofloxacin 400 mg IV
e) Insulin 10 unties SQ and Glucophage 500 mg PO now
121. 20 yr old male unconscious is brought to emergency department by medics. After ABC, the next
most appropriate intervention is:
a. Normal saline bolus 30 ml/kg
b. Thiamine 100 mg IV, Narcan 1 mg IV, D50 1 amp IV
c. Start CT brain
d. Intubation, activated charcoal
e. Drug toxscreen, ETOH level and gastric lavage

122. 50 yrs old burn victim develops fever, tachycardia, tachypnoea while receiving treatment on
burns ward. You make a diagnosis of SIRS. Which of the following is NOT an essential element
for diagnosis of SIRS (Systemic inflammatory response syndrome)?
a. Temp less than 36°C(96.8°F) or greater than 38°C(100.4°F)
b. Heart rate greater than 90 beats per minute
c. High lactate of 2 mmol/L
d. Tachypnea greater than 20 breaths per minute
e. Leukocytosis or leukopenia
123. A 33-year-old obese man comes to the emergency department because of abdominal pain
nausea and vomiting. O/E rebound tenderness and guarding in Rt iliac fossa. You provisionally
make a diagnosis of acute appendicitis. Which of the following will be essential before a patient
goes to OR?
a. Labs and overnight NPO
b. Labs and CT scan of abdomen and pelvis with contrast
c. Antibiotics, US abdomen, IVFs
d. SAMPLE history, exam, labs surgical consent and surgical consult
e. SAMPLE history, exam, CT scan and surgical consent
86. 40 years old farmer presents with reduced GCS, salivation, lacrimation, urination, diarrhea, GI
upset and emesis. He has been using a new insecticide spray today. O/E: P: 52/min, BP: 110/60mmHg,
RR: 36/min. Pupils are small on examination. What is the diagnosis?
a. Opioid poisoning
b. Organophosphate toxicity
c. Cocaine toxicity
d. Salicylate poisoning
e. Lead poisoning
124. 25 year old young man presents with sudden onset of complete weakness of one side of the
face. The clinical examination reveals as follows. Check this out

What is the most appropriate ER management?


a. Reassure, start acyclovir/steroids/artificial tears
b. Immediate CT scan of brain/ seniors thought this was the answer, however COMPLETE
weakness of one side of face = LMN injury, hence possibly Bell’s palsy which is treated
with steroids
c. Immediate MRI with referral to neurology
d. Stroke thrombolysis
e. Immediate referral to ENT
125. The optimal head position for intubating the patient under anesthesia is
a. Extension of the neck
b. Flexion of the neck and extension at atlanto-occipital joint.
c. Flexion of the neck, extension at the atlanto-occipital joint and jaw thrust
d. Extension of the neck and jaw thrust
126. 70 years old male presents to ED with acute symptoms of stroke. The patient is a candidate for
stroke thrombolysis (tPA) if:
a. Window of 6 hours
b. NIH stroke scale score >25
c. He has an INR > 3.5
d. Blood glucose is <50 mg/dl (2.7 mmol/L)
e. Head trauma 4 weeks ago

127. 80 yrs old man with chest pain collapsed in CCU. The cardiac monitor showed the
following rhythm

In shockable cardiorespiratory arrest resuscitation, which of the following drug combinations


are used:
a. Atropine & Adrenaline (epinephrine)
b. Atropine & Amiodarone
c. Vasopressin & bicarb
d. Adrenaline(epinephrine) & amiodarone
e. Adrenaline(epinephrine) & bicarb

128. You are the team leader in a cardiopulmonary resuscitation team attending an arrest in ER. The
patient is in the following rhythm on cardiac monitor

You decided this is a shockable rhythm and you start chest compression with ventilation. Which
of the following is most appropriate next step?
a. Defibrillation @ 100 Joules
b. Defibrillation @ 200 Joules
c. Synchronized cardioversion @ 100 Joules
d. Synchronized cardioversion @ 300 Joules
e. Periodic chest thump

129. 20 years old male with recent history of alcohol use presented to emergency room with sudden
loss of consciousness. On the monitor you note the following rhythm
A medication that may resolve this rhythm is:
a. Atropine
b. Adenosine
c. Magnesium sulphate
d. Calcium chloride
e. Epinephrine

130. 62 years old male attended ER in CardioRespiratory Arrest . Following Rhytm was seen on
Cardiac Monitor Which of the following Drug should be given during ACLS Resuscitation?
Adrenaline(Epinephrine)

131. In Shockable Adult CardioRespiratory Resuscitation which of the following is the correct ratio
between chest compression and ventilation plus type of defibrillation? 30:2/Asynchronous
Defibrillation.

132. A 21 years old male after a recent cruise holiday presents with fever, flu like symptoms, inguinal
lymph nodes and painful genital blisters and sores. Most prudent treatment option is
a. Metronidazole, doxycycline & ciprofloxacin
b. Diflucan
c. Azithromycin & rocephine
d. Famcyclovir
e. Clindamycin gel
133. All of the following are excited to read neurotransmitters that modulate pain except:
A. Substance P
B. ATP
C. Somatostatin
D. Glutamate
E. Aspartate
134. Patient rescued from fire, had soot in her mouth, what was given to her (it was cyanide
poisoning)? Hydroxocobalamin
135. A female presents to the ER with overdose of beta blocker and suicidal ideation, with unstable
heart block, which one of the following managements is least likely to help her:
A. Glucagon
B. Calcium gluconate
C. Hemodialysis MPA
D. Cardiac pacing (patient is unstable which excludes using pacing at this stage :/)
E. Dopamine
F. All the above can be done in BB toxicity apparently
136. A patient started having symptoms of stroke 2 hours ago. What is the time frame to get a non-
enhanced CT scan for this patient? Dr Wagleys lecture, there is a picture with the times)
A. 10 minutes
B. 15 minutes
C. 25 minutes
D. 45 minutes
E. 60 minutes
137. Patient asymptomatic had ingested acetaminophen tablets 4 hrs. ago, what’s next step
management?
A. Whole bowel irrigation
B. Check acetaminophen levels
138. A 4-year-old boy suddenly develops a limp? No other symptoms what is your diagnosis?
A. Transient synovitis
B. Growing pains
C. Avascular necrosis of femur
139. Patienthad Vfib was shocked, on third attempt showed rhythm: bradycardia. What do
you do next?
a. Give atropine
b. Check if he has pulse
140. What medication can help in bradycardia?
a. Atropine
141. When can nitrates not be given? CHF, Inferior, Last 24hr Viagra
142. Which medication improves morbidity/mortality post- MI? Beta-Blockers
Subdural hematoma: CT: hyperdense, crescent-shaped lesions

Epidural hematoma CT: hematomas appear lenticular or biconvex

X-ray Volvulus
X-ray Small bowel obstruction: multiple air fluid levels

X-ray of perforated hollow viscus: Air under diaphragm


Retinal detachment

Subarachnoid
Family medicine questions
1. Question describing a study with 2 groups of children with Pb exposure and without Pb
exposure and following them up to see who develops anemia, what type of study: Cohort
study
2. What is a cross-sectional study? Disease prevalence
3. What is a case-control study? Odds ratio.
4. EBM? Research+ experience + patient values
5. Number needed to treat?
6. Disease orientated? Disease-oriented evidence refers to the outcomes of studies that measure
physiologic or surrogate markers of health. ... Improvements in these outcomes do not always
lead to improvements in patient-oriented
7. If a drug is proven to lower LDL but does not affect patient’s life, this is an example of? Disease
oriented study (not patient oriented)
8. Strawberry tongue? Scarlet fever but ruleout Kawasi symptoms
9. Centor criteria? No cough, fever, tonsillar swelling, lymphadenopathy
10. Child with sore throat, fever, maculopapular rash all over, in hands and feet, conjunctival
injection, strawberry tongue. Dx? Kawasaki
11. Breast cancer hx treated, came with back pain symptoms  CT spine
12. Stretching exercise as initial management of a regular MSK complaint (I believe low back pain)
in a normal patient.
13. PDD + ve? Rule our active TB + prophylactic INH
14. Woman with pain at sole of the foot, worse in morning, better with long distance walking,
what is the Dx? Plantar fasciitis
15. Postmenopausal vaginal bleeding investigation? US
16. Screening detect 10 years earlier? Length time bias
17. 74 yo male per rectal blood? Colon cancer, rectal cancer
18. Seizure, non-blenching red pinpoint rash? Brain CT
19. Tertiary prevention for DM? retinopathy screening.
20. 35-year-old woman, who is not sure of her childhood immunization. Which vaccine should you
give her now?
A. HPV
B. Tetanus
C. Polio
D. Zoster
21. Female who lost her husband 4 months ago, and is sad, hallucinates about him. Mx?
A. SSRI
B. Bereavement clinic for 1 year
C. Antipsychotics
22. Suicide: what is true?
A. men widowed has highest risk
B. suicide is more common in winters
C. women are more successful
D. men attempt more
23. Primary dysmenorrhea: NSAIDs
24. Painless hematuria: Cytoscopy
25. Man, who is has prediabetes, pre-HTN, and High LDL, what will you do?
A. lower cholesterol, carb and reduce weight & exercise and F/U 3m
B. lower cholesterol, carb and reduce exercise schedule F/U in 1 year
26. What's true about depression? Need to verify this
• Maximum treatment is reached at 2-8 weeks of therapy
• No need for a follow up appointment after taking medications
27. Scabies picture with itching Hx  Scabies

28. Woman who wants osteoporosis scan (DEXA) > scan after age 65
29. HSV infection: Treatment for life
30. PPD positive: Isoniazid 6 months
31. Acid fast bacilli: TB pneumonia
32. Otitis externa most common organism: P. aeruginosa
33. 55 yr old man for follow-up: FOBT (55yr old)
34. Prostate cancer:

35. Iron deficiency Anemia: Ferritin


36. Fever + Pancytopenia: Bone Marrow Biopsy
37. Dysphagia old patient: (1) Esophageal cancer vs (2) Achalasia.
38. Combined OCPs are contraindicated in patients with Migraine with auras
39. Anal fissures vs fistula (loose stool or pus discharge)
40. Most cause of chest pain in family medicine: (1) MSK vs (2) GERD
41. Tension headache for which patient is now using around 200 pills a month: Chronic daily HA
(medication overuse headache)
42. Drug which reduces FBG and HBA1c. Is it disease or patient oriented?
43. Pregnancy: Sphincter tone decreases
44. Exercise: 5 times a week 30 mins. (40-60 minutes 5 times a week).
45. Respiratory syncytial virus transmission by droplets
46. Epiglottitis patients has characteristic presentation of drooling  Intubation in ED
47. cluster headache (stays for < 3 hr on daily basis)
48. Pregnant lady with cat exposure and lymphadenopathy > toxoplasma. (Bartonella henselae if
scratch).
49. Vitamin B12 deficiency in gastrectomy
50. What is the most common cause of UTI? E. Coli
51. What cause Infectious Mononucleosis? EBV
52. Most common cause of chronic cough? Post-nasal drip
53. Lyme disease after tick bite: presented with palsy, Borrelia burgdorferi
54. Facial nerve paralysis + Vesicles in ear; diagnosis + treatment: Ramsey hunter syndrome; Tx
with antiviral + analgesia. Check more.
55. 28 yo female not married (not sexually active), want to do cervical screening. What do you tell
her: do HPV test, she can't do it (out of the screening people)? Need to check this

56. Duodenal Ulcer cause episodic epigastric pain.


57. Agoraphobia: Fear of crowds
58. When to worry and send the patient to the neurologist: Weak planter flexion. MRI
59. Trams with burse to lower back, on CT abdomen there is a kidney cyst, labs are normal: Simple
renal cyst. Need to check this

60. 70 year old male had blunt trauma. On examination he has contusions on his lower back. CT
showed 1 kidney cyst on the right side measuring 2 cm. Both kidneys are of normal size. RFTs
all normal. Dx.? (a) PCKD (b) simple renal cyst (c) RCC (d) recurrent UTI (e) prostatic nodular
hyperplasia
61. child can't sleep well due to anal itching, it is due to: ascariasis, pinworms, giardiasis.
62. 60-year-old lady has MCP joints and PIP nodules (and stiffness I think)? (a) Rheumatoid
arthritis (b) osteoarthritis.
63. Lady feels fatigued for the past few years, swollen red PIP and DIP with nodules of elbow --> RA.

64. Changing bowel habits & Some Cramps with bleeding: Advice colonoscopy as the colon CA
risk.
65. Pregnant 32 wks with E. coli UTI: Treat with antibiotics
66. 55yr old patient with dyspepsia what is your first line investigation? (1) Urea breath test (2)
Endoscopy only if patient > 60y or atleast 1 alarm feature.

67. Deep neck space infection organism: Polymicrobial


68. Chronic Back pain; Evidence-based treatment: exercise
69. Red flag for back pain; weak leg dorsiflexion (I put this)? or saddle anesthesia? Confirm this.

70. Young male with bloating, cramps & changing bowel habits: IBS
71. A woman is anxious, and she had tried psychotherapy sessions, but she's still anxious. You will
give her medical therapy. The first line would be? SSRI
72. Adhesive material placed around wound edges needs how long to hold it before release? 30
seconds; At no time should the applicator tip be pressed into the wound; At least three layers
should be applied to ensure optimal strength.
73. (This case is for the next three questions) 25-year-old man went to Thailand and came back, he
is currently suffering from severe knee pain:
• Which of the followings is an essential part of history in this case? Having multiple
sexual partners
• What is the next step management that would determine what is wrong with his knee?
Knee aspiration
• What are you suspecting in this patient? Septic arthritis (N. Gonorrhea)
74. (This case is for the next two question) A 35-year-old female patient presenting with a complaint
of dizziness, she describes her dizziness as “like she is out of this world”, all results of labs and
investigations were negative except, however she has accepted a new job which is very
competitive and stressful:
• Which is the least next action to be taken in this condition? Lab work + imaging
• What is the most probable diagnosis? Non specific dizziness
75. (This case is for next two questions) Patient with DM, he developed dizziness.
• Which history question is essential in the start of history taking? (1) what do you
mean by dizziness (2) The dizziness happens in which circumstances
• What physical examination you do to narrow your differential diagnosis? (1) Measure
BP supine, sitting, standing
76. Which of the following is correct regarding Combined oral contraceptive pills:
A. Increases risk of endometrial cancer
B. Decreases incidence of cervical cancer
C. Increases Risk of DVT
77. Which medication induces ovulation and restores the monthly period in a patient with
Polycystic ovary syndrome? (a) Metformin (b) Misoprostol
78. CURB-65->used for assessment, abx and decision for admission/mortality
i. Confusion of new onset (defined as an AMTS of 8 or less)
ii. Blood Urea nitrogen greater than 7 mmol/l (19 mg/dL)
iii. Respiratory rate of 30 breaths per minute or greater
iv. Blood pressure less than 90 mmHg systolic or diastolic blood pressure 60 mmHg or less
v. Age 65 or older

79. Irritable bowel syndrome


• CBC normal = IBS
• IBS treatment: mebeverin (antispasmodic). Need to check this
• IBS will show: (a) mucus stool (b) hard to flush (c) float stool (d)pale/clary stool.
• IBS worsened by caffeine
• IBS in > 50yo, rule out ovarian pathology
• True about IBS? Better with defecation
80. Smoking cessation

• Norepinephrine reuptake inhibitor (bupropion)


• Tobacco clinics in Saudi Arabia: 70
• Smoker patient failed nicotine patches treatment, next step? (a) Counselling (b)
Bupropion.
• which of the following is most effective regarding smoking cessation? advise the patient
every visit.
81. Diabetes mellitus management
• How to increase insulin sensitivity: (1) thiazolidinediones
• Hypertensive patients & Risk of renal failure: ACE inhibitors
• For hypoglycemic coma, the treatment of choice is: 50 milligram glucoses/dextrose IV.


• Which of these diabetic drugs increase weight: (a) Metformin (b) Acarbose (c) Glyburide
(Sulfonylurea) (d) Sitagliptin
• Which of these diabetic drugs increase weight: Liraglutide (GLP1)
• Malignant otitis media or extrena common in diabetic.

• Glargine; longest duration available in KFSH, not sure.


• 1st line drug for the treatment of newly diagnosed DM -> Metformin
• Drug decrease insulin production? Octreotide

82. Hypertension management
• to what do you correct blood pressure: 140/90. Goal depends on patient age; check
table
• Which of the followings not present as an oral preparation for HTN? (Atenolol,
Nifedipine, Thiazide, Furosemide, Captopril). Sodium Nitroprusside
• Malignant HTN management: Sodium Nitroprusside
• Pregnant patient switch to: Methyl-Dopa
• routinely for newly diagnosed hypertension with no previous investigations: EKG
renin.
• Male patient diagnosed with uncomplicated HTN what is the 1st line treatment. (a)
Carvedilol (b) Amlodipine (c) Hydralazine
• African American patient with HTN: (1) Calcium Channel blocker (2) Thiazide
• Man, with HTN is on ACE i, develops chronic cough so doctor changes his
medication. Which medication did he change it to -> Losartan (ARB)
• Initial HTN routine investigation? Urinalysis
83. Asthma management
• Decrease allergens (a) decrease humidity (b) Not washing beds.
• Patient’s symptoms no longer control with short acting beta-agonist  ICS
• Patient’s symptoms no longer control with short acting beta-agonist (Albuterol) + High
dose ICS  Add LABA (Salbuterol) or oral steroids.
• Which antihypertensive should be avoided in bronchial asthma patients? Labetolol
• Uncontrolled severe asthma: (a) FEV1/FVC ratio less than 70%, (b) FEV1/FVC ratio is
normal.
• True about asthma? Increased with b-blocker and aspirin
84. COPD
• Diagnosis: Spirometry
• A patient with COPD. Check previous culture.
85. Degrees of prevention
86. What are IPSG (International patient safety goals) Goal 1 & 2
i. Goal 1 - Identify Patients Correctly
ii. Goal 2 - Improve Effective Communication
iii. Goal 3 - Improve the safety of high-Alert Medications
iv. Goal 4 Ensure correct Site, Correct Procedure, Correct Patient Surgery
v. Goal 5 - Reduce Risk of Health Care-Associates Infections
vi. Goal 6 - Reduce the Risk of Patient Harm resulting from Fall
87. Medical Trainee JC’s AMC Orientation Includes: Any Ongoing Required Education, International
Safety Goals, Infection Control Program, Medication Safety Program.
88. Which of the followings best define Never events:
• "adverse events that are serious, largely preventable, and of concern to both the public
and health care providers for the purpose of public accountability." Are 29 Particularly
Shocking Medical Errors Causing Death or Significant Injury of Patient or Staff, classified
to 7 categories. IPSG 4 is geared toward Never Event

https://psnet.ahrq.gov/primers/primer/3/never-events

89. Menopause criteria: Age greater than 50yrs & 12 months of amenorrhea

90. Secondary screening: (a) Cervical cancer (b) hypertension


91. Herpes zoster/varicella of ear. Need to check this
92. Mild otitis media management. Analgesia and Amoxicillin
93. Acute otitis media: (a) Amoxicillin (b) Augmentin
94. Malingering 15 yrs old girl claims she can’t see her mobile
95. Definition of malingering? Malingering: is the intentional faking or creating of illness in order to obtain
secondary gain (e.g. workers compensation, disability payments, avoiding work or jail time, pain medication,
96. next step in pts that has closed ear canal? Clean it out
97. Light receptor cells? Cone & rods

98. Recurrent epistaxis Pic of the mouth? Telengiectasia.

99. Young man with nasopharyngeal tumor +epistaxis what is the next step? Imaging.
100. 51 year old IUD + not having period for year? Menopause
101. What is true about bariatric surgeries? Nutritional deficiencies can be replaced with
supplements
102. Negative birefringence: Gout
103. Positive birefringence: Pseudogout
104. epistaxis with pic of lip & tongue have dots: hereditary hemorrhagic telangiectasia,
105. tongue: Kaposi sarcoma, hairy leukoplakia’s, SCC. (a) Surgery (b) surgery with neck
dissection
106. Acute vestibular neuritis: corticosteroids
107. Female with HTN and facial acne and hirsutism  Cushings (if no HTN, PCOS)
108. Vaccination to a woman in a prenatal visit with no rubella antibodies should be AFTER
delivery (right after)
109. Best treatment of low HDL: 1st exercise (best drug: niacin)
110. Best treatment of high LDL: statin
111. Megaloblastic anemia: (1) folic acid def (2) B12 deficiency
112. HBV + surface antibodies and negative antigens  good immunity against HBV
113. Stretching as a treatment to neck or back pain.
114. Suspect low PTH in post-thyroidectomy pt with hypocalcemia
115. DMPA Given every 3 months
116. Frozen shoulder presentation

117. Chronically constipated, anal pain and itching: (a) hemorrhoids (b) fissures. Fissure
more likely.
118. 21 lady just came from Africa and diagnosed with malaria what is the most
specific test to make the diagnosis of malaria: thick and thin blood film
119. 71 old man diabetic, feel dizziness when he wakes up from the bed for the last 7
days he had syncope , he is on aspirin , frusemide and beta blocker what is the cause :
orthostatic hypotension
120. Patient with cor pulmonale what is the cause: pulmonary fibrosis, pneumonia
121. familial cholesterolemia exercise not helpful, LDL more than 6 what to do?
Simvastatin
122. man, with fatigue and shortness of breath Hb is 7.5 microcytosis what type of
deficiency? Iron defiency anemia
123. Child with recurrent chest infections, malabsorption symptoms, chronic sinusitis,
and nasal polyps. What test would you do to confirm your suspected diagnosis?
Sweat chloride test
124. How can you be cost effective in family practice?
Options included: generic prescription, drug formulations with multiple drug.
125. When to remove copper coil -> 5 or 7 years? I guess copper = 10, progesterone – 3-5
yrs.
126. Risk for breast cancer -> alcohol consumption
127. Which of the following is true about mammograms -> low radiation, painful
128. Supplementation of which vitamin has known health benefits -> Vitamin D
129. Which of the following can be given to a 62 y/o woman to decrease fracture risk ->
alendronate (just to point out that the guideline is for >65yr old women)
130. CVS risk assessment in a low risk individual and some tests were already done and
asking what other tests to do? -> do nothing
131. First line treatment for 1ry dysmenorrhea -> NSAIDs
132. Pubmed is an example of -> 1ry source
133. First step in the management of OSA -> weight loss
134. Complication of OSA -> CHF
135. Having difficulty communicating with patient, what should you do? Prevent pt from
going off topic / open ended questions
136. Single blood test that would be expected to be the most sensitive for determining
whether the patient is euthyroid, hypothyroid or hyperthyroid? -> TSH
137. A nurse sustained a needle stick injury while assisting in a procedure at the clinic
yesterday. The patient’s status is unknown for any blood-borne illnesses. The nurse is
vaccinated against Hep B. Which of the following is the most appropriate next step? Investigate
both the nurse and patient, for blood-borne illnesses
138. Diagnostic test for gout -> joint aspiration
139. Lady with amenorrhea, weight gain, etc. -> Hypothyroidism vs Cushing’s Bc with
hypothyroidism it’s more of menorrhagia not amenorrhea
140. Which of the following would indicate poor DM control -> HbA1c of 10%
141. Kid has 3 months hx of abdominal pain, now he can’t attend school, which of the
following would hint towards an organic rather than a functional cause -> headache, anxiety,
suprapubic pain, family hx of abdominal pain??
142. The loss of a loved one results in grief as universal human behavior, the following
Is the most appropriate statement regarding the stages (denial, anger, bargaining,
depression, and acceptance):
a. Everyone must go through the stages in order
b. Medications should be used to shorten the grieving process
c. One may go through grieving process without passing through all stages of grieving
d. The stages of grief have been validated in randomized control trials
e. Normally, grief should be over by 1 month
143. Depression on SSRI, partial improvement? Increase dose
• If there is an inadequate response but good tolerability after four weeks of
treatment at the recommended minimum effective dose, the dose can be slowly
titrated upward. Further dose increases can generally be made at one to four-
week intervals as needed.
144. Mild depression? CBT
145. GAD? CBT
146. Lobar pneumonia case, management? 30 ml/kg IV fluids
147. Koilonychia because of iron deficiency
148. Anisocytosis? Electrophoresis, thyroid function test
149. Prevention of migraine? Betablockers & Topiramate.
150. Acute migraine: Sumatriptan or Aspirin + Metoclopramide
151. 4 yo limp, healthy? Trauma, growing pain
152. prediabetic: lose weight to not become diabetic/ or rescreen within one year
153. supraspinatus injury (can’t abduct 90 degrees) (Re confirm)
154. what’s the most common mistakes: wrong medication vs wrong patient?
155. Which of these scenarios reflects EBM: Using drug x instead of drug Y after
proven to be better from RCT
156. How to differentiate between Delirium and Dementia?
a. Visual hallucintions
b. Onset and speed of progression of symptoms
157. What is the most-potent and proven effective long-term treatment of Asthma?
a. Inhaled corticosterioids
b. SABA
c. LABA
158. Which has a proven benefit for chronic lower back pain?
a. Massage
b. Acupuncture
c. Aroma therapy
d. Exercise
Subacute is heat pad & massage

159. Patient for pre-employment screening has fatigue and weakness, sister has
anemia. What investigation would help? Hb electrophoresis
160. A man presents with suspicious cut on hand and claims he fell down. What is
your management? (suspected of fighting and cut is human bite)
161. A kid with a snake bite presented with diffuse redness and two puncture wounds
and bruising across body. How would you manage?

162. Patient with weakness and numbness in lower back and buttocks worsens when
he coughs or sneezes. What’s the most important investigation?
a. MRI of whole spine
b. CT scan
163. New test detects cancer earlier, but patient dies at same age so does not
increase life expectancy. This is an example of? (study)
a. Lead-time bias
164. Patient from India trip came back with diarrhea for last 4 days. Most likely
organism?
a. Rotavirus
b. Shigella
c. E coli
165. Immunocompromised patient admitted and given Ceftriaxone and blood
cultures taken. What’s the next step?
a. Wait for culture and consult ID for antibiotic regimen
b. Add antifungal Abs
c. Start on Vancomycin
166. Lyme disease after tick bite: presented with palsy, Borrelia burgdorferi
167. Ankylosing spondylitis: presented with sacroiliac stiffness
1. Which one is a IV hypnotic?  Propofol
2. Chemical arteritis can be caused by thiopental
3. Pain with Propofol? Add lidocaine
4. Morphine which of the following isn’t a side effect: hallucinations
5. Causes of difficult airway: arched palate, micrognathia, no teeth, denture wearing, beard, ect…
(I forgot the exact choices, but these are the correct answer).
What reduces the EtCO2?
A. Decrease minute ventilation
B. Increase tidal volume
C. Decrease RR (Increased EtCO2)
D. malignant hyperthermia

A towel can hold 100-150ml

70ml/kg
20ml

Normal EtCO2 is 35-45mmHg

Signs of severe hypothermia: No shivering, coma/decreased consciousness signs

1. More with delirium than dementia? Auditory hallucination


• Hallucination are more common with an acute cause, which delirium is.
2. Auditory hallucinations: Psychiatric (functional) Psychosis.
3. Visual hallucinations: Organic (medical, drugs, tumor) psychosis.
4. True about face exam? Bell’s palsy cause ptosis by orbicularis oris paralysis.
5. Multi organ dysfunction score (or sth like that not sure what was it) composed of?

6. Pulseless electrical activity management:


A. IV epinephrine
B. IV atropine
C. Defibrillation (It’s not non-shockable rhythm)
D. IV amiodarone
7. Facial never palsy  Steroids.
8. Facial palsy treatment: Corticosteroids and antiviral/acyclovir (ramsay-hunt Sx)

9. Snake bite what is the most important thing?


A. Bring the snake with you if possible to the ED
B. Time is the most important thing (DIC risk greatest within 4hrs)

10. Febrile neutropenia in a leukemic patient, who has a hickmans catheter, comes with infection.
They took cultures, she is started IV paracetamol/Iv cefipime. with Ce Tx? Need to confirm this
A. Add Vancomycin
B. Switch to piperacillin + tazobactom
C. add some other abx
11. Pt with mild hypercalcemia:
A. Treat with hemodialysis
B. Calculate corrected calcium with plasma protein
C. Give fluids, steroids, aldronic acid

12. Most common sexually transmitted Infection: HPV (as per CDC fact sheet)
13. Bacterial vaginosis; fishy odor of grayish white/yellow vaginal discharge; clue cells treatment:
Metronidazole
14. Pregnant woman with signs of PE/DVT > send for CTPE OR DOPPLER. Ideally would go with CT
but pregnancy makes that a problem

1. Pancreatitis: High lipase (more specific) & Amylase, Emergency.


2. First line in Beta-Blocker toxicity: (a) Calcium gluconate (b) glucagon.

1. Retinal detachment on fundoscopy: diagnosis


1. Hyphema presentation (Picture from lecture): Raise head+analgesia

Angle closure glaucoma; Loss of vision + pain  Pilocarpine

What is the most common cause of a sudden limp? Trauma


Eye trauma diagnostic assessment: Flurescein stain

Anticholenergic OD antidote: Physotigmine, glucose, benzo


Which electrolyte abnormality can’t cause cardiac arrest: hypocalcemia

1. Slit lamp showes pus accumulating in the ant chamber >> hypopyon

15. DM, friable with granulation >> Biopsy it (necrotizing otitis externa)
1. A diabetic with middle ear discharge, pain especially at night, and facial nerve paralysis as well
as a friable tissue and asking you for the best step in management which included: biopsy, oral
steroids + antibiotics, iv antibiotics, oral antibiotic
2. which cranial nerve Close eyes >> CN7
3. Patient presents with ptosis. If you suspect sympathetic damage as a cause (Horner’s
Syndrome), what another sign this patient should have? MLA: Miotic pupil
4. Drug cause adrenal insufficiency? Etomidate
5. Hypovolemic shock management: IV crystalloid (3 liters of crystalloid per 1 liter of blood loss)

16. Sepsis Care guidelines; Look at table


• First 3 hours: Measure lactate
• During 6hrs: Mean arterial pressure (MAP) ≥ 65mmHg
• As per Surviving Sepsis Campaign Care Bundles which one of the following is the first
line choice for inotropes? Norepinephrine
• As per Surviving Sepsis Campaign Care Bundles which one of the following is the
recommended initial dose of fluids? 30 mls/kg of Crystalloids (repeated!)

1. Trauma series X-rays include which of these: Cervical Spine – AP, Chest – AP, Pelvis – AP
2. DKA you give insulin and IV-fluid, but you have low K, low Na, high glucose? Hold insulin and
give K
3. 1st dug in MI? Aspirin0
Most common complication of atrial fibrillation? stroke
CPR Increases ventricular fibrillation time to shock

17. After performing a procedure in the emergency room department- the next most important
step is to:
A. Documentation and taking pictures
B. Inform the family
C. Teaching the staff
D. Checking if the procedure is done right
E. Taking a quick break before attending next patient
18. 50 yrs old burn victim develops fever, tachycardia, tachypnoea while receiving treatment on
burns ward. You make a diagnosis of SIRS. Which of the following is NOT an essential element
for diagnosis of SIRS (Systemic inflammatory response syndrome)?
a. Temp less than 36°C(96.8°F) or greater than 38°C(100.4°F)
b. Heart rate greater than 90 beats per minute
c. High lactate of 2 mmol/L
d. Tachypnea greater than 20 breaths per minute
e. Leukocytosis or leukopenia

19. 25 year old young man presents with sudden onset of complete weakness of one side of the
face. The clinical examination reveals as follows. Check this out

What is the most appropriate ER management?


a. Reassure, start acyclovir/steroids/artificial tears
b. Immediate CT scan of brain/ seniors thought this was the answer, however COMPLETE
weakness of one side of face = LMN injury, hence possibly Bell’s palsy which is treated
with steroids
c. Immediate MRI with referral to neurology
d. Stroke thrombolysis
e. Immediate referral to ENT
If the patient can close his eyes, and use forehead frontalis muscle, it means its only upper motor
neuron lesion which is a stroke and you need to do CT

20. The optimal head position for intubating the patient under anesthesia is
a. Extension of the neck
b. Flexion of the neck and extension at atlanto-occipital joint.
c. Flexion of the neck, extension at the atlanto-occipital joint and jaw thrust
d. Extension of the neck and jaw thrust

Prompt defibrillation is indicated in patients with hemodynamically unstable torsades de pointes.


In Shockable Adult CardioRespiratory Resuscitation which of the following is the correct ratio between
chest compression and ventilation plus type of defibrillation? 30:2/Asynchronous Defibrillation.

21. A female presents to the ER with overdose of beta blocker and suicidal ideation, with unstable
heart block, which one of the following managements is least likely to help her:
A. Glucagon
B. Calcium gluconate
C. Hemodialysis MPA
D. Cardiac pacing (patient is unstable which excludes using pacing at this stage :/)
E. Dopamine
F. All the above can be done in BB toxicity apparently

When can nitrates not be given? CHF, Inferior, Last 24hr Viagra

Subarachnoid
Otitis externa most common organism: S. aeruginosa
Combined OCPs are contraindicated in patients with Migraine with auras
cluster headache (stays for < 3 hr on daily basis)

Most common cause of chronic cough? Post-nasal drip

Facial nerve paralysis + Vesicles in ear; diagnosis + treatment: Ramsey hunter syndrome; Tx with
antiviral + analgesia. Check more.

60-year-old lady has MCP joints and PIP nodules (and stiffness I think)? (a) Rheumatoid arthritis (b)
osteoarthritis.
Elbow nodes =RA

Deep neck space infection organism: Polymicrobial

Adhesive material placed around wound edges needs how long to hold it before release? 30 seconds;
At no time should the applicator tip be pressed into the wound; At least three layers should be applied
to ensure optimal strength.

CURB-65->used for assessment, abx and decision for admission/mortality


i. Confusion of new onset (defined as an AMTS of 8 or less)
ii. Blood Urea nitrogen greater than 7 mmol/l (19 mg/dL)
iii. Respiratory rate of 30 breaths per minute or greater
iv. Blood pressure less than 90 mmHg systolic or diastolic blood pressure 60 mmHg or less
v. Age 65 or older

IBS treatment: mebeverin (antispasmodic).


Which of these diabetic drugs increase weight: Glyburide (Sulfonylurea), thiazolidinediones
Which of these diabetic drugs decrease weight: Liraglutide (GLP1), Gliptin (DDLP-4 inhibitors)
Glargine (Insulin); longest duration
Drug decrease insulin production? Octreotide
Patient’s symptoms no longer control with short acting beta-agonist  ICS
Patient’s symptoms no longer control with short acting beta-agonist (Albuterol) + High dose ICS  Add
LABA (Salbuterol)

primary prevention: identify risk factors for common diseases; counsel patients to promote healthy
behavior. prevent disease or injury before it ever occurs. Vaccination

Secondary prevention: presymptomatic detection of disease to allow early treatment and to prevent
disease progression. Aims to reduce the impact of a disease or injury that has already occurred.
Mammograms or Aspirin in a patient with stable angina

Tertiary prevention: soften the impact of an ongoing illness or injury that has lasting effects. Carotid
artery endarterectomy in patient that a stroke.

What are IPSG (International patient safety goals) Goal 1 & 2


i. Goal 1 - Identify Patients Correctly
ii. Goal 2 - Improve Effective Communication
iii. Goal 3 - Improve the safety of high-Alert Medications
iv. Goal 4 Ensure correct Site, Correct Procedure, Correct Patient Surgery
v. Goal 5 - Reduce Risk of Health Care-Associates Infections
vi. Goal 6 - Reduce the Risk of Patient Harm resulting from Fall
1. Menopause criteria: Age greater than 50yrs & 12 months of amenorrhea

2. Best treatment of low HDL: 1st exercise (best drug: niacin)


3. Best treatment of high LDL: statin

Lobar pneumonia case, management? 30 ml/kg IV fluids

supraspinatus injury (can’t abduct 90 degrees)

Lecture stuff

CXR of primary PTX (< 20% of the hemithorax)  observe these patients for 6 hours and then repeat
CXR  no increase in the size of the PTX  the patient can be discharged with follow-up in 24 hours.

Small Pleural Effusion (5-50ml) CXR positioning: lateral decubitus film with the affected side down.
They > 200ml fluid to be seen on PA supine.
Thermal burns: Singed nares  Pneumonitis.

Exudative fluid: (1) LDH > 200 units (2) LDH ratio > 0.6 (3) Protein ratio > 0.5
Alcoholic ketoacidosis Lab findings: β-hydroxybutyrate (β-OH)/acetyl acetate (AcAc) formation ratio
is 5:1

Small bowel obstruction


Clinical presentation: vomiting, intermittent crampy abdominal pain, abdominal distention,
hyperactive bowel sounds and general tenderness. No flatus or bowel movement.

Testicular torsion
• Absent cremasteric reflex
• Prehn sign: the physical lifting of the testicles relieves the pain of epididymitis but not pain
caused by testicular torsion
Epididymitis: Most common causative organism: E coli (Age >35) or C trachomatis & Neisseria
gonorrhoeae (Age <35). Ceftriaxone + Doxycycline

Blumberg sign: Occurrence of a sharp pain when the examiner presses his or her hand over McBurney
point. This sign is indicative of peritoneal inflammation.

1) Ceftriaxone or ciprofloxacin for N. gonorrhea (2) Azithromycin or Doxycycline for Chlamydia.

Standard anticonvulsants: (1) diazepam, (2) phenobarbital, & (3) phenytoin


Osmolar Gap

• Osmolar gap > 50 mOsm/L = Toxic alcohol ingestion

A nasal airway is placed into the nostril of a somnolent patient with an intact gag reflex.
An oral airway is a rigid instrument that is used to prevent the base of the tongue from occluding the
hypopharynx. It should be used to maintain the airway only in a patient with an absent gag reflex

Benign Positional vertigo: transient positional vertigo associated with nystagmus


Diagnostic test: Dix-Hallpike maneuver
Treatment: Epley Maneuver: a series of head and body turns that reposition the canalith.

Bronchiolitis most common cause is RSV. Drooling


Croup most common cause is Parainfluenza. “seal-like”/Barking cough. Steroids + Racemic
epinephrine

Which opioid is least likely to cause hypotension: Fentanyl


Which agent causes truncal & jaw muscle rigidity: Fentanyl
Which agents may precipitate bronchospasm in patients with reactive airway disease: Methohexital
(Barbiturate). ketamine may have a mild, transient bronchodilatory effect.
Which agent provides analgesia in addition to sedation: Ketamine

Narrow complex Tachycardia (Supraventricular tachycardia/SVT)

1. ECG findings: QRS complex duration < 100ms & ventricular rate > 100bpm. Absent P waves
but T waves present
2. management for stable patient:
• First try to Increase vagal tone via (1) Carotid sinus massage (2) Valsalva maneuver
(3) Facial immersion in cold water
• 2nd option is adenosine (6mg IV push, then 6mg again; 3rd time give 12mg)
• 3rd option is Class II or IV arrhythmias
3. Initial management for unstable patient: Synchronized Cardioversion (50-100 Joules)

Wide complex Tachycardia (Ventricular tachycardia)

1. ECG findings: QRS complex duration > 120ms & ventricular rate > 100bpm. Absent P waves
(only present with Bundle branch block)
2. Initial management for stable patient:
• Primary agent of choice: adenosine (6mg IV push, then 6mg again; can go upto 12mg)
• 2nd line agents Amiodarone or Procainamide.
3. Initial management for unstable patient: Synchronized Cardioversion 100 Joules

Ventricular Fibrillation: pulseless VT, these are nonperfusing rhythms


ECG
• Chaotic irregular deflections of varying amplitude
• No identifiable P waves, (irregular) QRS complexes, or T waves
• Rate 150 to 500 per minute
• Amplitude decreases with duration (coarse VF -> fine VF)
Management: Defibrillation Cardioversion (300 Joules)
Other considerations: Usually no pulse, If with pulse it’s an artifact.

Atypical MI: Epigastric discomfort, indigestion, or nausea & vomiting: Women, Elderly, diabetics,
Inferior wall MI
contraindications to the use of β-blockers include asthma/COPD, CHF, and third-trimester pregnancy,
cocaine, acute decompensated heart failure, hypotension
contraindications to the use of Nitroglycerin: (1) Right ventricular infract/Inferior myocardial
infraction, (2) Systolic heart failure, (3) Sildenafil/Viagra (within previous 24hrs)
Comatose patients in ER: L7
Primary survey
• Airway: Brainstem lesions.
• Breathing
• Circulation
• Disability: Focal neurological deficit.
• Exposure: Frost bite
• Vital Signs
• Blood glucose
• Brief History
• Secondary Survey: Head to Toe

Glasgow coma scale: assesses a person’s (1) eye, (2) verbal, and (3) motor responsiveness.

It is recommended to intubate patients with a GCS score of 8 or less for airway protection.
Exclusion criteria for the use of thrombolytics include systolic BP > 185 mm Hg or diastolic BP > 110 mm
Hg
Don’t treat HTN in ischemic stroke unless (1) Thrombolysis candidates (2) SBP > 185 or DPB> 100.
Labetalol.

Strep pharyngitis
Factors that increase MAC
• Hyperthermia
• Hypernatremia
• Chronic alcohol and opioid abuse
• Increase in pressure
Factors that Decrease MAC
• Hypothermia
• Increasing Age
• Hypoxemia
• Hypotension
• Anemia
• Pregnancy
• CNS depressant drugs (benzo, opioids)
• Acute Alcohol intoxication
• Local anesthetics (except cocaine)
No Change in MAC
• Sex
• Weight
• Duration of anesthesia
• Hypo/hyperthyroidism

Desflurane: Transient increase in heart rate and blood pressure (humans)

Propofol
• Hypoventilation and Apnea.
• Propofol Infusion Syndrome: Lactic acidosis after prolonging high dose >24hr,>75 μg/kg/min
• Bacterial Growth
• Pain with induction: Small vein, dorsum hand treats with large vein and lidocaine.
Etomidate
• Myoclonus movement.
• Preferable IV induction in Cardiovascular Unstable Patient
• Adrenocortical Suppression: (1) Does dependent inhibition of conversion cholesterol to cortisol
(2) Better to avoid in Sepsis or Hemorrhage (3) Increased 30 days mortality.
Thiopental
• Venous thrombosis
• Chemical arteritis with arterial occlusion and ischemia after accidental intra-arterial injection.
• Acute Intermittent Porphyria.
Ketamine
• Cataleptic state, eye open with slow nystagmic gaze, amnesia and analgesia
• Preservation of respirations, bronchodilation, ↑ HR

independent risk factors of difficult mask ventilation: (1) Age > 55 years of age (2) BMI > 26 (3) History
of snoring (obstruction) (4) Beard (5) Edentulous
Risk Factors for Difficult Intubation
o Mouth opening < 4 cm
o Thyromental distance < 6 cm
o Mallampati grade 3 or greater
o Neck movement < 80%
o Inability to advance mandible (prognathism)
o Body weight > 110 kg
o Positive history of difficult intubation
o < 9 cm from symphysis of mandible to mandible angle (Thyromental angle)
Mallampati Classification
• Class I: soft palate, tonsillar fauces, tonsillar pillars, and uvuala visualized
• Class II: soft palate, tonsillar fauces, and uvula visualized
• Class III: soft palate and base of uvula visualized
• Class IV: soft palate not visualized
• Class III and IV: Difficult to Intubate
• Traditional Pre-oxygenation: 3 minutes of tidal volume breathing at 5 ml/kg 100% O 2
• Rapid: 8 deep breaths within 60 seconds at 10 L/min

Experimental Event rate: Affected number/Total number in experimental arm


Control Event rate: Affected number/Total number in control arm
RR= EER/CER

NNT=Number needed to harm

You might also like